Sat Practise Test 2

  • May 2020
  • PDF

This document was uploaded by user and they confirmed that they have the permission to share it. If you are author or own the copyright of this book, please report to us by using this DMCA report form. Report DMCA


Overview

Download & View Sat Practise Test 2 as PDF for free.

More details

  • Words: 44,735
  • Pages: 82
Answer Sheets

1

A B D E C O O O O O

6

A B D E C O O O O O

11

A B D E C O O O O O

16

A B D E C O O O O O

2

A B D E C O O O O O

7

A B D E C O O O O O

12

A B D E C O O O O O

17

A B D E C O O O O O

3

A B D E C O O O O O

8

A B D E C O O O O O

13

A B D E C O O O O O

18

A B D E C O O O O O

4

A O

B D E C O O O O

9

A O

B D E C O O O O

14

A O

B D E C O O O O

19

A B D E C O O O O O

5

A B D E C O O O O O

10

A B D E C O O O O O

15

A B D E C O O O O O

20

A B D E C O O O O O

1

A B D E C O O O O O

7

A B D E C O O O O O

12

A B D E C O O O O O

17

A B D E C O O O O O

SECTION

2

A B D E C O O O O O

8

A B D E C O O O O O

13

A B D E C O O O O O

18

A B D E C O O O O O

2

3

A B D E C O O O O O

9

A B D E C O O O O O

14

A B D E C O O O O O

19

A B D E C O O O O O

4

A B D E C O O O O O

10

A B D E C O O O O O

15

A B D E C O O O O O

20

A B D E C O O O O O

5

A O

11

A O

E O

16

A O

E O

21

A B D E C O O O O O

6

A B D E C O O O O O

1

A B D E C O O O O O

9

A B D E C O O O O O

17

A B D E C O O O O O

24

A B D E C O O O O O

2

A B D E C O O O O O

10

A B D E C O O O O O

18

A B D E C O O O O O

25

A B D E C O O O O O

SECTION

3

A B D E C O O O O O

11

A B D E C O O O O O

19

A B D E C O O O O O

26

A B D E C O O O O O

3

4

A B D E C O O O O O

12

A B D E C O O O O O

20

A B D E C O O O O O

27

A B D E C O O O O O

5

A O

E O

13

A O

E O

21

A O

E O

28

A B D E C O O O O O

6

A B D E C O O O O O

14

A B D E C O O O O O

22

A B D E C O O O O O

29

A B D E C O O O O O

7

A B D E C O O O O O

15

A B D E C O O O O O

23

A B D E C O O O O O

30

A B D E C O O O O O

8

A B D E C O O O O O

16

A B D E C O O O O O

1

A B D E C O O O O O

8

A B D E C O O O O O

15

A B D E C O O O O O

22

A B D E C O O O O O

2

A B D E C O O O O O

9

A B D E C O O O O O

16

A B D E C O O O O O

23

A B D E C O O O O O

E O

17

A O

E O

24

A B D E C O O O O O

SECTION

1

SECTION

B O

B O

C O

C O

D O

D O

E O

B O

B O

C O

C O

D O

D O

B O

B O

C O

C O

D O

D O

3

A O

E O

10

A O

4

A B D E C O O O O O

11

A B D E C O O O O O

18

A B D E C O O O O O

25

A B D E C O O O O O

5

A B D E C O O O O O

12

A B D E C O O O O O

19

A B D E C O O O O O

26

A B D E C O O O O O

6

A O

B D E C O O O O

13

A O

B D E C O O O O

20

A O

27

A B D E C O O O O O

7

A B D E C O O O O O

14

A B D E C O O O O O

21

A B D E C O O O O O

1

A B D E C O O O O O

7

A B D E C O O O O O

12

A B D E C O O O O O

17

A B D E C O O O O O

SECTION

2

A B D E C O O O O O

8

A B D E C O O O O O

13

A B D E C O O O O O

18

A B D E C O O O O O

5

3

A B D E C O O O O O

9

A B D E C O O O O O

14

A B D E C O O O O O

19

A B D E C O O O O O

4

A B D E C O O O O O

10

A B D E C O O O O O

15

A B D E C O O O O O

20

A B D E C O O O O O

5

A O

11

A O

16

A O

21

A B D E C O O O O O

6

A B D E C O O O O O

4

B O

B O

C O

C O

D O

D O

E O

B O

B O

C O

C O

D O

D O

E O

B O

C O

D O

B D E C O O O O

B O

C O

D O

E O

Copyright © 2005 Thomson Peterson’s, a part of The Thomson Corporaton SAT is a registered trademark of the College Entrance Examination Board, which was not involved in the production of and does not endorse this product.

1

Answer Sheets

SECTION

1

A B D E C O O O O O

5

A B D E C O O O O O

9

A B D E C O O O O O

13

A B D E C O O O O O

6

2

A B D E C O O O O O

6

A B D E C O O O O O

10

A B D E C O O O O O

14

A B D E C O O O O O

3

A B D E C O O O O O

7

A B D E C O O O O O

11

A B D E C O O O O O

15

A B D E C O O O O O

4

A O

8

A O

12

A O

16

A B D E C O O O O O

B D E C O O O O

B D E C O O O O

B D E C O O O O

SECTION

7

2

For Questions 1–13: Only answers entered in the ovals in each grid area will be scored. You will not receive credit for anything written in the boxes above the ovals.

1

2

3

4

5

6

7

8

9

10

11

12

13

Copyright © 2005 Thomson Peterson’s, a part of The Thomson Corporaton SAT is a registered trademark of the College Entrance Examination Board, which was not involved in the production of and does not endorse this product.

Practice Test

2 Section 1 20 Questions j Time—25 Minutes 12345678901234567890123456789012123456789012345678901234567890121234567 7 12234567890123456789012345678901212345678901234567890123456789012123456 7 1234567890123456789012345678901212345678901234567890123456789012123456 7 1234567890123456789012345678901212345678901234567890123456789012123456 7 1234567890123456789012345678901212345678901234567890123456789012123456 Directions: Read each of the passages carefully, then answer the questions that come after them. 7 1234567890123456789012345678901212345678901234567890123456789012123456 34567890123456789012345678901212345678901234567890123456789012123456 7 1234567890123456789012345678901212345678901234567890123456789012123456 1 The answer to each question may be stated overtly or only implied. You will not have to use 7 7 12234567890123456789012345678901212345678901234567890123456789012123456 outside knowledge to answer the questions—all the material you will need will be in the passage 7 1234567890123456789012345678901212345678901234567890123456789012123456 7 1234567890123456789012345678901212345678901234567890123456789012123456 itself. In some cases, you will be asked to read two related passages and answer questions about 34567890123456789012345678901212345678901234567890123456789012123456 1234567890123456789012345678901212345678901234567890123456789012123456 77 1234567890123456789012345678901212345678901234567890123456789012123456 their relationship to one another. Mark the letter of your choice on your answer sheet. 7 1234567890123456789012345678901212345678901234567890123456789012123456 1234567890123456789012345678901212345678901234567890123456789012123456 77 1234567890123456789012345678901212345678901234567890123456789012123456 7 1234567890123456789012345678901212345678901234567890123456789012123456 7 1234567890123456789012345678901212345678901234567890123456789012123456 7 1234567890123456789012345678901212345678901234567890123456789012123456 A bill is the form used for most legislation in 1. From the passage, it can be inferred that a 7 1234567890123456789012345678901212345678901234567890123456789012123456 7 1 the United State Congress. Only constitutional bill that is designated as H.R. 1 is the 7 12234567890123456789012345678901212345678901234567890123456789012123456 7 1234567890123456789012345678901212345678901234567890123456789012123456 amendments and procedural issues affecting the first bill 1 34567890123456789012345678901212345678901234567890123456789012123456 7 7 1234567890123456789012345678901212345678901234567890123456789012123456 House and Senate are adopted by a resolution, (A) voted upon by the House of Repre7 12234567890123456789012345678901212345678901234567890123456789012123456 34567890123456789012345678901212345678901234567890123456789012123456 7 1234567890123456789012345678901212345678901234567890123456789012123456 rather than a bill. Bills can be written to be sentatives in a particular session of 7 1234567890123456789012345678901212345678901234567890123456789012123456 7 1 permanent or temporary, general or special. A Congress. 7 12234567890123456789012345678901212345678901234567890123456789012123456 bill originating in the House of Representatives 7 1234567890123456789012345678901212345678901234567890123456789012123456 (B) submitted to the House of Represen34567890123456789012345678901212345678901234567890123456789012123456 7 1234567890123456789012345678901212345678901234567890123456789012123456 is designated by the letters “H.R.,” signifying 7 1234567890123456789012345678901212345678901234567890123456789012123456 tatives in a particular session of 7 1 “House of Representatives,” followed by a 7 1234567890123456789012345678901212345678901234567890123456789012123456 Congress. 7 1234567890123456789012345678901212345678901234567890123456789012123456 number that it retains throughout all its 2 34567890123456789012345678901212345678901234567890123456789012123456 1234567890123456789012345678901212345678901234567890123456789012123456 (C) sent to the Senate from the House of 7 7 1234567890123456789012345678901212345678901234567890123456789012123456 parliamentary stages. The number on the bill is 7 1234567890123456789012345678901212345678901234567890123456789012123456 Representatives in a particular 7 1234567890123456789012345678901212345678901234567890123456789012123456 determined by the order in which it was 7 1 session of Congress. 2 34567890123456789012345678901212345678901234567890123456789012123456 7 1234567890123456789012345678901212345678901234567890123456789012123456 submitted during a particular session. Bills are 7 1234567890123456789012345678901212345678901234567890123456789012123456 (D) originating in the House of Repre7 1234567890123456789012345678901212345678901234567890123456789012123456 presented to the President for action when 7 1234567890123456789012345678901212345678901234567890123456789012123456 sentatives signed by the President in 7 1 approved in identical form by both the House 2 34567890123456789012345678901212345678901234567890123456789012123456 7 1234567890123456789012345678901212345678901234567890123456789012123456 a particular session of Congress. 7 1234567890123456789012345678901212345678901234567890123456789012123456 of Representatives and the Senate. 1234567890123456789012345678901212345678901234567890123456789012123456 (E) debated on the floor of the House of 7 1234567890123456789012345678901212345678901234567890123456789012123456 77 1 Representatives in a particular 7 12234567890123456789012345678901212345678901234567890123456789012123456 7 1 34567890123456789012345678901212345678901234567890123456789012123456 session of Congress. 7 12234567890123456789012345678901212345678901234567890123456789012123456 1 34567890123456789012345678901212345678901234567890123456789012123456 7 7 12234567890123456789012345678901212345678901234567890123456789012123456 1 34567890123456789012345678901212345678901234567890123456789012123456 7 1234567890123456789012345678901212345678901234567890123456789012123456 7 1234567890123456789012345678901212345678901234567890123456789012123456 7 1234567890123456789012345678901212345678901234567890123456789012123456 7 12345678901234567890123456789012123456789012345678901234567890121234567 Copyright © 2005 Thomson Peterson’s, a part of The Thomson Corporaton SAT is a registered trademark of the College Entrance Examination Board, which was not involved in the production of and does not endorse this product.

3

12345678901234567890123456789012123456789012345678901234567890121234567 1234567890123456789012345678901212345678901234567890123456789012123456 7 34567890123456789012345678901212345678901234567890123456789012123456 7 12 2. It is implied in the passage that once a bill 7 4. Given what the passage states about 1234567890123456789012345678901212345678901234567890123456789012123456 34567890123456789012345678901212345678901234567890123456789012123456 7 12 is passed in the House of Representatives Native American views of nature, which 34567890123456789012345678901212345678901234567890123456789012123456 7 12 7 1234567890123456789012345678901212345678901234567890123456789012123456 that it might be sent to which of the of the following scenarios most accords 34567890123456789012345678901212345678901234567890123456789012123456 7 12 2 34567890123456789012345678901212345678901234567890123456789012123456 7 1 following two places? with a Native American view? 34567890123456789012345678901212345678901234567890123456789012123456 7 12 34567890123456789012345678901212345678901234567890123456789012123456 7 12 34567890123456789012345678901212345678901234567890123456789012123456 7 12 (A) Senate, conference committee (A) Studying a microorganism removed 34567890123456789012345678901212345678901234567890123456789012123456 7 12 2 34567890123456789012345678901212345678901234567890123456789012123456 7 1 (B) Senate, House committee from its habitat. 34567890123456789012345678901212345678901234567890123456789012123456 7 12 34567890123456789012345678901212345678901234567890123456789012123456 7 12 (C) Senate, President (B) Studying Earth through satellite 34567890123456789012345678901212345678901234567890123456789012123456 7 12 34567890123456789012345678901212345678901234567890123456789012123456 7 12 (D) President, Supreme Court images. 7 1234567890123456789012345678901212345678901234567890123456789012123456 34567890123456789012345678901212345678901234567890123456789012123456 7 12 (E) President, Congress (C) Studying only animals and sub34567890123456789012345678901212345678901234567890123456789012123456 7 12 34567890123456789012345678901212345678901234567890123456789012123456 7 12 stances with spiritual symbolism. 34567890123456789012345678901212345678901234567890123456789012123456 7 12 Native American views of nature have impor2 34567890123456789012345678901212345678901234567890123456789012123456 7 1 (D) Studying a specific organism’s 34567890123456789012345678901212345678901234567890123456789012123456 7 12 tant parallels in contemporary ecology. 34567890123456789012345678901212345678901234567890123456789012123456 7 12 interrelationships with its habitat. 34567890123456789012345678901212345678901234567890123456789012123456 7 12 Through traditional customs and symbols like 34567890123456789012345678901212345678901234567890123456789012123456 7 12 (E) Studying a habitat as a whole. 2 34567890123456789012345678901212345678901234567890123456789012123456 7 1234567890123456789012345678901212345678901234567890123456789012123456 the medicine wheel, a circular arrangement of 7 1234567890123456789012345678901212345678901234567890123456789012123456 7 1234567890123456789012345678901212345678901234567890123456789012123456 stones often interpreted as representing the 7 1234567890123456789012345678901212345678901234567890123456789012123456 Questions 5–12 are based on the following 7 1234567890123456789012345678901212345678901234567890123456789012123456 relationship between Earth, air, water, and fire, 7 1234567890123456789012345678901212345678901234567890123456789012123456 passage. 7 1 Native Americans have long recognized and 34567890123456789012345678901212345678901234567890123456789012123456 7 12 34567890123456789012345678901212345678901234567890123456789012123456 12 This passage is about Aaron Copland, one of the 7 celebrated the connectedness among all natural 34567890123456789012345678901212345678901234567890123456789012123456 7 12 2 34567890123456789012345678901212345678901234567890123456789012123456 7 1234567890123456789012345678901212345678901234567890123456789012123456 most celebrated American composers. things. Indeed, the Native American view of the 7 1234567890123456789012345678901212345678901234567890123456789012123456 7 1234567890123456789012345678901212345678901234567890123456789012123456 world has always been consistent with that of Line Copland’s music of the late 1920s 7 1234567890123456789012345678901212345678901234567890123456789012123456 7 1234567890123456789012345678901212345678901234567890123456789012123456 Earth ecology—that Earth is a single system of culminates in two key works, both 1234567890123456789012345678901212345678901234567890123456789012123456 77 1234567890123456789012345678901212345678901234567890123456789012123456 interconnected parts. uncompromising in their modernism: the 7 1234567890123456789012345678901212345678901234567890123456789012123456 7 1234567890123456789012345678901212345678901234567890123456789012123456 Symphonic Ode of 1929 and the Piano 7 1 2 34567890123456789012345678901212345678901234567890123456789012123456 7 1234567890123456789012345678901212345678901234567890123456789012123456 (5) Variations of 1930. The fate of these 3. The symbol of the medicine wheel is given 7 1234567890123456789012345678901212345678901234567890123456789012123456 7 1 compositions contrasts sharply. While the as a(n) 1234567890123456789012345678901212345678901234567890123456789012123456 7 34567890123456789012345678901212345678901234567890123456789012123456 7 12 Piano Variations is not often performed 34567890123456789012345678901212345678901234567890123456789012123456 7 12 (A) illustration of how Native Americans 34567890123456789012345678901212345678901234567890123456789012123456 7 12 in concert, it is well known to pianists 7 1234567890123456789012345678901212345678901234567890123456789012123456 view the Earth as an interconnected 34567890123456789012345678901212345678901234567890123456789012123456 7 12 because, although it does contain 34567890123456789012345678901212345678901234567890123456789012123456 7 12 system. 2 34567890123456789012345678901212345678901234567890123456789012123456 7 1234567890123456789012345678901212345678901234567890123456789012123456 (10) virtuoso passages, even those of very 7 1234567890123456789012345678901212345678901234567890123456789012123456 (B) example of the Native American 7 1 modest ability can “play at” the work in 7 1234567890123456789012345678901212345678901234567890123456789012123456 understanding of the four elements. 7 1234567890123456789012345678901212345678901234567890123456789012123456 private. It represents the twentieth2 34567890123456789012345678901212345678901234567890123456789012123456 7 1234567890123456789012345678901212345678901234567890123456789012123456 (C) example of the interrelatedness of the 7 1234567890123456789012345678901212345678901234567890123456789012123456 century continuation of the great 7 1234567890123456789012345678901212345678901234567890123456789012123456 four basic elements. 7 1234567890123456789012345678901212345678901234567890123456789012123456 tradition of keyboard variations—the 7 1 (D) critique of contemporary ecological 2 34567890123456789012345678901212345678901234567890123456789012123456 7 1234567890123456789012345678901212345678901234567890123456789012123456 (15) tradition that produced such works as understandings of the Earth. 7 1234567890123456789012345678901212345678901234567890123456789012123456 7 1234567890123456789012345678901212345678901234567890123456789012123456 the Bach Goldberg Variations, and (E) contrast to contemporary ecological 7 1234567890123456789012345678901212345678901234567890123456789012123456 7 1 Beethoven’s Diabelli Variations. Copunderstandings of the Earth. 2 34567890123456789012345678901212345678901234567890123456789012123456 7 1234567890123456789012345678901212345678901234567890123456789012123456 land’s Symphonic Ode, on the other 7 1234567890123456789012345678901212345678901234567890123456789012123456 7 1234567890123456789012345678901212345678901234567890123456789012123456 hand, remains almost unknown: An 7 1234567890123456789012345678901212345678901234567890123456789012123456 7 1 (20) intense symphonic movement, it was 2 34567890123456789012345678901212345678901234567890123456789012123456 1234567890123456789012345678901212345678901234567890123456789012123456 77 1 considered unperformable by the 34567890123456789012345678901212345678901234567890123456789012123456 7 12 7 1234567890123456789012345678901212345678901234567890123456789012123456 conductor Serge Koussevitzky, otherwise 34567890123456789012345678901212345678901234567890123456789012123456 7 12 2 34567890123456789012345678901212345678901234567890123456789012123456 7 1 the most potent American champion of 7 1234567890123456789012345678901212345678901234567890123456789012123456 1234567890123456789012345678901212345678901234567890123456789012123456 77 1234567890123456789012345678901212345678901234567890123456789012123456 12345678901234567890123456789012123456789012345678901234567890121234567 4

Copyright © 2005 Thomson Peterson’s, a part of The Thomson Corporaton SAT is a registered trademark of the College Entrance Examination Board, which was not involved in the production of and does not endorse this product.

12345678901234567890123456789012123456789012345678901234567890121234567 1234567890123456789012345678901212345678901234567890123456789012123456 7 7 12234567890123456789012345678901212345678901234567890123456789012123456 Copland’s work during the first half of (Some of the folk music he heard in Rio 7 1 34567890123456789012345678901212345678901234567890123456789012123456 7 12234567890123456789012345678901212345678901234567890123456789012123456 (25) the century. Koussevitzky did perform a de Janeiro on this trip appears in his later 7 1234567890123456789012345678901212345678901234567890123456789012123456 7 1 34567890123456789012345678901212345678901234567890123456789012123456 revised version in 1932; but even with a works.) Copland in fact envisioned 7 12234567890123456789012345678901212345678901234567890123456789012123456 34567890123456789012345678901212345678901234567890123456789012123456 7 1 second, more extensive revision in 1955, (70) “American music” as being music of the 7 12234567890123456789012345678901212345678901234567890123456789012123456 7 1234567890123456789012345678901212345678901234567890123456789012123456 the Ode is seldom played. It is Copland’s Americas as a whole. His own use of 7 1234567890123456789012345678901212345678901234567890123456789012123456 7 1234567890123456789012345678901212345678901234567890123456789012123456 single longest orchestral movement. Mexican material in the mid-1930s 34567890123456789012345678901212345678901234567890123456789012123456 7 1 7 12234567890123456789012345678901212345678901234567890123456789012123456 (30) Perhaps as a reaction to the perforhelped make his style more accessible to 7 1234567890123456789012345678901212345678901234567890123456789012123456 7 1234567890123456789012345678901212345678901234567890123456789012123456 mance problems of the Symphonic Ode, listeners not willing to accept the 7 1234567890123456789012345678901212345678901234567890123456789012123456 34567890123456789012345678901212345678901234567890123456789012123456 1 (75) Copland’s next two orchestral works challenges of modern symphonic music. 7 7 12234567890123456789012345678901212345678901234567890123456789012123456 7 1234567890123456789012345678901212345678901234567890123456789012123456 deal in shorter units of time: the Short 7 1234567890123456789012345678901212345678901234567890123456789012123456 7 1234567890123456789012345678901212345678901234567890123456789012123456 Symphony of 1933 requires fifteen 5. What is the author’s tone toward Cop34567890123456789012345678901212345678901234567890123456789012123456 7 1 7 12234567890123456789012345678901212345678901234567890123456789012123456 (35) minutes for three movements and the six land’s music? 7 1234567890123456789012345678901212345678901234567890123456789012123456 7 1234567890123456789012345678901212345678901234567890123456789012123456 Statements for orchestra of 1935 last 7 1234567890123456789012345678901212345678901234567890123456789012123456 (A) Strident skepticism 34567890123456789012345678901212345678901234567890123456789012123456 7 1234567890123456789012345678901212345678901234567890123456789012123456 only nineteen minutes. Yet, in fact, these 7 1234567890123456789012345678901212345678901234567890123456789012123456 (B) Clinical objectivity 7 1234567890123456789012345678901212345678901234567890123456789012123456 works were more complex than the Ode; 7 1234567890123456789012345678901212345678901234567890123456789012123456 (C) Respectful description 7 1234567890123456789012345678901212345678901234567890123456789012123456 in particular, the wiry, agile rhythms of 7 1234567890123456789012345678901212345678901234567890123456789012123456 (D) Qualified enthusiasm 7 1 (40) the opening movement of the Short 7 12234567890123456789012345678901212345678901234567890123456789012123456 (E) Unqualified praise Symphony proved too much for both the 7 1234567890123456789012345678901212345678901234567890123456789012123456 7 1234567890123456789012345678901212345678901234567890123456789012123456 conductors Serge Koussevitzky and 34567890123456789012345678901212345678901234567890123456789012123456 1234567890123456789012345678901212345678901234567890123456789012123456 77 1234567890123456789012345678901212345678901234567890123456789012123456 6. The word “virtuoso” in line 10 could best Leopold Stokowski. In the end it was 7 1234567890123456789012345678901212345678901234567890123456789012123456 be replaced with 7 1234567890123456789012345678901212345678901234567890123456789012123456 Carlos Chávez and the Orquesta Sin1234567890123456789012345678901212345678901234567890123456789012123456 77 1234567890123456789012345678901212345678901234567890123456789012123456 (45) fónica de México who gave the Short (A) ostentatious. 7 1234567890123456789012345678901212345678901234567890123456789012123456 7 1234567890123456789012345678901212345678901234567890123456789012123456 Symphony its premiere. (B) intricate 1234567890123456789012345678901212345678901234567890123456789012123456 77 1 It may have been partly Copland’s (C) raucous. 7 12234567890123456789012345678901212345678901234567890123456789012123456 7 1234567890123456789012345678901212345678901234567890123456789012123456 friendship with Carlos Chávez that drew (D) abstruse. 1 34567890123456789012345678901212345678901234567890123456789012123456 7 7 1234567890123456789012345678901212345678901234567890123456789012123456 him to Mexico. Copland first visited (E) publicized. 7 12234567890123456789012345678901212345678901234567890123456789012123456 34567890123456789012345678901212345678901234567890123456789012123456 7 1234567890123456789012345678901212345678901234567890123456789012123456 (50) Mexico in 1932 and returned frequently 7 1234567890123456789012345678901212345678901234567890123456789012123456 7 1 in later years. His initial delight in the 7 12234567890123456789012345678901212345678901234567890123456789012123456 7 1234567890123456789012345678901212345678901234567890123456789012123456 country is related in his letter of January 34567890123456789012345678901212345678901234567890123456789012123456 1234567890123456789012345678901212345678901234567890123456789012123456 77 1234567890123456789012345678901212345678901234567890123456789012123456 13, 1933, to Mary Lescaze, in which he 7 1 7 1234567890123456789012345678901212345678901234567890123456789012123456 glowingly describes the Mexican people 1234567890123456789012345678901212345678901234567890123456789012123456 7 2 34567890123456789012345678901212345678901234567890123456789012123456 7 1234567890123456789012345678901212345678901234567890123456789012123456 (55) and the Mexican landscape. His interest 7 1234567890123456789012345678901212345678901234567890123456789012123456 7 1234567890123456789012345678901212345678901234567890123456789012123456 in Mexico is also reflected in his music, 1234567890123456789012345678901212345678901234567890123456789012123456 77 1 including El Salón México (1936) and 7 12234567890123456789012345678901212345678901234567890123456789012123456 the Three Latin American Sketches 7 1234567890123456789012345678901212345678901234567890123456789012123456 7 1234567890123456789012345678901212345678901234567890123456789012123456 (1972). 7 1234567890123456789012345678901212345678901234567890123456789012123456 7 1 34567890123456789012345678901212345678901234567890123456789012123456 (60) Mexico was not Copland’s only Latin 2 34567890123456789012345678901212345678901234567890123456789012123456 1234567890123456789012345678901212345678901234567890123456789012123456 77 1234567890123456789012345678901212345678901234567890123456789012123456 American interest. A 1941 trip to 7 1234567890123456789012345678901212345678901234567890123456789012123456 7 1234567890123456789012345678901212345678901234567890123456789012123456 Havana suggested his Danzón Cubano. 7 1 2 34567890123456789012345678901212345678901234567890123456789012123456 7 1234567890123456789012345678901212345678901234567890123456789012123456 By the early 1940s he was friends with 7 1 7 12234567890123456789012345678901212345678901234567890123456789012123456 South American composers such as 1 34567890123456789012345678901212345678901234567890123456789012123456 7 7 12234567890123456789012345678901212345678901234567890123456789012123456 (65) Jacobo Ficher, and in 1947 he toured 7 1 34567890123456789012345678901212345678901234567890123456789012123456 7 1234567890123456789012345678901212345678901234567890123456789012123456 South America for the State Department. 1234567890123456789012345678901212345678901234567890123456789012123456 77 1234567890123456789012345678901212345678901234567890123456789012123456 12345678901234567890123456789012123456789012345678901234567890121234567 Copyright © 2005 Thomson Peterson’s, a part of The Thomson Corporaton SAT is a registered trademark of the College Entrance Examination Board, which was not involved in the production of and does not endorse this product.

5

12345678901234567890123456789012123456789012345678901234567890121234567 1234567890123456789012345678901212345678901234567890123456789012123456 7 34567890123456789012345678901212345678901234567890123456789012123456 7 12 7. In the first paragraph the author states 7 9. The author of the passage believes that 1234567890123456789012345678901212345678901234567890123456789012123456 34567890123456789012345678901212345678901234567890123456789012123456 7 12 that Symphonic Ode and Piano Variations Copland’s works immediately subsequent 7 34567890123456789012345678901212345678901234567890123456789012123456 12 7 1234567890123456789012345678901212345678901234567890123456789012123456 had different fates in that to the Symphonic Ode were possibly 34567890123456789012345678901212345678901234567890123456789012123456 7 12 2 34567890123456789012345678901212345678901234567890123456789012123456 7 1 written 34567890123456789012345678901212345678901234567890123456789012123456 7 12 (A) one was largely ignored while the 34567890123456789012345678901212345678901234567890123456789012123456 7 12 34567890123456789012345678901212345678901234567890123456789012123456 7 12 other was almost universally praised. (A) for Copland’s new relationship with 34567890123456789012345678901212345678901234567890123456789012123456 7 12 2 34567890123456789012345678901212345678901234567890123456789012123456 7 1 (B) one, a simpler piece, won popular Carlos Chávez and the Orquesta 34567890123456789012345678901212345678901234567890123456789012123456 7 12 34567890123456789012345678901212345678901234567890123456789012123456 7 12 acclaim, while the other, a more Sinfónica de México. 34567890123456789012345678901212345678901234567890123456789012123456 7 12 34567890123456789012345678901212345678901234567890123456789012123456 7 12 complex piece, won critical acclaim. (B) to be simpler than the Symphonic 7 1234567890123456789012345678901212345678901234567890123456789012123456 34567890123456789012345678901212345678901234567890123456789012123456 7 12 (C) one, a simpler piece, became widely Ode, on account of its difficulty in 34567890123456789012345678901212345678901234567890123456789012123456 7 12 34567890123456789012345678901212345678901234567890123456789012123456 7 12 known by pianists, but the other, a being performed. 34567890123456789012345678901212345678901234567890123456789012123456 7 12 2 34567890123456789012345678901212345678901234567890123456789012123456 7 1 more complex piece, remained (C) to be shorter than the Symphonic 34567890123456789012345678901212345678901234567890123456789012123456 7 12 34567890123456789012345678901212345678901234567890123456789012123456 12 largely unknown. Ode, because the Ode was not being 7 34567890123456789012345678901212345678901234567890123456789012123456 7 12 34567890123456789012345678901212345678901234567890123456789012123456 7 12 (D) one, featuring Mexican influences, performed. 2 7 1234567890123456789012345678901212345678901234567890123456789012123456 7 1234567890123456789012345678901212345678901234567890123456789012123456 was popular in Latin America, and (D) to demand even more of conductors 7 1234567890123456789012345678901212345678901234567890123456789012123456 the other, a modernist piece, was 7 and musicians attempting to play 1234567890123456789012345678901212345678901234567890123456789012123456 7 1234567890123456789012345678901212345678901234567890123456789012123456 popular in the United States. Copland’s music. 34567890123456789012345678901212345678901234567890123456789012123456 1234567890123456789012345678901212345678901234567890123456789012123456 77 1 (E) both were initially acclaimed but (E) to reflect Copland’s new interest in 34567890123456789012345678901212345678901234567890123456789012123456 7 12 34567890123456789012345678901212345678901234567890123456789012123456 7 12 only one became part of Copland’s Latin America. 34567890123456789012345678901212345678901234567890123456789012123456 7 12 2 34567890123456789012345678901212345678901234567890123456789012123456 7 1234567890123456789012345678901212345678901234567890123456789012123456 corpus of beloved works. 7 1234567890123456789012345678901212345678901234567890123456789012123456 7 1234567890123456789012345678901212345678901234567890123456789012123456 10. In the sentence beginning “Yet, in fact, 1234567890123456789012345678901212345678901234567890123456789012123456 77 1234567890123456789012345678901212345678901234567890123456789012123456 8. Koussevitzky is mentioned as an example these works. . .” in lines 37–43 [second 7 1234567890123456789012345678901212345678901234567890123456789012123456 7 1234567890123456789012345678901212345678901234567890123456789012123456 of a(n) paragraph], the author suggests that 1234567890123456789012345678901212345678901234567890123456789012123456 77 1234567890123456789012345678901212345678901234567890123456789012123456 7 1 (A) American conductor who admired (A) parts of the Short Symphony simply 34567890123456789012345678901212345678901234567890123456789012123456 7 12 34567890123456789012345678901212345678901234567890123456789012123456 7 Copland’s work, but nonetheless 12 weren’t melodic enough to engage 7 1234567890123456789012345678901212345678901234567890123456789012123456 found some pieces too difficult to audiences. 7 1234567890123456789012345678901212345678901234567890123456789012123456 34567890123456789012345678901212345678901234567890123456789012123456 7 12 perform. (B) the Statements were too brief to 2 34567890123456789012345678901212345678901234567890123456789012123456 1234567890123456789012345678901212345678901234567890123456789012123456 77 1234567890123456789012345678901212345678901234567890123456789012123456 (B) friend of Copland’s who agreed to warrant a formal performance. 7 1 34567890123456789012345678901212345678901234567890123456789012123456 7 12 perform his less popular works. (C) even those who admired Copland’s 34567890123456789012345678901212345678901234567890123456789012123456 7 12 2 34567890123456789012345678901212345678901234567890123456789012123456 7 1234567890123456789012345678901212345678901234567890123456789012123456 (C) European composer who took issue work lost patience with the Short 7 1234567890123456789012345678901212345678901234567890123456789012123456 7 1 with the difficulty of Copland’s Symphony and Statements. 1234567890123456789012345678901212345678901234567890123456789012123456 7 7 1234567890123456789012345678901212345678901234567890123456789012123456 early work. (D) the Statements and Short Symphony 34567890123456789012345678901212345678901234567890123456789012123456 7 12 34567890123456789012345678901212345678901234567890123456789012123456 7 12 (D) musician who appreciated Copland’s determined which performers were 34567890123456789012345678901212345678901234567890123456789012123456 7 12 34567890123456789012345678901212345678901234567890123456789012123456 7 12 work but was unable to play it. truly excellent and which were 7 1234567890123456789012345678901212345678901234567890123456789012123456 2 34567890123456789012345678901212345678901234567890123456789012123456 7 1234567890123456789012345678901212345678901234567890123456789012123456 (E) European conductor who performed mediocre. 7 1234567890123456789012345678901212345678901234567890123456789012123456 7 1234567890123456789012345678901212345678901234567890123456789012123456 Copland’s work. (E) the Short Symphony had melodies 1234567890123456789012345678901212345678901234567890123456789012123456 77 1 that were too quick to be played 34567890123456789012345678901212345678901234567890123456789012123456 7 12 34567890123456789012345678901212345678901234567890123456789012123456 7 12 even by famous musicians. 34567890123456789012345678901212345678901234567890123456789012123456 7 12 34567890123456789012345678901212345678901234567890123456789012123456 7 12 1234567890123456789012345678901212345678901234567890123456789012123456 7 2 7 1234567890123456789012345678901212345678901234567890123456789012123456 1 34567890123456789012345678901212345678901234567890123456789012123456 7 34567890123456789012345678901212345678901234567890123456789012123456 7 12 1234567890123456789012345678901212345678901234567890123456789012123456 7 34567890123456789012345678901212345678901234567890123456789012123456 7 12 1234567890123456789012345678901212345678901234567890123456789012123456 7 1234567890123456789012345678901212345678901234567890123456789012123456 7 1234567890123456789012345678901212345678901234567890123456789012123456 7 1234567890123456789012345678901212345678901234567890123456789012123456 7 12345678901234567890123456789012123456789012345678901234567890121234567 6

Copyright © 2005 Thomson Peterson’s, a part of The Thomson Corporaton SAT is a registered trademark of the College Entrance Examination Board, which was not involved in the production of and does not endorse this product.

12345678901234567890123456789012123456789012345678901234567890121234567 1234567890123456789012345678901212345678901234567890123456789012123456 7 7 12234567890123456789012345678901212345678901234567890123456789012123456 11. The author suggests that Copland believed round, we are still just barely skimming 7 1 34567890123456789012345678901212345678901234567890123456789012123456 7 12234567890123456789012345678901212345678901234567890123456789012123456 (10) Latin American music the surface when you consider that the 7 1234567890123456789012345678901212345678901234567890123456789012123456 7 1 34567890123456789012345678901212345678901234567890123456789012123456 diameter of the Earth is over 8,000 miles. 7 12234567890123456789012345678901212345678901234567890123456789012123456 (A) was unfamiliar enough to a North 34567890123456789012345678901212345678901234567890123456789012123456 7 1 So how much of the Earth can we see 7 12234567890123456789012345678901212345678901234567890123456789012123456 American audience that he needed to 7 1234567890123456789012345678901212345678901234567890123456789012123456 at one time? When you are standing on 7 1234567890123456789012345678901212345678901234567890123456789012123456 introduce them to it. 7 1234567890123456789012345678901212345678901234567890123456789012123456 the ground, the horizon is a few miles 34567890123456789012345678901212345678901234567890123456789012123456 7 1 (B) was different enough from North 7 12234567890123456789012345678901212345678901234567890123456789012123456 (15) away. When in a tall building, the 7 1234567890123456789012345678901212345678901234567890123456789012123456 American music that incorporating 7 1234567890123456789012345678901212345678901234567890123456789012123456 horizon can be as far as about 40 miles. 7 1234567890123456789012345678901212345678901234567890123456789012123456 aspects of it would make his music 34567890123456789012345678901212345678901234567890123456789012123456 1 From the International Space Station, the 7 7 12234567890123456789012345678901212345678901234567890123456789012123456 unique and exciting. 7 1234567890123456789012345678901212345678901234567890123456789012123456 distance to the horizon is over 1,000 7 1234567890123456789012345678901212345678901234567890123456789012123456 (C) influenced and was influenced by 7 1234567890123456789012345678901212345678901234567890123456789012123456 miles. So from horizon to horizon, the 34567890123456789012345678901212345678901234567890123456789012123456 7 1 North American music. 7 12234567890123456789012345678901212345678901234567890123456789012123456 (20) section of the Earth you can see at any 7 1234567890123456789012345678901212345678901234567890123456789012123456 (D) primarily originated in Mexico 7 1234567890123456789012345678901212345678901234567890123456789012123456 one time is a patch about 2,000 miles 7 1234567890123456789012345678901212345678901234567890123456789012123456 and Cuba. 34567890123456789012345678901212345678901234567890123456789012123456 7 1234567890123456789012345678901212345678901234567890123456789012123456 across, almost enough to see the entire 7 1234567890123456789012345678901212345678901234567890123456789012123456 (E) embodied the polar opposite of 7 1234567890123456789012345678901212345678901234567890123456789012123456 United States at once. It isn’t exactly modernist aesthetics. 7 1234567890123456789012345678901212345678901234567890123456789012123456 7 1234567890123456789012345678901212345678901234567890123456789012123456 seeing the Earth like a big blue marble, 1234567890123456789012345678901212345678901234567890123456789012123456 77 1 12. The sentence beginning “His own use of (25) it’s more like having your face up against 7 12234567890123456789012345678901212345678901234567890123456789012123456 a big blue beach ball. When I look out a 7 1234567890123456789012345678901212345678901234567890123456789012123456 Mexican material. . . “ in lines 71–75 sug7 1234567890123456789012345678901212345678901234567890123456789012123456 window that faces straight down, it is 7 1234567890123456789012345678901212345678901234567890123456789012123456 gests that the modernist music which also 7 1234567890123456789012345678901212345678901234567890123456789012123456 actually pretty hard to see the horizon— 7 1234567890123456789012345678901212345678901234567890123456789012123456 influenced Copland’s compositions was 7 1234567890123456789012345678901212345678901234567890123456789012123456 you need to get your face very close to 7 1234567890123456789012345678901212345678901234567890123456789012123456 34567890123456789012345678901212345678901234567890123456789012123456 7 1234567890123456789012345678901212345678901234567890123456789012123456 (A) superior in quality to his Latin (30) the window. So what you see out a 7 1234567890123456789012345678901212345678901234567890123456789012123456 7 1234567890123456789012345678901212345678901234567890123456789012123456 American influences. window like that is a moving patch of 1234567890123456789012345678901212345678901234567890123456789012123456 77 1 (B) dry and passionless. ground (or water). 7 12234567890123456789012345678901212345678901234567890123456789012123456 7 (C) technically more challenging to 1234567890123456789012345678901212345678901234567890123456789012123456 From the time a place on the ground 7 1 34567890123456789012345678901212345678901234567890123456789012123456 perform. 7 1234567890123456789012345678901212345678901234567890123456789012123456 comes into view until it disappears over 7 12234567890123456789012345678901212345678901234567890123456789012123456 (D) inaccessible but rewarding. 34567890123456789012345678901212345678901234567890123456789012123456 7 1234567890123456789012345678901212345678901234567890123456789012123456 (35) the horizon is only a few minutes, since 7 1234567890123456789012345678901212345678901234567890123456789012123456 (E) outmoded by the 1930s. 7 1 we are traveling 300 miles per minute. 7 12234567890123456789012345678901212345678901234567890123456789012123456 7 1234567890123456789012345678901212345678901234567890123456789012123456 When looking out a sideward facing 34567890123456789012345678901212345678901234567890123456789012123456 1234567890123456789012345678901212345678901234567890123456789012123456 77 1234567890123456789012345678901212345678901234567890123456789012123456 window, you can see the horizon of the 7 1 Questions 13–20 are based on the following 7 1234567890123456789012345678901212345678901234567890123456789012123456 passage. Earth against the black background of 1234567890123456789012345678901212345678901234567890123456789012123456 7 2 34567890123456789012345678901212345678901234567890123456789012123456 7 1234567890123456789012345678901212345678901234567890123456789012123456 (40) space. The horizon is distinctly curved. The following passage was written by Ed Lu, an 7 1234567890123456789012345678901212345678901234567890123456789012123456 7 1234567890123456789012345678901212345678901234567890123456789012123456 The edge of the Earth isn’t distinct but astronaut, while a crew member of the Interna7 1234567890123456789012345678901212345678901234567890123456789012123456 7 1 tional Space Station. rather is smeared out due to the atmo2 34567890123456789012345678901212345678901234567890123456789012123456 7 1234567890123456789012345678901212345678901234567890123456789012123456 sphere. Here you can get a feel for how 7 1234567890123456789012345678901212345678901234567890123456789012123456 7 1234567890123456789012345678901212345678901234567890123456789012123456 Line Whenever I get a chance, I spend time relatively thin the atmosphere is com7 1234567890123456789012345678901212345678901234567890123456789012123456 7 1 just observing the planet below. It turns (45) pared to the Earth as a whole. I can see 2 34567890123456789012345678901212345678901234567890123456789012123456 1234567890123456789012345678901212345678901234567890123456789012123456 77 1234567890123456789012345678901212345678901234567890123456789012123456 out you can see a lot more from up here that the width of the atmosphere on the 7 1234567890123456789012345678901212345678901234567890123456789012123456 1234567890123456789012345678901212345678901234567890123456789012123456 than you might expect. First off, we horizon is about 1 degree in angular size, 7 7 1 2 34567890123456789012345678901212345678901234567890123456789012123456 7 1234567890123456789012345678901212345678901234567890123456789012123456 which is about the width of your index 7 1 (5) aren’t as far away as some people 7 12234567890123456789012345678901212345678901234567890123456789012123456 think—our orbit is only about 240 miles finger held out at arms length. There 1 34567890123456789012345678901212345678901234567890123456789012123456 7 7 12234567890123456789012345678901212345678901234567890123456789012123456 above the surface of the Earth. While this (50) really isn’t a sharp boundary to the 7 1 34567890123456789012345678901212345678901234567890123456789012123456 7 1234567890123456789012345678901212345678901234567890123456789012123456 is high enough to see that the Earth is atmosphere, but it gets rapidly thinner 1234567890123456789012345678901212345678901234567890123456789012123456 77 1234567890123456789012345678901212345678901234567890123456789012123456 12345678901234567890123456789012123456789012345678901234567890121234567 Copyright © 2005 Thomson Peterson’s, a part of The Thomson Corporaton SAT is a registered trademark of the College Entrance Examination Board, which was not involved in the production of and does not endorse this product.

7

12345678901234567890123456789012123456789012345678901234567890121234567 1234567890123456789012345678901212345678901234567890123456789012123456 7 34567890123456789012345678901212345678901234567890123456789012123456 7 12 the higher you go. Not many airplanes 7 14. The second half of the second paragraph 1234567890123456789012345678901212345678901234567890123456789012123456 34567890123456789012345678901212345678901234567890123456789012123456 7 12 can fly higher than about 10 miles, and is primarily concerned with 34567890123456789012345678901212345678901234567890123456789012123456 7 12 7 1234567890123456789012345678901212345678901234567890123456789012123456 the highest mountains are only about 6 34567890123456789012345678901212345678901234567890123456789012123456 7 12 (A) how one’s location affects one’s 2 34567890123456789012345678901212345678901234567890123456789012123456 7 1 (55) miles high. Above about 30 miles there is 34567890123456789012345678901212345678901234567890123456789012123456 7 12 visual horizon. 34567890123456789012345678901212345678901234567890123456789012123456 7 12 very little air to speak of, but at night 34567890123456789012345678901212345678901234567890123456789012123456 7 12 (B) the thickness and density of the 34567890123456789012345678901212345678901234567890123456789012123456 7 12 you can see a faint glow from what little 2 34567890123456789012345678901212345678901234567890123456789012123456 7 1 atmosphere. 34567890123456789012345678901212345678901234567890123456789012123456 7 12 air there is at that height. 34567890123456789012345678901212345678901234567890123456789012123456 7 12 (C) the speed of the International Space 34567890123456789012345678901212345678901234567890123456789012123456 7 12 Since we orbit at an altitude about 40 34567890123456789012345678901212345678901234567890123456789012123456 7 12 Station. 2 34567890123456789012345678901212345678901234567890123456789012123456 7 1 (60) times higher than the tallest mountain, 34567890123456789012345678901212345678901234567890123456789012123456 7 12 (D) the visual horizon from atop a 34567890123456789012345678901212345678901234567890123456789012123456 7 12 the surface of the Earth is pretty smooth 34567890123456789012345678901212345678901234567890123456789012123456 7 12 tall building. 34567890123456789012345678901212345678901234567890123456789012123456 7 12 from our perspective. A good way to 2 34567890123456789012345678901212345678901234567890123456789012123456 1 (E) being able to see all the Earth at once. 7 34567890123456789012345678901212345678901234567890123456789012123456 7 12 imagine our view is to stand up and look 34567890123456789012345678901212345678901234567890123456789012123456 7 12 34567890123456789012345678901212345678901234567890123456789012123456 7 12 down at your feet. Imagine that your eyes 15. The author compares the view of the 34567890123456789012345678901212345678901234567890123456789012123456 7 12 2 34567890123456789012345678901212345678901234567890123456789012123456 7 1234567890123456789012345678901212345678901234567890123456789012123456 (65) are where the International Space Station Earth from a downward-facing window in 7 1234567890123456789012345678901212345678901234567890123456789012123456 7 1234567890123456789012345678901212345678901234567890123456789012123456 is orbiting, and the floor is the surface of the International Space Station to 7 1234567890123456789012345678901212345678901234567890123456789012123456 7 1234567890123456789012345678901212345678901234567890123456789012123456 the Earth. The atmosphere would be 7 1234567890123456789012345678901212345678901234567890123456789012123456 (A) holding a blue marble at arm’s 7 1 about 6 inches high, and the height of the 34567890123456789012345678901212345678901234567890123456789012123456 7 12 length. tallest mountain is less than 2 inches, or 34567890123456789012345678901212345678901234567890123456789012123456 7 12 34567890123456789012345678901212345678901234567890123456789012123456 7 12 (B) having your face up-close to a big (70) about the height of the tops of your feet. 2 34567890123456789012345678901212345678901234567890123456789012123456 1234567890123456789012345678901212345678901234567890123456789012123456 77 1234567890123456789012345678901212345678901234567890123456789012123456 blue beach ball. Almost all of the people below you 7 1234567890123456789012345678901212345678901234567890123456789012123456 7 1234567890123456789012345678901212345678901234567890123456789012123456 (C) looking at the tips of your shoes would live in the first one quarter of an 1234567890123456789012345678901212345678901234567890123456789012123456 77 1234567890123456789012345678901212345678901234567890123456789012123456 when standing up. inch from the floor. The horizon of the 7 1234567890123456789012345678901212345678901234567890123456789012123456 7 1234567890123456789012345678901212345678901234567890123456789012123456 (D) looking at an object that is on the Earth is a little over 20 feet away from 1234567890123456789012345678901212345678901234567890123456789012123456 77 1 (75) where you are standing. If you are ground fifteen feet away when you 34567890123456789012345678901212345678901234567890123456789012123456 7 12 34567890123456789012345678901212345678901234567890123456789012123456 7 12 are standing up. standing on top of Denver, then about 15 7 1234567890123456789012345678901212345678901234567890123456789012123456 (E) the view from a high-flying plane. 7 1234567890123456789012345678901212345678901234567890123456789012123456 feet to one side you can see San Fran34567890123456789012345678901212345678901234567890123456789012123456 7 12 2 34567890123456789012345678901212345678901234567890123456789012123456 7 1234567890123456789012345678901212345678901234567890123456789012123456 cisco, and about 15 feet to the other side 7 1234567890123456789012345678901212345678901234567890123456789012123456 16. In the passage, the author contrasts the 7 1 you can see Chicago. 34567890123456789012345678901212345678901234567890123456789012123456 7 12 view from a window looking “straight 34567890123456789012345678901212345678901234567890123456789012123456 7 12 2 34567890123456789012345678901212345678901234567890123456789012123456 7 1234567890123456789012345678901212345678901234567890123456789012123456 down” with the view from 7 1234567890123456789012345678901212345678901234567890123456789012123456 7 1 13. The primary purpose of this passage is to 7 1234567890123456789012345678901212345678901234567890123456789012123456 (A) the observational deck. 7 1234567890123456789012345678901212345678901234567890123456789012123456 (A) provide a layperson’s account of the 2 34567890123456789012345678901212345678901234567890123456789012123456 7 1234567890123456789012345678901212345678901234567890123456789012123456 (B) a sideward-facing window. 7 1234567890123456789012345678901212345678901234567890123456789012123456 Space Station’s motion over the Earth. 7 1234567890123456789012345678901212345678901234567890123456789012123456 (C) a passenger airliner. 7 1234567890123456789012345678901212345678901234567890123456789012123456 (B) explain the relationship between the 7 1 (D) a window looking “straight up.” 2 34567890123456789012345678901212345678901234567890123456789012123456 7 1234567890123456789012345678901212345678901234567890123456789012123456 diameter of the Earth and the (E) the circular windows on the 7 1234567890123456789012345678901212345678901234567890123456789012123456 7 1234567890123456789012345678901212345678901234567890123456789012123456 thickness of the Earth’s atmosphere. space station. 7 1234567890123456789012345678901212345678901234567890123456789012123456 7 1 (C) answer the imagined question, “What 2 34567890123456789012345678901212345678901234567890123456789012123456 1234567890123456789012345678901212345678901234567890123456789012123456 77 1234567890123456789012345678901212345678901234567890123456789012123456 do astronauts see from space?” 7 1234567890123456789012345678901212345678901234567890123456789012123456 7 1234567890123456789012345678901212345678901234567890123456789012123456 (D) give a glimpse of some of the daily 7 1 2 34567890123456789012345678901212345678901234567890123456789012123456 7 1234567890123456789012345678901212345678901234567890123456789012123456 activities of astronauts in space. 7 1 34567890123456789012345678901212345678901234567890123456789012123456 7 12 (E) discuss the thickness and composi7 1234567890123456789012345678901212345678901234567890123456789012123456 tion of the atmosphere. 34567890123456789012345678901212345678901234567890123456789012123456 7 12 2 34567890123456789012345678901212345678901234567890123456789012123456 7 1 1234567890123456789012345678901212345678901234567890123456789012123456 7 1234567890123456789012345678901212345678901234567890123456789012123456 7 1234567890123456789012345678901212345678901234567890123456789012123456 7 12345678901234567890123456789012123456789012345678901234567890121234567 8

Copyright © 2005 Thomson Peterson’s, a part of The Thomson Corporaton SAT is a registered trademark of the College Entrance Examination Board, which was not involved in the production of and does not endorse this product.

12345678901234567890123456789012123456789012345678901234567890121234567 1234567890123456789012345678901212345678901234567890123456789012123456 7 7 12234567890123456789012345678901212345678901234567890123456789012123456 17. The “faint glow” at night that the author 19. The tone of the passage is best described as 7 1 34567890123456789012345678901212345678901234567890123456789012123456 7 12234567890123456789012345678901212345678901234567890123456789012123456 speaks of in the passage comes from 7 1234567890123456789012345678901212345678901234567890123456789012123456 (A) fairly technical. 1 34567890123456789012345678901212345678901234567890123456789012123456 7 7 12234567890123456789012345678901212345678901234567890123456789012123456 (A) low-lying atmosphere. B. highly professional. 7 1 34567890123456789012345678901212345678901234567890123456789012123456 7 12234567890123456789012345678901212345678901234567890123456789012123456 (B) the outer edges of the atmosphere. (C) refreshingly irreverent. 7 1234567890123456789012345678901212345678901234567890123456789012123456 7 1234567890123456789012345678901212345678901234567890123456789012123456 (C) the eastern horizon of the Earth just (D) engagingly conversational. 7 1234567890123456789012345678901212345678901234567890123456789012123456 34567890123456789012345678901212345678901234567890123456789012123456 7 1 before sunrise. (E) lyrically impassioned. 7 12234567890123456789012345678901212345678901234567890123456789012123456 7 1234567890123456789012345678901212345678901234567890123456789012123456 (D) haze from foreign particulates in the 7 1234567890123456789012345678901212345678901234567890123456789012123456 7 1234567890123456789012345678901212345678901234567890123456789012123456 20. From the passage as a whole, it can be atmosphere. 34567890123456789012345678901212345678901234567890123456789012123456 7 1 7 12234567890123456789012345678901212345678901234567890123456789012123456 inferred that the astronauts’ training (E) the sun reflecting off aircraft in the 7 1234567890123456789012345678901212345678901234567890123456789012123456 7 1234567890123456789012345678901212345678901234567890123456789012123456 high atmosphere. (A) did not prepare them for their free 7 1234567890123456789012345678901212345678901234567890123456789012123456 34567890123456789012345678901212345678901234567890123456789012123456 7 1 time in space. 7 12234567890123456789012345678901212345678901234567890123456789012123456 18. In the last paragraph the author provides 7 1234567890123456789012345678901212345678901234567890123456789012123456 (B) included a great deal of zero-gravity 7 1234567890123456789012345678901212345678901234567890123456789012123456 the thought exercise with the reader’s 7 1234567890123456789012345678901212345678901234567890123456789012123456 exercises. 34567890123456789012345678901212345678901234567890123456789012123456 7 1234567890123456789012345678901212345678901234567890123456789012123456 height primarily to 7 1234567890123456789012345678901212345678901234567890123456789012123456 (C) was more physical than technical. 1234567890123456789012345678901212345678901234567890123456789012123456 77 1234567890123456789012345678901212345678901234567890123456789012123456 (A) demonstrate the distance from (D) involved a strong background 7 1234567890123456789012345678901212345678901234567890123456789012123456 7 1234567890123456789012345678901212345678901234567890123456789012123456 Denver to San Francisco. in math. 7 1 7 12234567890123456789012345678901212345678901234567890123456789012123456 (B) give the reader a concrete sense of (E) focused on the astronauts’ communi7 1234567890123456789012345678901212345678901234567890123456789012123456 7 1234567890123456789012345678901212345678901234567890123456789012123456 the proportions involved in looking cation procedures and abilities. 34567890123456789012345678901212345678901234567890123456789012123456 1234567890123456789012345678901212345678901234567890123456789012123456 77 1234567890123456789012345678901212345678901234567890123456789012123456 down from the space station. 7 1234567890123456789012345678901212345678901234567890123456789012123456 7 1234567890123456789012345678901212345678901234567890123456789012123456 (C) point out that most humans live at a 1234567890123456789012345678901212345678901234567890123456789012123456 77 1234567890123456789012345678901212345678901234567890123456789012123456 low altitude relative to the height of 7 1234567890123456789012345678901212345678901234567890123456789012123456 7 1234567890123456789012345678901212345678901234567890123456789012123456 the atmosphere. 1234567890123456789012345678901212345678901234567890123456789012123456 77 1 (D) illustrate the expansion of one’s 7 12234567890123456789012345678901212345678901234567890123456789012123456 7 horizon at high altitudes. 1234567890123456789012345678901212345678901234567890123456789012123456 7 1 34567890123456789012345678901212345678901234567890123456789012123456 (E) provide visual details of his activities 7 1234567890123456789012345678901212345678901234567890123456789012123456 7 12234567890123456789012345678901212345678901234567890123456789012123456 in space. 34567890123456789012345678901212345678901234567890123456789012123456 1234567890123456789012345678901212345678901234567890123456789012123456 77 1234567890123456789012345678901212345678901234567890123456789012123456 7 1 7 12234567890123456789012345678901212345678901234567890123456789012123456 7 1234567890123456789012345678901212345678901234567890123456789012123456 34567890123456789012345678901212345678901234567890123456789012123456 1234567890123456789012345678901212345678901234567890123456789012123456 77 1234567890123456789012345678901212345678901234567890123456789012123456 7 1 1234567890123456789012345678901212345678901234567890123456789012123456 7 1234567890123456789012345678901212345678901234567890123456789012123456 7 7 12234567890123456789012345678901212345678901234567890123456789012123456 7 1234567890123456789012345678901212345678901234567890123456789012123456 7 1234567890123456789012345678901212345678901234567890123456789012123456 7 1234567890123456789012345678901212345678901234567890123456789012123456 1 34567890123456789012345678901212345678901234567890123456789012123456 7 2 34567890123456789012345678901212345678901234567890123456789012123456 1234567890123456789012345678901212345678901234567890123456789012123456 77 1234567890123456789012345678901212345678901234567890123456789012123456 7 1234567890123456789012345678901212345678901234567890123456789012123456 7 1234567890123456789012345678901212345678901234567890123456789012123456 7 1 7 12234567890123456789012345678901212345678901234567890123456789012123456 7 1234567890123456789012345678901212345678901234567890123456789012123456 7 1234567890123456789012345678901212345678901234567890123456789012123456 7 1234567890123456789012345678901212345678901234567890123456789012123456 1 34567890123456789012345678901212345678901234567890123456789012123456 7 2 34567890123456789012345678901212345678901234567890123456789012123456 1234567890123456789012345678901212345678901234567890123456789012123456 77 1 7 12234567890123456789012345678901212345678901234567890123456789012123456 1 34567890123456789012345678901212345678901234567890123456789012123456 7 7 12234567890123456789012345678901212345678901234567890123456789012123456 Do not proceed to the next section until time is up. 7 1 34567890123456789012345678901212345678901234567890123456789012123456 1234567890123456789012345678901212345678901234567890123456789012123456 7 1234567890123456789012345678901212345678901234567890123456789012123456 7 1234567890123456789012345678901212345678901234567890123456789012123456 7 12345678901234567890123456789012123456789012345678901234567890121234567

STOP

Copyright © 2005 Thomson Peterson’s, a part of The Thomson Corporaton SAT is a registered trademark of the College Entrance Examination Board, which was not involved in the production of and does not endorse this product.

9

Section 2 21 Questions j Time—25 Minutes

Reference Information

12345678901234567890123456789012123456789012345678901234567890121234567 34567890123456789012345678901212345678901234567890123456789012123456 7 12 34567890123456789012345678901212345678901234567890123456789012123456 7 12 2 1 34567890123456789012345678901212345678901234567890123456789012123456 7 34567890123456789012345678901212345678901234567890123456789012123456 7 12 Directions: Solve the following problems using any available space on the page for scratchwork. 34567890123456789012345678901212345678901234567890123456789012123456 7 12 34567890123456789012345678901212345678901234567890123456789012123456 7 12 Mark the letter of your choice on the answer sheet that best corresponds to the correct answer. 34567890123456789012345678901212345678901234567890123456789012123456 7 12 2 34567890123456789012345678901212345678901234567890123456789012123456 1234567890123456789012345678901212345678901234567890123456789012123456 77 Notes: 1234567890123456789012345678901212345678901234567890123456789012123456 7 1234567890123456789012345678901212345678901234567890123456789012123456 1. You may use a calculator. All of the numbers used are real numbers. 7 1234567890123456789012345678901212345678901234567890123456789012123456 1234567890123456789012345678901212345678901234567890123456789012123456 77 1234567890123456789012345678901212345678901234567890123456789012123456 2. You may use the figures that accompany the problems to help you find the solution. Unless 7 1 34567890123456789012345678901212345678901234567890123456789012123456 7 12 the instructions say that a figure is not drawn to scale, assume that it has been drawn 34567890123456789012345678901212345678901234567890123456789012123456 7 12 34567890123456789012345678901212345678901234567890123456789012123456 7 12 accurately. Each figure lies in a plane unless the instructions say otherwise. 34567890123456789012345678901212345678901234567890123456789012123456 7 12 34567890123456789012345678901212345678901234567890123456789012123456 7 12 34567890123456789012345678901212345678901234567890123456789012123456 7 12 34567890123456789012345678901212345678901234567890123456789012123456 7 12 34567890123456789012345678901212345678901234567890123456789012123456 7 12 2 7 1234567890123456789012345678901212345678901234567890123456789012123456 2s r 2x 7 1234567890123456789012345678901212345678901234567890123456789012123456 w r s x 7 1234567890123456789012345678901212345678901234567890123456789012123456 c h h h b 7 1234567890123456789012345678901212345678901234567890123456789012123456 7 1 34567890123456789012345678901212345678901234567890123456789012123456 w 2 34567890123456789012345678901212345678901234567890123456789012123456 7 1234567890123456789012345678901212345678901234567890123456789012123456 s b 3x a A 5 pr2 7 1234567890123456789012345678901212345678901234567890123456789012123456 1 2 2 2 2 7 1 h c 5 a 1 b Special Right Triangles V 5 ,wh V 5 pr C 5 2pr A 5 ,w bh A 5 7 1234567890123456789012345678901212345678901234567890123456789012123456 2 34567890123456789012345678901212345678901234567890123456789012123456 7 12 2 34567890123456789012345678901212345678901234567890123456789012123456 7 1234567890123456789012345678901212345678901234567890123456789012123456 The number of degrees of arc in a circle is 360. 7 1234567890123456789012345678901212345678901234567890123456789012123456 The measure in degrees of a straight angle is 180. 7 1 34567890123456789012345678901212345678901234567890123456789012123456 7 12 The sum of the measures in degrees of the angles of a triangle is 180. 34567890123456789012345678901212345678901234567890123456789012123456 7 12 2 34567890123456789012345678901212345678901234567890123456789012123456 1234567890123456789012345678901212345678901234567890123456789012123456 77 1234567890123456789012345678901212345678901234567890123456789012123456 7 1 7 1234567890123456789012345678901212345678901234567890123456789012123456 1. If x 1 4y 5 3 and x 5 2y, then y 5 2. It takes Michael three hours to mow 2d 7 1234567890123456789012345678901212345678901234567890123456789012123456 2 34567890123456789012345678901212345678901234567890123456789012123456 1234567890123456789012345678901212345678901234567890123456789012123456 acres. If Michael mows d acres at the same 7 7 1234567890123456789012345678901212345678901234567890123456789012123456 (A) 21 7 1234567890123456789012345678901212345678901234567890123456789012123456 rate, how many minutes would it take? 7 1234567890123456789012345678901212345678901234567890123456789012123456 (B) 0 7 1 2 34567890123456789012345678901212345678901234567890123456789012123456 7 1234567890123456789012345678901212345678901234567890123456789012123456 (C) 1 (A) 75 7 1234567890123456789012345678901212345678901234567890123456789012123456 7 1234567890123456789012345678901212345678901234567890123456789012123456 (D) 2 (B) 90 1234567890123456789012345678901212345678901234567890123456789012123456 77 1 (E) 3 (C) 100 34567890123456789012345678901212345678901234567890123456789012123456 7 12 34567890123456789012345678901212345678901234567890123456789012123456 7 12 (D) 120 34567890123456789012345678901212345678901234567890123456789012123456 7 12 (E) 150 34567890123456789012345678901212345678901234567890123456789012123456 7 12 1234567890123456789012345678901212345678901234567890123456789012123456 7 2 34567890123456789012345678901212345678901234567890123456789012123456 1234567890123456789012345678901212345678901234567890123456789012123456 77 1 34567890123456789012345678901212345678901234567890123456789012123456 7 12 1234567890123456789012345678901212345678901234567890123456789012123456 7 34567890123456789012345678901212345678901234567890123456789012123456 7 12 1234567890123456789012345678901212345678901234567890123456789012123456 7 1234567890123456789012345678901212345678901234567890123456789012123456 7 1234567890123456789012345678901212345678901234567890123456789012123456 7 1234567890123456789012345678901212345678901234567890123456789012123456 7 12345678901234567890123456789012123456789012345678901234567890121234567 10

45

60

30

45

Copyright © 2005 Thomson Peterson’s, a part of The Thomson Corporaton SAT is a registered trademark of the College Entrance Examination Board, which was not involved in the production of and does not endorse this product.

12345678901234567890123456789012123456789012345678901234567890121234567 1234567890123456789012345678901212345678901234567890123456789012123456 7 7 12234567890123456789012345678901212345678901234567890123456789012123456 3. If 3f 1 15 5 27, then 3f 2 6 5 7. If =x2 5 2x 1 3, then x 5 7 1 34567890123456789012345678901212345678901234567890123456789012123456 7 12234567890123456789012345678901212345678901234567890123456789012123456 7 1234567890123456789012345678901212345678901234567890123456789012123456 (A) 2x (A) 6 1 34567890123456789012345678901212345678901234567890123456789012123456 7 7 12234567890123456789012345678901212345678901234567890123456789012123456 (B) 23 (B) 10 7 1 34567890123456789012345678901212345678901234567890123456789012123456 7 12234567890123456789012345678901212345678901234567890123456789012123456 (C) x (C) 12 7 1234567890123456789012345678901212345678901234567890123456789012123456 7 1234567890123456789012345678901212345678901234567890123456789012123456 (D) 21 (D) 17 7 1234567890123456789012345678901212345678901234567890123456789012123456 34567890123456789012345678901212345678901234567890123456789012123456 7 1 (E) 4 (E) 20 7 12234567890123456789012345678901212345678901234567890123456789012123456 7 1234567890123456789012345678901212345678901234567890123456789012123456 7 1234567890123456789012345678901212345678901234567890123456789012123456 1234567890123456789012345678901212345678901234567890123456789012123456 8. If |x 1 1| . |y| then which of the following 7 4. 34567890123456789012345678901212345678901234567890123456789012123456 7 1 7 12234567890123456789012345678901212345678901234567890123456789012123456 expresses the relationship between 7 1234567890123456789012345678901212345678901234567890123456789012123456 7 x and y? 1234567890123456789012345678901212345678901234567890123456789012123456 7 1234567890123456789012345678901212345678901234567890123456789012123456 34567890123456789012345678901212345678901234567890123456789012123456 7 1 (A) x 1 1 . y 7 12234567890123456789012345678901212345678901234567890123456789012123456 7 1234567890123456789012345678901212345678901234567890123456789012123456 (B) x 1 1 . y 7 1234567890123456789012345678901212345678901234567890123456789012123456 7 1234567890123456789012345678901212345678901234567890123456789012123456 (C) x , y 34567890123456789012345678901212345678901234567890123456789012123456 1234567890123456789012345678901212345678901234567890123456789012123456 77 1234567890123456789012345678901212345678901234567890123456789012123456 What is the value of x 1 y? (D) x . y 7 1234567890123456789012345678901212345678901234567890123456789012123456 7 1234567890123456789012345678901212345678901234567890123456789012123456 (E) It cannot be determined. 7 1234567890123456789012345678901212345678901234567890123456789012123456 (A) 7 1234567890123456789012345678901212345678901234567890123456789012123456 77 1 (B) 7=2 7 12234567890123456789012345678901212345678901234567890123456789012123456 9. 75% of 104 is the same value as 60% of 7 1234567890123456789012345678901212345678901234567890123456789012123456 3 7 (C) = 7 1234567890123456789012345678901212345678901234567890123456789012123456 what number? 34567890123456789012345678901212345678901234567890123456789012123456 1234567890123456789012345678901212345678901234567890123456789012123456 77 1234567890123456789012345678901212345678901234567890123456789012123456 (D) 14 (A) 130 7 1234567890123456789012345678901212345678901234567890123456789012123456 7 1234567890123456789012345678901212345678901234567890123456789012123456 (E) It cannot be determined. (B) 133 1234567890123456789012345678901212345678901234567890123456789012123456 77 1234567890123456789012345678901212345678901234567890123456789012123456 (C) 136 7 1234567890123456789012345678901212345678901234567890123456789012123456 7 1234567890123456789012345678901212345678901234567890123456789012123456 5. Steve bought a snack and a drink for (D) 140 1234567890123456789012345678901212345678901234567890123456789012123456 77 1 $1.30. If the snack costs twenty cents less (E) 144 7 12234567890123456789012345678901212345678901234567890123456789012123456 than the drink, how much does the 7 1234567890123456789012345678901212345678901234567890123456789012123456 7 1 34567890123456789012345678901212345678901234567890123456789012123456 drink cost? 7 1234567890123456789012345678901212345678901234567890123456789012123456 10. If 3y 2 x 5 12 is the equation of a line, 7 12234567890123456789012345678901212345678901234567890123456789012123456 34567890123456789012345678901212345678901234567890123456789012123456 7 1234567890123456789012345678901212345678901234567890123456789012123456 what is twice the value of this line’s (A) $0.50 7 1234567890123456789012345678901212345678901234567890123456789012123456 7 1 y-intercept? (B) $0.55 7 12234567890123456789012345678901212345678901234567890123456789012123456 7 1234567890123456789012345678901212345678901234567890123456789012123456 (C) $0.65 (A) 22 34567890123456789012345678901212345678901234567890123456789012123456 1234567890123456789012345678901212345678901234567890123456789012123456 77 1234567890123456789012345678901212345678901234567890123456789012123456 (D) $0.75 (B) 2 7 1 7 1234567890123456789012345678901212345678901234567890123456789012123456 (E) $0.80 (C) 4 1234567890123456789012345678901212345678901234567890123456789012123456 7 2 34567890123456789012345678901212345678901234567890123456789012123456 7 1234567890123456789012345678901212345678901234567890123456789012123456 (D) 8 7 1234567890123456789012345678901212345678901234567890123456789012123456 x 7 1234567890123456789012345678901212345678901234567890123456789012123456 6. If f(x) 5 x 1 x , when x 5 2, f(2x) 5 (E) 24 1234567890123456789012345678901212345678901234567890123456789012123456 77 1 (A) 6 7 12234567890123456789012345678901212345678901234567890123456789012123456 7 1234567890123456789012345678901212345678901234567890123456789012123456 (B) 20 7 1234567890123456789012345678901212345678901234567890123456789012123456 7 1234567890123456789012345678901212345678901234567890123456789012123456 (C) 200 1 34567890123456789012345678901212345678901234567890123456789012123456 7 2 34567890123456789012345678901212345678901234567890123456789012123456 7 1234567890123456789012345678901212345678901234567890123456789012123456 (D) 260 7 1234567890123456789012345678901212345678901234567890123456789012123456 7 1234567890123456789012345678901212345678901234567890123456789012123456 (E) 4096 1234567890123456789012345678901212345678901234567890123456789012123456 77 1 7 12234567890123456789012345678901212345678901234567890123456789012123456 1 34567890123456789012345678901212345678901234567890123456789012123456 7 7 12234567890123456789012345678901212345678901234567890123456789012123456 1 34567890123456789012345678901212345678901234567890123456789012123456 7 7 12234567890123456789012345678901212345678901234567890123456789012123456 1 34567890123456789012345678901212345678901234567890123456789012123456 7 1234567890123456789012345678901212345678901234567890123456789012123456 7 1234567890123456789012345678901212345678901234567890123456789012123456 7 1234567890123456789012345678901212345678901234567890123456789012123456 7 12345678901234567890123456789012123456789012345678901234567890121234567 Copyright © 2005 Thomson Peterson’s, a part of The Thomson Corporaton SAT is a registered trademark of the College Entrance Examination Board, which was not involved in the production of and does not endorse this product.

11

12345678901234567890123456789012123456789012345678901234567890121234567 1234567890123456789012345678901212345678901234567890123456789012123456 7 34567890123456789012345678901212345678901234567890123456789012123456 7 12 11. 14. If A . B . C . D . E, and each is a digit 7 1234567890123456789012345678901212345678901234567890123456789012123456 p q 34567890123456789012345678901212345678901234567890123456789012123456 7 12 ᐉ 1 through 9, then ABCD 2 { (ABCD) is 34567890123456789012345678901212345678901234567890123456789012123456 7 12 1234567890123456789012345678901212345678901234567890123456789012123456 7 34567890123456789012345678901212345678901234567890123456789012123456 7 12 (A) less than zero. 7 1234567890123456789012345678901212345678901234567890123456789012123456 34567890123456789012345678901212345678901234567890123456789012123456 7 12 (B) between zero and 100. 34567890123456789012345678901212345678901234567890123456789012123456 7 12 34567890123456789012345678901212345678901234567890123456789012123456 7 12 (C) between 100 and 500. 34567890123456789012345678901212345678901234567890123456789012123456 7 12 2 34567890123456789012345678901212345678901234567890123456789012123456 7 1 (D) between 500 and 1000. 34567890123456789012345678901212345678901234567890123456789012123456 7 12 34567890123456789012345678901212345678901234567890123456789012123456 7 12 (E) more than 1000. 34567890123456789012345678901212345678901234567890123456789012123456 7 12 34567890123456789012345678901212345678901234567890123456789012123456 7 12 7 1234567890123456789012345678901212345678901234567890123456789012123456 m 34567890123456789012345678901212345678901234567890123456789012123456 7 12 15. 34567890123456789012345678901212345678901234567890123456789012123456 7 12 34567890123456789012345678901212345678901234567890123456789012123456 7 12 34567890123456789012345678901212345678901234567890123456789012123456 7 12 2 7 1 34567890123456789012345678901212345678901234567890123456789012123456 Given the figure above, which of the 34567890123456789012345678901212345678901234567890123456789012123456 7 12 34567890123456789012345678901212345678901234567890123456789012123456 7 12 following must be a true statement? 34567890123456789012345678901212345678901234567890123456789012123456 7 12 34567890123456789012345678901212345678901234567890123456789012123456 7 12 34567890123456789012345678901212345678901234567890123456789012123456 7 12 (A) x . y 34567890123456789012345678901212345678901234567890123456789012123456 7 12 34567890123456789012345678901212345678901234567890123456789012123456 7 12 (B) x , y 34567890123456789012345678901212345678901234567890123456789012123456 7 12 34567890123456789012345678901212345678901234567890123456789012123456 7 12 (C) x 5 y 34567890123456789012345678901212345678901234567890123456789012123456 7 12 7 1234567890123456789012345678901212345678901234567890123456789012123456 (D) x 1 y 5 90 34567890123456789012345678901212345678901234567890123456789012123456 7 12 (E) x 2 y 5 90 34567890123456789012345678901212345678901234567890123456789012123456 7 12 34567890123456789012345678901212345678901234567890123456789012123456 7 12 2 34567890123456789012345678901212345678901234567890123456789012123456 1234567890123456789012345678901212345678901234567890123456789012123456 77 1234567890123456789012345678901212345678901234567890123456789012123456 7 1234567890123456789012345678901212345678901234567890123456789012123456 Questions 12–14 refer to the following 7 1234567890123456789012345678901212345678901234567890123456789012123456 7 1234567890123456789012345678901212345678901234567890123456789012123456 definition. 1234567890123456789012345678901212345678901234567890123456789012123456 77 1234567890123456789012345678901212345678901234567890123456789012123456 Let { m be defined for any positive integer m as 7 1234567890123456789012345678901212345678901234567890123456789012123456 7 1234567890123456789012345678901212345678901234567890123456789012123456 the number obtained when the first and last digit 7 1 2 34567890123456789012345678901212345678901234567890123456789012123456 7 1234567890123456789012345678901212345678901234567890123456789012123456 If RT 5 18, what is the area of the of m switch places. 7 1234567890123456789012345678901212345678901234567890123456789012123456 7 1 above circle? 7 1234567890123456789012345678901212345678901234567890123456789012123456 For example { 4 5 4, { 35 5 53, and 34567890123456789012345678901212345678901234567890123456789012123456 7 12 2 34567890123456789012345678901212345678901234567890123456789012123456 7 1234567890123456789012345678901212345678901234567890123456789012123456 (A) 6p { 2003 5 3002. 7 1234567890123456789012345678901212345678901234567890123456789012123456 (B) 12p 7 1 34567890123456789012345678901212345678901234567890123456789012123456 7 12 12. { 2323 2 { 2321 5 (C) 36p 34567890123456789012345678901212345678901234567890123456789012123456 7 12 2 34567890123456789012345678901212345678901234567890123456789012123456 7 1234567890123456789012345678901212345678901234567890123456789012123456 (D) 81p 7 1234567890123456789012345678901212345678901234567890123456789012123456 (A) 2 7 1 (E) 324p 7 1234567890123456789012345678901212345678901234567890123456789012123456 (B) 20 1234567890123456789012345678901212345678901234567890123456789012123456 7 2 34567890123456789012345678901212345678901234567890123456789012123456 7 1234567890123456789012345678901212345678901234567890123456789012123456 (C) 200 7 1234567890123456789012345678901212345678901234567890123456789012123456 7 1234567890123456789012345678901212345678901234567890123456789012123456 (D) 2000 1234567890123456789012345678901212345678901234567890123456789012123456 77 1 (E) 20,000 34567890123456789012345678901212345678901234567890123456789012123456 7 12 34567890123456789012345678901212345678901234567890123456789012123456 7 12 34567890123456789012345678901212345678901234567890123456789012123456 7 12 13. If A is a two-digit number between 10 and 34567890123456789012345678901212345678901234567890123456789012123456 7 12 7 1234567890123456789012345678901212345678901234567890123456789012123456 2 2 5 {(A ), then A 5 20 and ({ A) 2 34567890123456789012345678901212345678901234567890123456789012123456 1234567890123456789012345678901212345678901234567890123456789012123456 77 1234567890123456789012345678901212345678901234567890123456789012123456 7 1234567890123456789012345678901212345678901234567890123456789012123456 (A) 11 7 1234567890123456789012345678901212345678901234567890123456789012123456 7 1 (B) 13 2 34567890123456789012345678901212345678901234567890123456789012123456 1234567890123456789012345678901212345678901234567890123456789012123456 77 1 (C) 14 34567890123456789012345678901212345678901234567890123456789012123456 7 12 7 1234567890123456789012345678901212345678901234567890123456789012123456 (D) 16 34567890123456789012345678901212345678901234567890123456789012123456 7 12 2 34567890123456789012345678901212345678901234567890123456789012123456 7 1 (E) 18 7 1234567890123456789012345678901212345678901234567890123456789012123456 1234567890123456789012345678901212345678901234567890123456789012123456 77 1234567890123456789012345678901212345678901234567890123456789012123456 12345678901234567890123456789012123456789012345678901234567890121234567 12

Copyright © 2005 Thomson Peterson’s, a part of The Thomson Corporaton SAT is a registered trademark of the College Entrance Examination Board, which was not involved in the production of and does not endorse this product.

12345678901234567890123456789012123456789012345678901234567890121234567 1234567890123456789012345678901212345678901234567890123456789012123456 7 7 12234567890123456789012345678901212345678901234567890123456789012123456 16. 18. Which of the following is the product of 7 1 34567890123456789012345678901212345678901234567890123456789012123456 7 12234567890123456789012345678901212345678901234567890123456789012123456 two prime numbers whose difference is 7 1234567890123456789012345678901212345678901234567890123456789012123456 7 1 34567890123456789012345678901212345678901234567890123456789012123456 twenty one? 7 12234567890123456789012345678901212345678901234567890123456789012123456 34567890123456789012345678901212345678901234567890123456789012123456 7 1 7 12234567890123456789012345678901212345678901234567890123456789012123456 (A) 46 7 1234567890123456789012345678901212345678901234567890123456789012123456 7 1234567890123456789012345678901212345678901234567890123456789012123456 (B) 51 7 1234567890123456789012345678901212345678901234567890123456789012123456 (C) 55 34567890123456789012345678901212345678901234567890123456789012123456 7 1 7 12234567890123456789012345678901212345678901234567890123456789012123456 (D) 57 7 1234567890123456789012345678901212345678901234567890123456789012123456 7 1234567890123456789012345678901212345678901234567890123456789012123456 (E) 58 7 1234567890123456789012345678901212345678901234567890123456789012123456 34567890123456789012345678901212345678901234567890123456789012123456 7 1 7 12234567890123456789012345678901212345678901234567890123456789012123456 19. What is the distance from the midpoint of 7 1234567890123456789012345678901212345678901234567890123456789012123456 7 1234567890123456789012345678901212345678901234567890123456789012123456 7 1234567890123456789012345678901212345678901234567890123456789012123456 DE to the origin, if D(0,12) and E(5,0)? 34567890123456789012345678901212345678901234567890123456789012123456 7 1 7 12234567890123456789012345678901212345678901234567890123456789012123456 (A) 5 7 1234567890123456789012345678901212345678901234567890123456789012123456 According to the above table, which of the 7 1234567890123456789012345678901212345678901234567890123456789012123456 (B) 6.5 7 1234567890123456789012345678901212345678901234567890123456789012123456 following statements is true? (C) 8.2 34567890123456789012345678901212345678901234567890123456789012123456 1234567890123456789012345678901212345678901234567890123456789012123456 77 1234567890123456789012345678901212345678901234567890123456789012123456 (D) 12 7 1234567890123456789012345678901212345678901234567890123456789012123456 (A) September total sales are 3.5% 7 1234567890123456789012345678901212345678901234567890123456789012123456 (E) 13 7 1234567890123456789012345678901212345678901234567890123456789012123456 greater than August total sales. 1234567890123456789012345678901212345678901234567890123456789012123456 77 1 (B) September total sales are 2.5% less 12234567890123456789012345678901212345678901234567890123456789012123456 20. If 2b2 5 (b 2 7)(b 1 3) 2 (2b 1 2)(b 1 5), 7 7 than August total sales. 1234567890123456789012345678901212345678901234567890123456789012123456 then b equals 7 1234567890123456789012345678901212345678901234567890123456789012123456 (C) September total sales are 2.5% 34567890123456789012345678901212345678901234567890123456789012123456 1234567890123456789012345678901212345678901234567890123456789012123456 77 1234567890123456789012345678901212345678901234567890123456789012123456 15 greater than August total sales. 7 1234567890123456789012345678901212345678901234567890123456789012123456 (A) 21 7 1234567890123456789012345678901212345678901234567890123456789012123456 16 (D) September total sales are 5.4% less 1234567890123456789012345678901212345678901234567890123456789012123456 77 1234567890123456789012345678901212345678901234567890123456789012123456 3 than August total sales. 7 1234567890123456789012345678901212345678901234567890123456789012123456 (B) 1 7 1234567890123456789012345678901212345678901234567890123456789012123456 8 (E) September total sales are 5.4% 1234567890123456789012345678901212345678901234567890123456789012123456 77 1 (C) 2 greater than August total sales. 7 12234567890123456789012345678901212345678901234567890123456789012123456 7 1234567890123456789012345678901212345678901234567890123456789012123456 10 7 1 34567890123456789012345678901212345678901234567890123456789012123456 (D) 2 7 1234567890123456789012345678901212345678901234567890123456789012123456 17. 11 7 12234567890123456789012345678901212345678901234567890123456789012123456 34567890123456789012345678901212345678901234567890123456789012123456 7 1234567890123456789012345678901212345678901234567890123456789012123456 1 7 1234567890123456789012345678901212345678901234567890123456789012123456 (E) 5 7 1 3 7 12234567890123456789012345678901212345678901234567890123456789012123456 7 1234567890123456789012345678901212345678901234567890123456789012123456 34567890123456789012345678901212345678901234567890123456789012123456 1234567890123456789012345678901212345678901234567890123456789012123456 21. Between 1950 and 2000, the population of 7 7 1234567890123456789012345678901212345678901234567890123456789012123456 7 1 Cree County tripled every ten years, except 1234567890123456789012345678901212345678901234567890123456789012123456 7 1234567890123456789012345678901212345678901234567890123456789012123456 for the decade of the 1980s when the popu- 7 7 12234567890123456789012345678901212345678901234567890123456789012123456 7 1234567890123456789012345678901212345678901234567890123456789012123456 lation of the country fell by one-half. If the 7 1234567890123456789012345678901212345678901234567890123456789012123456 1234567890123456789012345678901212345678901234567890123456789012123456 population of Cree country was 1000 people 7 7 1 34567890123456789012345678901212345678901234567890123456789012123456 What is the perimeter of the above figure? 7 12234567890123456789012345678901212345678901234567890123456789012123456 in 1950, what is the population now? 7 1234567890123456789012345678901212345678901234567890123456789012123456 7 1234567890123456789012345678901212345678901234567890123456789012123456 (A) 24 (A) 40,500 7 1234567890123456789012345678901212345678901234567890123456789012123456 7 1 34567890123456789012345678901212345678901234567890123456789012123456 (B) 25 (B) 72,900 2 34567890123456789012345678901212345678901234567890123456789012123456 1234567890123456789012345678901212345678901234567890123456789012123456 77 1234567890123456789012345678901212345678901234567890123456789012123456 (C) 121,500 (C) 26 7 1234567890123456789012345678901212345678901234567890123456789012123456 (D) 218,700 7 1234567890123456789012345678901212345678901234567890123456789012123456 (D) 27 7 1 (E) 243,000 2 34567890123456789012345678901212345678901234567890123456789012123456 7 1234567890123456789012345678901212345678901234567890123456789012123456 (E) 28 7 1 7 12234567890123456789012345678901212345678901234567890123456789012123456 1 34567890123456789012345678901212345678901234567890123456789012123456 7 7 12234567890123456789012345678901212345678901234567890123456789012123456 Do not proceed to the next section until time is up. 7 1 34567890123456789012345678901212345678901234567890123456789012123456 1234567890123456789012345678901212345678901234567890123456789012123456 7 1234567890123456789012345678901212345678901234567890123456789012123456 7 1234567890123456789012345678901212345678901234567890123456789012123456 7 12345678901234567890123456789012123456789012345678901234567890121234567

STOP

Copyright © 2005 Thomson Peterson’s, a part of The Thomson Corporaton SAT is a registered trademark of the College Entrance Examination Board, which was not involved in the production of and does not endorse this product.

13

Section 3 30 Questions j Time—25 Minutes 12345678901234567890123456789012123456789012345678901234567890121234567 34567890123456789012345678901212345678901234567890123456789012123456 7 12 34567890123456789012345678901212345678901234567890123456789012123456 7 12 2 7 1 34567890123456789012345678901212345678901234567890123456789012123456 Identifying Sentence Errors 34567890123456789012345678901212345678901234567890123456789012123456 7 12 34567890123456789012345678901212345678901234567890123456789012123456 7 12 34567890123456789012345678901212345678901234567890123456789012123456 7 12 34567890123456789012345678901212345678901234567890123456789012123456 7 12 Directions: Mark the letter of your choice on the answer sheet that best corresponds to the 2 7 1234567890123456789012345678901212345678901234567890123456789012123456 7 correct answer. 1234567890123456789012345678901212345678901234567890123456789012123456 7 1234567890123456789012345678901212345678901234567890123456789012123456 7 1234567890123456789012345678901212345678901234567890123456789012123456 Notes: 7 1234567890123456789012345678901212345678901234567890123456789012123456 34567890123456789012345678901212345678901234567890123456789012123456 1234567890123456789012345678901212345678901234567890123456789012123456 77 1 1. The following questions test your knowledge of the rules of English grammar, as well as 34567890123456789012345678901212345678901234567890123456789012123456 7 12 34567890123456789012345678901212345678901234567890123456789012123456 7 12 word usage, word choice, and idioms. 34567890123456789012345678901212345678901234567890123456789012123456 7 12 2 34567890123456789012345678901212345678901234567890123456789012123456 1234567890123456789012345678901212345678901234567890123456789012123456 77 1234567890123456789012345678901212345678901234567890123456789012123456 2. Some sentences are correct, but others contain a single error. No sentence contains more 7 1234567890123456789012345678901212345678901234567890123456789012123456 7 1234567890123456789012345678901212345678901234567890123456789012123456 than one error. 1234567890123456789012345678901212345678901234567890123456789012123456 77 1234567890123456789012345678901212345678901234567890123456789012123456 3. Any errors that occur will be found in the underlined portion of the sentence. Choose the 7 1234567890123456789012345678901212345678901234567890123456789012123456 7 1234567890123456789012345678901212345678901234567890123456789012123456 letter underneath the error to indicate the part of the sentence that must be changed. 7 1234567890123456789012345678901212345678901234567890123456789012123456 7 1 2 34567890123456789012345678901212345678901234567890123456789012123456 7 1234567890123456789012345678901212345678901234567890123456789012123456 4. If there is no error, pick answer choice (E). 7 1234567890123456789012345678901212345678901234567890123456789012123456 7 1 7 1234567890123456789012345678901212345678901234567890123456789012123456 5. There will be no change in any parts of the sentence that are not underlined. 34567890123456789012345678901212345678901234567890123456789012123456 7 12 2 34567890123456789012345678901212345678901234567890123456789012123456 1234567890123456789012345678901212345678901234567890123456789012123456 77 1234567890123456789012345678901212345678901234567890123456789012123456 7 1 34567890123456789012345678901212345678901234567890123456789012123456 7 12 34567890123456789012345678901212345678901234567890123456789012123456 7 12 3. The team, which is composed of four 1. The scientists found that there were less 2 7 1234567890123456789012345678901212345678901234567890123456789012123456 7 1234567890123456789012345678901212345678901234567890123456789012123456 A A 1 34567890123456789012345678901212345678901234567890123456789012123456 7 7 1234567890123456789012345678901212345678901234567890123456789012123456 strands of the mold they were studying cyclists, compete against other cycling 7 1234567890123456789012345678901212345678901234567890123456789012123456 2 34567890123456789012345678901212345678901234567890123456789012123456 7 1234567890123456789012345678901212345678901234567890123456789012123456 B C B 7 1234567890123456789012345678901212345678901234567890123456789012123456 than they needed to conduct the experiteams from around the nation and around 1234567890123456789012345678901212345678901234567890123456789012123456 77 1234567890123456789012345678901212345678901234567890123456789012123456 D C D 7 1 34567890123456789012345678901212345678901234567890123456789012123456 7 12 ment. No error the world. No error 34567890123456789012345678901212345678901234567890123456789012123456 7 12 34567890123456789012345678901212345678901234567890123456789012123456 7 12 E E 34567890123456789012345678901212345678901234567890123456789012123456 7 12 1234567890123456789012345678901212345678901234567890123456789012123456 7 2 34567890123456789012345678901212345678901234567890123456789012123456 7 1234567890123456789012345678901212345678901234567890123456789012123456 2. The painter gave a fantastic demonstration 4. Depending on which historian you read, 7 1234567890123456789012345678901212345678901234567890123456789012123456 7 1234567890123456789012345678901212345678901234567890123456789012123456 A A B 1234567890123456789012345678901212345678901234567890123456789012123456 77 1 for Ada and I and we vowed to utilize the the Pax Romana was either as long as five 34567890123456789012345678901212345678901234567890123456789012123456 7 12 7 1234567890123456789012345678901212345678901234567890123456789012123456 B C C 34567890123456789012345678901212345678901234567890123456789012123456 7 12 new technique in our own work . No error centuries or as short as three . No error 1234567890123456789012345678901212345678901234567890123456789012123456 7 34567890123456789012345678901212345678901234567890123456789012123456 7 12 D E D E 7 1234567890123456789012345678901212345678901234567890123456789012123456 1234567890123456789012345678901212345678901234567890123456789012123456 7 1234567890123456789012345678901212345678901234567890123456789012123456 7 1234567890123456789012345678901212345678901234567890123456789012123456 7 12345678901234567890123456789012123456789012345678901234567890121234567 14

Copyright © 2005 Thomson Peterson’s, a part of The Thomson Corporaton SAT is a registered trademark of the College Entrance Examination Board, which was not involved in the production of and does not endorse this product.

12345678901234567890123456789012123456789012345678901234567890121234567 1234567890123456789012345678901212345678901234567890123456789012123456 7 7 12234567890123456789012345678901212345678901234567890123456789012123456 5. Because they have extensively surveyed the 8. The census statistics was a complex source 7 1 34567890123456789012345678901212345678901234567890123456789012123456 7 12234567890123456789012345678901212345678901234567890123456789012123456 7 A A 1234567890123456789012345678901212345678901234567890123456789012123456 7 1 34567890123456789012345678901212345678901234567890123456789012123456 Lower Ohio Valley , Louis and Clark of information, in that the demographers 7 12234567890123456789012345678901212345678901234567890123456789012123456 34567890123456789012345678901212345678901234567890123456789012123456 7 1 B B 7 12234567890123456789012345678901212345678901234567890123456789012123456 7 1234567890123456789012345678901212345678901234567890123456789012123456 were well prepared to navigate uncharted knew that certain groups of people were 7 1234567890123456789012345678901212345678901234567890123456789012123456 7 1234567890123456789012345678901212345678901234567890123456789012123456 C C 34567890123456789012345678901212345678901234567890123456789012123456 7 1 territory along the Missouri River. No error more likely to respond than others. 7 12234567890123456789012345678901212345678901234567890123456789012123456 7 1234567890123456789012345678901212345678901234567890123456789012123456 D E D 7 1234567890123456789012345678901212345678901234567890123456789012123456 7 1234567890123456789012345678901212345678901234567890123456789012123456 No error 34567890123456789012345678901212345678901234567890123456789012123456 7 1 7 12234567890123456789012345678901212345678901234567890123456789012123456 6. The administrator suggested that the staff E 7 1234567890123456789012345678901212345678901234567890123456789012123456 7 1234567890123456789012345678901212345678901234567890123456789012123456 7 1234567890123456789012345678901212345678901234567890123456789012123456 9. Because of the large traffic jam, scarcely first focus on contacting 34567890123456789012345678901212345678901234567890123456789012123456 7 1 7 12234567890123456789012345678901212345678901234567890123456789012123456 A A 7 1234567890123456789012345678901212345678901234567890123456789012123456 7 1234567890123456789012345678901212345678901234567890123456789012123456 prospective participants for the upcoming no one from the group made it to the 7 1234567890123456789012345678901212345678901234567890123456789012123456 34567890123456789012345678901212345678901234567890123456789012123456 7 1234567890123456789012345678901212345678901234567890123456789012123456 B B C 7 1234567890123456789012345678901212345678901234567890123456789012123456 seminar and then on follow-up with airport in time to catch the flight to the 7 1234567890123456789012345678901212345678901234567890123456789012123456 7 1234567890123456789012345678901212345678901234567890123456789012123456 C D D 1234567890123456789012345678901212345678901234567890123456789012123456 77 1234567890123456789012345678901212345678901234567890123456789012123456 previous speakers. No error conference in San Antonio. No error 7 1 7 12234567890123456789012345678901212345678901234567890123456789012123456 E E 7 1234567890123456789012345678901212345678901234567890123456789012123456 7 1234567890123456789012345678901212345678901234567890123456789012123456 34567890123456789012345678901212345678901234567890123456789012123456 7 1234567890123456789012345678901212345678901234567890123456789012123456 7. The Ottoman Empire initiated better trade 10. Even though he had the title of 7 1234567890123456789012345678901212345678901234567890123456789012123456 7 1234567890123456789012345678901212345678901234567890123456789012123456 A B A 1234567890123456789012345678901212345678901234567890123456789012123456 77 1234567890123456789012345678901212345678901234567890123456789012123456 relations with the Austrian Empire when Vice-President of Operations, his duties 7 1234567890123456789012345678901212345678901234567890123456789012123456 7 1234567890123456789012345678901212345678901234567890123456789012123456 B 7 1234567890123456789012345678901212345678901234567890123456789012123456 and responsibilities were not much greater 7 its border disputes with Russia 1234567890123456789012345678901212345678901234567890123456789012123456 7 1 C C 7 12234567890123456789012345678901212345678901234567890123456789012123456 7 1234567890123456789012345678901212345678901234567890123456789012123456 were resolved (D). No error than a midlevel manager . No error 1 34567890123456789012345678901212345678901234567890123456789012123456 7 7 1234567890123456789012345678901212345678901234567890123456789012123456 D E D E 7 12234567890123456789012345678901212345678901234567890123456789012123456 34567890123456789012345678901212345678901234567890123456789012123456 1234567890123456789012345678901212345678901234567890123456789012123456 77 1234567890123456789012345678901212345678901234567890123456789012123456 7 1 7 12234567890123456789012345678901212345678901234567890123456789012123456 7 1234567890123456789012345678901212345678901234567890123456789012123456 34567890123456789012345678901212345678901234567890123456789012123456 1234567890123456789012345678901212345678901234567890123456789012123456 77 1234567890123456789012345678901212345678901234567890123456789012123456 7 1 1234567890123456789012345678901212345678901234567890123456789012123456 7 1234567890123456789012345678901212345678901234567890123456789012123456 7 7 12234567890123456789012345678901212345678901234567890123456789012123456 7 1234567890123456789012345678901212345678901234567890123456789012123456 7 1234567890123456789012345678901212345678901234567890123456789012123456 7 1234567890123456789012345678901212345678901234567890123456789012123456 1 34567890123456789012345678901212345678901234567890123456789012123456 7 2 34567890123456789012345678901212345678901234567890123456789012123456 1234567890123456789012345678901212345678901234567890123456789012123456 77 1234567890123456789012345678901212345678901234567890123456789012123456 7 1234567890123456789012345678901212345678901234567890123456789012123456 7 1234567890123456789012345678901212345678901234567890123456789012123456 7 1 7 12234567890123456789012345678901212345678901234567890123456789012123456 7 1234567890123456789012345678901212345678901234567890123456789012123456 7 1234567890123456789012345678901212345678901234567890123456789012123456 7 1234567890123456789012345678901212345678901234567890123456789012123456 1 34567890123456789012345678901212345678901234567890123456789012123456 7 2 34567890123456789012345678901212345678901234567890123456789012123456 1234567890123456789012345678901212345678901234567890123456789012123456 77 1 7 12234567890123456789012345678901212345678901234567890123456789012123456 1 34567890123456789012345678901212345678901234567890123456789012123456 7 7 12234567890123456789012345678901212345678901234567890123456789012123456 1 34567890123456789012345678901212345678901234567890123456789012123456 7 1234567890123456789012345678901212345678901234567890123456789012123456 7 1234567890123456789012345678901212345678901234567890123456789012123456 7 1234567890123456789012345678901212345678901234567890123456789012123456 7 12345678901234567890123456789012123456789012345678901234567890121234567 Copyright © 2005 Thomson Peterson’s, a part of The Thomson Corporaton SAT is a registered trademark of the College Entrance Examination Board, which was not involved in the production of and does not endorse this product.

15

12345678901234567890123456789012123456789012345678901234567890121234567 1234567890123456789012345678901212345678901234567890123456789012123456 7 34567890123456789012345678901212345678901234567890123456789012123456 7 12 7 1234567890123456789012345678901212345678901234567890123456789012123456 Improving Sentences 34567890123456789012345678901212345678901234567890123456789012123456 7 12 34567890123456789012345678901212345678901234567890123456789012123456 7 12 1234567890123456789012345678901212345678901234567890123456789012123456 7 34567890123456789012345678901212345678901234567890123456789012123456 7 12 Directions: 7 1234567890123456789012345678901212345678901234567890123456789012123456 34567890123456789012345678901212345678901234567890123456789012123456 7 12 1. The following questions test your knowledge of English grammar, word usage, word 34567890123456789012345678901212345678901234567890123456789012123456 7 12 34567890123456789012345678901212345678901234567890123456789012123456 7 12 choice, sentence construction, and punctuation. 34567890123456789012345678901212345678901234567890123456789012123456 7 12 2 34567890123456789012345678901212345678901234567890123456789012123456 7 1 34567890123456789012345678901212345678901234567890123456789012123456 7 12 2. Every sentence contains a portion that is underlined. 34567890123456789012345678901212345678901234567890123456789012123456 7 12 34567890123456789012345678901212345678901234567890123456789012123456 7 12 3. Any errors that occur will be found in the underlined portion of the sentence. If you 34567890123456789012345678901212345678901234567890123456789012123456 7 12 2 34567890123456789012345678901212345678901234567890123456789012123456 7 1 believe there is an error, choose the answer choice that corrects the original mistake. 34567890123456789012345678901212345678901234567890123456789012123456 7 12 34567890123456789012345678901212345678901234567890123456789012123456 7 12 Answer choices (B), (C), (D), and (E) contain alternative phrasings of the underlined 34567890123456789012345678901212345678901234567890123456789012123456 7 12 34567890123456789012345678901212345678901234567890123456789012123456 7 12 portion. If the sentence contains an error, one of these alternate phrasings will correct it. 7 1234567890123456789012345678901212345678901234567890123456789012123456 34567890123456789012345678901212345678901234567890123456789012123456 7 12 4. Choice (A) repeats the original underlined portion. If you believe the underlined portion 34567890123456789012345678901212345678901234567890123456789012123456 7 12 34567890123456789012345678901212345678901234567890123456789012123456 7 12 does not contain any errors, select answer choice (A). 34567890123456789012345678901212345678901234567890123456789012123456 7 12 2 34567890123456789012345678901212345678901234567890123456789012123456 1234567890123456789012345678901212345678901234567890123456789012123456 77 1234567890123456789012345678901212345678901234567890123456789012123456 5. There will be no change in any parts of the sentence that are not underlined. 7 1234567890123456789012345678901212345678901234567890123456789012123456 1234567890123456789012345678901212345678901234567890123456789012123456 77 1234567890123456789012345678901212345678901234567890123456789012123456 7 1234567890123456789012345678901212345678901234567890123456789012123456 7 1 34567890123456789012345678901212345678901234567890123456789012123456 7 12 11. In an effort to make the Constitution both 12. Widely considered one of the most 34567890123456789012345678901212345678901234567890123456789012123456 7 12 34567890123456789012345678901212345678901234567890123456789012123456 7 12 more accessible and understandable to the original poets of all time, Gerard Manley 2 34567890123456789012345678901212345678901234567890123456789012123456 1234567890123456789012345678901212345678901234567890123456789012123456 77 1234567890123456789012345678901212345678901234567890123456789012123456 public, the House of Representatives has Hopkins’s poems display utterly noncon7 1234567890123456789012345678901212345678901234567890123456789012123456 7 1234567890123456789012345678901212345678901234567890123456789012123456 authorized the publication of a series of ventional systems of rhyme. 1234567890123456789012345678901212345678901234567890123456789012123456 77 1234567890123456789012345678901212345678901234567890123456789012123456 pamphlets about the Constitution. (A) Gerard Manley Hopkins’s poems 7 1234567890123456789012345678901212345678901234567890123456789012123456 7 1234567890123456789012345678901212345678901234567890123456789012123456 (A) both more accessible and underdisplay utterly nonconventional 7 1234567890123456789012345678901212345678901234567890123456789012123456 7 1 standable to the public systems of rhyme. 2 34567890123456789012345678901212345678901234567890123456789012123456 1234567890123456789012345678901212345678901234567890123456789012123456 77 1234567890123456789012345678901212345678901234567890123456789012123456 (B) more both accessible and under(B) Gerard Manley Hopkins’s poems 7 1 7 1234567890123456789012345678901212345678901234567890123456789012123456 standable to the public displayed utterly nonconventional 34567890123456789012345678901212345678901234567890123456789012123456 7 12 2 34567890123456789012345678901212345678901234567890123456789012123456 7 1234567890123456789012345678901212345678901234567890123456789012123456 (C) more accessible to the public and systems of rhyme. 7 1234567890123456789012345678901212345678901234567890123456789012123456 7 1 more understandable for it (C) Gerard Manley Hopkins’s poems 34567890123456789012345678901212345678901234567890123456789012123456 7 12 34567890123456789012345678901212345678901234567890123456789012123456 7 12 (D) both more accessible and more have systems of rhyme that are 34567890123456789012345678901212345678901234567890123456789012123456 7 12 34567890123456789012345678901212345678901234567890123456789012123456 7 12 understandable to the public utterly nonconventional. 1234567890123456789012345678901212345678901234567890123456789012123456 7 7 1234567890123456789012345678901212345678901234567890123456789012123456 (E) accessible to the public and under(D) Gerard Manley Hopkins wrote 7 1234567890123456789012345678901212345678901234567890123456789012123456 2 34567890123456789012345678901212345678901234567890123456789012123456 1234567890123456789012345678901212345678901234567890123456789012123456 standable poems using utterly nonconventional 7 7 1234567890123456789012345678901212345678901234567890123456789012123456 7 1234567890123456789012345678901212345678901234567890123456789012123456 systems of rhyme. 1234567890123456789012345678901212345678901234567890123456789012123456 77 1 (E) Gerard Manley Hopkins had written 34567890123456789012345678901212345678901234567890123456789012123456 7 12 34567890123456789012345678901212345678901234567890123456789012123456 7 12 poems that were displaying utterly 34567890123456789012345678901212345678901234567890123456789012123456 7 12 34567890123456789012345678901212345678901234567890123456789012123456 7 12 nonconventional systems of rhyme. 7 1234567890123456789012345678901212345678901234567890123456789012123456 2 34567890123456789012345678901212345678901234567890123456789012123456 1234567890123456789012345678901212345678901234567890123456789012123456 77 1234567890123456789012345678901212345678901234567890123456789012123456 7 1234567890123456789012345678901212345678901234567890123456789012123456 7 1234567890123456789012345678901212345678901234567890123456789012123456 7 1 34567890123456789012345678901212345678901234567890123456789012123456 7 12 1234567890123456789012345678901212345678901234567890123456789012123456 7 34567890123456789012345678901212345678901234567890123456789012123456 7 12 1234567890123456789012345678901212345678901234567890123456789012123456 7 34567890123456789012345678901212345678901234567890123456789012123456 7 12 1234567890123456789012345678901212345678901234567890123456789012123456 7 1234567890123456789012345678901212345678901234567890123456789012123456 7 1234567890123456789012345678901212345678901234567890123456789012123456 7 1234567890123456789012345678901212345678901234567890123456789012123456 7 12345678901234567890123456789012123456789012345678901234567890121234567 16

Copyright © 2005 Thomson Peterson’s, a part of The Thomson Corporaton SAT is a registered trademark of the College Entrance Examination Board, which was not involved in the production of and does not endorse this product.

12345678901234567890123456789012123456789012345678901234567890121234567 1234567890123456789012345678901212345678901234567890123456789012123456 7 7 12234567890123456789012345678901212345678901234567890123456789012123456 13. Eleanor Roosevelt, the wife of President 14. Sputnik was the first artificial satellite 7 1 34567890123456789012345678901212345678901234567890123456789012123456 7 12234567890123456789012345678901212345678901234567890123456789012123456 successfully propelled into orbit and began 7 Franklin D. Roosevelt, made an active 1234567890123456789012345678901212345678901234567890123456789012123456 7 1 34567890123456789012345678901212345678901234567890123456789012123456 the space race between the United States contribution to many political organiza7 12234567890123456789012345678901212345678901234567890123456789012123456 34567890123456789012345678901212345678901234567890123456789012123456 7 1 tions, this included the Human Rights and the Soviet Union, in 1957 it was 7 12234567890123456789012345678901212345678901234567890123456789012123456 7 1234567890123456789012345678901212345678901234567890123456789012123456 Commission. launched by the Soviets. 7 1234567890123456789012345678901212345678901234567890123456789012123456 7 1234567890123456789012345678901212345678901234567890123456789012123456 34567890123456789012345678901212345678901234567890123456789012123456 7 1 (A) organizations, this included the (A) Sputnik was the first artificial 7 12234567890123456789012345678901212345678901234567890123456789012123456 7 1234567890123456789012345678901212345678901234567890123456789012123456 Human Rights Commission. satellite successfully propelled into 7 1234567890123456789012345678901212345678901234567890123456789012123456 7 1234567890123456789012345678901212345678901234567890123456789012123456 (B) organizations, being included the orbit and began the space race 34567890123456789012345678901212345678901234567890123456789012123456 7 1 7 12234567890123456789012345678901212345678901234567890123456789012123456 Human Rights Commission. between the United States and the 7 1234567890123456789012345678901212345678901234567890123456789012123456 7 1234567890123456789012345678901212345678901234567890123456789012123456 (C) organizations, whose participation Soviet Union, in 1957 it was 7 1234567890123456789012345678901212345678901234567890123456789012123456 34567890123456789012345678901212345678901234567890123456789012123456 7 1 included the Human Rights launched by the Soviets. 7 12234567890123456789012345678901212345678901234567890123456789012123456 7 1234567890123456789012345678901212345678901234567890123456789012123456 Commission. (B) In 1957, the first satellite and space 7 1234567890123456789012345678901212345678901234567890123456789012123456 7 1234567890123456789012345678901212345678901234567890123456789012123456 (D) organizations; this including the race were beginning when Sputnik 34567890123456789012345678901212345678901234567890123456789012123456 1234567890123456789012345678901212345678901234567890123456789012123456 77 1234567890123456789012345678901212345678901234567890123456789012123456 Human Rights Commission. was launched. 7 1234567890123456789012345678901212345678901234567890123456789012123456 (E) organizations, including the Human (C) Launched by the Soviets in 1957, 7 1234567890123456789012345678901212345678901234567890123456789012123456 7 1234567890123456789012345678901212345678901234567890123456789012123456 Rights Commission. Sputnik was the first artificial 1234567890123456789012345678901212345678901234567890123456789012123456 77 1 satellite successfully propelled into 7 12234567890123456789012345678901212345678901234567890123456789012123456 7 1234567890123456789012345678901212345678901234567890123456789012123456 orbit, beginning the space race 7 1234567890123456789012345678901212345678901234567890123456789012123456 34567890123456789012345678901212345678901234567890123456789012123456 7 1234567890123456789012345678901212345678901234567890123456789012123456 between the United States and the 7 1234567890123456789012345678901212345678901234567890123456789012123456 7 1234567890123456789012345678901212345678901234567890123456789012123456 Soviet Union. 1234567890123456789012345678901212345678901234567890123456789012123456 77 1234567890123456789012345678901212345678901234567890123456789012123456 (D) The launching of Sputnik was in 7 1234567890123456789012345678901212345678901234567890123456789012123456 7 1234567890123456789012345678901212345678901234567890123456789012123456 1957, the first artificial satellite was 1234567890123456789012345678901212345678901234567890123456789012123456 77 1234567890123456789012345678901212345678901234567890123456789012123456 successfully propelled and the space 7 1 7 12234567890123456789012345678901212345678901234567890123456789012123456 race between the United States and 7 1234567890123456789012345678901212345678901234567890123456789012123456 7 1 34567890123456789012345678901212345678901234567890123456789012123456 the Soviet Union was begun. 1234567890123456789012345678901212345678901234567890123456789012123456 7 7 12234567890123456789012345678901212345678901234567890123456789012123456 (E) The first artificial satellite success7 1234567890123456789012345678901212345678901234567890123456789012123456 7 1234567890123456789012345678901212345678901234567890123456789012123456 fully propelled into orbit was when 1 34567890123456789012345678901212345678901234567890123456789012123456 7 7 12234567890123456789012345678901212345678901234567890123456789012123456 Sputnik was launched in 1957, and 7 1234567890123456789012345678901212345678901234567890123456789012123456 34567890123456789012345678901212345678901234567890123456789012123456 7 1234567890123456789012345678901212345678901234567890123456789012123456 the space race between the United 7 1234567890123456789012345678901212345678901234567890123456789012123456 7 1 States and the Soviet Union was 1234567890123456789012345678901212345678901234567890123456789012123456 7 7 1234567890123456789012345678901212345678901234567890123456789012123456 begun as well. 7 12234567890123456789012345678901212345678901234567890123456789012123456 7 1234567890123456789012345678901212345678901234567890123456789012123456 7 1234567890123456789012345678901212345678901234567890123456789012123456 7 1234567890123456789012345678901212345678901234567890123456789012123456 1 34567890123456789012345678901212345678901234567890123456789012123456 7 2 34567890123456789012345678901212345678901234567890123456789012123456 1234567890123456789012345678901212345678901234567890123456789012123456 77 1234567890123456789012345678901212345678901234567890123456789012123456 7 1234567890123456789012345678901212345678901234567890123456789012123456 7 1234567890123456789012345678901212345678901234567890123456789012123456 7 1 7 12234567890123456789012345678901212345678901234567890123456789012123456 7 1234567890123456789012345678901212345678901234567890123456789012123456 7 1234567890123456789012345678901212345678901234567890123456789012123456 7 1234567890123456789012345678901212345678901234567890123456789012123456 1 34567890123456789012345678901212345678901234567890123456789012123456 7 2 34567890123456789012345678901212345678901234567890123456789012123456 1234567890123456789012345678901212345678901234567890123456789012123456 77 1 7 12234567890123456789012345678901212345678901234567890123456789012123456 1 34567890123456789012345678901212345678901234567890123456789012123456 7 7 12234567890123456789012345678901212345678901234567890123456789012123456 1 34567890123456789012345678901212345678901234567890123456789012123456 7 1234567890123456789012345678901212345678901234567890123456789012123456 7 1234567890123456789012345678901212345678901234567890123456789012123456 7 1234567890123456789012345678901212345678901234567890123456789012123456 7 12345678901234567890123456789012123456789012345678901234567890121234567 Copyright © 2005 Thomson Peterson’s, a part of The Thomson Corporaton SAT is a registered trademark of the College Entrance Examination Board, which was not involved in the production of and does not endorse this product.

17

12345678901234567890123456789012123456789012345678901234567890121234567 1234567890123456789012345678901212345678901234567890123456789012123456 7 34567890123456789012345678901212345678901234567890123456789012123456 7 12 15. The newly hired CEO stated clearly in her 17. Domesticated over 5,000 years ago, the 7 1234567890123456789012345678901212345678901234567890123456789012123456 34567890123456789012345678901212345678901234567890123456789012123456 7 12 opening address to the company that her camel is a useful pack animal because of 34567890123456789012345678901212345678901234567890123456789012123456 7 12 7 1234567890123456789012345678901212345678901234567890123456789012123456 its tolerance for hot sand, extreme plans for reinvigorating the company were 34567890123456789012345678901212345678901234567890123456789012123456 7 12 2 34567890123456789012345678901212345678901234567890123456789012123456 7 1 temperatures, and it needs little drinkto cut back on discretionary spending, 34567890123456789012345678901212345678901234567890123456789012123456 7 12 34567890123456789012345678901212345678901234567890123456789012123456 7 12 ing water. refinance the company’s largest loans, and 34567890123456789012345678901212345678901234567890123456789012123456 7 12 34567890123456789012345678901212345678901234567890123456789012123456 7 12 her plans of keeping the company’s 1234567890123456789012345678901212345678901234567890123456789012123456 (A) of its tolerance for hot sand, extreme 7 34567890123456789012345678901212345678901234567890123456789012123456 7 12 holdings in the stock market. 34567890123456789012345678901212345678901234567890123456789012123456 7 12 temperatures, and it needs little 34567890123456789012345678901212345678901234567890123456789012123456 7 12 34567890123456789012345678901212345678901234567890123456789012123456 7 12 (A) her plans of keeping the company’s drinking water. 7 1234567890123456789012345678901212345678901234567890123456789012123456 34567890123456789012345678901212345678901234567890123456789012123456 7 12 holdings in the stock market. (B) of its tolerance for hot sand and 34567890123456789012345678901212345678901234567890123456789012123456 7 12 34567890123456789012345678901212345678901234567890123456789012123456 7 12 (B) keep the company’s holdings in the extreme temperatures and its need 34567890123456789012345678901212345678901234567890123456789012123456 7 12 2 34567890123456789012345678901212345678901234567890123456789012123456 7 1 stock market. for water is very small. 34567890123456789012345678901212345678901234567890123456789012123456 7 12 34567890123456789012345678901212345678901234567890123456789012123456 7 12 (C) to get the company to keep its (C) it can tolerate hot sand, extreme 34567890123456789012345678901212345678901234567890123456789012123456 7 12 34567890123456789012345678901212345678901234567890123456789012123456 7 12 holdings in the stock market. temperatures, and a lack of drink34567890123456789012345678901212345678901234567890123456789012123456 7 12 34567890123456789012345678901212345678901234567890123456789012123456 7 12 (D) her plans to keep the company’s ing water. 34567890123456789012345678901212345678901234567890123456789012123456 7 12 holdings in the stock market. (D) it can tolerate hot stand, withstand34567890123456789012345678901212345678901234567890123456789012123456 7 12 34567890123456789012345678901212345678901234567890123456789012123456 7 12 (E) keeping the company’s holdings in ing extreme temperatures, and needs 2 34567890123456789012345678901212345678901234567890123456789012123456 1234567890123456789012345678901212345678901234567890123456789012123456 77 1 the stock market. little water. 34567890123456789012345678901212345678901234567890123456789012123456 7 12 34567890123456789012345678901212345678901234567890123456789012123456 7 12 (E) it can tolerate hot sand and extreme 34567890123456789012345678901212345678901234567890123456789012123456 7 12 2 34567890123456789012345678901212345678901234567890123456789012123456 7 1234567890123456789012345678901212345678901234567890123456789012123456 16. Typically, a restaurant’s kitchen is divided temperatures, and needs little water. 7 1234567890123456789012345678901212345678901234567890123456789012123456 7 1234567890123456789012345678901212345678901234567890123456789012123456 into a number of sections, each with a 1234567890123456789012345678901212345678901234567890123456789012123456 77 1234567890123456789012345678901212345678901234567890123456789012123456 particular aspect of food preparation 18. Legally, an agreement among two people 7 1234567890123456789012345678901212345678901234567890123456789012123456 7 1234567890123456789012345678901212345678901234567890123456789012123456 performed there. to commit a crime or concealing it 1234567890123456789012345678901212345678901234567890123456789012123456 77 1234567890123456789012345678901212345678901234567890123456789012123456 constitutes a criminal conspiracy. 7 1 (A) each with a particular aspect of food 2 34567890123456789012345678901212345678901234567890123456789012123456 1234567890123456789012345678901212345678901234567890123456789012123456 77 (A) among two people to commit a crime preparation performed there. 1234567890123456789012345678901212345678901234567890123456789012123456 7 1 or concealing it (B) each corresponding to a particular 7 1234567890123456789012345678901212345678901234567890123456789012123456 34567890123456789012345678901212345678901234567890123456789012123456 7 12 (B) among two people to commit a crime aspect of food preparation. 2 34567890123456789012345678901212345678901234567890123456789012123456 1234567890123456789012345678901212345678901234567890123456789012123456 77 1234567890123456789012345678901212345678901234567890123456789012123456 or agreeing to conceal it (C) where they each have their particu7 1 34567890123456789012345678901212345678901234567890123456789012123456 7 12 (C) among two people committing a lar aspect of food preparation to 34567890123456789012345678901212345678901234567890123456789012123456 7 12 2 34567890123456789012345678901212345678901234567890123456789012123456 7 1234567890123456789012345678901212345678901234567890123456789012123456 crime or concealing it perform. 7 1234567890123456789012345678901212345678901234567890123456789012123456 7 1 (D) between two people to commit a (D) which has a particular aspect of food 1234567890123456789012345678901212345678901234567890123456789012123456 7 7 1234567890123456789012345678901212345678901234567890123456789012123456 crime or conceal it preparation performed there. 34567890123456789012345678901212345678901234567890123456789012123456 7 12 34567890123456789012345678901212345678901234567890123456789012123456 7 12 (E) between two people in the commit(E) they each correspond to a particular 34567890123456789012345678901212345678901234567890123456789012123456 7 12 34567890123456789012345678901212345678901234567890123456789012123456 7 12 ting of a crime or the concealment aspect of food preparation. 7 1234567890123456789012345678901212345678901234567890123456789012123456 2 34567890123456789012345678901212345678901234567890123456789012123456 7 1234567890123456789012345678901212345678901234567890123456789012123456 of it 7 1234567890123456789012345678901212345678901234567890123456789012123456 1234567890123456789012345678901212345678901234567890123456789012123456 77 1234567890123456789012345678901212345678901234567890123456789012123456 7 1 34567890123456789012345678901212345678901234567890123456789012123456 7 12 34567890123456789012345678901212345678901234567890123456789012123456 7 12 34567890123456789012345678901212345678901234567890123456789012123456 7 12 34567890123456789012345678901212345678901234567890123456789012123456 7 12 1234567890123456789012345678901212345678901234567890123456789012123456 7 2 7 1234567890123456789012345678901212345678901234567890123456789012123456 1 34567890123456789012345678901212345678901234567890123456789012123456 7 34567890123456789012345678901212345678901234567890123456789012123456 7 12 1234567890123456789012345678901212345678901234567890123456789012123456 7 34567890123456789012345678901212345678901234567890123456789012123456 7 12 1234567890123456789012345678901212345678901234567890123456789012123456 7 1234567890123456789012345678901212345678901234567890123456789012123456 7 1234567890123456789012345678901212345678901234567890123456789012123456 7 1234567890123456789012345678901212345678901234567890123456789012123456 7 12345678901234567890123456789012123456789012345678901234567890121234567 18

Copyright © 2005 Thomson Peterson’s, a part of The Thomson Corporaton SAT is a registered trademark of the College Entrance Examination Board, which was not involved in the production of and does not endorse this product.

12345678901234567890123456789012123456789012345678901234567890121234567 1234567890123456789012345678901212345678901234567890123456789012123456 7 7 12234567890123456789012345678901212345678901234567890123456789012123456 19. Not only was Sir Isaac Newton famous 20. If the high levels of stock market invest7 1 34567890123456789012345678901212345678901234567890123456789012123456 7 12234567890123456789012345678901212345678901234567890123456789012123456 ment are to continue it will depend upon for his pioneering work in Physics, he was 7 1234567890123456789012345678901212345678901234567890123456789012123456 7 1 34567890123456789012345678901212345678901234567890123456789012123456 also a talented and well-respected ecoboth how long the stock market remains 7 12234567890123456789012345678901212345678901234567890123456789012123456 34567890123456789012345678901212345678901234567890123456789012123456 7 1 nomic advisor to the king. stable and its long-term durability. 7 12234567890123456789012345678901212345678901234567890123456789012123456 7 1234567890123456789012345678901212345678901234567890123456789012123456 7 1234567890123456789012345678901212345678901234567890123456789012123456 (A) he was also a talented and well(A) If the high levels of stock market 7 1234567890123456789012345678901212345678901234567890123456789012123456 34567890123456789012345678901212345678901234567890123456789012123456 7 1 respected economic advisor to the king. investment are to continue it 7 12234567890123456789012345678901212345678901234567890123456789012123456 7 1234567890123456789012345678901212345678901234567890123456789012123456 (B) he had also been a talented and will depend 7 1234567890123456789012345678901212345678901234567890123456789012123456 7 1234567890123456789012345678901212345678901234567890123456789012123456 well-respected economic advisor to (B) If the high levels of stock market 34567890123456789012345678901212345678901234567890123456789012123456 7 1 7 12234567890123456789012345678901212345678901234567890123456789012123456 the king. investment are to continue, depend7 1234567890123456789012345678901212345678901234567890123456789012123456 7 1234567890123456789012345678901212345678901234567890123456789012123456 (C) but he was also a talented and ing upon 7 1234567890123456789012345678901212345678901234567890123456789012123456 34567890123456789012345678901212345678901234567890123456789012123456 7 1 well-respected economic advisor to (C) If the high levels of stock market 7 12234567890123456789012345678901212345678901234567890123456789012123456 7 1234567890123456789012345678901212345678901234567890123456789012123456 the king. investment continue, it will depend 7 1234567890123456789012345678901212345678901234567890123456789012123456 7 1234567890123456789012345678901212345678901234567890123456789012123456 (D) as he also was a talented and (D) Whether the high levels of stock 34567890123456789012345678901212345678901234567890123456789012123456 1234567890123456789012345678901212345678901234567890123456789012123456 77 1234567890123456789012345678901212345678901234567890123456789012123456 well-respected economic advisor to market investment continue will 7 1234567890123456789012345678901212345678901234567890123456789012123456 the king. depend 7 1234567890123456789012345678901212345678901234567890123456789012123456 7 1234567890123456789012345678901212345678901234567890123456789012123456 (E) but he had also been a talented and (E) Whether the high levels of stock 1234567890123456789012345678901212345678901234567890123456789012123456 77 1 well-respected economic advisor to market investment continue it 7 12234567890123456789012345678901212345678901234567890123456789012123456 7 1234567890123456789012345678901212345678901234567890123456789012123456 the king. will depend 7 1234567890123456789012345678901212345678901234567890123456789012123456 34567890123456789012345678901212345678901234567890123456789012123456 1234567890123456789012345678901212345678901234567890123456789012123456 77 1234567890123456789012345678901212345678901234567890123456789012123456 7 1234567890123456789012345678901212345678901234567890123456789012123456 7 1234567890123456789012345678901212345678901234567890123456789012123456 7 1234567890123456789012345678901212345678901234567890123456789012123456 7 1234567890123456789012345678901212345678901234567890123456789012123456 7 1234567890123456789012345678901212345678901234567890123456789012123456 7 1234567890123456789012345678901212345678901234567890123456789012123456 7 1234567890123456789012345678901212345678901234567890123456789012123456 7 1 7 12234567890123456789012345678901212345678901234567890123456789012123456 7 1234567890123456789012345678901212345678901234567890123456789012123456 1 34567890123456789012345678901212345678901234567890123456789012123456 7 1234567890123456789012345678901212345678901234567890123456789012123456 7 7 12234567890123456789012345678901212345678901234567890123456789012123456 7 1234567890123456789012345678901212345678901234567890123456789012123456 7 1234567890123456789012345678901212345678901234567890123456789012123456 1 34567890123456789012345678901212345678901234567890123456789012123456 7 7 12234567890123456789012345678901212345678901234567890123456789012123456 7 1234567890123456789012345678901212345678901234567890123456789012123456 34567890123456789012345678901212345678901234567890123456789012123456 1234567890123456789012345678901212345678901234567890123456789012123456 77 1234567890123456789012345678901212345678901234567890123456789012123456 7 1 1234567890123456789012345678901212345678901234567890123456789012123456 7 1234567890123456789012345678901212345678901234567890123456789012123456 7 7 12234567890123456789012345678901212345678901234567890123456789012123456 7 1234567890123456789012345678901212345678901234567890123456789012123456 7 1234567890123456789012345678901212345678901234567890123456789012123456 7 1234567890123456789012345678901212345678901234567890123456789012123456 1 34567890123456789012345678901212345678901234567890123456789012123456 7 2 34567890123456789012345678901212345678901234567890123456789012123456 1234567890123456789012345678901212345678901234567890123456789012123456 77 1234567890123456789012345678901212345678901234567890123456789012123456 7 1234567890123456789012345678901212345678901234567890123456789012123456 7 1234567890123456789012345678901212345678901234567890123456789012123456 7 1 7 12234567890123456789012345678901212345678901234567890123456789012123456 7 1234567890123456789012345678901212345678901234567890123456789012123456 7 1234567890123456789012345678901212345678901234567890123456789012123456 7 1234567890123456789012345678901212345678901234567890123456789012123456 1 34567890123456789012345678901212345678901234567890123456789012123456 7 2 34567890123456789012345678901212345678901234567890123456789012123456 1234567890123456789012345678901212345678901234567890123456789012123456 77 1 7 12234567890123456789012345678901212345678901234567890123456789012123456 1 34567890123456789012345678901212345678901234567890123456789012123456 7 7 12234567890123456789012345678901212345678901234567890123456789012123456 1 34567890123456789012345678901212345678901234567890123456789012123456 7 1234567890123456789012345678901212345678901234567890123456789012123456 7 1234567890123456789012345678901212345678901234567890123456789012123456 7 1234567890123456789012345678901212345678901234567890123456789012123456 7 12345678901234567890123456789012123456789012345678901234567890121234567 Copyright © 2005 Thomson Peterson’s, a part of The Thomson Corporaton SAT is a registered trademark of the College Entrance Examination Board, which was not involved in the production of and does not endorse this product.

19

12345678901234567890123456789012123456789012345678901234567890121234567 1234567890123456789012345678901212345678901234567890123456789012123456 7 34567890123456789012345678901212345678901234567890123456789012123456 7 12 7 1234567890123456789012345678901212345678901234567890123456789012123456 Improving Paragraphs 34567890123456789012345678901212345678901234567890123456789012123456 7 12 34567890123456789012345678901212345678901234567890123456789012123456 7 12 1234567890123456789012345678901212345678901234567890123456789012123456 7 34567890123456789012345678901212345678901234567890123456789012123456 7 12 Directions: 7 1234567890123456789012345678901212345678901234567890123456789012123456 34567890123456789012345678901212345678901234567890123456789012123456 7 12 1. The following questions test your knowledge of paragraph and sentence construction. 34567890123456789012345678901212345678901234567890123456789012123456 7 12 34567890123456789012345678901212345678901234567890123456789012123456 7 12 34567890123456789012345678901212345678901234567890123456789012123456 7 12 2. The following passage is a rough draft of an essay. This rough draft contains various 7 1234567890123456789012345678901212345678901234567890123456789012123456 34567890123456789012345678901212345678901234567890123456789012123456 7 12 errors. 34567890123456789012345678901212345678901234567890123456789012123456 7 12 34567890123456789012345678901212345678901234567890123456789012123456 7 12 34567890123456789012345678901212345678901234567890123456789012123456 7 12 3. Read the rough draft and then answer the questions that follow. Some questions will focus 2 7 1 34567890123456789012345678901212345678901234567890123456789012123456 34567890123456789012345678901212345678901234567890123456789012123456 7 12 on specific sentences and ask if there are any problems with that sentence’s word choice, 34567890123456789012345678901212345678901234567890123456789012123456 7 12 34567890123456789012345678901212345678901234567890123456789012123456 7 12 word usage, or overall structure. Other questions will ask about the paragraph itself. 34567890123456789012345678901212345678901234567890123456789012123456 7 12 2 34567890123456789012345678901212345678901234567890123456789012123456 7 1 These questions will focus on paragraph organization and development. 34567890123456789012345678901212345678901234567890123456789012123456 7 12 34567890123456789012345678901212345678901234567890123456789012123456 7 12 4. Select the answer that best reflects the rules of English grammar and proper essay and 34567890123456789012345678901212345678901234567890123456789012123456 7 12 34567890123456789012345678901212345678901234567890123456789012123456 7 12 paragraph writing. 2 34567890123456789012345678901212345678901234567890123456789012123456 1234567890123456789012345678901212345678901234567890123456789012123456 77 1234567890123456789012345678901212345678901234567890123456789012123456 7 1234567890123456789012345678901212345678901234567890123456789012123456 7 1234567890123456789012345678901212345678901234567890123456789012123456 7 1234567890123456789012345678901212345678901234567890123456789012123456 7 1234567890123456789012345678901212345678901234567890123456789012123456 century after when the philosophers 7 1 Questions 21–25 are based on the following 34567890123456789012345678901212345678901234567890123456789012123456 7 12 passage. reckon the beginning of modern times. 34567890123456789012345678901212345678901234567890123456789012123456 7 12 34567890123456789012345678901212345678901234567890123456789012123456 7 12 (25) (11) The philosophers and the historians 2 34567890123456789012345678901212345678901234567890123456789012123456 7 1234567890123456789012345678901212345678901234567890123456789012123456 The following passage is part of an essay about 7 1234567890123456789012345678901212345678901234567890123456789012123456 have a bit of a discrepancy here. 7 1234567890123456789012345678901212345678901234567890123456789012123456 the different meanings of the word “modern.” 7 1234567890123456789012345678901212345678901234567890123456789012123456 (12) Of course, you might think both 1234567890123456789012345678901212345678901234567890123456789012123456 77 1234567890123456789012345678901212345678901234567890123456789012123456 Line (1) The word modern is a curious word the philosophers and the historians are a 7 1234567890123456789012345678901212345678901234567890123456789012123456 7 1234567890123456789012345678901212345678901234567890123456789012123456 (2) It is curious for all its different bit off on this whole modernity thing. 7 1234567890123456789012345678901212345678901234567890123456789012123456 7 1 meanings. (3) If you take a freshman (30) (13) Who thinks of the mud and cobble2 34567890123456789012345678901212345678901234567890123456789012123456 1234567890123456789012345678901212345678901234567890123456789012123456 77 1234567890123456789012345678901212345678901234567890123456789012123456 class in philosophy, for instance, your stone streets of Paris in the early decades 7 1 7 1234567890123456789012345678901212345678901234567890123456789012123456 (5) professor might tell you about Rene of the seventeenth century as modern? 34567890123456789012345678901212345678901234567890123456789012123456 7 12 2 34567890123456789012345678901212345678901234567890123456789012123456 7 1234567890123456789012345678901212345678901234567890123456789012123456 Descartes. (4) Descartes was a French (14) Then again, London a century later 7 1234567890123456789012345678901212345678901234567890123456789012123456 philosopher from the seventeenth 7 1 with the power of steam harnessed does 34567890123456789012345678901212345678901234567890123456789012123456 7 12 century. (5) Many consider him as the 34567890123456789012345678901212345678901234567890123456789012123456 7 12 (35) not strike most as the picture of modern 2 34567890123456789012345678901212345678901234567890123456789012123456 7 1234567890123456789012345678901212345678901234567890123456789012123456 first figure in modern philosophy. (6) So 1234567890123456789012345678901212345678901234567890123456789012123456 times. (15) Most of us, when we think of 7 7 1 (10) if you are talking to a philosophy 7 1234567890123456789012345678901212345678901234567890123456789012123456 what modern means, we are thinking 7 1234567890123456789012345678901212345678901234567890123456789012123456 professor the word modern denotes 2 34567890123456789012345678901212345678901234567890123456789012123456 7 1234567890123456789012345678901212345678901234567890123456789012123456 about computers and cell phones and 7 1234567890123456789012345678901212345678901234567890123456789012123456 anytime between about 1615 and now. wireless networks. (16) We are not 7 1234567890123456789012345678901212345678901234567890123456789012123456 7 1234567890123456789012345678901212345678901234567890123456789012123456 (7) (Of course, some philosophers think (40) thinking about some philosophical 7 1 2 34567890123456789012345678901212345678901234567890123456789012123456 7 1234567890123456789012345678901212345678901234567890123456789012123456 that modern times ended a few decades discourse that a Frenchman wrote four 7 1234567890123456789012345678901212345678901234567890123456789012123456 7 1234567890123456789012345678901212345678901234567890123456789012123456 (15) back and that we are already in to centuries back. 1234567890123456789012345678901212345678901234567890123456789012123456 77 1 postmodern times.) (8) Other academics 34567890123456789012345678901212345678901234567890123456789012123456 7 12 34567890123456789012345678901212345678901234567890123456789012123456 7 12 have a different timeline for the birth of 34567890123456789012345678901212345678901234567890123456789012123456 7 12 34567890123456789012345678901212345678901234567890123456789012123456 7 12 modern times, or modernity. (9) And 7 1234567890123456789012345678901212345678901234567890123456789012123456 2 34567890123456789012345678901212345678901234567890123456789012123456 7 1234567890123456789012345678901212345678901234567890123456789012123456 some historians point to the Industrial 7 1 34567890123456789012345678901212345678901234567890123456789012123456 7 12 (20) Revolution in England as the beginning 1234567890123456789012345678901212345678901234567890123456789012123456 7 34567890123456789012345678901212345678901234567890123456789012123456 7 12 of modernity. (10) This dating technique 7 1234567890123456789012345678901212345678901234567890123456789012123456 7 1234567890123456789012345678901212345678901234567890123456789012123456 puts the beginning of modernity at least a 1234567890123456789012345678901212345678901234567890123456789012123456 77 1234567890123456789012345678901212345678901234567890123456789012123456 12345678901234567890123456789012123456789012345678901234567890121234567 20

Copyright © 2005 Thomson Peterson’s, a part of The Thomson Corporaton SAT is a registered trademark of the College Entrance Examination Board, which was not involved in the production of and does not endorse this product.

12345678901234567890123456789012123456789012345678901234567890121234567 1234567890123456789012345678901212345678901234567890123456789012123456 7 7 12234567890123456789012345678901212345678901234567890123456789012123456 21. Which of the following is the best 23. Which of the following would be the best 7 1 34567890123456789012345678901212345678901234567890123456789012123456 7 12234567890123456789012345678901212345678901234567890123456789012123456 combination of sentences 1 and 2 replacement for “And” at the beginning of 7 1234567890123456789012345678901212345678901234567890123456789012123456 7 1 34567890123456789012345678901212345678901234567890123456789012123456 (reproduced below)? sentence 9 (reproduced below)? 7 12234567890123456789012345678901212345678901234567890123456789012123456 34567890123456789012345678901212345678901234567890123456789012123456 7 1 7 12234567890123456789012345678901212345678901234567890123456789012123456 The word modern is a curious word. It is And some historians point to the Indus7 1234567890123456789012345678901212345678901234567890123456789012123456 7 1234567890123456789012345678901212345678901234567890123456789012123456 curious for all its different meanings. trial Revolution in England as the 7 1234567890123456789012345678901212345678901234567890123456789012123456 34567890123456789012345678901212345678901234567890123456789012123456 7 1 beginning of modernity. 7 12234567890123456789012345678901212345678901234567890123456789012123456 (A) The word modern is a curious word. It 7 1234567890123456789012345678901212345678901234567890123456789012123456 7 1234567890123456789012345678901212345678901234567890123456789012123456 is curious for all its different meanings. (A) However, 7 1234567890123456789012345678901212345678901234567890123456789012123456 34567890123456789012345678901212345678901234567890123456789012123456 7 1 (B) Modern is a curious word because it (B) Moreover, 7 12234567890123456789012345678901212345678901234567890123456789012123456 7 1234567890123456789012345678901212345678901234567890123456789012123456 has all its different meanings. (C) Even so, 7 1234567890123456789012345678901212345678901234567890123456789012123456 7 1234567890123456789012345678901212345678901234567890123456789012123456 (C) It is curious that the word modern (D) Considering this, 34567890123456789012345678901212345678901234567890123456789012123456 7 1 7 12234567890123456789012345678901212345678901234567890123456789012123456 has various different meanings. (E) For example, 7 1234567890123456789012345678901212345678901234567890123456789012123456 7 (D) Curiously, the word modern has 1234567890123456789012345678901212345678901234567890123456789012123456 7 1234567890123456789012345678901212345678901234567890123456789012123456 24. Which of the following is the best revision many different meanings. 34567890123456789012345678901212345678901234567890123456789012123456 1234567890123456789012345678901212345678901234567890123456789012123456 77 1234567890123456789012345678901212345678901234567890123456789012123456 of the underlined portions of sentence 10 7 (E) Modernity is a curious word for all 1234567890123456789012345678901212345678901234567890123456789012123456 7 1234567890123456789012345678901212345678901234567890123456789012123456 (reproduced below)? its various different meanings. 7 1234567890123456789012345678901212345678901234567890123456789012123456 1234567890123456789012345678901212345678901234567890123456789012123456 77 1 This dating technique puts the beginning 7 12234567890123456789012345678901212345678901234567890123456789012123456 22. Which of the following is the best way to 7 1234567890123456789012345678901212345678901234567890123456789012123456 of modernity at least a century after when 7 1234567890123456789012345678901212345678901234567890123456789012123456 revise sentences 4 and 5 (reproduced 34567890123456789012345678901212345678901234567890123456789012123456 7 1234567890123456789012345678901212345678901234567890123456789012123456 the philosophers reckon the beginning of 7 1234567890123456789012345678901212345678901234567890123456789012123456 below) so that they are condensed into 7 1234567890123456789012345678901212345678901234567890123456789012123456 modern times. 7 1234567890123456789012345678901212345678901234567890123456789012123456 one sentence? 7 1234567890123456789012345678901212345678901234567890123456789012123456 (A) As it is now. 1234567890123456789012345678901212345678901234567890123456789012123456 77 1234567890123456789012345678901212345678901234567890123456789012123456 Descartes was a French philosopher from (B) at least a century after when the 7 1234567890123456789012345678901212345678901234567890123456789012123456 the seventeenth century. Many consider him 7 1234567890123456789012345678901212345678901234567890123456789012123456 philosophers reckon it. 7 1 as the first figure in modern philosophy. 2 34567890123456789012345678901212345678901234567890123456789012123456 7 1234567890123456789012345678901212345678901234567890123456789012123456 (C) at least a century after when the 7 1234567890123456789012345678901212345678901234567890123456789012123456 7 1 philosopher’s date modernity’s (A) Descartes was a French philosopher 1234567890123456789012345678901212345678901234567890123456789012123456 7 7 12234567890123456789012345678901212345678901234567890123456789012123456 from the seventeenth century, inception. 7 1234567890123456789012345678901212345678901234567890123456789012123456 7 1234567890123456789012345678901212345678901234567890123456789012123456 considering him the first figure in (D) at least one century after when the 1 34567890123456789012345678901212345678901234567890123456789012123456 7 7 12234567890123456789012345678901212345678901234567890123456789012123456 modern philosophy. philosophers had reckoned the 7 1234567890123456789012345678901212345678901234567890123456789012123456 34567890123456789012345678901212345678901234567890123456789012123456 7 1234567890123456789012345678901212345678901234567890123456789012123456 (B) Descartes was a French philosopher inception of modernity. 7 1234567890123456789012345678901212345678901234567890123456789012123456 7 1 from the seventeenth century, whom (E) at least a century after its having 1234567890123456789012345678901212345678901234567890123456789012123456 7 7 1234567890123456789012345678901212345678901234567890123456789012123456 many consider the first figure in been reckoned by the philosophers. 7 12234567890123456789012345678901212345678901234567890123456789012123456 7 modern philosophy. 1234567890123456789012345678901212345678901234567890123456789012123456 7 1234567890123456789012345678901212345678901234567890123456789012123456 (C) Descartes was a French philosopher 7 1234567890123456789012345678901212345678901234567890123456789012123456 7 1 34567890123456789012345678901212345678901234567890123456789012123456 from the seventeenth century, and 2 34567890123456789012345678901212345678901234567890123456789012123456 1234567890123456789012345678901212345678901234567890123456789012123456 77 1234567890123456789012345678901212345678901234567890123456789012123456 many consider him the first figure in 7 1234567890123456789012345678901212345678901234567890123456789012123456 7 1234567890123456789012345678901212345678901234567890123456789012123456 modern philosophy. 7 1 2 34567890123456789012345678901212345678901234567890123456789012123456 7 1234567890123456789012345678901212345678901234567890123456789012123456 (D) Descartes was a French philosopher 7 1234567890123456789012345678901212345678901234567890123456789012123456 7 1234567890123456789012345678901212345678901234567890123456789012123456 from the seventeenth century, so he was 1234567890123456789012345678901212345678901234567890123456789012123456 77 1 the first figure in modern philosophy. 7 12234567890123456789012345678901212345678901234567890123456789012123456 7 1 34567890123456789012345678901212345678901234567890123456789012123456 (E) Descartes was a French philosopher 7 12234567890123456789012345678901212345678901234567890123456789012123456 7 1 34567890123456789012345678901212345678901234567890123456789012123456 from the seventeenth century and 7 12234567890123456789012345678901212345678901234567890123456789012123456 34567890123456789012345678901212345678901234567890123456789012123456 7 1 figuring him the first figure in 7 1234567890123456789012345678901212345678901234567890123456789012123456 modern philosophy. 7 1234567890123456789012345678901212345678901234567890123456789012123456 1234567890123456789012345678901212345678901234567890123456789012123456 7 12345678901234567890123456789012123456789012345678901234567890121234567 Copyright © 2005 Thomson Peterson’s, a part of The Thomson Corporaton SAT is a registered trademark of the College Entrance Examination Board, which was not involved in the production of and does not endorse this product.

21

12345678901234567890123456789012123456789012345678901234567890121234567 1234567890123456789012345678901212345678901234567890123456789012123456 7 34567890123456789012345678901212345678901234567890123456789012123456 7 12 25. Which of the following is the best revision larger group of people. (6) Those people 7 1234567890123456789012345678901212345678901234567890123456789012123456 34567890123456789012345678901212345678901234567890123456789012123456 7 12 (15) of sentence 15 (reproduced below)? could be potential consumers. 34567890123456789012345678901212345678901234567890123456789012123456 7 12 7 1234567890123456789012345678901212345678901234567890123456789012123456 (7) Many of our most famous 34567890123456789012345678901212345678901234567890123456789012123456 7 12 Most of us, when we think of what 2 34567890123456789012345678901212345678901234567890123456789012123456 7 1 department store chains first expanded 34567890123456789012345678901212345678901234567890123456789012123456 7 12 modern means, we are thinking about 34567890123456789012345678901212345678901234567890123456789012123456 7 12 during this time. (8) These include Sears, 34567890123456789012345678901212345678901234567890123456789012123456 7 12 computers and cell phones and wireless 34567890123456789012345678901212345678901234567890123456789012123456 7 12 Roebuck; Woolworth’s; the Great 2 34567890123456789012345678901212345678901234567890123456789012123456 7 1 networks. 34567890123456789012345678901212345678901234567890123456789012123456 7 12 (20) Atlantic and Pacific Tea Company (the 34567890123456789012345678901212345678901234567890123456789012123456 7 12 34567890123456789012345678901212345678901234567890123456789012123456 7 12 (A) As it is now. A&P); and Walgreen Drug. (9) Among 34567890123456789012345678901212345678901234567890123456789012123456 7 12 2 34567890123456789012345678901212345678901234567890123456789012123456 7 1 (B) Most of us, when we think of what the most successful department stores 34567890123456789012345678901212345678901234567890123456789012123456 7 12 34567890123456789012345678901212345678901234567890123456789012123456 7 12 modern means, computers and cell was Filene’s in Boston and Macy’s in 34567890123456789012345678901212345678901234567890123456789012123456 7 12 34567890123456789012345678901212345678901234567890123456789012123456 7 12 phones and wireless networks New York. (10) Initially, department 7 1234567890123456789012345678901212345678901234567890123456789012123456 34567890123456789012345678901212345678901234567890123456789012123456 7 12 coming to mind. (25) stores were more like the malls of today. 34567890123456789012345678901212345678901234567890123456789012123456 7 12 34567890123456789012345678901212345678901234567890123456789012123456 7 (C) When we think of what modern 12 (11) Each department was leased to an 34567890123456789012345678901212345678901234567890123456789012123456 7 12 means, think about computers and 2 34567890123456789012345678901212345678901234567890123456789012123456 7 1234567890123456789012345678901212345678901234567890123456789012123456 individual owner. (12) Nowadays, 7 1234567890123456789012345678901212345678901234567890123456789012123456 cell phones and wireless networks. 7 1234567890123456789012345678901212345678901234567890123456789012123456 virtually all departments are run by the 7 1234567890123456789012345678901212345678901234567890123456789012123456 (D) Most of us, when we think of what 7 1234567890123456789012345678901212345678901234567890123456789012123456 larger company, including restaurants. 7 1234567890123456789012345678901212345678901234567890123456789012123456 modern means, we are thinking 7 1 (30) (13) Also, with their radio cam34567890123456789012345678901212345678901234567890123456789012123456 7 12 about computers, cell phones, paigns, the new department stores of the 7 34567890123456789012345678901212345678901234567890123456789012123456 12 34567890123456789012345678901212345678901234567890123456789012123456 7 12 wireless networks. 1920s put on extravagant advertising 34567890123456789012345678901212345678901234567890123456789012123456 7 12 34567890123456789012345678901212345678901234567890123456789012123456 7 12 (E) Most of us, when we think of what spectacles. (14) Sometimes, they even 34567890123456789012345678901212345678901234567890123456789012123456 7 12 34567890123456789012345678901212345678901234567890123456789012123456 7 12 modern means, think about computhosted entertainment events to attract 34567890123456789012345678901212345678901234567890123456789012123456 7 12 2 34567890123456789012345678901212345678901234567890123456789012123456 7 1234567890123456789012345678901212345678901234567890123456789012123456 ers and cell phones and wireless (35) consumers. (15) The Macy’s Thanksgiv7 1234567890123456789012345678901212345678901234567890123456789012123456 7 1234567890123456789012345678901212345678901234567890123456789012123456 networks. ing Day Parade, an attempt to capture 1234567890123456789012345678901212345678901234567890123456789012123456 77 1 the children’s toy market, is one example 7 34567890123456789012345678901212345678901234567890123456789012123456 12 34567890123456789012345678901212345678901234567890123456789012123456 7 12 of popular merchandising. (16) BloomQuestions 26–30 are based on the following 7 1234567890123456789012345678901212345678901234567890123456789012123456 7 1234567890123456789012345678901212345678901234567890123456789012123456 ingdale’s posted ads on all New York passage. 34567890123456789012345678901212345678901234567890123456789012123456 7 12 2 34567890123456789012345678901212345678901234567890123456789012123456 7 1234567890123456789012345678901212345678901234567890123456789012123456 (40) public transit, pronouncing, “All Cars 7 1234567890123456789012345678901212345678901234567890123456789012123456 The following is a first draft of an essay about 7 1 Transfer to Bloomingdale’s.” 34567890123456789012345678901212345678901234567890123456789012123456 7 12 the growth of the department store industry in 34567890123456789012345678901212345678901234567890123456789012123456 7 12 2 34567890123456789012345678901212345678901234567890123456789012123456 7 1234567890123456789012345678901212345678901234567890123456789012123456 the 1920s. 7 1234567890123456789012345678901212345678901234567890123456789012123456 7 1 7 1234567890123456789012345678901212345678901234567890123456789012123456 Line (1) Automobiles and radios became far 1234567890123456789012345678901212345678901234567890123456789012123456 7 2 34567890123456789012345678901212345678901234567890123456789012123456 7 1234567890123456789012345678901212345678901234567890123456789012123456 more affordable in the 1920s. (2) By 7 1234567890123456789012345678901212345678901234567890123456789012123456 7 1234567890123456789012345678901212345678901234567890123456789012123456 1925 there was one automobile for every 1234567890123456789012345678901212345678901234567890123456789012123456 77 1 six people in the United States, by 1930 34567890123456789012345678901212345678901234567890123456789012123456 7 12 (5) this had increased to one for every 4.6 34567890123456789012345678901212345678901234567890123456789012123456 7 12 34567890123456789012345678901212345678901234567890123456789012123456 7 12 people. (3) Also by 1930, about 4 in 10 34567890123456789012345678901212345678901234567890123456789012123456 7 12 7 1234567890123456789012345678901212345678901234567890123456789012123456 American families owned radios. (4) The 2 34567890123456789012345678901212345678901234567890123456789012123456 1234567890123456789012345678901212345678901234567890123456789012123456 77 1234567890123456789012345678901212345678901234567890123456789012123456 popularity of automobiles and radios led 7 1234567890123456789012345678901212345678901234567890123456789012123456 7 1234567890123456789012345678901212345678901234567890123456789012123456 to the spread of chain stores of all kinds. 7 1 2 34567890123456789012345678901212345678901234567890123456789012123456 7 1234567890123456789012345678901212345678901234567890123456789012123456 7 1 (10) (5) Automobiles allowed consumers to 34567890123456789012345678901212345678901234567890123456789012123456 7 12 travel further in search of the right item 1234567890123456789012345678901212345678901234567890123456789012123456 7 34567890123456789012345678901212345678901234567890123456789012123456 7 12 for the right price, while radios allowed 7 1234567890123456789012345678901212345678901234567890123456789012123456 7 1234567890123456789012345678901212345678901234567890123456789012123456 businesses to advertise their products to a 1234567890123456789012345678901212345678901234567890123456789012123456 77 1234567890123456789012345678901212345678901234567890123456789012123456 12345678901234567890123456789012123456789012345678901234567890121234567 22

Copyright © 2005 Thomson Peterson’s, a part of The Thomson Corporaton SAT is a registered trademark of the College Entrance Examination Board, which was not involved in the production of and does not endorse this product.

12345678901234567890123456789012123456789012345678901234567890121234567 1234567890123456789012345678901212345678901234567890123456789012123456 7 7 12234567890123456789012345678901212345678901234567890123456789012123456 26. Which of the following is the best revision 7 28. If you were to combine sentences 10 and 1 34567890123456789012345678901212345678901234567890123456789012123456 7 12234567890123456789012345678901212345678901234567890123456789012123456 of the underlined portions of sentences 5 11 (reproduced below), which would be 7 1234567890123456789012345678901212345678901234567890123456789012123456 7 1 34567890123456789012345678901212345678901234567890123456789012123456 and 6 (reproduced below)? the most appropriate and precise punctua7 12234567890123456789012345678901212345678901234567890123456789012123456 34567890123456789012345678901212345678901234567890123456789012123456 7 1 tion mark to use? 7 12234567890123456789012345678901212345678901234567890123456789012123456 Automobiles allowed consumers to travel 7 1234567890123456789012345678901212345678901234567890123456789012123456 7 1234567890123456789012345678901212345678901234567890123456789012123456 further in search of the right item for the Initially, department stores were more like 7 1234567890123456789012345678901212345678901234567890123456789012123456 34567890123456789012345678901212345678901234567890123456789012123456 7 1 right price, while radios allowed busithe malls of today. Each department was 7 12234567890123456789012345678901212345678901234567890123456789012123456 7 1234567890123456789012345678901212345678901234567890123456789012123456 nesses to advertise their products to a leased to an individual owner. 7 1234567890123456789012345678901212345678901234567890123456789012123456 7 1234567890123456789012345678901212345678901234567890123456789012123456 larger group of people. Those people 34567890123456789012345678901212345678901234567890123456789012123456 7 1 (A) today ‘each. . . owner.’ 7 12234567890123456789012345678901212345678901234567890123456789012123456 could be potential consumers. 7 1234567890123456789012345678901212345678901234567890123456789012123456 (B) today; each 7 1234567890123456789012345678901212345678901234567890123456789012123456 7 1234567890123456789012345678901212345678901234567890123456789012123456 (A) As it is now. (C) today: each 34567890123456789012345678901212345678901234567890123456789012123456 7 1 7 12234567890123456789012345678901212345678901234567890123456789012123456 (B) a larger group of people, who could (D) today (each. . . owner). 7 1234567890123456789012345678901212345678901234567890123456789012123456 be potential consumers. 7 (E) today, each 1234567890123456789012345678901212345678901234567890123456789012123456 7 1234567890123456789012345678901212345678901234567890123456789012123456 (C) a larger group of people, being 34567890123456789012345678901212345678901234567890123456789012123456 1234567890123456789012345678901212345678901234567890123456789012123456 77 1234567890123456789012345678901212345678901234567890123456789012123456 29. Which of the following is the best revision 7 potential consumers. 1234567890123456789012345678901212345678901234567890123456789012123456 7 1234567890123456789012345678901212345678901234567890123456789012123456 of the underlined portion of sentence 13 (D) a larger group of people, whom they 7 1234567890123456789012345678901212345678901234567890123456789012123456 7 1234567890123456789012345678901212345678901234567890123456789012123456 (reproduced below)? made into potential consumers. 7 1 7 12234567890123456789012345678901212345678901234567890123456789012123456 (E) a larger group of people, having the 7 1234567890123456789012345678901212345678901234567890123456789012123456 Also, with their radio campaigns, the new 7 1234567890123456789012345678901212345678901234567890123456789012123456 potential to become consumers. 34567890123456789012345678901212345678901234567890123456789012123456 7 1234567890123456789012345678901212345678901234567890123456789012123456 department stores of the 1920s put on 7 1234567890123456789012345678901212345678901234567890123456789012123456 7 1234567890123456789012345678901212345678901234567890123456789012123456 extravagant advertising spectacles. 7 1234567890123456789012345678901212345678901234567890123456789012123456 27. “This time” in sentence 7 (reproduced 7 1234567890123456789012345678901212345678901234567890123456789012123456 (A) Moreover 7 1234567890123456789012345678901212345678901234567890123456789012123456 below) is best made more specific. Which 7 1234567890123456789012345678901212345678901234567890123456789012123456 (B) Too 7 1234567890123456789012345678901212345678901234567890123456789012123456 of the following phrases is the best 7 1234567890123456789012345678901212345678901234567890123456789012123456 (C) What’s more to their radio campaigns 7 1 revision? 2 34567890123456789012345678901212345678901234567890123456789012123456 1234567890123456789012345678901212345678901234567890123456789012123456 (D) In addition to their radio campaigns 7 7 1234567890123456789012345678901212345678901234567890123456789012123456 Many of our most famous department 7 1 (E) The radio campaigns being included 7 1234567890123456789012345678901212345678901234567890123456789012123456 store chains first expanded during 7 12234567890123456789012345678901212345678901234567890123456789012123456 34567890123456789012345678901212345678901234567890123456789012123456 7 1234567890123456789012345678901212345678901234567890123456789012123456 this time. 7 1234567890123456789012345678901212345678901234567890123456789012123456 30. Which of the following sentences, if added 7 1 7 12234567890123456789012345678901212345678901234567890123456789012123456 at the end of paragraph 3, is the best (A) these years 7 1234567890123456789012345678901212345678901234567890123456789012123456 34567890123456789012345678901212345678901234567890123456789012123456 7 1234567890123456789012345678901212345678901234567890123456789012123456 concluding sentence for the passage? (B) the twentieth century 7 1234567890123456789012345678901212345678901234567890123456789012123456 7 1 (C) the same when automobile sales and (A) In the 20s, shopping and advertising 7 1234567890123456789012345678901212345678901234567890123456789012123456 7 1234567890123456789012345678901212345678901234567890123456789012123456 radio sales were also on the rise started to look a lot like they do now. 7 12234567890123456789012345678901212345678901234567890123456789012123456 7 1234567890123456789012345678901212345678901234567890123456789012123456 (D) the years described in the previous (B) Other stores also had extravaganzas. 7 1234567890123456789012345678901212345678901234567890123456789012123456 1234567890123456789012345678901212345678901234567890123456789012123456 paragraph (C) So now get in your car and drive to a 7 7 1 34567890123456789012345678901212345678901234567890123456789012123456 2 34567890123456789012345678901212345678901234567890123456789012123456 7 1234567890123456789012345678901212345678901234567890123456789012123456 (E) the 1920s department store! 7 1234567890123456789012345678901212345678901234567890123456789012123456 7 1234567890123456789012345678901212345678901234567890123456789012123456 (D) Department Stores having become 1234567890123456789012345678901212345678901234567890123456789012123456 77 1 huge successes. 7 12234567890123456789012345678901212345678901234567890123456789012123456 7 1234567890123456789012345678901212345678901234567890123456789012123456 (E) Macy’s Thanksgiving Day Parade 7 1234567890123456789012345678901212345678901234567890123456789012123456 7 1234567890123456789012345678901212345678901234567890123456789012123456 was always a big hit. 1 34567890123456789012345678901212345678901234567890123456789012123456 7 2 34567890123456789012345678901212345678901234567890123456789012123456 1234567890123456789012345678901212345678901234567890123456789012123456 77 1 7 12234567890123456789012345678901212345678901234567890123456789012123456 1 34567890123456789012345678901212345678901234567890123456789012123456 7 7 12234567890123456789012345678901212345678901234567890123456789012123456 Do not proceed to the next section until time is up. 7 1 34567890123456789012345678901212345678901234567890123456789012123456 1234567890123456789012345678901212345678901234567890123456789012123456 7 1234567890123456789012345678901212345678901234567890123456789012123456 7 1234567890123456789012345678901212345678901234567890123456789012123456 7 12345678901234567890123456789012123456789012345678901234567890121234567

STOP

Copyright © 2005 Thomson Peterson’s, a part of The Thomson Corporaton SAT is a registered trademark of the College Entrance Examination Board, which was not involved in the production of and does not endorse this product.

23

Section 4 27 Questions j Time—25 Minutes 12345678901234567890123456789012123456789012345678901234567890121234567 34567890123456789012345678901212345678901234567890123456789012123456 7 12 34567890123456789012345678901212345678901234567890123456789012123456 7 12 2 1 34567890123456789012345678901212345678901234567890123456789012123456 7 34567890123456789012345678901212345678901234567890123456789012123456 7 12 Directions: Each sentence below has either one or two blanks in it and is followed by five 34567890123456789012345678901212345678901234567890123456789012123456 7 12 34567890123456789012345678901212345678901234567890123456789012123456 7 12 choices, labeled (A) through (E). These choices represent words or phrases that have been left 34567890123456789012345678901212345678901234567890123456789012123456 7 12 2 34567890123456789012345678901212345678901234567890123456789012123456 7 1234567890123456789012345678901212345678901234567890123456789012123456 out. Choose the word or phrase that, if inserted into the sentence, would best fit the meaning of 7 1234567890123456789012345678901212345678901234567890123456789012123456 7 1234567890123456789012345678901212345678901234567890123456789012123456 the sentence as a whole. 1234567890123456789012345678901212345678901234567890123456789012123456 77 1234567890123456789012345678901212345678901234567890123456789012123456 Example: 7 1234567890123456789012345678901212345678901234567890123456789012123456 7 1 34567890123456789012345678901212345678901234567890123456789012123456 7 12 Canine massage is a veterinary technique 34567890123456789012345678901212345678901234567890123456789012123456 7 12 34567890123456789012345678901212345678901234567890123456789012123456 7 12 for calming dogs that are extremely __________. 34567890123456789012345678901212345678901234567890123456789012123456 7 12 34567890123456789012345678901212345678901234567890123456789012123456 7 12 34567890123456789012345678901212345678901234567890123456789012123456 7 12 (A) inept 34567890123456789012345678901212345678901234567890123456789012123456 7 12 34567890123456789012345678901212345678901234567890123456789012123456 7 12 (B) disciplined 2 34567890123456789012345678901212345678901234567890123456789012123456 1234567890123456789012345678901212345678901234567890123456789012123456 77 1234567890123456789012345678901212345678901234567890123456789012123456 (C) controlled 7 1234567890123456789012345678901212345678901234567890123456789012123456 7 1234567890123456789012345678901212345678901234567890123456789012123456 (D) stressed 7 1 2 34567890123456789012345678901212345678901234567890123456789012123456 7 1234567890123456789012345678901212345678901234567890123456789012123456 (E) restrained 7 1234567890123456789012345678901212345678901234567890123456789012123456 7 1 A B C E O Þ Þ O Þ O Þ O Þ 7 1234567890123456789012345678901212345678901234567890123456789012123456 34567890123456789012345678901212345678901234567890123456789012123456 7 12 2 34567890123456789012345678901212345678901234567890123456789012123456 1234567890123456789012345678901212345678901234567890123456789012123456 77 1234567890123456789012345678901212345678901234567890123456789012123456 7 1 34567890123456789012345678901212345678901234567890123456789012123456 7 12 1. Jerome is a true __________ ; he rarely 2. The property tax hike in Colson County 34567890123456789012345678901212345678901234567890123456789012123456 7 12 2 34567890123456789012345678901212345678901234567890123456789012123456 7 1234567890123456789012345678901212345678901234567890123456789012123456 buys anything other than food, and even was not just __________ by the county 7 1234567890123456789012345678901212345678901234567890123456789012123456 7 1 his food is plain and minimal. residents; a series of protests were even 1234567890123456789012345678901212345678901234567890123456789012123456 7 7 1234567890123456789012345678901212345678901234567890123456789012123456 __________. 34567890123456789012345678901212345678901234567890123456789012123456 7 12 (A) hermit 34567890123456789012345678901212345678901234567890123456789012123456 7 12 34567890123456789012345678901212345678901234567890123456789012123456 7 12 (B) teetotaler (A) disliked. .organized 34567890123456789012345678901212345678901234567890123456789012123456 7 12 7 1234567890123456789012345678901212345678901234567890123456789012123456 (C) gourmand (B) espoused. .planned 34567890123456789012345678901212345678901234567890123456789012123456 7 12 34567890123456789012345678901212345678901234567890123456789012123456 7 12 (D) ascetic (C) discouraged. .theorized 34567890123456789012345678901212345678901234567890123456789012123456 7 12 34567890123456789012345678901212345678901234567890123456789012123456 7 12 (E) fanatic (D) bolstered. .analyzed 7 1234567890123456789012345678901212345678901234567890123456789012123456 2 34567890123456789012345678901212345678901234567890123456789012123456 7 1234567890123456789012345678901212345678901234567890123456789012123456 (E) detested. .negotiated 7 1234567890123456789012345678901212345678901234567890123456789012123456 1234567890123456789012345678901212345678901234567890123456789012123456 77 1234567890123456789012345678901212345678901234567890123456789012123456 7 1 34567890123456789012345678901212345678901234567890123456789012123456 7 12 1234567890123456789012345678901212345678901234567890123456789012123456 7 34567890123456789012345678901212345678901234567890123456789012123456 7 12 1234567890123456789012345678901212345678901234567890123456789012123456 7 34567890123456789012345678901212345678901234567890123456789012123456 7 12 1234567890123456789012345678901212345678901234567890123456789012123456 7 1234567890123456789012345678901212345678901234567890123456789012123456 7 1234567890123456789012345678901212345678901234567890123456789012123456 7 1234567890123456789012345678901212345678901234567890123456789012123456 7 12345678901234567890123456789012123456789012345678901234567890121234567 24

Copyright © 2005 Thomson Peterson’s, a part of The Thomson Corporaton SAT is a registered trademark of the College Entrance Examination Board, which was not involved in the production of and does not endorse this product.

12345678901234567890123456789012123456789012345678901234567890121234567 1234567890123456789012345678901212345678901234567890123456789012123456 7 7 12234567890123456789012345678901212345678901234567890123456789012123456 3. There was criticism that the councilman 7 7. Despite the longstanding __________ 1 34567890123456789012345678901212345678901234567890123456789012123456 7 12234567890123456789012345678901212345678901234567890123456789012123456 was __________ when he seized the between the clans, both clans __________ 7 1234567890123456789012345678901212345678901234567890123456789012123456 7 1 34567890123456789012345678901212345678901234567890123456789012123456 ceremony __________ by the girl’s tragic each other in the aftermath of the disaster. 7 12234567890123456789012345678901212345678901234567890123456789012123456 34567890123456789012345678901212345678901234567890123456789012123456 7 1 death to speak out against his opponent. 7 12234567890123456789012345678901212345678901234567890123456789012123456 (A) feud. .assisted 7 1234567890123456789012345678901212345678901234567890123456789012123456 7 1234567890123456789012345678901212345678901234567890123456789012123456 (A) militaristic. . negated (B) grudge. .maligned 7 1234567890123456789012345678901212345678901234567890123456789012123456 34567890123456789012345678901212345678901234567890123456789012123456 7 1 (B) opportunistic. .afforded (C) detente. .withstood 7 12234567890123456789012345678901212345678901234567890123456789012123456 7 1234567890123456789012345678901212345678901234567890123456789012123456 (C) unreceptive. .preempted (D) skirmish. . ameliorated 7 1234567890123456789012345678901212345678901234567890123456789012123456 7 1234567890123456789012345678901212345678901234567890123456789012123456 (D) passive. . created (E) alliance. . discounted 34567890123456789012345678901212345678901234567890123456789012123456 7 1 7 12234567890123456789012345678901212345678901234567890123456789012123456 (E) defeatist. .overshadowed 7 1234567890123456789012345678901212345678901234567890123456789012123456 1234567890123456789012345678901212345678901234567890123456789012123456 8. The oral tradition of the Bambara people 7 7 1234567890123456789012345678901212345678901234567890123456789012123456 4. The computer expert underscored that the of West Africa is rich with humor and 7 1 34567890123456789012345678901212345678901234567890123456789012123456 7 12234567890123456789012345678901212345678901234567890123456789012123456 new software would _____ the prior __________, characteristics which are 7 1234567890123456789012345678901212345678901234567890123456789012123456 7 1234567890123456789012345678901212345678901234567890123456789012123456 version; users could simply __________ evident in the merriment of their every7 1234567890123456789012345678901212345678901234567890123456789012123456 34567890123456789012345678901212345678901234567890123456789012123456 7 1234567890123456789012345678901212345678901234567890123456789012123456 the old one. day life. 7 1234567890123456789012345678901212345678901234567890123456789012123456 1234567890123456789012345678901212345678901234567890123456789012123456 77 1234567890123456789012345678901212345678901234567890123456789012123456 (A) preclude. .destroy (A) irony 7 1234567890123456789012345678901212345678901234567890123456789012123456 7 1234567890123456789012345678901212345678901234567890123456789012123456 (B) outdo. .implement (B) mirth 7 1 7 12234567890123456789012345678901212345678901234567890123456789012123456 (C) infect. .disregard (C) cynicism 7 1234567890123456789012345678901212345678901234567890123456789012123456 7 1234567890123456789012345678901212345678901234567890123456789012123456 (D) undermine. .detach (D) history 34567890123456789012345678901212345678901234567890123456789012123456 1234567890123456789012345678901212345678901234567890123456789012123456 77 1234567890123456789012345678901212345678901234567890123456789012123456 (E) supercede. .discard (E) mystery 7 1234567890123456789012345678901212345678901234567890123456789012123456 1234567890123456789012345678901212345678901234567890123456789012123456 77 1234567890123456789012345678901212345678901234567890123456789012123456 5. Romania has a long and __________ 9. Although Astropithicus has more subspe- 7 1234567890123456789012345678901212345678901234567890123456789012123456 7 1234567890123456789012345678901212345678901234567890123456789012123456 tradition of activist-poets, who through cies than Pthicalitius, the latter populates 7 1234567890123456789012345678901212345678901234567890123456789012123456 7 1234567890123456789012345678901212345678901234567890123456789012123456 poetry have __________ the dignity and the Earth with __________ numbers and 7 1 2 34567890123456789012345678901212345678901234567890123456789012123456 7 1234567890123456789012345678901212345678901234567890123456789012123456 equality of humanity. in more __________ geographic regions. 7 1234567890123456789012345678901212345678901234567890123456789012123456 7 1 7 1234567890123456789012345678901212345678901234567890123456789012123456 (A) reputable. .initiated (A) greater. .diverse 7 12234567890123456789012345678901212345678901234567890123456789012123456 34567890123456789012345678901212345678901234567890123456789012123456 7 1234567890123456789012345678901212345678901234567890123456789012123456 (B) storied. .articulated (B) lesser. .secluded 7 1234567890123456789012345678901212345678901234567890123456789012123456 7 1 (C) tenuous. .supported (C) milder. .remote 7 12234567890123456789012345678901212345678901234567890123456789012123456 7 1234567890123456789012345678901212345678901234567890123456789012123456 (D) tactile. .decried (D) scanter. .familiar 34567890123456789012345678901212345678901234567890123456789012123456 1234567890123456789012345678901212345678901234567890123456789012123456 77 1234567890123456789012345678901212345678901234567890123456789012123456 (E) exemplary. .countered (E) larger. .ominous 7 1 1234567890123456789012345678901212345678901234567890123456789012123456 7 7 1234567890123456789012345678901212345678901234567890123456789012123456 6. Dr. Patel expected the surgery to be 10. It is generally __________ by medical 7 12234567890123456789012345678901212345678901234567890123456789012123456 7 1234567890123456789012345678901212345678901234567890123456789012123456 __________ and laborious, but it turned practitioners that the last few weeks of a 7 1234567890123456789012345678901212345678901234567890123456789012123456 7 1234567890123456789012345678901212345678901234567890123456789012123456 out to be speedy and __________. pregnancy are crucial in the __________ of 1 34567890123456789012345678901212345678901234567890123456789012123456 7 2 34567890123456789012345678901212345678901234567890123456789012123456 7 1234567890123456789012345678901212345678901234567890123456789012123456 the fetus. 7 1234567890123456789012345678901212345678901234567890123456789012123456 (A) fragile. .simple 1234567890123456789012345678901212345678901234567890123456789012123456 77 1234567890123456789012345678901212345678901234567890123456789012123456 (B) compelling. .forthcoming (A) acknowledged. .progression 7 1 7 12234567890123456789012345678901212345678901234567890123456789012123456 hedged. .health (C) intricate. .straightforward (B) 7 1234567890123456789012345678901212345678901234567890123456789012123456 7 1234567890123456789012345678901212345678901234567890123456789012123456 (C) endorsed. .birthing (D) complicated. .locatable 7 1234567890123456789012345678901212345678901234567890123456789012123456 7 1 34567890123456789012345678901212345678901234567890123456789012123456 (D) accepted. .development (E) hard. .mechanical 2 34567890123456789012345678901212345678901234567890123456789012123456 1234567890123456789012345678901212345678901234567890123456789012123456 77 1 (E) negated. .vitality 7 12234567890123456789012345678901212345678901234567890123456789012123456 1 34567890123456789012345678901212345678901234567890123456789012123456 7 7 12234567890123456789012345678901212345678901234567890123456789012123456 1 34567890123456789012345678901212345678901234567890123456789012123456 7 1234567890123456789012345678901212345678901234567890123456789012123456 7 1234567890123456789012345678901212345678901234567890123456789012123456 7 1234567890123456789012345678901212345678901234567890123456789012123456 7 12345678901234567890123456789012123456789012345678901234567890121234567 Copyright © 2005 Thomson Peterson’s, a part of The Thomson Corporaton SAT is a registered trademark of the College Entrance Examination Board, which was not involved in the production of and does not endorse this product.

25

12345678901234567890123456789012123456789012345678901234567890121234567 1234567890123456789012345678901212345678901234567890123456789012123456 7 34567890123456789012345678901212345678901234567890123456789012123456 7 12 11. Though the organization espoused 7 15. Technology, instead of alleviating the 1234567890123456789012345678901212345678901234567890123456789012123456 34567890123456789012345678901212345678901234567890123456789012123456 7 12 outward-focused ideals, in practice, it was demands upon our time, has made the 34567890123456789012345678901212345678901234567890123456789012123456 7 12 7 1234567890123456789012345678901212345678901234567890123456789012123456 quite __________. pace of modern-day life increase to a near 34567890123456789012345678901212345678901234567890123456789012123456 7 12 2 34567890123456789012345678901212345678901234567890123456789012123456 7 1 __________ pace. 34567890123456789012345678901212345678901234567890123456789012123456 7 12 (A) gregarious 34567890123456789012345678901212345678901234567890123456789012123456 7 12 34567890123456789012345678901212345678901234567890123456789012123456 7 12 (B) vigilant (A) elicit 34567890123456789012345678901212345678901234567890123456789012123456 7 12 2 34567890123456789012345678901212345678901234567890123456789012123456 7 1 (C) insular (B) frenetic 34567890123456789012345678901212345678901234567890123456789012123456 7 12 34567890123456789012345678901212345678901234567890123456789012123456 7 12 (D) hermetic (C) lethargic 34567890123456789012345678901212345678901234567890123456789012123456 7 12 34567890123456789012345678901212345678901234567890123456789012123456 7 12 (E) idiosyncratic (D) dilatory 7 1234567890123456789012345678901212345678901234567890123456789012123456 34567890123456789012345678901212345678901234567890123456789012123456 7 12 (E) cavalier 34567890123456789012345678901212345678901234567890123456789012123456 7 12 34567890123456789012345678901212345678901234567890123456789012123456 7 12. The prosecuting attorney described the 12 34567890123456789012345678901212345678901234567890123456789012123456 7 12 defendant’s character as __________ and 2 34567890123456789012345678901212345678901234567890123456789012123456 7 1 Questions 16–27 are based on the following 34567890123456789012345678901212345678901234567890123456789012123456 7 12 base, but the defense attorney rejoined 34567890123456789012345678901212345678901234567890123456789012123456 7 12 passage. 34567890123456789012345678901212345678901234567890123456789012123456 7 12 that the prosecution was __________ the 34567890123456789012345678901212345678901234567890123456789012123456 7 12 The following passage tells the story of Juan 2 34567890123456789012345678901212345678901234567890123456789012123456 7 1234567890123456789012345678901212345678901234567890123456789012123456 testimonials about the defendant. 7 1234567890123456789012345678901212345678901234567890123456789012123456 Bautista Rael, an Hispanic-American scholar. It 7 1234567890123456789012345678901212345678901234567890123456789012123456 7 1234567890123456789012345678901212345678901234567890123456789012123456 (A) dark. .misunderstanding is adapted from a biography written by Enrique 7 1234567890123456789012345678901212345678901234567890123456789012123456 7 1234567890123456789012345678901212345678901234567890123456789012123456 (B) nefarious. .misconstruing R. Lamadrid. 7 1 34567890123456789012345678901212345678901234567890123456789012123456 7 12 (C) lackluster. .misinterpreting 34567890123456789012345678901212345678901234567890123456789012123456 7 12 Line Linguist and folklorist Juan Bautista Rael 34567890123456789012345678901212345678901234567890123456789012123456 7 12 (D) blustery. .misapplying 2 34567890123456789012345678901212345678901234567890123456789012123456 7 1234567890123456789012345678901212345678901234567890123456789012123456 was a highly regarded academic pioneer. 7 1234567890123456789012345678901212345678901234567890123456789012123456 (E) motley. .misrepresenting 7 1234567890123456789012345678901212345678901234567890123456789012123456 His major contribution to linguistics was 1234567890123456789012345678901212345678901234567890123456789012123456 77 1234567890123456789012345678901212345678901234567890123456789012123456 collecting and documenting the Hispano 13. Many believe that the new drug regime 7 1234567890123456789012345678901212345678901234567890123456789012123456 7 1234567890123456789012345678901212345678901234567890123456789012123456 (5) folk stories, plays, and religious tradiwill be a __________ for helping cure 7 1234567890123456789012345678901212345678901234567890123456789012123456 7 1234567890123456789012345678901212345678901234567890123456789012123456 tions of northern New Mexico and diverse ailments related to spinal dysfunc7 1 2 34567890123456789012345678901212345678901234567890123456789012123456 7 1234567890123456789012345678901212345678901234567890123456789012123456 southern Colorado. tions. 7 1234567890123456789012345678901212345678901234567890123456789012123456 7 1 Rael was born on August 14, 1900, 7 1234567890123456789012345678901212345678901234567890123456789012123456 (A) bane 34567890123456789012345678901212345678901234567890123456789012123456 12 in Arroyo Hondo, New Mexico. Famous 7 2 34567890123456789012345678901212345678901234567890123456789012123456 7 1234567890123456789012345678901212345678901234567890123456789012123456 (B) panacea (10) for its spectacular setting north of Taos, 7 1234567890123456789012345678901212345678901234567890123456789012123456 7 1 (C) module the village lies in a deep, narrow valley 34567890123456789012345678901212345678901234567890123456789012123456 7 12 34567890123456789012345678901212345678901234567890123456789012123456 7 12 (D) fledgling between Taos mountain and the gorge of 2 34567890123456789012345678901212345678901234567890123456789012123456 1234567890123456789012345678901212345678901234567890123456789012123456 77 1234567890123456789012345678901212345678901234567890123456789012123456 (E) pariah the Rio Grande to the west. His family 7 1 7 1234567890123456789012345678901212345678901234567890123456789012123456 prospered in sheep and cattle ranching 7 1234567890123456789012345678901212345678901234567890123456789012123456 14. The executive charged that the whistle2 34567890123456789012345678901212345678901234567890123456789012123456 7 1234567890123456789012345678901212345678901234567890123456789012123456 (15) and owned a mercantile business that 7 1234567890123456789012345678901212345678901234567890123456789012123456 blower’s actions were so self-centered that 7 1234567890123456789012345678901212345678901234567890123456789012123456 served surrounding Hispano communities 7 1234567890123456789012345678901212345678901234567890123456789012123456 they were not just __________ but even 7 1 as well as nearby Taos Pueblo. 2 34567890123456789012345678901212345678901234567890123456789012123456 7 1234567890123456789012345678901212345678901234567890123456789012123456 __________. 7 1234567890123456789012345678901212345678901234567890123456789012123456 Juan’s parents, José Ignacio Rael and 1234567890123456789012345678901212345678901234567890123456789012123456 77 1234567890123456789012345678901212345678901234567890123456789012123456 Soledad Santistevan, raised a family of (A) self-involved. .altruistic 7 1 34567890123456789012345678901212345678901234567890123456789012123456 7 12 (20) four sons and a daughter. José Ignacio (B) ill-tempered. .narcissistic 34567890123456789012345678901212345678901234567890123456789012123456 7 12 34567890123456789012345678901212345678901234567890123456789012123456 7 12 had the foresight to recognize the (C) egocentric. .solipsistic 34567890123456789012345678901212345678901234567890123456789012123456 7 12 7 1234567890123456789012345678901212345678901234567890123456789012123456 changes that were coming with the (D) unduly. .respectable 34567890123456789012345678901212345678901234567890123456789012123456 7 12 7 1234567890123456789012345678901212345678901234567890123456789012123456 increasing Americanization of New (E) erratic. .destructive 34567890123456789012345678901212345678901234567890123456789012123456 7 12 7 1234567890123456789012345678901212345678901234567890123456789012123456 Mexico and realized that a fluent 34567890123456789012345678901212345678901234567890123456789012123456 7 12 2 34567890123456789012345678901212345678901234567890123456789012123456 7 1 (25) knowledge of English and a good 7 1234567890123456789012345678901212345678901234567890123456789012123456 7 1234567890123456789012345678901212345678901234567890123456789012123456 education would be necessary for his 1234567890123456789012345678901212345678901234567890123456789012123456 7 12345678901234567890123456789012123456789012345678901234567890121234567 26

Copyright © 2005 Thomson Peterson’s, a part of The Thomson Corporaton SAT is a registered trademark of the College Entrance Examination Board, which was not involved in the production of and does not endorse this product.

12345678901234567890123456789012123456789012345678901234567890121234567 1234567890123456789012345678901212345678901234567890123456789012123456 7 7 12234567890123456789012345678901212345678901234567890123456789012123456 family to excel. Since local schools were (70) 7 relinquished his family inheritance in 1 34567890123456789012345678901212345678901234567890123456789012123456 7 12234567890123456789012345678901212345678901234567890123456789012123456 rudimentary at best, the family relied land, cattle, and sheep to his three 7 1234567890123456789012345678901212345678901234567890123456789012123456 7 1 34567890123456789012345678901212345678901234567890123456789012123456 upon its own resources to get the best brothers and his sister. 7 12234567890123456789012345678901212345678901234567890123456789012123456 34567890123456789012345678901212345678901234567890123456789012123456 7 1 (30) possible education for the children. Juan Rael realized that the wealth in 7 12234567890123456789012345678901212345678901234567890123456789012123456 7 1234567890123456789012345678901212345678901234567890123456789012123456 was a dedicated student from his earliest northern New Mexico that interested 7 1234567890123456789012345678901212345678901234567890123456789012123456 7 1234567890123456789012345678901212345678901234567890123456789012123456 years, and his father’s ambition was for (75) him was the vast repertory of folk 34567890123456789012345678901212345678901234567890123456789012123456 7 1 7 12234567890123456789012345678901212345678901234567890123456789012123456 him to become a lawyer and tend to the narrative, song, and custom that had 7 1234567890123456789012345678901212345678901234567890123456789012123456 7 1234567890123456789012345678901212345678901234567890123456789012123456 family lands and business. Juan’s scarcely been documented. While 7 1234567890123456789012345678901212345678901234567890123456789012123456 34567890123456789012345678901212345678901234567890123456789012123456 1 (35) elementary schooling was at Saint teaching at the University of Oregon, he 7 7 12234567890123456789012345678901212345678901234567890123456789012123456 7 1234567890123456789012345678901212345678901234567890123456789012123456 Michael’s College in Santa Fe, and his returned to Arroyo Hondo in the summer 7 1234567890123456789012345678901212345678901234567890123456789012123456 7 1234567890123456789012345678901212345678901234567890123456789012123456 (80) high school studies were at the Christian of 1930 to begin compiling his famous 34567890123456789012345678901212345678901234567890123456789012123456 7 1 7 12234567890123456789012345678901212345678901234567890123456789012123456 Brothers’ College in St. Louis, Missouri. collection of over five hundred Nuevo 7 1234567890123456789012345678901212345678901234567890123456789012123456 7 1234567890123456789012345678901212345678901234567890123456789012123456 The boy’s semester-long absences from Mexicano folk tales. 7 1234567890123456789012345678901212345678901234567890123456789012123456 34567890123456789012345678901212345678901234567890123456789012123456 7 1234567890123456789012345678901212345678901234567890123456789012123456 (40) his family led him to treasure the simple By then his work had attracted the 7 1234567890123456789012345678901212345678901234567890123456789012123456 7 1234567890123456789012345678901212345678901234567890123456789012123456 pleasures of village life. Summers are attention of pioneer Hispano folklorist 1234567890123456789012345678901212345678901234567890123456789012123456 77 1234567890123456789012345678901212345678901234567890123456789012123456 (85) especially beautiful in Arroyo Hondo, Aurelio Espinosa who invited Rael to 7 1234567890123456789012345678901212345678901234567890123456789012123456 7 1 and Christmas and Easter vacations were Stanford in 1933. Rael completed his 7 12234567890123456789012345678901212345678901234567890123456789012123456 filled with colorful festivities and solemn doctoral studies in 1937 there with a 7 1234567890123456789012345678901212345678901234567890123456789012123456 7 1234567890123456789012345678901212345678901234567890123456789012123456 (45) ceremony. Rael later reminisced about dissertation on the phonology and 34567890123456789012345678901212345678901234567890123456789012123456 1234567890123456789012345678901212345678901234567890123456789012123456 77 1234567890123456789012345678901212345678901234567890123456789012123456 how much the Pastores, or Shepherds’ morphology of New Mexico Spanish that 7 1234567890123456789012345678901212345678901234567890123456789012123456 7 1234567890123456789012345678901212345678901234567890123456789012123456 (90) plays, impressed him as a child. Unamplified the work of Espinosa with the 7 1234567890123456789012345678901212345678901234567890123456789012123456 1234567890123456789012345678901212345678901234567890123456789012123456 doubtedly, the instincts and sympathies huge corpus of folk tales, later published 7 7 1234567890123456789012345678901212345678901234567890123456789012123456 7 1234567890123456789012345678901212345678901234567890123456789012123456 of Rael the folklorist can be traced to as, Cuentos Españoles de Colorado y 1234567890123456789012345678901212345678901234567890123456789012123456 77 1 (50) these beginnings—watching rehearsals Nuevo Mexico: Spanish Folk Tales of 7 12234567890123456789012345678901212345678901234567890123456789012123456 7 1234567890123456789012345678901212345678901234567890123456789012123456 and performances depicting shepherds, Colorado and New Mexico. 1 34567890123456789012345678901212345678901234567890123456789012123456 7 1234567890123456789012345678901212345678901234567890123456789012123456 (95) hermits, and the rich ensemble of Well versed in the historic-geographic 7 7 12234567890123456789012345678901212345678901234567890123456789012123456 34567890123456789012345678901212345678901234567890123456789012123456 7 1234567890123456789012345678901212345678901234567890123456789012123456 pastoral characters. theory of transmission and diffusion of 7 1234567890123456789012345678901212345678901234567890123456789012123456 7 1 What became clear in his postmotifs, tale types, and genres, Dr. Rael 7 12234567890123456789012345678901212345678901234567890123456789012123456 7 1234567890123456789012345678901212345678901234567890123456789012123456 (55) secondary studies is that he was much set out on the formidable, almost 34567890123456789012345678901212345678901234567890123456789012123456 1234567890123456789012345678901212345678901234567890123456789012123456 77 1234567890123456789012345678901212345678901234567890123456789012123456 more attracted to literature, philology, quixotic task of gathering all the possible 7 1 7 1234567890123456789012345678901212345678901234567890123456789012123456 (100) versions and texts of the tales, hymns, and the emerging disciplines of linguistics 1234567890123456789012345678901212345678901234567890123456789012123456 7 2 34567890123456789012345678901212345678901234567890123456789012123456 7 1234567890123456789012345678901212345678901234567890123456789012123456 and folklore. His Bachelor’s degree, from and plays he was studying. The vast 7 1234567890123456789012345678901212345678901234567890123456789012123456 7 1234567890123456789012345678901212345678901234567890123456789012123456 St. Mary’s College in Oakland in 1923, majority of tales are of European 1234567890123456789012345678901212345678901234567890123456789012123456 77 1 (60) led to a Master’s degree from the provenance, with only minimal local 7 12234567890123456789012345678901212345678901234567890123456789012123456 University of California at Berkeley in references. He meticulously traced the 7 1234567890123456789012345678901212345678901234567890123456789012123456 7 1234567890123456789012345678901212345678901234567890123456789012123456 (105) shepherds’ plays to several root sources 1927. In the meantime, in 1923, he 7 1234567890123456789012345678901212345678901234567890123456789012123456 7 1 34567890123456789012345678901212345678901234567890123456789012123456 married Quirina Espinoza of Antonito, in Mexico, and his study, The Sources 2 34567890123456789012345678901212345678901234567890123456789012123456 1234567890123456789012345678901212345678901234567890123456789012123456 77 1234567890123456789012345678901212345678901234567890123456789012123456 Colorado. Rael’s first inclination was to and Diffusion of the Mexican Shepherds’ 7 1234567890123456789012345678901212345678901234567890123456789012123456 7 1234567890123456789012345678901212345678901234567890123456789012123456 (65) become an English teacher, but his bride Plays, is a standard reference on the 7 1 2 34567890123456789012345678901212345678901234567890123456789012123456 1234567890123456789012345678901212345678901234567890123456789012123456 helped convince him that his opportunisubject. His ground-breaking study of the 7 7 1 7 12234567890123456789012345678901212345678901234567890123456789012123456 (110) ties and strengths would be as an alabado hymn, The New Mexican 1 34567890123456789012345678901212345678901234567890123456789012123456 7 7 12234567890123456789012345678901212345678901234567890123456789012123456 Hispanist. After deciding on a university Alabado, is also a prime resource. But 7 1 34567890123456789012345678901212345678901234567890123456789012123456 7 1234567890123456789012345678901212345678901234567890123456789012123456 career of teaching and research, Rael inevitably the historic-geographic 1234567890123456789012345678901212345678901234567890123456789012123456 77 1234567890123456789012345678901212345678901234567890123456789012123456 12345678901234567890123456789012123456789012345678901234567890121234567 Copyright © 2005 Thomson Peterson’s, a part of The Thomson Corporaton SAT is a registered trademark of the College Entrance Examination Board, which was not involved in the production of and does not endorse this product.

27

12345678901234567890123456789012123456789012345678901234567890121234567 1234567890123456789012345678901212345678901234567890123456789012123456 7 34567890123456789012345678901212345678901234567890123456789012123456 7 12 approach led more to collection building 7 19. The author probably uses the word 1234567890123456789012345678901212345678901234567890123456789012123456 34567890123456789012345678901212345678901234567890123456789012123456 7 12 than to analysis. Later generations of “relinquished” in line 70 to emphasize 34567890123456789012345678901212345678901234567890123456789012123456 7 12 7 1234567890123456789012345678901212345678901234567890123456789012123456 (115) scholars would develop interests in that 34567890123456789012345678901212345678901234567890123456789012123456 7 12 2 34567890123456789012345678901212345678901234567890123456789012123456 7 1 performance-centered studies, but the 34567890123456789012345678901212345678901234567890123456789012123456 7 12 (A) Rael had had friction with his 34567890123456789012345678901212345678901234567890123456789012123456 7 12 collections of Rael continue to be an 34567890123456789012345678901212345678901234567890123456789012123456 7 12 siblings. 34567890123456789012345678901212345678901234567890123456789012123456 7 12 indispensable landmark in the field. 2 34567890123456789012345678901212345678901234567890123456789012123456 7 1 (B) Rael’s family was very wealthy. 34567890123456789012345678901212345678901234567890123456789012123456 7 12 34567890123456789012345678901212345678901234567890123456789012123456 7 12 (C) Rael had tried to be a rancher for 34567890123456789012345678901212345678901234567890123456789012123456 7 12 34567890123456789012345678901212345678901234567890123456789012123456 7 12 16. The author’s attitude towards Dr. Rael’s some time. 7 1234567890123456789012345678901212345678901234567890123456789012123456 34567890123456789012345678901212345678901234567890123456789012123456 7 12 work can best be described as (D) Rael was relieved to be free of his 34567890123456789012345678901212345678901234567890123456789012123456 7 12 34567890123456789012345678901212345678901234567890123456789012123456 7 12 family duties. (A) laudatory. 34567890123456789012345678901212345678901234567890123456789012123456 7 12 2 34567890123456789012345678901212345678901234567890123456789012123456 7 1 (E) Rael’s career path was difficult and (B) engaged. 34567890123456789012345678901212345678901234567890123456789012123456 7 12 34567890123456789012345678901212345678901234567890123456789012123456 7 12 not lucrative. (C) ambivalent. 34567890123456789012345678901212345678901234567890123456789012123456 7 12 34567890123456789012345678901212345678901234567890123456789012123456 7 12 (D) disinterested. 34567890123456789012345678901212345678901234567890123456789012123456 7 12 20. It can be inferred from the passage that 34567890123456789012345678901212345678901234567890123456789012123456 7 12 (E) condescending. 34567890123456789012345678901212345678901234567890123456789012123456 7 12 northern New Mexico in the early decades 34567890123456789012345678901212345678901234567890123456789012123456 7 12 34567890123456789012345678901212345678901234567890123456789012123456 7 12 of the twentieth century 34567890123456789012345678901212345678901234567890123456789012123456 7 12 17. The passage primarily 7 1234567890123456789012345678901212345678901234567890123456789012123456 34567890123456789012345678901212345678901234567890123456789012123456 7 12 (A) had a ranch-driven economy. (A) analyzes the academic contributions 34567890123456789012345678901212345678901234567890123456789012123456 7 12 34567890123456789012345678901212345678901234567890123456789012123456 7 12 (B) was suffering severe economic of Dr. Rael. 2 34567890123456789012345678901212345678901234567890123456789012123456 1234567890123456789012345678901212345678901234567890123456789012123456 77 1234567890123456789012345678901212345678901234567890123456789012123456 depression. (B) contrasts Dr. Rael’s work with the 7 1234567890123456789012345678901212345678901234567890123456789012123456 7 1234567890123456789012345678901212345678901234567890123456789012123456 (C) was economically booming because work of Dr. Espinosa. 1234567890123456789012345678901212345678901234567890123456789012123456 77 1234567890123456789012345678901212345678901234567890123456789012123456 of a newly opened southern railroad (C) tells the story of Dr. Rael’s life and 7 1234567890123456789012345678901212345678901234567890123456789012123456 7 1234567890123456789012345678901212345678901234567890123456789012123456 route to California. work. 1234567890123456789012345678901212345678901234567890123456789012123456 77 1 (D) had a strong public education system. (D) discusses the basic assumptions of 34567890123456789012345678901212345678901234567890123456789012123456 7 12 34567890123456789012345678901212345678901234567890123456789012123456 12 (E) was not a primarily English-speaking 7 Hispano scholarship. 7 1234567890123456789012345678901212345678901234567890123456789012123456 region. 7 1234567890123456789012345678901212345678901234567890123456789012123456 (E) relates the story of the Rael family. 34567890123456789012345678901212345678901234567890123456789012123456 7 12 2 34567890123456789012345678901212345678901234567890123456789012123456 1234567890123456789012345678901212345678901234567890123456789012123456 77 1234567890123456789012345678901212345678901234567890123456789012123456 21. Which of the following best describes Dr. 7 1 18. The passage implies that Rael decided that 34567890123456789012345678901212345678901234567890123456789012123456 7 12 Espinosa’s relationship to Dr. Rael? 34567890123456789012345678901212345678901234567890123456789012123456 7 12 he would have more opportunity as a 2 34567890123456789012345678901212345678901234567890123456789012123456 7 1234567890123456789012345678901212345678901234567890123456789012123456 Hispanist than as an English teacher 7 1234567890123456789012345678901212345678901234567890123456789012123456 (A) Boss 7 1 because 7 1234567890123456789012345678901212345678901234567890123456789012123456 (B) Critic 1234567890123456789012345678901212345678901234567890123456789012123456 7 2 34567890123456789012345678901212345678901234567890123456789012123456 7 1234567890123456789012345678901212345678901234567890123456789012123456 (C) Mentor (A) Hispanics weren’t often hired to 7 1234567890123456789012345678901212345678901234567890123456789012123456 7 1234567890123456789012345678901212345678901234567890123456789012123456 (D) Pastor teach English. 1234567890123456789012345678901212345678901234567890123456789012123456 77 1 (E) Father (B) Hispanic folklore would soon vanish. 34567890123456789012345678901212345678901234567890123456789012123456 7 12 34567890123456789012345678901212345678901234567890123456789012123456 7 12 (C) he would have to live far from his 34567890123456789012345678901212345678901234567890123456789012123456 7 12 22. The word “corpus” in line 91 most closely 34567890123456789012345678901212345678901234567890123456789012123456 7 12 family to teach English. 7 1234567890123456789012345678901212345678901234567890123456789012123456 means 2 34567890123456789012345678901212345678901234567890123456789012123456 7 1234567890123456789012345678901212345678901234567890123456789012123456 (D) his English skills were mediocre. 7 1234567890123456789012345678901212345678901234567890123456789012123456 7 1234567890123456789012345678901212345678901234567890123456789012123456 (E) becoming an Hispanist was a nearer (A) religious text. 1234567890123456789012345678901212345678901234567890123456789012123456 77 1 match to his educational back(B) oeuvre. 34567890123456789012345678901212345678901234567890123456789012123456 7 12 7 1234567890123456789012345678901212345678901234567890123456789012123456 published collection. ground. (C) 34567890123456789012345678901212345678901234567890123456789012123456 7 12 7 1234567890123456789012345678901212345678901234567890123456789012123456 (D) dissertation. 34567890123456789012345678901212345678901234567890123456789012123456 7 12 2 34567890123456789012345678901212345678901234567890123456789012123456 7 1 (E) dialect. 7 1234567890123456789012345678901212345678901234567890123456789012123456 1234567890123456789012345678901212345678901234567890123456789012123456 77 1234567890123456789012345678901212345678901234567890123456789012123456 12345678901234567890123456789012123456789012345678901234567890121234567 28

Copyright © 2005 Thomson Peterson’s, a part of The Thomson Corporaton SAT is a registered trademark of the College Entrance Examination Board, which was not involved in the production of and does not endorse this product.

12345678901234567890123456789012123456789012345678901234567890121234567 1234567890123456789012345678901212345678901234567890123456789012123456 7 7 12234567890123456789012345678901212345678901234567890123456789012123456 23. According to the passage, which of the 26. The main criticism the author offers of Dr. 7 1 34567890123456789012345678901212345678901234567890123456789012123456 7 12234567890123456789012345678901212345678901234567890123456789012123456 following of Dr. Rael’s activities as a Rael’s work is that it 7 1234567890123456789012345678901212345678901234567890123456789012123456 7 1 34567890123456789012345678901212345678901234567890123456789012123456 young man was most important for the 7 12234567890123456789012345678901212345678901234567890123456789012123456 (A) focuses too much on the Pastores. 34567890123456789012345678901212345678901234567890123456789012123456 7 1 development of his later academic 7 12234567890123456789012345678901212345678901234567890123456789012123456 (B) privileges Spanish-language stories 7 1234567890123456789012345678901212345678901234567890123456789012123456 interests? 7 1234567890123456789012345678901212345678901234567890123456789012123456 over English-language stories. 7 1234567890123456789012345678901212345678901234567890123456789012123456 34567890123456789012345678901212345678901234567890123456789012123456 7 1 (A) Working on his family’s ranch (C) focuses too much on European 7 12234567890123456789012345678901212345678901234567890123456789012123456 7 1234567890123456789012345678901212345678901234567890123456789012123456 (B) Watching Pastores as a young man stories and not enough on Mexican 7 1234567890123456789012345678901212345678901234567890123456789012123456 7 1234567890123456789012345678901212345678901234567890123456789012123456 (C) Studying in a religious school stories. 34567890123456789012345678901212345678901234567890123456789012123456 7 1 7 12234567890123456789012345678901212345678901234567890123456789012123456 (D) Reading books on the shepherds of (D) doesn’t offer enough analysis of the 7 1234567890123456789012345678901212345678901234567890123456789012123456 7 1234567890123456789012345678901212345678901234567890123456789012123456 northern New Mexico folklore. 7 1234567890123456789012345678901212345678901234567890123456789012123456 34567890123456789012345678901212345678901234567890123456789012123456 1 (E) Struggling to retain his Spanish when (E) includes too many materials, without 7 7 12234567890123456789012345678901212345678901234567890123456789012123456 7 1234567890123456789012345678901212345678901234567890123456789012123456 his schooling was in English differentiating between good and 7 1234567890123456789012345678901212345678901234567890123456789012123456 7 1234567890123456789012345678901212345678901234567890123456789012123456 bad. 34567890123456789012345678901212345678901234567890123456789012123456 7 1234567890123456789012345678901212345678901234567890123456789012123456 24. The phrase “diffusion of motifs” in lines 7 1234567890123456789012345678901212345678901234567890123456789012123456 7 1234567890123456789012345678901212345678901234567890123456789012123456 96–97 refers to 27. The passage suggests, in lines 97–101 that 1234567890123456789012345678901212345678901234567890123456789012123456 77 1234567890123456789012345678901212345678901234567890123456789012123456 Dr. Rael 7 1234567890123456789012345678901212345678901234567890123456789012123456 (A) variations in the same stories that 7 1 7 12234567890123456789012345678901212345678901234567890123456789012123456 occur over time or by region. (A) was mistaken about how much 7 1234567890123456789012345678901212345678901234567890123456789012123456 7 1234567890123456789012345678901212345678901234567890123456789012123456 (B) adherence to standard literary folklore was circulating. 34567890123456789012345678901212345678901234567890123456789012123456 1234567890123456789012345678901212345678901234567890123456789012123456 77 1234567890123456789012345678901212345678901234567890123456789012123456 structures. (B) traveled extensively as he gathered 7 1234567890123456789012345678901212345678901234567890123456789012123456 7 1234567890123456789012345678901212345678901234567890123456789012123456 (C) variations in language use by region. folklore. 1234567890123456789012345678901212345678901234567890123456789012123456 77 1234567890123456789012345678901212345678901234567890123456789012123456 (D) the surprising similarity of stories in (C) was disorganized but intelligent in 7 1234567890123456789012345678901212345678901234567890123456789012123456 7 1234567890123456789012345678901212345678901234567890123456789012123456 different cultures. his methods. 1234567890123456789012345678901212345678901234567890123456789012123456 77 1 (E) the loss of folklore in more industri(D) had an historic insight about the 7 12234567890123456789012345678901212345678901234567890123456789012123456 7 alized societies. 1234567890123456789012345678901212345678901234567890123456789012123456 source of New Mexican folklore. 7 1 34567890123456789012345678901212345678901234567890123456789012123456 (E) developed an excessively technical 7 1234567890123456789012345678901212345678901234567890123456789012123456 7 12234567890123456789012345678901212345678901234567890123456789012123456 25. The word “provenance,” line 103, could model for the development of 34567890123456789012345678901212345678901234567890123456789012123456 1234567890123456789012345678901212345678901234567890123456789012123456 77 1234567890123456789012345678901212345678901234567890123456789012123456 best be replaced with which of the folklore. 7 1 7 12234567890123456789012345678901212345678901234567890123456789012123456 following words? 7 1234567890123456789012345678901212345678901234567890123456789012123456 34567890123456789012345678901212345678901234567890123456789012123456 1234567890123456789012345678901212345678901234567890123456789012123456 77 1234567890123456789012345678901212345678901234567890123456789012123456 (A) Authority 7 1 7 1234567890123456789012345678901212345678901234567890123456789012123456 (B) Origin 1234567890123456789012345678901212345678901234567890123456789012123456 7 2 34567890123456789012345678901212345678901234567890123456789012123456 7 1234567890123456789012345678901212345678901234567890123456789012123456 (C) Location 7 1234567890123456789012345678901212345678901234567890123456789012123456 7 1234567890123456789012345678901212345678901234567890123456789012123456 (D) Destiny 1234567890123456789012345678901212345678901234567890123456789012123456 77 1 (E) Ethos 7 12234567890123456789012345678901212345678901234567890123456789012123456 7 1234567890123456789012345678901212345678901234567890123456789012123456 7 1234567890123456789012345678901212345678901234567890123456789012123456 7 1234567890123456789012345678901212345678901234567890123456789012123456 1 34567890123456789012345678901212345678901234567890123456789012123456 7 2 34567890123456789012345678901212345678901234567890123456789012123456 1234567890123456789012345678901212345678901234567890123456789012123456 77 1234567890123456789012345678901212345678901234567890123456789012123456 7 1234567890123456789012345678901212345678901234567890123456789012123456 7 1234567890123456789012345678901212345678901234567890123456789012123456 7 1 7 12234567890123456789012345678901212345678901234567890123456789012123456 1 34567890123456789012345678901212345678901234567890123456789012123456 7 7 12234567890123456789012345678901212345678901234567890123456789012123456 1 34567890123456789012345678901212345678901234567890123456789012123456 7 7 12234567890123456789012345678901212345678901234567890123456789012123456 Do not proceed to the next section until time is up. 7 1 34567890123456789012345678901212345678901234567890123456789012123456 1234567890123456789012345678901212345678901234567890123456789012123456 7 1234567890123456789012345678901212345678901234567890123456789012123456 7 1234567890123456789012345678901212345678901234567890123456789012123456 7 12345678901234567890123456789012123456789012345678901234567890121234567

STOP

Copyright © 2005 Thomson Peterson’s, a part of The Thomson Corporaton SAT is a registered trademark of the College Entrance Examination Board, which was not involved in the production of and does not endorse this product.

29

Section 5 21 Questions j Time—25 Minutes

Reference Information

12345678901234567890123456789012123456789012345678901234567890121234567 34567890123456789012345678901212345678901234567890123456789012123456 7 12 34567890123456789012345678901212345678901234567890123456789012123456 7 12 2 1 34567890123456789012345678901212345678901234567890123456789012123456 7 34567890123456789012345678901212345678901234567890123456789012123456 7 12 Directions: Solve the following problems using any available space on the page for scratchwork. 34567890123456789012345678901212345678901234567890123456789012123456 7 12 34567890123456789012345678901212345678901234567890123456789012123456 7 12 Mark the letter of your choice on the answer sheet that best corresponds to the correct answer. 34567890123456789012345678901212345678901234567890123456789012123456 7 12 2 34567890123456789012345678901212345678901234567890123456789012123456 1234567890123456789012345678901212345678901234567890123456789012123456 77 Notes: 1234567890123456789012345678901212345678901234567890123456789012123456 7 1234567890123456789012345678901212345678901234567890123456789012123456 1. You may use a calculator. All of the numbers used are real numbers. 7 1234567890123456789012345678901212345678901234567890123456789012123456 1234567890123456789012345678901212345678901234567890123456789012123456 77 1234567890123456789012345678901212345678901234567890123456789012123456 2. You may use the figures that accompany the problems to help you find the solution. Unless 7 1 34567890123456789012345678901212345678901234567890123456789012123456 7 12 the instructions say that a figure is not drawn to scale, assume that it has been drawn 34567890123456789012345678901212345678901234567890123456789012123456 7 12 34567890123456789012345678901212345678901234567890123456789012123456 7 12 accurately. Each figure lies in a plane unless the instructions say otherwise. 34567890123456789012345678901212345678901234567890123456789012123456 7 12 34567890123456789012345678901212345678901234567890123456789012123456 7 12 34567890123456789012345678901212345678901234567890123456789012123456 7 12 34567890123456789012345678901212345678901234567890123456789012123456 7 12 34567890123456789012345678901212345678901234567890123456789012123456 7 12 2 7 1234567890123456789012345678901212345678901234567890123456789012123456 2s r 2x 7 1234567890123456789012345678901212345678901234567890123456789012123456 w r s x 7 1234567890123456789012345678901212345678901234567890123456789012123456 c h h h b 7 1234567890123456789012345678901212345678901234567890123456789012123456 7 1 34567890123456789012345678901212345678901234567890123456789012123456 w 2 34567890123456789012345678901212345678901234567890123456789012123456 7 1234567890123456789012345678901212345678901234567890123456789012123456 s b 3x a A 5 pr2 7 1234567890123456789012345678901212345678901234567890123456789012123456 1 2 2 2 2 7 1 h c 5 a 1 b Special Right Triangles V 5 ,wh V 5 pr C 5 2pr A 5 ,w bh A 5 7 1234567890123456789012345678901212345678901234567890123456789012123456 2 34567890123456789012345678901212345678901234567890123456789012123456 7 12 2 34567890123456789012345678901212345678901234567890123456789012123456 7 1234567890123456789012345678901212345678901234567890123456789012123456 The number of degrees of arc in a circle is 360. 7 1234567890123456789012345678901212345678901234567890123456789012123456 The measure in degrees of a straight angle is 180. 7 1 34567890123456789012345678901212345678901234567890123456789012123456 7 12 The sum of the measures in degrees of the angles of a triangle is 180. 34567890123456789012345678901212345678901234567890123456789012123456 7 12 2 34567890123456789012345678901212345678901234567890123456789012123456 1234567890123456789012345678901212345678901234567890123456789012123456 77 1234567890123456789012345678901212345678901234567890123456789012123456 7 1 2x 7 1234567890123456789012345678901212345678901234567890123456789012123456 1. At full capacity, Thompson Paper Factory 2. If Z 5 and Z 5 3, then x 5 7 1234567890123456789012345678901212345678901234567890123456789012123456 5 2 34567890123456789012345678901212345678901234567890123456789012123456 7 1234567890123456789012345678901212345678901234567890123456789012123456 produces 200 sheets of paper per second. 7 1234567890123456789012345678901212345678901234567890123456789012123456 5 7 1234567890123456789012345678901212345678901234567890123456789012123456 If the factory is operating at a quarter of (A) 7 1234567890123456789012345678901212345678901234567890123456789012123456 3 7 1 its full capacity, how many sheets of paper 2 34567890123456789012345678901212345678901234567890123456789012123456 1234567890123456789012345678901212345678901234567890123456789012123456 77 1234567890123456789012345678901212345678901234567890123456789012123456 (B) 2 will the factory produce in twelve sec7 1234567890123456789012345678901212345678901234567890123456789012123456 7 1234567890123456789012345678901212345678901234567890123456789012123456 onds? (C) 5 7 1 2 34567890123456789012345678901212345678901234567890123456789012123456 1234567890123456789012345678901212345678901234567890123456789012123456 77 1234567890123456789012345678901212345678901234567890123456789012123456 (A) 600 19 7 1234567890123456789012345678901212345678901234567890123456789012123456 (D) (B) 900 7 1234567890123456789012345678901212345678901234567890123456789012123456 3 7 1 (C) 1200 2 34567890123456789012345678901212345678901234567890123456789012123456 1234567890123456789012345678901212345678901234567890123456789012123456 77 1 15 (D) 2000 34567890123456789012345678901212345678901234567890123456789012123456 7 12 (E) 7 1234567890123456789012345678901212345678901234567890123456789012123456 2 (E) 2400 34567890123456789012345678901212345678901234567890123456789012123456 7 12 2 34567890123456789012345678901212345678901234567890123456789012123456 7 1 1234567890123456789012345678901212345678901234567890123456789012123456 7 1234567890123456789012345678901212345678901234567890123456789012123456 7 1234567890123456789012345678901212345678901234567890123456789012123456 7 12345678901234567890123456789012123456789012345678901234567890121234567 30

45

60

30

45

Copyright © 2005 Thomson Peterson’s, a part of The Thomson Corporaton SAT is a registered trademark of the College Entrance Examination Board, which was not involved in the production of and does not endorse this product.

12345678901234567890123456789012123456789012345678901234567890121234567 1234567890123456789012345678901212345678901234567890123456789012123456 7 7 12234567890123456789012345678901212345678901234567890123456789012123456 Questions 5–6 refer to the following table. 3. Which of the following is the greatest 7 1 34567890123456789012345678901212345678901234567890123456789012123456 7 12234567890123456789012345678901212345678901234567890123456789012123456 common factor of 32 and 42? 7 1234567890123456789012345678901212345678901234567890123456789012123456 7 1 34567890123456789012345678901212345678901234567890123456789012123456 5. 7 12234567890123456789012345678901212345678901234567890123456789012123456 (A) 2 34567890123456789012345678901212345678901234567890123456789012123456 7 1 7 12234567890123456789012345678901212345678901234567890123456789012123456 (B) 3 7 1234567890123456789012345678901212345678901234567890123456789012123456 7 1234567890123456789012345678901212345678901234567890123456789012123456 (C) 6 7 1234567890123456789012345678901212345678901234567890123456789012123456 34567890123456789012345678901212345678901234567890123456789012123456 7 1 (D) 8 7 12234567890123456789012345678901212345678901234567890123456789012123456 7 1234567890123456789012345678901212345678901234567890123456789012123456 (E) 12 7 1234567890123456789012345678901212345678901234567890123456789012123456 7 1234567890123456789012345678901212345678901234567890123456789012123456 34567890123456789012345678901212345678901234567890123456789012123456 7 1 7 12234567890123456789012345678901212345678901234567890123456789012123456 4. 7 1234567890123456789012345678901212345678901234567890123456789012123456 7 1234567890123456789012345678901212345678901234567890123456789012123456 7 1234567890123456789012345678901212345678901234567890123456789012123456 34567890123456789012345678901212345678901234567890123456789012123456 7 1 According to the table, how many 7 12234567890123456789012345678901212345678901234567890123456789012123456 rd th 7 1234567890123456789012345678901212345678901234567890123456789012123456 and 4 grade at students are there in 3 7 1234567890123456789012345678901212345678901234567890123456789012123456 7 1234567890123456789012345678901212345678901234567890123456789012123456 Hyde Park Elementary? 34567890123456789012345678901212345678901234567890123456789012123456 1234567890123456789012345678901212345678901234567890123456789012123456 77 1234567890123456789012345678901212345678901234567890123456789012123456 7 1234567890123456789012345678901212345678901234567890123456789012123456 (A) 30 7 1234567890123456789012345678901212345678901234567890123456789012123456 (B) 32 7 1234567890123456789012345678901212345678901234567890123456789012123456 7 1234567890123456789012345678901212345678901234567890123456789012123456 (C) 50 7 1 7 12234567890123456789012345678901212345678901234567890123456789012123456 (D) 60 7 1234567890123456789012345678901212345678901234567890123456789012123456 7 1234567890123456789012345678901212345678901234567890123456789012123456 (E) 62 34567890123456789012345678901212345678901234567890123456789012123456 1234567890123456789012345678901212345678901234567890123456789012123456 77 1234567890123456789012345678901212345678901234567890123456789012123456 7 1234567890123456789012345678901212345678901234567890123456789012123456 7 1234567890123456789012345678901212345678901234567890123456789012123456 6. If there are 18 girls in 4th grade, how 7 1234567890123456789012345678901212345678901234567890123456789012123456 rd th 7 1234567890123456789012345678901212345678901234567890123456789012123456 many boys are there in 3 and 4 grade 7 1234567890123456789012345678901212345678901234567890123456789012123456 7 1234567890123456789012345678901212345678901234567890123456789012123456 at Hyde Park Elementary? 1234567890123456789012345678901212345678901234567890123456789012123456 77 1 7 12234567890123456789012345678901212345678901234567890123456789012123456 (A) 18 7 1234567890123456789012345678901212345678901234567890123456789012123456 (B) 22 7 1 34567890123456789012345678901212345678901234567890123456789012123456 What is the slope of the above line? 7 1234567890123456789012345678901212345678901234567890123456789012123456 (C) 26 7 12234567890123456789012345678901212345678901234567890123456789012123456 34567890123456789012345678901212345678901234567890123456789012123456 7 1234567890123456789012345678901212345678901234567890123456789012123456 3 (D) 30 7 1234567890123456789012345678901212345678901234567890123456789012123456 2 A. 7 1 2 (E) 32 7 12234567890123456789012345678901212345678901234567890123456789012123456 7 1234567890123456789012345678901212345678901234567890123456789012123456 2 34567890123456789012345678901212345678901234567890123456789012123456 7 1234567890123456789012345678901212345678901234567890123456789012123456 B. 2 7 1234567890123456789012345678901212345678901234567890123456789012123456 3 7 1 1234567890123456789012345678901212345678901234567890123456789012123456 7 7 1234567890123456789012345678901212345678901234567890123456789012123456 2 7 12234567890123456789012345678901212345678901234567890123456789012123456 C. 7 1234567890123456789012345678901212345678901234567890123456789012123456 3 7 1234567890123456789012345678901212345678901234567890123456789012123456 7 1234567890123456789012345678901212345678901234567890123456789012123456 D. 1 1 34567890123456789012345678901212345678901234567890123456789012123456 7 2 34567890123456789012345678901212345678901234567890123456789012123456 1234567890123456789012345678901212345678901234567890123456789012123456 77 1234567890123456789012345678901212345678901234567890123456789012123456 3 7 1234567890123456789012345678901212345678901234567890123456789012123456 E. 7 1234567890123456789012345678901212345678901234567890123456789012123456 2 7 1 2 34567890123456789012345678901212345678901234567890123456789012123456 1234567890123456789012345678901212345678901234567890123456789012123456 77 1234567890123456789012345678901212345678901234567890123456789012123456 7 1234567890123456789012345678901212345678901234567890123456789012123456 7 1234567890123456789012345678901212345678901234567890123456789012123456 7 1 7 12234567890123456789012345678901212345678901234567890123456789012123456 1 34567890123456789012345678901212345678901234567890123456789012123456 7 7 12234567890123456789012345678901212345678901234567890123456789012123456 1 34567890123456789012345678901212345678901234567890123456789012123456 7 7 12234567890123456789012345678901212345678901234567890123456789012123456 1 34567890123456789012345678901212345678901234567890123456789012123456 7 1234567890123456789012345678901212345678901234567890123456789012123456 7 1234567890123456789012345678901212345678901234567890123456789012123456 7 1234567890123456789012345678901212345678901234567890123456789012123456 7 12345678901234567890123456789012123456789012345678901234567890121234567 Copyright © 2005 Thomson Peterson’s, a part of The Thomson Corporaton SAT is a registered trademark of the College Entrance Examination Board, which was not involved in the production of and does not endorse this product.

31

12345678901234567890123456789012123456789012345678901234567890121234567 7 1234567890123456789012345678901212345678901234567890123456789012123456 3x2 34567890123456789012345678901212345678901234567890123456789012123456 7 12 x 5 , for what values If f 7. Which of the following is the value of the 10. 2 34567890123456789012345678901212345678901234567890123456789012123456 7 1 ~ ! x2 1 3x 2 18 34567890123456789012345678901212345678901234567890123456789012123456 7 12 3 22 34567890123456789012345678901212345678901234567890123456789012123456 7 12 of x is the function undefined? 4 ~ ! m 7 1234567890123456789012345678901212345678901234567890123456789012123456 is exponent when the expression 34567890123456789012345678901212345678901234567890123456789012123456 7 12 5 2 34567890123456789012345678901212345678901234567890123456789012123456 7 1 (A) 0 m4 34567890123456789012345678901212345678901234567890123456789012123456 7 12 34567890123456789012345678901212345678901234567890123456789012123456 7 12 simplified? (B) 23,6 34567890123456789012345678901212345678901234567890123456789012123456 7 12 34567890123456789012345678901212345678901234567890123456789012123456 7 12 (C) 23,3 11 7 1234567890123456789012345678901212345678901234567890123456789012123456 (A) 2 34567890123456789012345678901212345678901234567890123456789012123456 7 12 (D) 2,5 4 34567890123456789012345678901212345678901234567890123456789012123456 7 12 (E) 3,26 34567890123456789012345678901212345678901234567890123456789012123456 7 12 29 34567890123456789012345678901212345678901234567890123456789012123456 7 12 2 2 34567890123456789012345678901212345678901234567890123456789012123456 7 1 (B) 34567890123456789012345678901212345678901234567890123456789012123456 7 16 12 11. 34567890123456789012345678901212345678901234567890123456789012123456 7 12 34567890123456789012345678901212345678901234567890123456789012123456 7 12 3 34567890123456789012345678901212345678901234567890123456789012123456 7 12 2 (C) 2 34567890123456789012345678901212345678901234567890123456789012123456 7 1 4 34567890123456789012345678901212345678901234567890123456789012123456 7 12 34567890123456789012345678901212345678901234567890123456789012123456 7 12 11 34567890123456789012345678901212345678901234567890123456789012123456 7 12 (D) 34567890123456789012345678901212345678901234567890123456789012123456 7 12 4 2 34567890123456789012345678901212345678901234567890123456789012123456 1234567890123456789012345678901212345678901234567890123456789012123456 77 1234567890123456789012345678901212345678901234567890123456789012123456 7 1234567890123456789012345678901212345678901234567890123456789012123456 29 7 1234567890123456789012345678901212345678901234567890123456789012123456 (E) 7 1234567890123456789012345678901212345678901234567890123456789012123456 6 1234567890123456789012345678901212345678901234567890123456789012123456 77 1 34567890123456789012345678901212345678901234567890123456789012123456 7 12 8. 34567890123456789012345678901212345678901234567890123456789012123456 7 12 34567890123456789012345678901212345678901234567890123456789012123456 7 12 2 34567890123456789012345678901212345678901234567890123456789012123456 1234567890123456789012345678901212345678901234567890123456789012123456 77 1234567890123456789012345678901212345678901234567890123456789012123456 7 1234567890123456789012345678901212345678901234567890123456789012123456 If f(x) is graphed above, then f(x) 5 7 1234567890123456789012345678901212345678901234567890123456789012123456 1234567890123456789012345678901212345678901234567890123456789012123456 77 1234567890123456789012345678901212345678901234567890123456789012123456 (A) x2 1 2 7 1234567890123456789012345678901212345678901234567890123456789012123456 7 1234567890123456789012345678901212345678901234567890123456789012123456 2 2 2 (B) x In the right triangle above, what is the 7 1234567890123456789012345678901212345678901234567890123456789012123456 2 7 1 1 2 (C) 2x value of x? 2 34567890123456789012345678901212345678901234567890123456789012123456 7 1234567890123456789012345678901212345678901234567890123456789012123456 2 7 1234567890123456789012345678901212345678901234567890123456789012123456 1 2) (D) 2(x 7 1 (A) 15 2 7 1234567890123456789012345678901212345678901234567890123456789012123456 (E) 2(x 12) 34567890123456789012345678901212345678901234567890123456789012123456 7 12 (B) 16 2 34567890123456789012345678901212345678901234567890123456789012123456 1234567890123456789012345678901212345678901234567890123456789012123456 77 1234567890123456789012345678901212345678901234567890123456789012123456 (C) 18 7 1 12. 34567890123456789012345678901212345678901234567890123456789012123456 7 12 (D) 24 34567890123456789012345678901212345678901234567890123456789012123456 7 12 2 34567890123456789012345678901212345678901234567890123456789012123456 7 1234567890123456789012345678901212345678901234567890123456789012123456 (E) 30 7 1234567890123456789012345678901212345678901234567890123456789012123456 7 1 7 1234567890123456789012345678901212345678901234567890123456789012123456 9. Taking the highway from Easton to 7 1234567890123456789012345678901212345678901234567890123456789012123456 2 34567890123456789012345678901212345678901234567890123456789012123456 7 1234567890123456789012345678901212345678901234567890123456789012123456 Bethsaida is 7 miles longer than taking 7 1234567890123456789012345678901212345678901234567890123456789012123456 7 1234567890123456789012345678901212345678901234567890123456789012123456 surface streets from Easton to Bethsaida. 7 1234567890123456789012345678901212345678901234567890123456789012123456 If the above triangles are congruent, what 7 1 It is 31 miles total if you travel from 2 34567890123456789012345678901212345678901234567890123456789012123456 7 1234567890123456789012345678901212345678901234567890123456789012123456 is the value of y? 7 1234567890123456789012345678901212345678901234567890123456789012123456 Easton to Bethsaida via highway and 7 1234567890123456789012345678901212345678901234567890123456789012123456 7 1234567890123456789012345678901212345678901234567890123456789012123456 (A) 5 return via surface streets. How many miles 7 1 2 34567890123456789012345678901212345678901234567890123456789012123456 7 1234567890123456789012345678901212345678901234567890123456789012123456 then, is the highway route? (B) =50 7 1234567890123456789012345678901212345678901234567890123456789012123456 1234567890123456789012345678901212345678901234567890123456789012123456 77 1234567890123456789012345678901212345678901234567890123456789012123456 (C) 8 (A) 12 7 1 2 34567890123456789012345678901212345678901234567890123456789012123456 7 1234567890123456789012345678901212345678901234567890123456789012123456 (B) 13 (D) =75 7 1 34567890123456789012345678901212345678901234567890123456789012123456 7 12 (C) 15 (E) 9 7 1234567890123456789012345678901212345678901234567890123456789012123456 34567890123456789012345678901212345678901234567890123456789012123456 7 12 (D) 17 2 34567890123456789012345678901212345678901234567890123456789012123456 7 1 7 1234567890123456789012345678901212345678901234567890123456789012123456 (E) 19 1234567890123456789012345678901212345678901234567890123456789012123456 77 1234567890123456789012345678901212345678901234567890123456789012123456 12345678901234567890123456789012123456789012345678901234567890121234567 32

Copyright © 2005 Thomson Peterson’s, a part of The Thomson Corporaton SAT is a registered trademark of the College Entrance Examination Board, which was not involved in the production of and does not endorse this product.

12345678901234567890123456789012123456789012345678901234567890121234567 1234567890123456789012345678901212345678901234567890123456789012123456 7 7 12234567890123456789012345678901212345678901234567890123456789012123456 13. (y 1 2)2 5 (y 2 4)2 is true when y equals (B) 7 1 34567890123456789012345678901212345678901234567890123456789012123456 7 12234567890123456789012345678901212345678901234567890123456789012123456 7 1234567890123456789012345678901212345678901234567890123456789012123456 (A) 1 only 1 34567890123456789012345678901212345678901234567890123456789012123456 7 7 12234567890123456789012345678901212345678901234567890123456789012123456 (B) 1 and 21 7 1 34567890123456789012345678901212345678901234567890123456789012123456 7 12234567890123456789012345678901212345678901234567890123456789012123456 (C) 2 and 22 7 1234567890123456789012345678901212345678901234567890123456789012123456 7 1234567890123456789012345678901212345678901234567890123456789012123456 (D) 1 and 2 7 1234567890123456789012345678901212345678901234567890123456789012123456 34567890123456789012345678901212345678901234567890123456789012123456 7 1 (E) 2 and 4 7 12234567890123456789012345678901212345678901234567890123456789012123456 7 1234567890123456789012345678901212345678901234567890123456789012123456 7 1234567890123456789012345678901212345678901234567890123456789012123456 7 1234567890123456789012345678901212345678901234567890123456789012123456 14. Z is the set of numbers 1 through 50 34567890123456789012345678901212345678901234567890123456789012123456 7 1 7 12234567890123456789012345678901212345678901234567890123456789012123456 inclusive. How many members of Z are 7 1234567890123456789012345678901212345678901234567890123456789012123456 7 evenly divisible by 2 and 3? 1234567890123456789012345678901212345678901234567890123456789012123456 7 1234567890123456789012345678901212345678901234567890123456789012123456 (C) 34567890123456789012345678901212345678901234567890123456789012123456 7 1 (A) 6 7 12234567890123456789012345678901212345678901234567890123456789012123456 7 1234567890123456789012345678901212345678901234567890123456789012123456 (B) 8 7 1234567890123456789012345678901212345678901234567890123456789012123456 7 1234567890123456789012345678901212345678901234567890123456789012123456 (C) 14 34567890123456789012345678901212345678901234567890123456789012123456 1234567890123456789012345678901212345678901234567890123456789012123456 77 1234567890123456789012345678901212345678901234567890123456789012123456 (D) 16 7 1234567890123456789012345678901212345678901234567890123456789012123456 7 1234567890123456789012345678901212345678901234567890123456789012123456 (E) 25 1234567890123456789012345678901212345678901234567890123456789012123456 77 1234567890123456789012345678901212345678901234567890123456789012123456 7 1 7 12234567890123456789012345678901212345678901234567890123456789012123456 15. 7 1234567890123456789012345678901212345678901234567890123456789012123456 7 1234567890123456789012345678901212345678901234567890123456789012123456 34567890123456789012345678901212345678901234567890123456789012123456 1234567890123456789012345678901212345678901234567890123456789012123456 77 1234567890123456789012345678901212345678901234567890123456789012123456 7 1234567890123456789012345678901212345678901234567890123456789012123456 7 1234567890123456789012345678901212345678901234567890123456789012123456 7 1234567890123456789012345678901212345678901234567890123456789012123456 7 1234567890123456789012345678901212345678901234567890123456789012123456 (D) 7 1234567890123456789012345678901212345678901234567890123456789012123456 1234567890123456789012345678901212345678901234567890123456789012123456 77 1234567890123456789012345678901212345678901234567890123456789012123456 7 1 7 12234567890123456789012345678901212345678901234567890123456789012123456 7 1234567890123456789012345678901212345678901234567890123456789012123456 1 34567890123456789012345678901212345678901234567890123456789012123456 7 1234567890123456789012345678901212345678901234567890123456789012123456 7 7 12234567890123456789012345678901212345678901234567890123456789012123456 7 1234567890123456789012345678901212345678901234567890123456789012123456 7 1234567890123456789012345678901212345678901234567890123456789012123456 1 34567890123456789012345678901212345678901234567890123456789012123456 7 7 12234567890123456789012345678901212345678901234567890123456789012123456 7 1234567890123456789012345678901212345678901234567890123456789012123456 Which of the following figures is similar 34567890123456789012345678901212345678901234567890123456789012123456 1234567890123456789012345678901212345678901234567890123456789012123456 77 1234567890123456789012345678901212345678901234567890123456789012123456 to the shape shown above? 7 1 1234567890123456789012345678901212345678901234567890123456789012123456 7 7 1234567890123456789012345678901212345678901234567890123456789012123456 (A) 7 12234567890123456789012345678901212345678901234567890123456789012123456 7 1234567890123456789012345678901212345678901234567890123456789012123456 (E) 7 1234567890123456789012345678901212345678901234567890123456789012123456 7 1234567890123456789012345678901212345678901234567890123456789012123456 1 34567890123456789012345678901212345678901234567890123456789012123456 7 2 34567890123456789012345678901212345678901234567890123456789012123456 1234567890123456789012345678901212345678901234567890123456789012123456 77 1234567890123456789012345678901212345678901234567890123456789012123456 7 1234567890123456789012345678901212345678901234567890123456789012123456 7 1234567890123456789012345678901212345678901234567890123456789012123456 7 1 7 12234567890123456789012345678901212345678901234567890123456789012123456 7 1234567890123456789012345678901212345678901234567890123456789012123456 7 1234567890123456789012345678901212345678901234567890123456789012123456 7 1234567890123456789012345678901212345678901234567890123456789012123456 1 34567890123456789012345678901212345678901234567890123456789012123456 7 2 34567890123456789012345678901212345678901234567890123456789012123456 1234567890123456789012345678901212345678901234567890123456789012123456 77 1 7 12234567890123456789012345678901212345678901234567890123456789012123456 1 34567890123456789012345678901212345678901234567890123456789012123456 7 7 12234567890123456789012345678901212345678901234567890123456789012123456 1 34567890123456789012345678901212345678901234567890123456789012123456 7 1234567890123456789012345678901212345678901234567890123456789012123456 7 1234567890123456789012345678901212345678901234567890123456789012123456 7 1234567890123456789012345678901212345678901234567890123456789012123456 7 12345678901234567890123456789012123456789012345678901234567890121234567 Copyright © 2005 Thomson Peterson’s, a part of The Thomson Corporaton SAT is a registered trademark of the College Entrance Examination Board, which was not involved in the production of and does not endorse this product.

33

12345678901234567890123456789012123456789012345678901234567890121234567 1234567890123456789012345678901212345678901234567890123456789012123456 7 34567890123456789012345678901212345678901234567890123456789012123456 7 12 16. 18. 7 1234567890123456789012345678901212345678901234567890123456789012123456 34567890123456789012345678901212345678901234567890123456789012123456 7 12 34567890123456789012345678901212345678901234567890123456789012123456 7 12 1234567890123456789012345678901212345678901234567890123456789012123456 7 34567890123456789012345678901212345678901234567890123456789012123456 7 12 1234567890123456789012345678901212345678901234567890123456789012123456 7 34567890123456789012345678901212345678901234567890123456789012123456 7 12 34567890123456789012345678901212345678901234567890123456789012123456 7 12 34567890123456789012345678901212345678901234567890123456789012123456 7 12 34567890123456789012345678901212345678901234567890123456789012123456 7 12 2 1 34567890123456789012345678901212345678901234567890123456789012123456 7 34567890123456789012345678901212345678901234567890123456789012123456 7 12 34567890123456789012345678901212345678901234567890123456789012123456 7 12 ABC is an equilateral triangle and DEFG 34567890123456789012345678901212345678901234567890123456789012123456 7 12 34567890123456789012345678901212345678901234567890123456789012123456 7 12 AB 5 DE, how many is a square. If 2 34567890123456789012345678901212345678901234567890123456789012123456 7 1 34567890123456789012345678901212345678901234567890123456789012123456 7 12 different ways can ABC be placed in 34567890123456789012345678901212345678901234567890123456789012123456 7 12 34567890123456789012345678901212345678901234567890123456789012123456 7 12 DEFG such that two vertices of the 34567890123456789012345678901212345678901234567890123456789012123456 7 12 What is the area of the above figure? 2 34567890123456789012345678901212345678901234567890123456789012123456 7 1 triangle coincide with two corners of 34567890123456789012345678901212345678901234567890123456789012123456 7 12 34567890123456789012345678901212345678901234567890123456789012123456 7 12 the square? (A) 1 34567890123456789012345678901212345678901234567890123456789012123456 7 12 34567890123456789012345678901212345678901234567890123456789012123456 7 12 (A) 4 34567890123456789012345678901212345678901234567890123456789012123456 7 12 (B) =2 34567890123456789012345678901212345678901234567890123456789012123456 7 12 (B) 6 34567890123456789012345678901212345678901234567890123456789012123456 7 12 (C) 2 34567890123456789012345678901212345678901234567890123456789012123456 7 12 (C) 8 34567890123456789012345678901212345678901234567890123456789012123456 7 12 2 1 (D) = 2 34567890123456789012345678901212345678901234567890123456789012123456 7 1234567890123456789012345678901212345678901234567890123456789012123456 (D) 10 7 1 34567890123456789012345678901212345678901234567890123456789012123456 7 12 (E) 4 (E) 12 34567890123456789012345678901212345678901234567890123456789012123456 7 12 34567890123456789012345678901212345678901234567890123456789012123456 7 12 34567890123456789012345678901212345678901234567890123456789012123456 7 12 17. The sum of eight positive even integers is 34567890123456789012345678901212345678901234567890123456789012123456 7 12 19. G, S, and T are three points that lie on a 34567890123456789012345678901212345678901234567890123456789012123456 7 12 50. If no integer can appear more than 34567890123456789012345678901212345678901234567890123456789012123456 7 12 plane. If the distance between G and S is 34567890123456789012345678901212345678901234567890123456789012123456 7 12 twice in the set, what is the greatest 2 34567890123456789012345678901212345678901234567890123456789012123456 7 1234567890123456789012345678901212345678901234567890123456789012123456 9, and the distance between S and T is 5, 7 1234567890123456789012345678901212345678901234567890123456789012123456 possible value of one of the integers. 7 1234567890123456789012345678901212345678901234567890123456789012123456 which of the following are possible 7 1234567890123456789012345678901212345678901234567890123456789012123456 distances between G and T? 7 1 (A) 8 2 34567890123456789012345678901212345678901234567890123456789012123456 1234567890123456789012345678901212345678901234567890123456789012123456 77 1234567890123456789012345678901212345678901234567890123456789012123456 (B) 18 I. 3 7 1 7 1234567890123456789012345678901212345678901234567890123456789012123456 (C) 22 II. 5 34567890123456789012345678901212345678901234567890123456789012123456 7 12 2 34567890123456789012345678901212345678901234567890123456789012123456 7 1234567890123456789012345678901212345678901234567890123456789012123456 (D) 24 III. 14 7 1234567890123456789012345678901212345678901234567890123456789012123456 7 1 (E) 32 34567890123456789012345678901212345678901234567890123456789012123456 7 12 (A) I only 34567890123456789012345678901212345678901234567890123456789012123456 7 12 2 34567890123456789012345678901212345678901234567890123456789012123456 7 1234567890123456789012345678901212345678901234567890123456789012123456 (B) II only 7 1234567890123456789012345678901212345678901234567890123456789012123456 7 1 (C) I and II only 1234567890123456789012345678901212345678901234567890123456789012123456 7 7 1234567890123456789012345678901212345678901234567890123456789012123456 (D) II and III only 34567890123456789012345678901212345678901234567890123456789012123456 7 12 34567890123456789012345678901212345678901234567890123456789012123456 7 12 (E) I, II, and III 34567890123456789012345678901212345678901234567890123456789012123456 7 12 34567890123456789012345678901212345678901234567890123456789012123456 7 12 1234567890123456789012345678901212345678901234567890123456789012123456 7 2 7 1234567890123456789012345678901212345678901234567890123456789012123456 7 1234567890123456789012345678901212345678901234567890123456789012123456 7 1234567890123456789012345678901212345678901234567890123456789012123456 7 1234567890123456789012345678901212345678901234567890123456789012123456 1 34567890123456789012345678901212345678901234567890123456789012123456 7 2 34567890123456789012345678901212345678901234567890123456789012123456 1234567890123456789012345678901212345678901234567890123456789012123456 77 1234567890123456789012345678901212345678901234567890123456789012123456 7 1234567890123456789012345678901212345678901234567890123456789012123456 7 1234567890123456789012345678901212345678901234567890123456789012123456 7 1 34567890123456789012345678901212345678901234567890123456789012123456 7 12 1234567890123456789012345678901212345678901234567890123456789012123456 7 34567890123456789012345678901212345678901234567890123456789012123456 7 12 1234567890123456789012345678901212345678901234567890123456789012123456 7 34567890123456789012345678901212345678901234567890123456789012123456 7 12 1234567890123456789012345678901212345678901234567890123456789012123456 7 1234567890123456789012345678901212345678901234567890123456789012123456 7 1234567890123456789012345678901212345678901234567890123456789012123456 7 1234567890123456789012345678901212345678901234567890123456789012123456 7 12345678901234567890123456789012123456789012345678901234567890121234567 34

Copyright © 2005 Thomson Peterson’s, a part of The Thomson Corporaton SAT is a registered trademark of the College Entrance Examination Board, which was not involved in the production of and does not endorse this product.

12345678901234567890123456789012123456789012345678901234567890121234567 1234567890123456789012345678901212345678901234567890123456789012123456 7 7 12234567890123456789012345678901212345678901234567890123456789012123456 21. 20. 7 1 34567890123456789012345678901212345678901234567890123456789012123456 7 12234567890123456789012345678901212345678901234567890123456789012123456 7 1234567890123456789012345678901212345678901234567890123456789012123456 1 34567890123456789012345678901212345678901234567890123456789012123456 7 7 12234567890123456789012345678901212345678901234567890123456789012123456 1 34567890123456789012345678901212345678901234567890123456789012123456 7 7 12234567890123456789012345678901212345678901234567890123456789012123456 7 1234567890123456789012345678901212345678901234567890123456789012123456 7 1234567890123456789012345678901212345678901234567890123456789012123456 7 1234567890123456789012345678901212345678901234567890123456789012123456 34567890123456789012345678901212345678901234567890123456789012123456 7 1 7 12234567890123456789012345678901212345678901234567890123456789012123456 7 1234567890123456789012345678901212345678901234567890123456789012123456 7 1234567890123456789012345678901212345678901234567890123456789012123456 7 1234567890123456789012345678901212345678901234567890123456789012123456 34567890123456789012345678901212345678901234567890123456789012123456 7 1 7 12234567890123456789012345678901212345678901234567890123456789012123456 7 1234567890123456789012345678901212345678901234567890123456789012123456 If AB 5 BC 5 AC 5 6, and D is the 7 1234567890123456789012345678901212345678901234567890123456789012123456 7 1234567890123456789012345678901212345678901234567890123456789012123456 BD 5 halfway between A and C, then 34567890123456789012345678901212345678901234567890123456789012123456 7 1 7 12234567890123456789012345678901212345678901234567890123456789012123456 7 1234567890123456789012345678901212345678901234567890123456789012123456 3 2 (A) = 7 1234567890123456789012345678901212345678901234567890123456789012123456 7 1234567890123456789012345678901212345678901234567890123456789012123456 (B) 3 34567890123456789012345678901212345678901234567890123456789012123456 1234567890123456789012345678901212345678901234567890123456789012123456 77 1234567890123456789012345678901212345678901234567890123456789012123456 7 1234567890123456789012345678901212345678901234567890123456789012123456 (C) 3=3 7 1234567890123456789012345678901212345678901234567890123456789012123456 7 1234567890123456789012345678901212345678901234567890123456789012123456 The figure above is the diagram of an 2 4 (D) = 1234567890123456789012345678901212345678901234567890123456789012123456 77 1 industrial fan blade. If the fan’s maximum 7 12234567890123456789012345678901212345678901234567890123456789012123456 (E) 4=3 7 1234567890123456789012345678901212345678901234567890123456789012123456 blade speed is 100 revolutions per 10 7 1234567890123456789012345678901212345678901234567890123456789012123456 34567890123456789012345678901212345678901234567890123456789012123456 7 1234567890123456789012345678901212345678901234567890123456789012123456 seconds, what is the greatest distance (in 7 1234567890123456789012345678901212345678901234567890123456789012123456 7 1234567890123456789012345678901212345678901234567890123456789012123456 feet) that any point on the blade could 1234567890123456789012345678901212345678901234567890123456789012123456 77 1234567890123456789012345678901212345678901234567890123456789012123456 travel in 30 seconds? 7 1234567890123456789012345678901212345678901234567890123456789012123456 1234567890123456789012345678901212345678901234567890123456789012123456 77 1234567890123456789012345678901212345678901234567890123456789012123456 (A) 100=2p 7 1234567890123456789012345678901212345678901234567890123456789012123456 7 1 3p 200 (B) = 2 34567890123456789012345678901212345678901234567890123456789012123456 1234567890123456789012345678901212345678901234567890123456789012123456 77 1234567890123456789012345678901212345678901234567890123456789012123456 7 1 (C) 600=2p 7 1234567890123456789012345678901212345678901234567890123456789012123456 7 12234567890123456789012345678901212345678901234567890123456789012123456 3p 600 (D) = 34567890123456789012345678901212345678901234567890123456789012123456 1234567890123456789012345678901212345678901234567890123456789012123456 77 1234567890123456789012345678901212345678901234567890123456789012123456 (E) 1200=2p 7 1 7 12234567890123456789012345678901212345678901234567890123456789012123456 7 1234567890123456789012345678901212345678901234567890123456789012123456 34567890123456789012345678901212345678901234567890123456789012123456 1234567890123456789012345678901212345678901234567890123456789012123456 77 1234567890123456789012345678901212345678901234567890123456789012123456 7 1 1234567890123456789012345678901212345678901234567890123456789012123456 7 1234567890123456789012345678901212345678901234567890123456789012123456 7 7 12234567890123456789012345678901212345678901234567890123456789012123456 7 1234567890123456789012345678901212345678901234567890123456789012123456 7 1234567890123456789012345678901212345678901234567890123456789012123456 7 1234567890123456789012345678901212345678901234567890123456789012123456 1 34567890123456789012345678901212345678901234567890123456789012123456 7 2 34567890123456789012345678901212345678901234567890123456789012123456 1234567890123456789012345678901212345678901234567890123456789012123456 77 1234567890123456789012345678901212345678901234567890123456789012123456 7 1234567890123456789012345678901212345678901234567890123456789012123456 7 1234567890123456789012345678901212345678901234567890123456789012123456 7 1 7 12234567890123456789012345678901212345678901234567890123456789012123456 7 1234567890123456789012345678901212345678901234567890123456789012123456 7 1234567890123456789012345678901212345678901234567890123456789012123456 7 1234567890123456789012345678901212345678901234567890123456789012123456 1 34567890123456789012345678901212345678901234567890123456789012123456 7 2 34567890123456789012345678901212345678901234567890123456789012123456 1234567890123456789012345678901212345678901234567890123456789012123456 77 1 7 12234567890123456789012345678901212345678901234567890123456789012123456 1 34567890123456789012345678901212345678901234567890123456789012123456 7 7 12234567890123456789012345678901212345678901234567890123456789012123456 Do not proceed to the next section until time is up. 7 1 34567890123456789012345678901212345678901234567890123456789012123456 1234567890123456789012345678901212345678901234567890123456789012123456 7 1234567890123456789012345678901212345678901234567890123456789012123456 7 1234567890123456789012345678901212345678901234567890123456789012123456 7 12345678901234567890123456789012123456789012345678901234567890121234567

STOP

Copyright © 2005 Thomson Peterson’s, a part of The Thomson Corporaton SAT is a registered trademark of the College Entrance Examination Board, which was not involved in the production of and does not endorse this product.

35

Section 6 16 Questions j Time—20 Minutes 12345678901234567890123456789012123456789012345678901234567890121234567 34567890123456789012345678901212345678901234567890123456789012123456 7 12 34567890123456789012345678901212345678901234567890123456789012123456 7 12 2 1 34567890123456789012345678901212345678901234567890123456789012123456 7 34567890123456789012345678901212345678901234567890123456789012123456 7 12 Directions: Each passage below is followed by a set of questions. Read each passage, then 34567890123456789012345678901212345678901234567890123456789012123456 7 12 34567890123456789012345678901212345678901234567890123456789012123456 7 12 answer the accompanying questions, basing your answers on what is stated or implied in the 34567890123456789012345678901212345678901234567890123456789012123456 7 12 2 34567890123456789012345678901212345678901234567890123456789012123456 7 1234567890123456789012345678901212345678901234567890123456789012123456 passage and any introductory material provided. Mark the letter of your choice on the answer 7 1234567890123456789012345678901212345678901234567890123456789012123456 7 1234567890123456789012345678901212345678901234567890123456789012123456 sheet that best corresponds to the correct answer. 1234567890123456789012345678901212345678901234567890123456789012123456 77 1234567890123456789012345678901212345678901234567890123456789012123456 7 1234567890123456789012345678901212345678901234567890123456789012123456 7 1 34567890123456789012345678901212345678901234567890123456789012123456 7 12 34567890123456789012345678901212345678901234567890123456789012123456 7 12 Line Frederic Remington (1861–1909) has Line The question of what counts as literature 34567890123456789012345678901212345678901234567890123456789012123456 7 12 2 34567890123456789012345678901212345678901234567890123456789012123456 7 1234567890123456789012345678901212345678901234567890123456789012123456 long been celebrated as one of the most has been strongly debated over the last 7 1234567890123456789012345678901212345678901234567890123456789012123456 gifted interpreters of the American West. few decades both in and out of academia. 7 1234567890123456789012345678901212345678901234567890123456789012123456 7 1234567890123456789012345678901212345678901234567890123456789012123456 Initially, his western images appeared as Some argue that only the test of time 7 1234567890123456789012345678901212345678901234567890123456789012123456 7 1234567890123456789012345678901212345678901234567890123456789012123456 (5) (5) illustrations in popular journals. As he ultimately vindicates a fictional work’s 7 1234567890123456789012345678901212345678901234567890123456789012123456 7 1234567890123456789012345678901212345678901234567890123456789012123456 matured, however, Remington turned his claim to the status of literature. Their 7 1234567890123456789012345678901212345678901234567890123456789012123456 7 1 attention away from illustration, argument runs like this: if people still 2 34567890123456789012345678901212345678901234567890123456789012123456 1234567890123456789012345678901212345678901234567890123456789012123456 77 1234567890123456789012345678901212345678901234567890123456789012123456 concentrating instead on painting and read, still reference, still care about a 7 1 7 1234567890123456789012345678901212345678901234567890123456789012123456 sculpture. About 1900 he began a series work of fiction decades or even centuries 34567890123456789012345678901212345678901234567890123456789012123456 7 12 2 34567890123456789012345678901212345678901234567890123456789012123456 7 1234567890123456789012345678901212345678901234567890123456789012123456 (10) (10) of paintings that took as their subject the after its original publication, then that 7 1234567890123456789012345678901212345678901234567890123456789012123456 7 1 color of night. Before his premature work clearly rises to the auspicious status 34567890123456789012345678901212345678901234567890123456789012123456 7 12 34567890123456789012345678901212345678901234567890123456789012123456 12 death in 1909 at age 48, Remington of literature. Critics of this view, though, 7 34567890123456789012345678901212345678901234567890123456789012123456 7 12 34567890123456789012345678901212345678901234567890123456789012123456 7 12 completed more than seventy paintings in point out that this method of determining 1234567890123456789012345678901212345678901234567890123456789012123456 7 1234567890123456789012345678901212345678901234567890123456789012123456 which he explored the technical and what is and is not literature by definition 7 7 1234567890123456789012345678901212345678901234567890123456789012123456 2 34567890123456789012345678901212345678901234567890123456789012123456 7 1234567890123456789012345678901212345678901234567890123456789012123456 (15) (15) aesthetic difficulties of painting darkness. excludes contemporary works from 7 1234567890123456789012345678901212345678901234567890123456789012123456 7 1234567890123456789012345678901212345678901234567890123456789012123456 consideration. We do not know, they 1234567890123456789012345678901212345678901234567890123456789012123456 77 1 1. The passage suggests Remington’s major rightfully contend, if a novel published in 34567890123456789012345678901212345678901234567890123456789012123456 7 12 34567890123456789012345678901212345678901234567890123456789012123456 7 12 the last few years will be read in a artistic accomplishments were 34567890123456789012345678901212345678901234567890123456789012123456 7 12 34567890123456789012345678901212345678901234567890123456789012123456 7 12 hundred years or not. And so they ask, 7 1234567890123456789012345678901212345678901234567890123456789012123456 (A) magazine illustrations. (20) does this mean we cannot meaningfully 2 34567890123456789012345678901212345678901234567890123456789012123456 1234567890123456789012345678901212345678901234567890123456789012123456 77 1234567890123456789012345678901212345678901234567890123456789012123456 (B) sculptures. discuss whether the work is important, or 7 1234567890123456789012345678901212345678901234567890123456789012123456 7 1234567890123456789012345678901212345678901234567890123456789012123456 (C) paintings of nocturnal cityscapes. influential, or of great merit? 7 1 2 34567890123456789012345678901212345678901234567890123456789012123456 7 1234567890123456789012345678901212345678901234567890123456789012123456 (D) paintings of nocturnal landscapes. 7 1 34567890123456789012345678901212345678901234567890123456789012123456 7 12 (E) color studies. 1234567890123456789012345678901212345678901234567890123456789012123456 7 34567890123456789012345678901212345678901234567890123456789012123456 7 12 1234567890123456789012345678901212345678901234567890123456789012123456 7 1234567890123456789012345678901212345678901234567890123456789012123456 7 1234567890123456789012345678901212345678901234567890123456789012123456 7 1234567890123456789012345678901212345678901234567890123456789012123456 7 12345678901234567890123456789012123456789012345678901234567890121234567 36

Copyright © 2005 Thomson Peterson’s, a part of The Thomson Corporaton SAT is a registered trademark of the College Entrance Examination Board, which was not involved in the production of and does not endorse this product.

12345678901234567890123456789012123456789012345678901234567890121234567 1234567890123456789012345678901212345678901234567890123456789012123456 7 7 12234567890123456789012345678901212345678901234567890123456789012123456 2. The author uses the word “vindicates” to these were considerations that led the 7 1 34567890123456789012345678901212345678901234567890123456789012123456 7 12234567890123456789012345678901212345678901234567890123456789012123456 emphasize that National Aeronautics and Space Admin- 7 1234567890123456789012345678901212345678901234567890123456789012123456 7 1 34567890123456789012345678901212345678901234567890123456789012123456 istration to Mars. Project Viking’s goal, 7 12234567890123456789012345678901212345678901234567890123456789012123456 (A) all works of fiction claim to be 34567890123456789012345678901212345678901234567890123456789012123456 7 1 (20) after making a soft landing on Mars, was 7 12234567890123456789012345678901212345678901234567890123456789012123456 literature. 1234567890123456789012345678901212345678901234567890123456789012123456 to execute a set of scientific investigations 7 7 1234567890123456789012345678901212345678901234567890123456789012123456 (B) all works of fiction are, in some 7 1234567890123456789012345678901212345678901234567890123456789012123456 that would not only provide data on the 34567890123456789012345678901212345678901234567890123456789012123456 7 1 sense, literature. 7 12234567890123456789012345678901212345678901234567890123456789012123456 physical nature of the planet but also 7 1234567890123456789012345678901212345678901234567890123456789012123456 (C) literature is a much more prestigious 7 1234567890123456789012345678901212345678901234567890123456789012123456 make a first attempt at determining if 7 1234567890123456789012345678901212345678901234567890123456789012123456 category than fiction. 34567890123456789012345678901212345678901234567890123456789012123456 7 1 (25) detectable life forms were present. 7 12234567890123456789012345678901212345678901234567890123456789012123456 (D) the debate regarding what is litera7 1234567890123456789012345678901212345678901234567890123456789012123456 7 1234567890123456789012345678901212345678901234567890123456789012123456 ture is excessively erudite. 7 1234567890123456789012345678901212345678901234567890123456789012123456 4. Which of the following does the para34567890123456789012345678901212345678901234567890123456789012123456 7 1 (E) for a work to establish itself as 7 12234567890123456789012345678901212345678901234567890123456789012123456 graph most emphasize as the motivation 7 1234567890123456789012345678901212345678901234567890123456789012123456 literature is an incredible feat. 7 1234567890123456789012345678901212345678901234567890123456789012123456 for the Viking trip to Mars? 7 1234567890123456789012345678901212345678901234567890123456789012123456 34567890123456789012345678901212345678901234567890123456789012123456 7 1234567890123456789012345678901212345678901234567890123456789012123456 3. The argument, given in the passage, 7 1234567890123456789012345678901212345678901234567890123456789012123456 (A) Fascination with chemistry on 7 1234567890123456789012345678901212345678901234567890123456789012123456 against the “test of time” approach is that 7 1234567890123456789012345678901212345678901234567890123456789012123456 another planet. 1234567890123456789012345678901212345678901234567890123456789012123456 77 1234567890123456789012345678901212345678901234567890123456789012123456 (A) it excludes by definition all writing (B) Four-century-old interest in the planet. 7 1 7 12234567890123456789012345678901212345678901234567890123456789012123456 that is not fictional. (C) The advancement of space explora7 1234567890123456789012345678901212345678901234567890123456789012123456 7 1234567890123456789012345678901212345678901234567890123456789012123456 (B) it does not take trends in critical tion. 34567890123456789012345678901212345678901234567890123456789012123456 1234567890123456789012345678901212345678901234567890123456789012123456 77 1234567890123456789012345678901212345678901234567890123456789012123456 interest into account. (D) Possibility of a space station on Mars. 7 1234567890123456789012345678901212345678901234567890123456789012123456 7 1234567890123456789012345678901212345678901234567890123456789012123456 (C) it excludes contemporary fiction (E) Possibility of life on Mars. 7 1234567890123456789012345678901212345678901234567890123456789012123456 7 1234567890123456789012345678901212345678901234567890123456789012123456 from the discussion. 7 1234567890123456789012345678901212345678901234567890123456789012123456 7 1234567890123456789012345678901212345678901234567890123456789012123456 (D) it allows contemporary works to be Questions 5–16 are based on the following two 1234567890123456789012345678901212345678901234567890123456789012123456 77 1 considered alongside the great works passages. 7 12234567890123456789012345678901212345678901234567890123456789012123456 7 of centuries past. 1234567890123456789012345678901212345678901234567890123456789012123456 The following two passages discuss the English 7 1 34567890123456789012345678901212345678901234567890123456789012123456 (E) it gives too much weight to popular 7 1234567890123456789012345678901212345678901234567890123456789012123456 and Metric systems of measurement. 7 12234567890123456789012345678901212345678901234567890123456789012123456 opinion. 34567890123456789012345678901212345678901234567890123456789012123456 1234567890123456789012345678901212345678901234567890123456789012123456 77 1234567890123456789012345678901212345678901234567890123456789012123456 7 1 Line Since the sixteenth century, astronomers Passage 1 7 12234567890123456789012345678901212345678901234567890123456789012123456 7 1234567890123456789012345678901212345678901234567890123456789012123456 have recognized Mars for what it is—a 34567890123456789012345678901212345678901234567890123456789012123456 7 1234567890123456789012345678901212345678901234567890123456789012123456 Line It is an oft-repeated tale that the English 7 1234567890123456789012345678901212345678901234567890123456789012123456 relatively nearby planet not so unlike our 7 1 measurement of the yard was standard7 1234567890123456789012345678901212345678901234567890123456789012123456 own. The fourth planet from the sun and 1234567890123456789012345678901212345678901234567890123456789012123456 ized when an English royal stepped into 7 2 34567890123456789012345678901212345678901234567890123456789012123456 7 1234567890123456789012345678901212345678901234567890123456789012123456 (5) Earth’s closest neighbor, Mars has been disputes about the measurement’s length 7 1234567890123456789012345678901212345678901234567890123456789012123456 7 1234567890123456789012345678901212345678901234567890123456789012123456 the subject of modern scientists’ careful (5) and declared the distance from his 7 1234567890123456789012345678901212345678901234567890123456789012123456 7 1 scrutiny with powerful telescopes, deep shoulder to the tip of his fingers as the 2 34567890123456789012345678901212345678901234567890123456789012123456 1234567890123456789012345678901212345678901234567890123456789012123456 77 1234567890123456789012345678901212345678901234567890123456789012123456 space probes, and orbiting spacecraft. In standard yard. Unlike many colorful 7 1234567890123456789012345678901212345678901234567890123456789012123456 7 1234567890123456789012345678901212345678901234567890123456789012123456 1976, Earth-bound scientists were anecdotes from history, this one is true. 7 1 2 34567890123456789012345678901212345678901234567890123456789012123456 7 1234567890123456789012345678901212345678901234567890123456789012123456 (10) brought significantly closer to their Early in the twelfth century, the English 7 1234567890123456789012345678901212345678901234567890123456789012123456 7 1234567890123456789012345678901212345678901234567890123456789012123456 subject of investigation when two Viking (10) king Henry I established the length of the 1234567890123456789012345678901212345678901234567890123456789012123456 77 probes touched down on that red soil. 1 yard as the distance from the tip of his 7 12234567890123456789012345678901212345678901234567890123456789012123456 The possibility of life on Mars, clues to 7 1 34567890123456789012345678901212345678901234567890123456789012123456 nose to the tip of his outstretched thumb. 7 12234567890123456789012345678901212345678901234567890123456789012123456 the evolution of the solar system, 7 1 34567890123456789012345678901212345678901234567890123456789012123456 In our scientific age such stories seem 7 12234567890123456789012345678901212345678901234567890123456789012123456 (15) fascination with the chemistry, geology, 34567890123456789012345678901212345678901234567890123456789012123456 7 1 earthy at best and ridiculous at worst. 7 1234567890123456789012345678901212345678901234567890123456789012123456 and meteorology of another planet— 7 1234567890123456789012345678901212345678901234567890123456789012123456 1234567890123456789012345678901212345678901234567890123456789012123456 7 12345678901234567890123456789012123456789012345678901234567890121234567 Copyright © 2005 Thomson Peterson’s, a part of The Thomson Corporaton SAT is a registered trademark of the College Entrance Examination Board, which was not involved in the production of and does not endorse this product.

37

12345678901234567890123456789012123456789012345678901234567890121234567 1234567890123456789012345678901212345678901234567890123456789012123456 7 34567890123456789012345678901212345678901234567890123456789012123456 7 12 (15) But not all ancient units of measure have 7 manageable size, it was defined as one 1234567890123456789012345678901212345678901234567890123456789012123456 34567890123456789012345678901212345678901234567890123456789012123456 7 12 such arbitrary origins. The mile is a good one-forty-millionth of the Earth’s 34567890123456789012345678901212345678901234567890123456789012123456 7 12 7 1234567890123456789012345678901212345678901234567890123456789012123456 example of this. Though the mile is today circumference. They employed the word 34567890123456789012345678901212345678901234567890123456789012123456 7 12 2 34567890123456789012345678901212345678901234567890123456789012123456 7 1 counted as part of the English system of meter to harken back to the ancient 34567890123456789012345678901212345678901234567890123456789012123456 7 12 34567890123456789012345678901212345678901234567890123456789012123456 7 12 (60) measurement, the unit dates back to Greek word metron, meaning measure. 34567890123456789012345678901212345678901234567890123456789012123456 7 12 34567890123456789012345678901212345678901234567890123456789012123456 7 12 (20) ancient Rome. The English word mile The rest of the metric system is even 7 1234567890123456789012345678901212345678901234567890123456789012123456 34567890123456789012345678901212345678901234567890123456789012123456 7 12 derives from the Latin term mille, which less arbitrary in origin. The other metric 34567890123456789012345678901212345678901234567890123456789012123456 7 12 34567890123456789012345678901212345678901234567890123456789012123456 7 12 means one thousand. For the Romans, units of length were generated by either 34567890123456789012345678901212345678901234567890123456789012123456 7 12 2 34567890123456789012345678901212345678901234567890123456789012123456 7 1 the mille was one thousand paces. A pace multiplying or dividing the meter by a 34567890123456789012345678901212345678901234567890123456789012123456 7 12 34567890123456789012345678901212345678901234567890123456789012123456 7 12 (65) was two steps, or five feet roughly. This factor of ten. Thus a kilometer is 1000 34567890123456789012345678901212345678901234567890123456789012123456 7 12 34567890123456789012345678901212345678901234567890123456789012123456 7 12 (25) meant the mille was 5,000 feet. In meters, and a centimeter is one one7 1234567890123456789012345678901212345678901234567890123456789012123456 34567890123456789012345678901212345678901234567890123456789012123456 7 12 medieval Europe, however, the 220-yard hundredths of a meter. It is the great asset 34567890123456789012345678901212345678901234567890123456789012123456 7 12 34567890123456789012345678901212345678901234567890123456789012123456 7 12 furlong became the dominant measureof the metric system, at least for scien34567890123456789012345678901212345678901234567890123456789012123456 7 12 2 34567890123456789012345678901212345678901234567890123456789012123456 1234567890123456789012345678901212345678901234567890123456789012123456 ment used. To reconcile the two measuretists, that units for measuring weight and 7 7 1234567890123456789012345678901212345678901234567890123456789012123456 7 1234567890123456789012345678901212345678901234567890123456789012123456 (70) ments, the mile was lengthened to be energy are also derived from the basic 1234567890123456789012345678901212345678901234567890123456789012123456 77 1234567890123456789012345678901212345678901234567890123456789012123456 (30) eight furlongs. This made the mile 5,280 unit of the meter. For instance, weight is 7 1234567890123456789012345678901212345678901234567890123456789012123456 1 feet. A sixteenth-century act of Parliameasured in grams, which are determined 7 34567890123456789012345678901212345678901234567890123456789012123456 7 12 ment fixed this measurement for the mile. by the weight of one cubic centimeter 34567890123456789012345678901212345678901234567890123456789012123456 7 12 34567890123456789012345678901212345678901234567890123456789012123456 7 12 It is true that the English system of of water. 2 34567890123456789012345678901212345678901234567890123456789012123456 1234567890123456789012345678901212345678901234567890123456789012123456 77 1234567890123456789012345678901212345678901234567890123456789012123456 (75) measurement, the system that includes France made use of the metric system 7 1234567890123456789012345678901212345678901234567890123456789012123456 7 1234567890123456789012345678901212345678901234567890123456789012123456 (35) the mile, the yard, the foot, and the inch, compulsory in 1840. Other countries 7 1234567890123456789012345678901212345678901234567890123456789012123456 7 1234567890123456789012345678901212345678901234567890123456789012123456 has a certain quirkiness to it because it rapidly followed suit. The adoption of 7 1234567890123456789012345678901212345678901234567890123456789012123456 7 1234567890123456789012345678901212345678901234567890123456789012123456 has evolved through human history. This the metric system, also known as the 1234567890123456789012345678901212345678901234567890123456789012123456 77 1 quirkiness might irritate scientists, but it international system, or S.I., coincided 34567890123456789012345678901212345678901234567890123456789012123456 7 12 34567890123456789012345678901212345678901234567890123456789012123456 7 12 (80) is part and parcel of the tradition that with great advances in science. By 1900, 1234567890123456789012345678901212345678901234567890123456789012123456 7 7 1234567890123456789012345678901212345678901234567890123456789012123456 (40) has been bequeathed, in its accumulated over 35 countries had officially adopted 34567890123456789012345678901212345678901234567890123456789012123456 7 12 2 34567890123456789012345678901212345678901234567890123456789012123456 7 1234567890123456789012345678901212345678901234567890123456789012123456 form, to the English-speaking world. its use. In the United States, the system 7 1234567890123456789012345678901212345678901234567890123456789012123456 7 1 has been dubbed “voluntary” and 34567890123456789012345678901212345678901234567890123456789012123456 7 12 34567890123456789012345678901212345678901234567890123456789012123456 7 12 Passage 2 “preferred,” but has never been made 34567890123456789012345678901212345678901234567890123456789012123456 7 12 34567890123456789012345678901212345678901234567890123456789012123456 7 12 (85) compulsory. The metric system was conceived by 7 1234567890123456789012345678901212345678901234567890123456789012123456 7 1234567890123456789012345678901212345678901234567890123456789012123456 The measure of the meter has been twelve French scientists during the 1234567890123456789012345678901212345678901234567890123456789012123456 7 2 34567890123456789012345678901212345678901234567890123456789012123456 1234567890123456789012345678901212345678901234567890123456789012123456 refined three times since its conception in 7 French Revolution. Like many innova7 1234567890123456789012345678901212345678901234567890123456789012123456 7 1234567890123456789012345678901212345678901234567890123456789012123456 1791. The latest was in 1983 when the (45) tions during the French Revolution, the 1234567890123456789012345678901212345678901234567890123456789012123456 77 1 speed of light was employed to give the metric system was formulated as a 34567890123456789012345678901212345678901234567890123456789012123456 7 12 (90) greatest precision for the measurement to 7 34567890123456789012345678901212345678901234567890123456789012123456 12 scientific system that would replace 34567890123456789012345678901212345678901234567890123456789012123456 7 12 date. The distance that light travels in a 34567890123456789012345678901212345678901234567890123456789012123456 7 12 traditional ways of ordering society. The 7 1234567890123456789012345678901212345678901234567890123456789012123456 vacuum in 1/299,792,458 of a second is 2 34567890123456789012345678901212345678901234567890123456789012123456 7 1234567890123456789012345678901212345678901234567890123456789012123456 revolutionaries did not see it as a 7 1234567890123456789012345678901212345678901234567890123456789012123456 the internationally accepted definition of 7 1234567890123456789012345678901212345678901234567890123456789012123456 (50) coincidence that length was meted in 7 1234567890123456789012345678901212345678901234567890123456789012123456 a meter. 7 1 measures based on the size of a medieval 2 34567890123456789012345678901212345678901234567890123456789012123456 1234567890123456789012345678901212345678901234567890123456789012123456 77 1 king. Instead of these arbitrary standards, 34567890123456789012345678901212345678901234567890123456789012123456 7 12 7 1234567890123456789012345678901212345678901234567890123456789012123456 the metric system’s basic unit of measure, 34567890123456789012345678901212345678901234567890123456789012123456 7 12 2 34567890123456789012345678901212345678901234567890123456789012123456 7 1 the meter, was based upon the circumfer7 1234567890123456789012345678901212345678901234567890123456789012123456 7 1234567890123456789012345678901212345678901234567890123456789012123456 (55) ence of the Earth. For the meter to be a 1234567890123456789012345678901212345678901234567890123456789012123456 7 12345678901234567890123456789012123456789012345678901234567890121234567 38

Copyright © 2005 Thomson Peterson’s, a part of The Thomson Corporaton SAT is a registered trademark of the College Entrance Examination Board, which was not involved in the production of and does not endorse this product.

12345678901234567890123456789012123456789012345678901234567890121234567 1234567890123456789012345678901212345678901234567890123456789012123456 7 7 12234567890123456789012345678901212345678901234567890123456789012123456 5. What is the author of Passage 1’s attitude 7 8. By “reconcile the two measurements,” 1 34567890123456789012345678901212345678901234567890123456789012123456 7 12234567890123456789012345678901212345678901234567890123456789012123456 towards the English system of measure(line 28–32) the author means 7 1234567890123456789012345678901212345678901234567890123456789012123456 7 1 34567890123456789012345678901212345678901234567890123456789012123456 ment? 7 12234567890123456789012345678901212345678901234567890123456789012123456 (A) determine which one was accurate. 34567890123456789012345678901212345678901234567890123456789012123456 7 1 7 12234567890123456789012345678901212345678901234567890123456789012123456 (A) Emphatic praise (B) develop a new system of measure7 1234567890123456789012345678901212345678901234567890123456789012123456 7 1234567890123456789012345678901212345678901234567890123456789012123456 (B) Qualified acceptance ment without the inaccuracies of 7 1234567890123456789012345678901212345678901234567890123456789012123456 34567890123456789012345678901212345678901234567890123456789012123456 7 1 (C) Neutrality the old. 7 12234567890123456789012345678901212345678901234567890123456789012123456 7 1234567890123456789012345678901212345678901234567890123456789012123456 (D) Strong criticism (C) settle the public’s disagreement over 7 1234567890123456789012345678901212345678901234567890123456789012123456 7 1234567890123456789012345678901212345678901234567890123456789012123456 (E) Antipathy which was better. 34567890123456789012345678901212345678901234567890123456789012123456 7 1 7 12234567890123456789012345678901212345678901234567890123456789012123456 (D) find a metric equivalent. 7 1234567890123456789012345678901212345678901234567890123456789012123456 7 6. In Passage 1, the story of Henry I is 1234567890123456789012345678901212345678901234567890123456789012123456 (E) cease using two different systems. 7 1234567890123456789012345678901212345678901234567890123456789012123456 offered primarily to 34567890123456789012345678901212345678901234567890123456789012123456 7 1 7 12234567890123456789012345678901212345678901234567890123456789012123456 9. The author refers to the English system’s 7 1234567890123456789012345678901212345678901234567890123456789012123456 (A) demonstrate that the reader’s 7 1234567890123456789012345678901212345678901234567890123456789012123456 “accumulated form” line 40–41 primarily 7 1234567890123456789012345678901212345678901234567890123456789012123456 preconceptions about the English 34567890123456789012345678901212345678901234567890123456789012123456 7 1234567890123456789012345678901212345678901234567890123456789012123456 to emphasize that the system 7 1234567890123456789012345678901212345678901234567890123456789012123456 system are wrong. 1234567890123456789012345678901212345678901234567890123456789012123456 77 1234567890123456789012345678901212345678901234567890123456789012123456 (B) illustrate the role of the English (A) ceased to change once officially 7 1234567890123456789012345678901212345678901234567890123456789012123456 7 1234567890123456789012345678901212345678901234567890123456789012123456 monarchy in the development of the adopted. 7 1 7 12234567890123456789012345678901212345678901234567890123456789012123456 English system. (B) derives from a variety of sources. 7 1234567890123456789012345678901212345678901234567890123456789012123456 7 1234567890123456789012345678901212345678901234567890123456789012123456 (C) reveal how far back in time the (C) stretches back further than reliable 34567890123456789012345678901212345678901234567890123456789012123456 1234567890123456789012345678901212345678901234567890123456789012123456 77 1234567890123456789012345678901212345678901234567890123456789012123456 English system goes. written history. 7 1234567890123456789012345678901212345678901234567890123456789012123456 7 1234567890123456789012345678901212345678901234567890123456789012123456 (D) provide a concrete example of how (D) continues to evolve. 1234567890123456789012345678901212345678901234567890123456789012123456 77 1234567890123456789012345678901212345678901234567890123456789012123456 the arbitrariness of the English (E) was adopted wholesale. 7 1234567890123456789012345678901212345678901234567890123456789012123456 7 1234567890123456789012345678901212345678901234567890123456789012123456 system developed. 7 1234567890123456789012345678901212345678901234567890123456789012123456 10. The authors of the two passages would be 7 1 (E) suggest the practicality of the English 2 34567890123456789012345678901212345678901234567890123456789012123456 1234567890123456789012345678901212345678901234567890123456789012123456 most likely to agree that the metric system 7 7 system. 1234567890123456789012345678901212345678901234567890123456789012123456 7 1 7 1234567890123456789012345678901212345678901234567890123456789012123456 (A) has a shorter but equally interesting 7 12234567890123456789012345678901212345678901234567890123456789012123456 7. The word “earthy” in line 14 most closely 34567890123456789012345678901212345678901234567890123456789012123456 7 1234567890123456789012345678901212345678901234567890123456789012123456 history as the English system. 7 1234567890123456789012345678901212345678901234567890123456789012123456 means 7 1 (B) has a history that reaches back as far 7 12234567890123456789012345678901212345678901234567890123456789012123456 7 1234567890123456789012345678901212345678901234567890123456789012123456 (A) unrefined. as the English system’s. 34567890123456789012345678901212345678901234567890123456789012123456 1234567890123456789012345678901212345678901234567890123456789012123456 77 1234567890123456789012345678901212345678901234567890123456789012123456 (B) musky. (C) has a longer history than the history 7 1 7 1234567890123456789012345678901212345678901234567890123456789012123456 (C) impractical. of the English system. 7 1234567890123456789012345678901212345678901234567890123456789012123456 2 34567890123456789012345678901212345678901234567890123456789012123456 1234567890123456789012345678901212345678901234567890123456789012123456 (D) old-fashioned. (D) should not be thought of historically. 7 7 1234567890123456789012345678901212345678901234567890123456789012123456 1234567890123456789012345678901212345678901234567890123456789012123456 (E) baffling. (E) has a history that is equally long but 7 1234567890123456789012345678901212345678901234567890123456789012123456 77 1 less colorful than the English 7 12234567890123456789012345678901212345678901234567890123456789012123456 system’s. 7 1234567890123456789012345678901212345678901234567890123456789012123456 7 1234567890123456789012345678901212345678901234567890123456789012123456 7 1234567890123456789012345678901212345678901234567890123456789012123456 1 34567890123456789012345678901212345678901234567890123456789012123456 7 2 34567890123456789012345678901212345678901234567890123456789012123456 1234567890123456789012345678901212345678901234567890123456789012123456 77 1234567890123456789012345678901212345678901234567890123456789012123456 7 1234567890123456789012345678901212345678901234567890123456789012123456 7 1234567890123456789012345678901212345678901234567890123456789012123456 7 1 7 12234567890123456789012345678901212345678901234567890123456789012123456 1 34567890123456789012345678901212345678901234567890123456789012123456 7 7 12234567890123456789012345678901212345678901234567890123456789012123456 1 34567890123456789012345678901212345678901234567890123456789012123456 7 7 12234567890123456789012345678901212345678901234567890123456789012123456 1 34567890123456789012345678901212345678901234567890123456789012123456 7 1234567890123456789012345678901212345678901234567890123456789012123456 7 1234567890123456789012345678901212345678901234567890123456789012123456 7 1234567890123456789012345678901212345678901234567890123456789012123456 7 12345678901234567890123456789012123456789012345678901234567890121234567 Copyright © 2005 Thomson Peterson’s, a part of The Thomson Corporaton SAT is a registered trademark of the College Entrance Examination Board, which was not involved in the production of and does not endorse this product.

39

12345678901234567890123456789012123456789012345678901234567890121234567 1234567890123456789012345678901212345678901234567890123456789012123456 7 34567890123456789012345678901212345678901234567890123456789012123456 7 12 11. According to Passage 2, the invention of 14. In Passage 2, the reason for the 1983 defini- 7 1234567890123456789012345678901212345678901234567890123456789012123456 34567890123456789012345678901212345678901234567890123456789012123456 7 12 the metric system was tion of the meter is probably that scientists 7 34567890123456789012345678901212345678901234567890123456789012123456 12 1234567890123456789012345678901212345678901234567890123456789012123456 7 34567890123456789012345678901212345678901234567890123456789012123456 7 12 (A) one of the greatest accomplishments (A) have determined that the new meter 7 1234567890123456789012345678901212345678901234567890123456789012123456 34567890123456789012345678901212345678901234567890123456789012123456 7 12 of the French Revolution. is a more manageable length. 34567890123456789012345678901212345678901234567890123456789012123456 7 12 34567890123456789012345678901212345678901234567890123456789012123456 7 12 (B) in contradiction to many of the other (B) have more sophisticated data on the 34567890123456789012345678901212345678901234567890123456789012123456 7 12 2 34567890123456789012345678901212345678901234567890123456789012123456 7 1 goals of the French Revolution. circumference of the earth. 34567890123456789012345678901212345678901234567890123456789012123456 7 12 34567890123456789012345678901212345678901234567890123456789012123456 7 12 (C) a side-effect of the French Revolu(C) needed a way to bring the meter’s 34567890123456789012345678901212345678901234567890123456789012123456 7 12 34567890123456789012345678901212345678901234567890123456789012123456 7 12 tion’s new calendar system. length closer to the yard’s. 7 1234567890123456789012345678901212345678901234567890123456789012123456 34567890123456789012345678901212345678901234567890123456789012123456 7 12 (D) one of many anti-traditionalist (D) have developed more accurate ways 34567890123456789012345678901212345678901234567890123456789012123456 7 12 34567890123456789012345678901212345678901234567890123456789012123456 7 12 undertakings of the French Revoluto calculate the original fraction. 34567890123456789012345678901212345678901234567890123456789012123456 7 12 2 34567890123456789012345678901212345678901234567890123456789012123456 7 1 tion. (E) wanted to disassociate the meter 34567890123456789012345678901212345678901234567890123456789012123456 7 12 34567890123456789012345678901212345678901234567890123456789012123456 7 12 (E) left incomplete at the end of the with the French Revolution. 34567890123456789012345678901212345678901234567890123456789012123456 7 12 34567890123456789012345678901212345678901234567890123456789012123456 7 12 French Revolution. 34567890123456789012345678901212345678901234567890123456789012123456 7 12 15. In at least one of the passages all of the 34567890123456789012345678901212345678901234567890123456789012123456 7 12 34567890123456789012345678901212345678901234567890123456789012123456 7 12 12. In the sentence beginning “In the United following are mentioned EXCEPT 34567890123456789012345678901212345678901234567890123456789012123456 7 12 34567890123456789012345678901212345678901234567890123456789012123456 7 12 States. . .” (line 82) the writer suggests 34567890123456789012345678901212345678901234567890123456789012123456 7 12 (A) the kings who ruled during the 7 1234567890123456789012345678901212345678901234567890123456789012123456 that the United States 34567890123456789012345678901212345678901234567890123456789012123456 7 12 standardization of measurements. 34567890123456789012345678901212345678901234567890123456789012123456 7 12 34567890123456789012345678901212345678901234567890123456789012123456 7 12 (A) has never seriously attempted to (B) the contemporary standing of the 34567890123456789012345678901212345678901234567890123456789012123456 7 12 34567890123456789012345678901212345678901234567890123456789012123456 7 12 implement the metric system. measuring system discussed. 34567890123456789012345678901212345678901234567890123456789012123456 7 12 34567890123456789012345678901212345678901234567890123456789012123456 7 12 (B) is likely to adopt the metric system (C) terms from ancient languages. 34567890123456789012345678901212345678901234567890123456789012123456 7 12 2 34567890123456789012345678901212345678901234567890123456789012123456 7 1234567890123456789012345678901212345678901234567890123456789012123456 fairly soon. (D) the refinement of measurement 7 1234567890123456789012345678901212345678901234567890123456789012123456 7 1234567890123456789012345678901212345678901234567890123456789012123456 (C) has created official policies regarding standards in recent years. 1234567890123456789012345678901212345678901234567890123456789012123456 77 1 use of the metric system. (E) the cultural heritage of each measur34567890123456789012345678901212345678901234567890123456789012123456 7 12 34567890123456789012345678901212345678901234567890123456789012123456 7 (D) has attempted to require use of the 12 ing system. 7 1234567890123456789012345678901212345678901234567890123456789012123456 metric system, but has been unable 7 1234567890123456789012345678901212345678901234567890123456789012123456 34567890123456789012345678901212345678901234567890123456789012123456 7 12 16. The author of Passage 2 conveys an to enforce its policies. 2 34567890123456789012345678901212345678901234567890123456789012123456 1234567890123456789012345678901212345678901234567890123456789012123456 77 1234567890123456789012345678901212345678901234567890123456789012123456 implicit belief that the (E) reflects a clear bias for the superior7 1 34567890123456789012345678901212345678901234567890123456789012123456 7 12 ity of the English system. 34567890123456789012345678901212345678901234567890123456789012123456 7 12 (A) metric system facilitates scientific 2 34567890123456789012345678901212345678901234567890123456789012123456 1234567890123456789012345678901212345678901234567890123456789012123456 77 1234567890123456789012345678901212345678901234567890123456789012123456 endeavors. 7 1 13. The word “refined” in line 87 most 1234567890123456789012345678901212345678901234567890123456789012123456 (B) United States has damaged its reputa- 7 7 1234567890123456789012345678901212345678901234567890123456789012123456 closely means 34567890123456789012345678901212345678901234567890123456789012123456 12 tion in the international community by 7 34567890123456789012345678901212345678901234567890123456789012123456 7 12 34567890123456789012345678901212345678901234567890123456789012123456 7 12 (A) processed. refusing to adopt the metric system. 34567890123456789012345678901212345678901234567890123456789012123456 7 12 7 1234567890123456789012345678901212345678901234567890123456789012123456 (B) renegotiated. (C) metric system is best confined to 34567890123456789012345678901212345678901234567890123456789012123456 7 12 (C) made smaller. scientific use. 34567890123456789012345678901212345678901234567890123456789012123456 7 12 34567890123456789012345678901212345678901234567890123456789012123456 7 12 (D) challenged. (D) French Revolution was a high point 34567890123456789012345678901212345678901234567890123456789012123456 7 12 7 1234567890123456789012345678901212345678901234567890123456789012123456 (E) modified. in the history of science. 2 34567890123456789012345678901212345678901234567890123456789012123456 1234567890123456789012345678901212345678901234567890123456789012123456 77 1234567890123456789012345678901212345678901234567890123456789012123456 (E) metric system is a more fitting system 7 1234567890123456789012345678901212345678901234567890123456789012123456 7 1234567890123456789012345678901212345678901234567890123456789012123456 for a democratic society. 7 1 2 34567890123456789012345678901212345678901234567890123456789012123456 1234567890123456789012345678901212345678901234567890123456789012123456 77 1 34567890123456789012345678901212345678901234567890123456789012123456 7 12 1234567890123456789012345678901212345678901234567890123456789012123456 7 34567890123456789012345678901212345678901234567890123456789012123456 7 12 Do not proceed to the next section until time is up. 7 1234567890123456789012345678901212345678901234567890123456789012123456 1234567890123456789012345678901212345678901234567890123456789012123456 7 1234567890123456789012345678901212345678901234567890123456789012123456 7 1234567890123456789012345678901212345678901234567890123456789012123456 7 12345678901234567890123456789012123456789012345678901234567890121234567

STOP

40

Copyright © 2005 Thomson Peterson’s, a part of The Thomson Corporaton SAT is a registered trademark of the College Entrance Examination Board, which was not involved in the production of and does not endorse this product.

Section 7 13 Questions j Time—20 Minutes 12345678901234567890123456789012123456789012345678901234567890121234567 7 12234567890123456789012345678901212345678901234567890123456789012123456 7 1234567890123456789012345678901212345678901234567890123456789012123456 34567890123456789012345678901212345678901234567890123456789012123456 7 1 7 12234567890123456789012345678901212345678901234567890123456789012123456 Notes: 7 1234567890123456789012345678901212345678901234567890123456789012123456 7 1234567890123456789012345678901212345678901234567890123456789012123456 7 1234567890123456789012345678901212345678901234567890123456789012123456 1. All numbers used are real numbers. 34567890123456789012345678901212345678901234567890123456789012123456 1234567890123456789012345678901212345678901234567890123456789012123456 77 1234567890123456789012345678901212345678901234567890123456789012123456 2. All angle measurements can be assumed to be positive unless otherwise noted. 7 1234567890123456789012345678901212345678901234567890123456789012123456 1234567890123456789012345678901212345678901234567890123456789012123456 77 1234567890123456789012345678901212345678901234567890123456789012123456 3. All figures lie in the same plane unless otherwise noted. 7 1234567890123456789012345678901212345678901234567890123456789012123456 7 1 7 12234567890123456789012345678901212345678901234567890123456789012123456 4. Drawings that accompany questions are intended to provide information useful in 7 1234567890123456789012345678901212345678901234567890123456789012123456 7 1234567890123456789012345678901212345678901234567890123456789012123456 answering the question. The figures are drawn closely to scale unless otherwise noted. 34567890123456789012345678901212345678901234567890123456789012123456 1234567890123456789012345678901212345678901234567890123456789012123456 77 1234567890123456789012345678901212345678901234567890123456789012123456 7 1234567890123456789012345678901212345678901234567890123456789012123456 7 1234567890123456789012345678901212345678901234567890123456789012123456 7 1234567890123456789012345678901212345678901234567890123456789012123456 Directions for Student Produced Responses 7 1234567890123456789012345678901212345678901234567890123456789012123456 1234567890123456789012345678901212345678901234567890123456789012123456 77 1234567890123456789012345678901212345678901234567890123456789012123456 Enter your responses to questions 1–13 in the Answer: 1.4 7 1234567890123456789012345678901212345678901234567890123456789012123456 7 1 special grids provided on your answer sheet. Either position is correct. 2 34567890123456789012345678901212345678901234567890123456789012123456 1234567890123456789012345678901212345678901234567890123456789012123456 77 1234567890123456789012345678901212345678901234567890123456789012123456 7 1 Input your answers as indicated in the directions 1 . 4 1 . 4 7 1234567890123456789012345678901212345678901234567890123456789012123456 below. 7 12234567890123456789012345678901212345678901234567890123456789012123456 O O O O 34567890123456789012345678901212345678901234567890123456789012123456 7 1234567890123456789012345678901212345678901234567890123456789012123456 O O Þ O O Þ O O 4 Decimal → 7 1234567890123456789012345678901212345678901234567890123456789012123456 Answer: or 4/9 7 1 O O O point O O O 9 7 12234567890123456789012345678901212345678901234567890123456789012123456 O Þ O O Þ O O O 7 1234567890123456789012345678901212345678901234567890123456789012123456 34567890123456789012345678901212345678901234567890123456789012123456 7 1234567890123456789012345678901212345678901234567890123456789012123456 O O O O O O O O Write answer → 4 / 9 7 1234567890123456789012345678901212345678901234567890123456789012123456 O O O O O O O O 7 1 in boxes. O O O Þ O O Þ O Þ O ← Fraction line 7 1234567890123456789012345678901212345678901234567890123456789012123456 7 1234567890123456789012345678901212345678901234567890123456789012123456 O O O O 2 34567890123456789012345678901212345678901234567890123456789012123456 7 1234567890123456789012345678901212345678901234567890123456789012123456 O O O Note: You may begin your answer in any col- 7 1234567890123456789012345678901212345678901234567890123456789012123456 7 1234567890123456789012345678901212345678901234567890123456789012123456 O O O ⎧O umn, space permitting. Leave blank any columns 7 1234567890123456789012345678901212345678901234567890123456789012123456 O O O O 7 1 O O O O ⎪Þ not needed. 7 12234567890123456789012345678901212345678901234567890123456789012123456 O O O 7 1234567890123456789012345678901212345678901234567890123456789012123456 Grid in → 7 1234567890123456789012345678901212345678901234567890123456789012123456 O O O O ⎨ • Writing your answer in the boxes at the top result. 7 1234567890123456789012345678901212345678901234567890123456789012123456 O O O O 7 1 34567890123456789012345678901212345678901234567890123456789012123456 of the columns will help you accurately grid O O O O ⎪ 2 34567890123456789012345678901212345678901234567890123456789012123456 7 1234567890123456789012345678901212345678901234567890123456789012123456 O O O O 7 1234567890123456789012345678901212345678901234567890123456789012123456 your answer, but it is not required. You will 7 1234567890123456789012345678901212345678901234567890123456789012123456 O O Þ O ⎩ 7 1234567890123456789012345678901212345678901234567890123456789012123456 only receive credit for an answer if the ovals 7 1 2 34567890123456789012345678901212345678901234567890123456789012123456 7 1234567890123456789012345678901212345678901234567890123456789012123456 are filled in properly. 7 1 7 12234567890123456789012345678901212345678901234567890123456789012123456 7 1 34567890123456789012345678901212345678901234567890123456789012123456 • Only fill in one oval in each column. 7 12234567890123456789012345678901212345678901234567890123456789012123456 34567890123456789012345678901212345678901234567890123456789012123456 7 1 1234567890123456789012345678901212345678901234567890123456789012123456 7 1234567890123456789012345678901212345678901234567890123456789012123456 7 1234567890123456789012345678901212345678901234567890123456789012123456 7 12345678901234567890123456789012123456789012345678901234567890121234567 /

.

/

.

.

.

.

0

0

0

1

1

1

1

2

2

2

2

3

3

3

3

4

4

4

5

5

5

5

6

6

6

6

7

7

7

7

8

8

8

8

9

9

9

/

/

.

.

.

0

0

0

1

1

1

1

2

2

2

2

2

3

3

3

3

4

4

4

/

.

0

.

0

0

1

1

2

2

2

3

3

3

3

4

4

4

12345678901234567890123456789012123456789012345678901234567890121234567 1234567890123456789012345678901212345678901234567890123456789012123456 7 34567890123456789012345678901212345678901234567890123456789012123456 7 12 • If a problem has several correct answers, Decimal Accuracy 7 1234567890123456789012345678901212345678901234567890123456789012123456 34567890123456789012345678901212345678901234567890123456789012123456 7 12 just grid in one of them. 34567890123456789012345678901212345678901234567890123456789012123456 7 12 Decimal answers must be gridded as accu1234567890123456789012345678901212345678901234567890123456789012123456 7 34567890123456789012345678901212345678901234567890123456789012123456 12 • There are no negative answers. rately as possible. The answer 0.3333 . . . 7 7 1234567890123456789012345678901212345678901234567890123456789012123456 34567890123456789012345678901212345678901234567890123456789012123456 7 12 must be gridded as .333. 34567890123456789012345678901212345678901234567890123456789012123456 7 12 • Never grid in mixed numbers. The answer 34567890123456789012345678901212345678901234567890123456789012123456 7 12 34567890123456789012345678901212345678901234567890123456789012123456 12 1 Less accurate values, such as .33 are not 7 7 1234567890123456789012345678901212345678901234567890123456789012123456 3 must be gridded as 16/5 or 3.2. If 34567890123456789012345678901212345678901234567890123456789012123456 7 12 acceptable. 5 34567890123456789012345678901212345678901234567890123456789012123456 7 12 is gridded, it will be read 34567890123456789012345678901212345678901234567890123456789012123456 7 12 3 1 / 5 34567890123456789012345678901212345678901234567890123456789012123456 7 12 1 1 31 2 34567890123456789012345678901212345678901234567890123456789012123456 7 1 Acceptable ways to grid 5 .3333 . . . O Þ as , not 3 . 34567890123456789012345678901212345678901234567890123456789012123456 7 12 3 O O O O 5 5 34567890123456789012345678901212345678901234567890123456789012123456 7 12 34567890123456789012345678901212345678901234567890123456789012123456 7 12 O O O 34567890123456789012345678901212345678901234567890123456789012123456 7 12 . 3 3 3 1 / 3 O Þ O O 2 34567890123456789012345678901212345678901234567890123456789012123456 7 1 34567890123456789012345678901212345678901234567890123456789012123456 7 12 O O O O O O Þ O 34567890123456789012345678901212345678901234567890123456789012123456 7 12 Þ O O O Þ O O O O O O O 34567890123456789012345678901212345678901234567890123456789012123456 7 12 O O O O 34567890123456789012345678901212345678901234567890123456789012123456 7 12 O O O O O O O O O Þ 2 34567890123456789012345678901212345678901234567890123456789012123456 7 1234567890123456789012345678901212345678901234567890123456789012123456 O O O O Þ O O O 7 1234567890123456789012345678901212345678901234567890123456789012123456 O O O O O O O O 7 1234567890123456789012345678901212345678901234567890123456789012123456 O Þ Þ Þ O O Þ O 7 1234567890123456789012345678901212345678901234567890123456789012123456 7 1234567890123456789012345678901212345678901234567890123456789012123456 O O O O O O O O 7 1234567890123456789012345678901212345678901234567890123456789012123456 O O O O O O O O 7 1 O O O O O O O O 34567890123456789012345678901212345678901234567890123456789012123456 7 12 34567890123456789012345678901212345678901234567890123456789012123456 7 12 34567890123456789012345678901212345678901234567890123456789012123456 7 12 2 7 1234567890123456789012345678901212345678901234567890123456789012123456 7 1234567890123456789012345678901212345678901234567890123456789012123456 7 1234567890123456789012345678901212345678901234567890123456789012123456 7 1234567890123456789012345678901212345678901234567890123456789012123456 7 1234567890123456789012345678901212345678901234567890123456789012123456 1. If 2=x 1 =x 5 y and 2y 5 12, then 5. If the area of a circle is 16p, what is the 34567890123456789012345678901212345678901234567890123456789012123456 1234567890123456789012345678901212345678901234567890123456789012123456 77 1234567890123456789012345678901212345678901234567890123456789012123456 what does x equal? diameter of the circle? 1234567890123456789012345678901212345678901234567890123456789012123456 77 1234567890123456789012345678901212345678901234567890123456789012123456 7 1 2. What is the product of the first five even 6. Lisa has three pizza toppings from which 34567890123456789012345678901212345678901234567890123456789012123456 7 12 34567890123456789012345678901212345678901234567890123456789012123456 7 12 integers? to choose: pepperoni, anchovies, and red 1234567890123456789012345678901212345678901234567890123456789012123456 7 7 1234567890123456789012345678901212345678901234567890123456789012123456 peppers. If she can choose as many 34567890123456789012345678901212345678901234567890123456789012123456 7 12 2 34567890123456789012345678901212345678901234567890123456789012123456 7 1234567890123456789012345678901212345678901234567890123456789012123456 3. Rider High has 400 students. A student toppings as three and as few toppings as 7 1234567890123456789012345678901212345678901234567890123456789012123456 7 1 will be picked at random from the student zero, how many different pizza orders are 34567890123456789012345678901212345678901234567890123456789012123456 7 12 34567890123456789012345678901212345678901234567890123456789012123456 7 12 body. If the probability that a senior possible? (Lisa cannot order the same 34567890123456789012345678901212345678901234567890123456789012123456 7 12 34567890123456789012345678901212345678901234567890123456789012123456 7 12 would be picked is three-eighths, how topping twice.) 1234567890123456789012345678901212345678901234567890123456789012123456 7 7 1234567890123456789012345678901212345678901234567890123456789012123456 many seniors are there? 7 1234567890123456789012345678901212345678901234567890123456789012123456 7. The average (arithmetic mean) of five 2 34567890123456789012345678901212345678901234567890123456789012123456 1234567890123456789012345678901212345678901234567890123456789012123456 77 1234567890123456789012345678901212345678901234567890123456789012123456 4. numbers is 16. If one number is taken 7 1234567890123456789012345678901212345678901234567890123456789012123456 7 1234567890123456789012345678901212345678901234567890123456789012123456 from the set, the new average is 14. What 7 1 2 34567890123456789012345678901212345678901234567890123456789012123456 7 1234567890123456789012345678901212345678901234567890123456789012123456 number was taken from the set? 7 1234567890123456789012345678901212345678901234567890123456789012123456 1234567890123456789012345678901212345678901234567890123456789012123456 77 1234567890123456789012345678901212345678901234567890123456789012123456 7 1 34567890123456789012345678901212345678901234567890123456789012123456 7 12 34567890123456789012345678901212345678901234567890123456789012123456 7 12 34567890123456789012345678901212345678901234567890123456789012123456 7 12 34567890123456789012345678901212345678901234567890123456789012123456 7 12 1234567890123456789012345678901212345678901234567890123456789012123456 7 2 7 1234567890123456789012345678901212345678901234567890123456789012123456 7 1 34567890123456789012345678901212345678901234567890123456789012123456 If the area of the striped side of the 34567890123456789012345678901212345678901234567890123456789012123456 7 12 7 1234567890123456789012345678901212345678901234567890123456789012123456 rectangular solid is 24, what is the volume 34567890123456789012345678901212345678901234567890123456789012123456 7 12 2 34567890123456789012345678901212345678901234567890123456789012123456 7 1 of the box? 7 1234567890123456789012345678901212345678901234567890123456789012123456 1234567890123456789012345678901212345678901234567890123456789012123456 77 1234567890123456789012345678901212345678901234567890123456789012123456 12345678901234567890123456789012123456789012345678901234567890121234567 /

.

.

.

.

0

0

0

1

1

2

2

2

3

3

3

4

4

4

4

5

5

5

1 2

42

/

/

.

.

.

0

0

0

1

1

1

2

2

2

4

4

4

4

5

5

5

5

6

6

6

6

/

.

.

.

.

0

0

0

1

1

1

1

2

2

2

2

2

3

3

3

3

4

4

4

4

5

5

5

5

6

6

6

6

Copyright © 2005 Thomson Peterson’s, a part of The Thomson Corporaton SAT is a registered trademark of the College Entrance Examination Board, which was not involved in the production of and does not endorse this product.

12345678901234567890123456789012123456789012345678901234567890121234567 1234567890123456789012345678901212345678901234567890123456789012123456 7 7 12234567890123456789012345678901212345678901234567890123456789012123456 8. 12. 7 1 34567890123456789012345678901212345678901234567890123456789012123456 7 12234567890123456789012345678901212345678901234567890123456789012123456 7 1234567890123456789012345678901212345678901234567890123456789012123456 1 34567890123456789012345678901212345678901234567890123456789012123456 7 7 12234567890123456789012345678901212345678901234567890123456789012123456 1 34567890123456789012345678901212345678901234567890123456789012123456 7 7 12234567890123456789012345678901212345678901234567890123456789012123456 7 1234567890123456789012345678901212345678901234567890123456789012123456 7 1234567890123456789012345678901212345678901234567890123456789012123456 7 1234567890123456789012345678901212345678901234567890123456789012123456 34567890123456789012345678901212345678901234567890123456789012123456 7 1 7 12234567890123456789012345678901212345678901234567890123456789012123456 7 1234567890123456789012345678901212345678901234567890123456789012123456 7 1234567890123456789012345678901212345678901234567890123456789012123456 If the figure above is composed of a 7 1234567890123456789012345678901212345678901234567890123456789012123456 34567890123456789012345678901212345678901234567890123456789012123456 7 1 square and a triangle, what is the area of 7 12234567890123456789012345678901212345678901234567890123456789012123456 7 1234567890123456789012345678901212345678901234567890123456789012123456 the square? 7 1234567890123456789012345678901212345678901234567890123456789012123456 7 1234567890123456789012345678901212345678901234567890123456789012123456 34567890123456789012345678901212345678901234567890123456789012123456 7 1 9. What is the slope a line that is defined by 7 12234567890123456789012345678901212345678901234567890123456789012123456 What is the sum of the y-intercepts in the 7 1234567890123456789012345678901212345678901234567890123456789012123456 the following two points, (21, 22) and 7 1234567890123456789012345678901212345678901234567890123456789012123456 above graph? 7 1234567890123456789012345678901212345678901234567890123456789012123456 (3,1)? 34567890123456789012345678901212345678901234567890123456789012123456 1234567890123456789012345678901212345678901234567890123456789012123456 77 1234567890123456789012345678901212345678901234567890123456789012123456 13. The sum of four positive distinct prime 7 1234567890123456789012345678901212345678901234567890123456789012123456 7 1234567890123456789012345678901212345678901234567890123456789012123456 10. Tom is twelve years older than Susan, and numbers is 21. What is the greatest 1234567890123456789012345678901212345678901234567890123456789012123456 77 1234567890123456789012345678901212345678901234567890123456789012123456 Susan is three times the age of Gina, and possible value of one of those prime 7 1 7 12234567890123456789012345678901212345678901234567890123456789012123456 Gina is five years younger than Bo. If Bo is numbers? 7 1234567890123456789012345678901212345678901234567890123456789012123456 7 1234567890123456789012345678901212345678901234567890123456789012123456 15, how old is Tom? 34567890123456789012345678901212345678901234567890123456789012123456 1234567890123456789012345678901212345678901234567890123456789012123456 77 1234567890123456789012345678901212345678901234567890123456789012123456 7 1234567890123456789012345678901212345678901234567890123456789012123456 11. For all positive integers, let m* equal the 7 1234567890123456789012345678901212345678901234567890123456789012123456 7 1234567890123456789012345678901212345678901234567890123456789012123456 greatest prime divisor of m. What does 1234567890123456789012345678901212345678901234567890123456789012123456 77 1234567890123456789012345678901212345678901234567890123456789012123456 (15*)(12*) equal? 7 1234567890123456789012345678901212345678901234567890123456789012123456 1234567890123456789012345678901212345678901234567890123456789012123456 77 1 7 12234567890123456789012345678901212345678901234567890123456789012123456 7 1234567890123456789012345678901212345678901234567890123456789012123456 1 34567890123456789012345678901212345678901234567890123456789012123456 7 1234567890123456789012345678901212345678901234567890123456789012123456 7 7 12234567890123456789012345678901212345678901234567890123456789012123456 7 1234567890123456789012345678901212345678901234567890123456789012123456 7 1234567890123456789012345678901212345678901234567890123456789012123456 1 34567890123456789012345678901212345678901234567890123456789012123456 7 7 12234567890123456789012345678901212345678901234567890123456789012123456 7 1234567890123456789012345678901212345678901234567890123456789012123456 34567890123456789012345678901212345678901234567890123456789012123456 1234567890123456789012345678901212345678901234567890123456789012123456 77 1234567890123456789012345678901212345678901234567890123456789012123456 7 1 1234567890123456789012345678901212345678901234567890123456789012123456 7 1234567890123456789012345678901212345678901234567890123456789012123456 7 7 12234567890123456789012345678901212345678901234567890123456789012123456 7 1234567890123456789012345678901212345678901234567890123456789012123456 7 1234567890123456789012345678901212345678901234567890123456789012123456 7 1234567890123456789012345678901212345678901234567890123456789012123456 1 34567890123456789012345678901212345678901234567890123456789012123456 7 2 34567890123456789012345678901212345678901234567890123456789012123456 1234567890123456789012345678901212345678901234567890123456789012123456 77 1234567890123456789012345678901212345678901234567890123456789012123456 7 1234567890123456789012345678901212345678901234567890123456789012123456 7 1234567890123456789012345678901212345678901234567890123456789012123456 7 1 7 12234567890123456789012345678901212345678901234567890123456789012123456 7 1234567890123456789012345678901212345678901234567890123456789012123456 7 1234567890123456789012345678901212345678901234567890123456789012123456 7 1234567890123456789012345678901212345678901234567890123456789012123456 1 34567890123456789012345678901212345678901234567890123456789012123456 7 2 34567890123456789012345678901212345678901234567890123456789012123456 1234567890123456789012345678901212345678901234567890123456789012123456 77 1 7 12234567890123456789012345678901212345678901234567890123456789012123456 1 34567890123456789012345678901212345678901234567890123456789012123456 7 7 12234567890123456789012345678901212345678901234567890123456789012123456 Do not proceed to the next section until time is up. 7 1 34567890123456789012345678901212345678901234567890123456789012123456 1234567890123456789012345678901212345678901234567890123456789012123456 7 1234567890123456789012345678901212345678901234567890123456789012123456 7 1234567890123456789012345678901212345678901234567890123456789012123456 7 12345678901234567890123456789012123456789012345678901234567890121234567

STOP

Copyright © 2005 Thomson Peterson’s, a part of The Thomson Corporaton SAT is a registered trademark of the College Entrance Examination Board, which was not involved in the production of and does not endorse this product.

43

Section 8 Time—25 Minutes 12345678901234567890123456789012123456789012345678901234567890121234567 34567890123456789012345678901212345678901234567890123456789012123456 7 12 34567890123456789012345678901212345678901234567890123456789012123456 7 12 2 1 34567890123456789012345678901212345678901234567890123456789012123456 7 34567890123456789012345678901212345678901234567890123456789012123456 7 12 Directions: Think carefully about the statement below and the assignment that follows it. 34567890123456789012345678901212345678901234567890123456789012123456 7 12 34567890123456789012345678901212345678901234567890123456789012123456 7 12 34567890123456789012345678901212345678901234567890123456789012123456 7 12 34567890123456789012345678901212345678901234567890123456789012123456 7 12 Consider the following statement. 34567890123456789012345678901212345678901234567890123456789012123456 7 12 34567890123456789012345678901212345678901234567890123456789012123456 7 12 34567890123456789012345678901212345678901234567890123456789012123456 12 Innovation is primarily accomplished by an individual, though groups often do work out the 7 34567890123456789012345678901212345678901234567890123456789012123456 7 12 2 34567890123456789012345678901212345678901234567890123456789012123456 7 1234567890123456789012345678901212345678901234567890123456789012123456 7 1 details of an individual’s innovations. 34567890123456789012345678901212345678901234567890123456789012123456 7 12 34567890123456789012345678901212345678901234567890123456789012123456 12 Assignment: Write an essay in which you argue for or against the preceding statement. Develop your 7 34567890123456789012345678901212345678901234567890123456789012123456 7 12 2 34567890123456789012345678901212345678901234567890123456789012123456 7 1234567890123456789012345678901212345678901234567890123456789012123456 point of view on this statement and be sure to support your stance with sufficient details. 7 1234567890123456789012345678901212345678901234567890123456789012123456 1234567890123456789012345678901212345678901234567890123456789012123456 77 1234567890123456789012345678901212345678901234567890123456789012123456 7 1234567890123456789012345678901212345678901234567890123456789012123456 7 1234567890123456789012345678901212345678901234567890123456789012123456 7 1234567890123456789012345678901212345678901234567890123456789012123456 7 1234567890123456789012345678901212345678901234567890123456789012123456 7 1234567890123456789012345678901212345678901234567890123456789012123456 7 1 34567890123456789012345678901212345678901234567890123456789012123456 7 12 34567890123456789012345678901212345678901234567890123456789012123456 7 12 1234567890123456789012345678901212345678901234567890123456789012123456 7 1234567890123456789012345678901212345678901234567890123456789012123456 7 34567890123456789012345678901212345678901234567890123456789012123456 7 12 34567890123456789012345678901212345678901234567890123456789012123456 7 12 34567890123456789012345678901212345678901234567890123456789012123456 7 12 1234567890123456789012345678901212345678901234567890123456789012123456 7 34567890123456789012345678901212345678901234567890123456789012123456 7 12 34567890123456789012345678901212345678901234567890123456789012123456 7 12 2 7 1234567890123456789012345678901212345678901234567890123456789012123456 7 1234567890123456789012345678901212345678901234567890123456789012123456 1 34567890123456789012345678901212345678901234567890123456789012123456 7 1234567890123456789012345678901212345678901234567890123456789012123456 7 1234567890123456789012345678901212345678901234567890123456789012123456 7 34567890123456789012345678901212345678901234567890123456789012123456 7 12 34567890123456789012345678901212345678901234567890123456789012123456 7 12 34567890123456789012345678901212345678901234567890123456789012123456 7 12 34567890123456789012345678901212345678901234567890123456789012123456 7 12 1234567890123456789012345678901212345678901234567890123456789012123456 7 2 7 1234567890123456789012345678901212345678901234567890123456789012123456 7 1234567890123456789012345678901212345678901234567890123456789012123456 7 1234567890123456789012345678901212345678901234567890123456789012123456 7 1234567890123456789012345678901212345678901234567890123456789012123456 1 34567890123456789012345678901212345678901234567890123456789012123456 7 2 34567890123456789012345678901212345678901234567890123456789012123456 1234567890123456789012345678901212345678901234567890123456789012123456 77 1234567890123456789012345678901212345678901234567890123456789012123456 7 1234567890123456789012345678901212345678901234567890123456789012123456 7 1234567890123456789012345678901212345678901234567890123456789012123456 7 1 34567890123456789012345678901212345678901234567890123456789012123456 7 12 1234567890123456789012345678901212345678901234567890123456789012123456 7 34567890123456789012345678901212345678901234567890123456789012123456 7 12 1234567890123456789012345678901212345678901234567890123456789012123456 7 34567890123456789012345678901212345678901234567890123456789012123456 7 12 1234567890123456789012345678901212345678901234567890123456789012123456 7 1234567890123456789012345678901212345678901234567890123456789012123456 7 1234567890123456789012345678901212345678901234567890123456789012123456 7 1234567890123456789012345678901212345678901234567890123456789012123456 7 12345678901234567890123456789012123456789012345678901234567890121234567 44

Copyright © 2005 Thomson Peterson’s, a part of The Thomson Corporaton SAT is a registered trademark of the College Entrance Examination Board, which was not involved in the production of and does not endorse this product.

12345678901234567890123456789012123456789012345678901234567890121234567 1234567890123456789012345678901212345678901234567890123456789012123456 7 7 12234567890123456789012345678901212345678901234567890123456789012123456 1 34567890123456789012345678901212345678901234567890123456789012123456 7 7 12234567890123456789012345678901212345678901234567890123456789012123456 7 1234567890123456789012345678901212345678901234567890123456789012123456 1 34567890123456789012345678901212345678901234567890123456789012123456 7 7 12234567890123456789012345678901212345678901234567890123456789012123456 1 34567890123456789012345678901212345678901234567890123456789012123456 7 7 12234567890123456789012345678901212345678901234567890123456789012123456 7 1234567890123456789012345678901212345678901234567890123456789012123456 7 1234567890123456789012345678901212345678901234567890123456789012123456 7 1234567890123456789012345678901212345678901234567890123456789012123456 34567890123456789012345678901212345678901234567890123456789012123456 7 1 7 12234567890123456789012345678901212345678901234567890123456789012123456 7 1234567890123456789012345678901212345678901234567890123456789012123456 7 1234567890123456789012345678901212345678901234567890123456789012123456 7 1234567890123456789012345678901212345678901234567890123456789012123456 34567890123456789012345678901212345678901234567890123456789012123456 7 1 7 12234567890123456789012345678901212345678901234567890123456789012123456 7 1234567890123456789012345678901212345678901234567890123456789012123456 7 1234567890123456789012345678901212345678901234567890123456789012123456 7 1234567890123456789012345678901212345678901234567890123456789012123456 34567890123456789012345678901212345678901234567890123456789012123456 7 1 7 12234567890123456789012345678901212345678901234567890123456789012123456 7 1234567890123456789012345678901212345678901234567890123456789012123456 7 1234567890123456789012345678901212345678901234567890123456789012123456 7 1234567890123456789012345678901212345678901234567890123456789012123456 34567890123456789012345678901212345678901234567890123456789012123456 1234567890123456789012345678901212345678901234567890123456789012123456 77 1234567890123456789012345678901212345678901234567890123456789012123456 7 1234567890123456789012345678901212345678901234567890123456789012123456 7 1234567890123456789012345678901212345678901234567890123456789012123456 7 1234567890123456789012345678901212345678901234567890123456789012123456 7 1234567890123456789012345678901212345678901234567890123456789012123456 7 1 7 12234567890123456789012345678901212345678901234567890123456789012123456 7 1234567890123456789012345678901212345678901234567890123456789012123456 7 1234567890123456789012345678901212345678901234567890123456789012123456 34567890123456789012345678901212345678901234567890123456789012123456 1234567890123456789012345678901212345678901234567890123456789012123456 77 1234567890123456789012345678901212345678901234567890123456789012123456 7 1234567890123456789012345678901212345678901234567890123456789012123456 7 1234567890123456789012345678901212345678901234567890123456789012123456 7 1234567890123456789012345678901212345678901234567890123456789012123456 7 1234567890123456789012345678901212345678901234567890123456789012123456 7 1234567890123456789012345678901212345678901234567890123456789012123456 7 1234567890123456789012345678901212345678901234567890123456789012123456 7 1234567890123456789012345678901212345678901234567890123456789012123456 7 1 7 12234567890123456789012345678901212345678901234567890123456789012123456 7 1234567890123456789012345678901212345678901234567890123456789012123456 1 34567890123456789012345678901212345678901234567890123456789012123456 7 1234567890123456789012345678901212345678901234567890123456789012123456 7 7 12234567890123456789012345678901212345678901234567890123456789012123456 7 1234567890123456789012345678901212345678901234567890123456789012123456 7 1234567890123456789012345678901212345678901234567890123456789012123456 1 34567890123456789012345678901212345678901234567890123456789012123456 7 7 12234567890123456789012345678901212345678901234567890123456789012123456 7 1234567890123456789012345678901212345678901234567890123456789012123456 34567890123456789012345678901212345678901234567890123456789012123456 1234567890123456789012345678901212345678901234567890123456789012123456 77 1234567890123456789012345678901212345678901234567890123456789012123456 7 1 1234567890123456789012345678901212345678901234567890123456789012123456 7 1234567890123456789012345678901212345678901234567890123456789012123456 7 7 12234567890123456789012345678901212345678901234567890123456789012123456 7 1234567890123456789012345678901212345678901234567890123456789012123456 7 1234567890123456789012345678901212345678901234567890123456789012123456 7 1234567890123456789012345678901212345678901234567890123456789012123456 1 34567890123456789012345678901212345678901234567890123456789012123456 7 2 34567890123456789012345678901212345678901234567890123456789012123456 1234567890123456789012345678901212345678901234567890123456789012123456 77 1234567890123456789012345678901212345678901234567890123456789012123456 7 1234567890123456789012345678901212345678901234567890123456789012123456 7 1234567890123456789012345678901212345678901234567890123456789012123456 7 1 7 12234567890123456789012345678901212345678901234567890123456789012123456 7 1234567890123456789012345678901212345678901234567890123456789012123456 7 1234567890123456789012345678901212345678901234567890123456789012123456 7 1234567890123456789012345678901212345678901234567890123456789012123456 1 34567890123456789012345678901212345678901234567890123456789012123456 7 2 34567890123456789012345678901212345678901234567890123456789012123456 1234567890123456789012345678901212345678901234567890123456789012123456 77 1 7 12234567890123456789012345678901212345678901234567890123456789012123456 1 34567890123456789012345678901212345678901234567890123456789012123456 7 7 12234567890123456789012345678901212345678901234567890123456789012123456 1 34567890123456789012345678901212345678901234567890123456789012123456 7 1234567890123456789012345678901212345678901234567890123456789012123456 7 1234567890123456789012345678901212345678901234567890123456789012123456 7 1234567890123456789012345678901212345678901234567890123456789012123456 7 12345678901234567890123456789012123456789012345678901234567890121234567 Copyright © 2005 Thomson Peterson’s, a part of The Thomson Corporaton SAT is a registered trademark of the College Entrance Examination Board, which was not involved in the production of and does not endorse this product.

45

12345678901234567890123456789012123456789012345678901234567890121234567 1234567890123456789012345678901212345678901234567890123456789012123456 7 34567890123456789012345678901212345678901234567890123456789012123456 7 12 1234567890123456789012345678901212345678901234567890123456789012123456 7 34567890123456789012345678901212345678901234567890123456789012123456 7 12 34567890123456789012345678901212345678901234567890123456789012123456 7 12 1234567890123456789012345678901212345678901234567890123456789012123456 7 34567890123456789012345678901212345678901234567890123456789012123456 7 12 1234567890123456789012345678901212345678901234567890123456789012123456 7 34567890123456789012345678901212345678901234567890123456789012123456 7 12 34567890123456789012345678901212345678901234567890123456789012123456 7 12 34567890123456789012345678901212345678901234567890123456789012123456 7 12 34567890123456789012345678901212345678901234567890123456789012123456 7 12 2 1 34567890123456789012345678901212345678901234567890123456789012123456 7 34567890123456789012345678901212345678901234567890123456789012123456 7 12 34567890123456789012345678901212345678901234567890123456789012123456 7 12 34567890123456789012345678901212345678901234567890123456789012123456 7 12 34567890123456789012345678901212345678901234567890123456789012123456 7 12 2 1 34567890123456789012345678901212345678901234567890123456789012123456 7 34567890123456789012345678901212345678901234567890123456789012123456 7 12 34567890123456789012345678901212345678901234567890123456789012123456 7 12 34567890123456789012345678901212345678901234567890123456789012123456 7 12 34567890123456789012345678901212345678901234567890123456789012123456 7 12 2 1 34567890123456789012345678901212345678901234567890123456789012123456 7 34567890123456789012345678901212345678901234567890123456789012123456 7 12 34567890123456789012345678901212345678901234567890123456789012123456 7 12 34567890123456789012345678901212345678901234567890123456789012123456 7 12 34567890123456789012345678901212345678901234567890123456789012123456 7 12 2 7 1234567890123456789012345678901212345678901234567890123456789012123456 7 1234567890123456789012345678901212345678901234567890123456789012123456 7 1234567890123456789012345678901212345678901234567890123456789012123456 7 1234567890123456789012345678901212345678901234567890123456789012123456 7 1234567890123456789012345678901212345678901234567890123456789012123456 34567890123456789012345678901212345678901234567890123456789012123456 1234567890123456789012345678901212345678901234567890123456789012123456 77 1 34567890123456789012345678901212345678901234567890123456789012123456 7 12 34567890123456789012345678901212345678901234567890123456789012123456 7 12 34567890123456789012345678901212345678901234567890123456789012123456 7 12 2 7 1234567890123456789012345678901212345678901234567890123456789012123456 7 1234567890123456789012345678901212345678901234567890123456789012123456 7 1234567890123456789012345678901212345678901234567890123456789012123456 7 1234567890123456789012345678901212345678901234567890123456789012123456 7 1234567890123456789012345678901212345678901234567890123456789012123456 34567890123456789012345678901212345678901234567890123456789012123456 1234567890123456789012345678901212345678901234567890123456789012123456 77 1234567890123456789012345678901212345678901234567890123456789012123456 7 1234567890123456789012345678901212345678901234567890123456789012123456 7 1234567890123456789012345678901212345678901234567890123456789012123456 7 1 34567890123456789012345678901212345678901234567890123456789012123456 7 12 34567890123456789012345678901212345678901234567890123456789012123456 7 12 1234567890123456789012345678901212345678901234567890123456789012123456 7 1234567890123456789012345678901212345678901234567890123456789012123456 7 34567890123456789012345678901212345678901234567890123456789012123456 7 12 34567890123456789012345678901212345678901234567890123456789012123456 7 12 34567890123456789012345678901212345678901234567890123456789012123456 7 12 1234567890123456789012345678901212345678901234567890123456789012123456 7 34567890123456789012345678901212345678901234567890123456789012123456 7 12 34567890123456789012345678901212345678901234567890123456789012123456 7 12 2 7 1234567890123456789012345678901212345678901234567890123456789012123456 7 1234567890123456789012345678901212345678901234567890123456789012123456 1 34567890123456789012345678901212345678901234567890123456789012123456 7 1234567890123456789012345678901212345678901234567890123456789012123456 7 1234567890123456789012345678901212345678901234567890123456789012123456 7 34567890123456789012345678901212345678901234567890123456789012123456 7 12 34567890123456789012345678901212345678901234567890123456789012123456 7 12 34567890123456789012345678901212345678901234567890123456789012123456 7 12 34567890123456789012345678901212345678901234567890123456789012123456 7 12 1234567890123456789012345678901212345678901234567890123456789012123456 7 2 7 1234567890123456789012345678901212345678901234567890123456789012123456 7 1234567890123456789012345678901212345678901234567890123456789012123456 7 1234567890123456789012345678901212345678901234567890123456789012123456 7 1234567890123456789012345678901212345678901234567890123456789012123456 1 34567890123456789012345678901212345678901234567890123456789012123456 7 2 34567890123456789012345678901212345678901234567890123456789012123456 1234567890123456789012345678901212345678901234567890123456789012123456 77 1234567890123456789012345678901212345678901234567890123456789012123456 7 1234567890123456789012345678901212345678901234567890123456789012123456 7 1234567890123456789012345678901212345678901234567890123456789012123456 7 1 34567890123456789012345678901212345678901234567890123456789012123456 7 12 1234567890123456789012345678901212345678901234567890123456789012123456 7 34567890123456789012345678901212345678901234567890123456789012123456 7 12 1234567890123456789012345678901212345678901234567890123456789012123456 7 34567890123456789012345678901212345678901234567890123456789012123456 7 12 When you are finished with your essay put your pencil down until the 7 1234567890123456789012345678901212345678901234567890123456789012123456 7 1234567890123456789012345678901212345678901234567890123456789012123456 time allotted is over. 1234567890123456789012345678901212345678901234567890123456789012123456 77 1234567890123456789012345678901212345678901234567890123456789012123456 12345678901234567890123456789012123456789012345678901234567890121234567

STOP

46

Copyright © 2005 Thomson Peterson’s, a part of The Thomson Corporaton SAT is a registered trademark of the College Entrance Examination Board, which was not involved in the production of and does not endorse this product.

Answers and Explanations 1234567890123456789012345678901212345678901234567890123456789 9 123456789012345678901234567890121234567890123456789012345678 9 123456789012345678901234567890121234567890123456789012345678 9 123456789012345678901234567890121234567890123456789012345678 Section 1 9 123456789012345678901234567890121234567890123456789012345678 9 123456789012345678901234567890121234567890123456789012345678 23456789012345678901234567890121234567890123456789012345678 9 123456789012345678901234567890121234567890123456789012345678 1. B The passage says that H.R. means House of Representatives and that 9 1 9 123456789012345678901234567890121234567890123456789012345678 the number is given according to, “the order it was submitted in a 9 123456789012345678901234567890121234567890123456789012345678 9 123456789012345678901234567890121234567890123456789012345678 particular session.” (lines 12–13) So the 1 means that it was submitted 23456789012345678901234567890121234567890123456789012345678 123456789012345678901234567890121234567890123456789012345678 99 123456789012345678901234567890121234567890123456789012345678 first in a particular session. Choice (B) is the correct answer. 9 123456789012345678901234567890121234567890123456789012345678 9 123456789012345678901234567890121234567890123456789012345678 2. C This is a tricky question. The major clue is that only three entities are 9 123456789012345678901234567890121234567890123456789012345678 9 123456789012345678901234567890121234567890123456789012345678 mentioned in the passage: the House of Representatives, the Senate, and 9 123456789012345678901234567890121234567890123456789012345678 9 123456789012345678901234567890121234567890123456789012345678 the President. You should suspect then that the answer will be the 9 123456789012345678901234567890121234567890123456789012345678 9 1 Senate and the President since those are the only other two entities 23456789012345678901234567890121234567890123456789012345678 123456789012345678901234567890121234567890123456789012345678 99 123456789012345678901234567890121234567890123456789012345678 mentioned in the passage. You can get confirmation for this hunch in 9 1 9 123456789012345678901234567890121234567890123456789012345678 the sentence that says “Bills are presented to the President for action 9 123456789012345678901234567890121234567890123456789012345678 23456789012345678901234567890121234567890123456789012345678 9 123456789012345678901234567890121234567890123456789012345678 when approved in identical form by both the House of Representatives 9 123456789012345678901234567890121234567890123456789012345678 9 1 and the Senate.” Bills must pass both houses, so if the bills originate in 9 123456789012345678901234567890121234567890123456789012345678 9 123456789012345678901234567890121234567890123456789012345678 the house, they have to go to the Senate before reaching the President. 23456789012345678901234567890121234567890123456789012345678 123456789012345678901234567890121234567890123456789012345678 99 123456789012345678901234567890121234567890123456789012345678 The answer is (C). 9 1 123456789012345678901234567890121234567890123456789012345678 9 9 123456789012345678901234567890121234567890123456789012345678 3. A The paragraph is about how Native Americans have “long recognized 9 123456789012345678901234567890121234567890123456789012345678 9 123456789012345678901234567890121234567890123456789012345678 and celebrated the connectedness among all natural things.” The 9 123456789012345678901234567890121234567890123456789012345678 9 123456789012345678901234567890121234567890123456789012345678 medicine wheel illustrates this point because it emphasizes the 9 123456789012345678901234567890121234567890123456789012345678 connectedness of the four basic elements. So (A) is the best choice. 23456789012345678901234567890121234567890123456789012345678 123456789012345678901234567890121234567890123456789012345678 99 123456789012345678901234567890121234567890123456789012345678 9 123456789012345678901234567890121234567890123456789012345678 4. E Remember the Native American view stated in the passage is that all 9 123456789012345678901234567890121234567890123456789012345678 9 1 natural things are connected; so we should expect that the scenario will 23456789012345678901234567890121234567890123456789012345678 123456789012345678901234567890121234567890123456789012345678 99 123456789012345678901234567890121234567890123456789012345678 demonstrate this belief. (A) is specifically opposed to that. (C) is also, to 9 123456789012345678901234567890121234567890123456789012345678 9 123456789012345678901234567890121234567890123456789012345678 a lesser degree. (B) is neutral toward the Native American idea—it 9 1 23456789012345678901234567890121234567890123456789012345678 9 123456789012345678901234567890121234567890123456789012345678 neither contradicts it nor realizes it. (D) and (E) are in accord with the 9 1 9 123456789012345678901234567890121234567890123456789012345678 Native American view because they recognize the connectedness of an 123456789012345678901234567890121234567890123456789012345678 9 9 123456789012345678901234567890121234567890123456789012345678 organism and its habitat. However, (D) mention studying a specific 9 123456789012345678901234567890121234567890123456789012345678 123456789012345678901234567890121234567890123456789012345678 9 123456789012345678901234567890121234567890123456789012345678 9 123456789012345678901234567890121234567890123456789012345678 9 1234567890123456789012345678901212345678901234567890123456789 Copyright © 2005 Thomson Peterson’s, a part of The Thomson Corporaton SAT is a registered trademark of the College Entrance Examination Board, which was not involved in the production of and does not endorse this product.

47

1234567890123456789012345678901212345678901234567890123456789 123456789012345678901234567890121234567890123456789012345678 9 9 123456789012345678901234567890121234567890123456789012345678 123456789012345678901234567890121234567890123456789012345678 9 9 123456789012345678901234567890121234567890123456789012345678 organism in its habitat, while (E) mentions studying a habitat as a 9 123456789012345678901234567890121234567890123456789012345678 9 123456789012345678901234567890121234567890123456789012345678 whole. So, (E) is broader and, in this case, broader is better. The best 9 123456789012345678901234567890121234567890123456789012345678 23456789012345678901234567890121234567890123456789012345678 9 1 answer is (E). 9 123456789012345678901234567890121234567890123456789012345678 9 123456789012345678901234567890121234567890123456789012345678 9 123456789012345678901234567890121234567890123456789012345678 5. C Does the author portray Copland’s music in a very negative or very 9 123456789012345678901234567890121234567890123456789012345678 23456789012345678901234567890121234567890123456789012345678 9 1 positive light? Neither, so (A) and (E) are out. The author actually just 9 123456789012345678901234567890121234567890123456789012345678 9 123456789012345678901234567890121234567890123456789012345678 describes the reactions of others to Copland’s music and interjects little 9 123456789012345678901234567890121234567890123456789012345678 9 123456789012345678901234567890121234567890123456789012345678 opinion of his or her own. That eliminates (D). Then again, the author 23456789012345678901234567890121234567890123456789012345678 9 1 9 123456789012345678901234567890121234567890123456789012345678 is not clinically objective in tone (just think, the passage does not sound 9 123456789012345678901234567890121234567890123456789012345678 9 like a science passage). That makes (C) the best choice. 123456789012345678901234567890121234567890123456789012345678 9 123456789012345678901234567890121234567890123456789012345678 23456789012345678901234567890121234567890123456789012345678 9 1 6. C The virtuoso passages are passages only a virtuoso could play— 9 123456789012345678901234567890121234567890123456789012345678 9 123456789012345678901234567890121234567890123456789012345678 essentially, difficult. Eliminate (E). There is no negative judgment of 9 123456789012345678901234567890121234567890123456789012345678 9 123456789012345678901234567890121234567890123456789012345678 that quality in the passage. That eliminates all but (C), which is the 23456789012345678901234567890121234567890123456789012345678 123456789012345678901234567890121234567890123456789012345678 9 9 123456789012345678901234567890121234567890123456789012345678 answer. 9 123456789012345678901234567890121234567890123456789012345678 123456789012345678901234567890121234567890123456789012345678 9 9 123456789012345678901234567890121234567890123456789012345678 7. C The different fates were that the Piano Variation was known by many 9 123456789012345678901234567890121234567890123456789012345678 9 1 pianists, but played by few in concert, and the Symphonic Ode was 9 123456789012345678901234567890121234567890123456789012345678 9 123456789012345678901234567890121234567890123456789012345678 largely unknown because it was too difficult. Choice (C) says as much, 9 123456789012345678901234567890121234567890123456789012345678 23456789012345678901234567890121234567890123456789012345678 9 123456789012345678901234567890121234567890123456789012345678 and so is the answer. 9 123456789012345678901234567890121234567890123456789012345678 9 123456789012345678901234567890121234567890123456789012345678 8. A In the passage, Koussevitzky is first mentioned in connection to the 9 123456789012345678901234567890121234567890123456789012345678 9 123456789012345678901234567890121234567890123456789012345678 difficulty of playing the Symphonic Ode. The passage tells you that 123456789012345678901234567890121234567890123456789012345678 9 9 123456789012345678901234567890121234567890123456789012345678 even he, a “champion” of Copland’s music, did not conduct the piece 9 123456789012345678901234567890121234567890123456789012345678 9 123456789012345678901234567890121234567890123456789012345678 until it had been simplified. Choice (A) correctly points this out. Don’t 9 1 23456789012345678901234567890121234567890123456789012345678 9 123456789012345678901234567890121234567890123456789012345678 be tricked by the difficult name into answering that he was European. 9 123456789012345678901234567890121234567890123456789012345678 9 1 9 123456789012345678901234567890121234567890123456789012345678 9. C The second paragraph begins, “Perhaps as a reaction to the perfor9 123456789012345678901234567890121234567890123456789012345678 23456789012345678901234567890121234567890123456789012345678 9 123456789012345678901234567890121234567890123456789012345678 mance problems of the Symphonic Ode, Copland’s next two orchestral 9 123456789012345678901234567890121234567890123456789012345678 9 1 works deal in shorter units of time.” So the author thinks that the pieces 9 123456789012345678901234567890121234567890123456789012345678 9 123456789012345678901234567890121234567890123456789012345678 immediately subsequent to the Symphonic Ode were made shorter 23456789012345678901234567890121234567890123456789012345678 123456789012345678901234567890121234567890123456789012345678 9 9 123456789012345678901234567890121234567890123456789012345678 because the Ode was not being performed. Choice (C) says the same. 9 1 9 123456789012345678901234567890121234567890123456789012345678 (B) is tempting but the passage explicitly states that it was the length 123456789012345678901234567890121234567890123456789012345678 9 23456789012345678901234567890121234567890123456789012345678 9 123456789012345678901234567890121234567890123456789012345678 and not the complexity that Copland adjusted in the subsequent pieces. 9 123456789012345678901234567890121234567890123456789012345678 9 123456789012345678901234567890121234567890123456789012345678 10. E This sentence is essentially saying that, although brief, these works were 9 123456789012345678901234567890121234567890123456789012345678 9 1 also very difficult to perform. The answer is (E). This is essentially a 23456789012345678901234567890121234567890123456789012345678 123456789012345678901234567890121234567890123456789012345678 9 9 123456789012345678901234567890121234567890123456789012345678 vocabulary question, asking you to define “agile.” 9 123456789012345678901234567890121234567890123456789012345678 123456789012345678901234567890121234567890123456789012345678 9 9 1 11. C First off, what does the passage state about Copland’s views of the 9 123456789012345678901234567890121234567890123456789012345678 9 123456789012345678901234567890121234567890123456789012345678 Americas and music? It says that Copland, “envisioned ‘American 9 123456789012345678901234567890121234567890123456789012345678 9 123456789012345678901234567890121234567890123456789012345678 music’ as being music of the Americas.” In other words, he thought 123456789012345678901234567890121234567890123456789012345678 9 23456789012345678901234567890121234567890123456789012345678 9 123456789012345678901234567890121234567890123456789012345678 they were related, or should be. (A), (B), and (E) run contrary to this 9 1 9 123456789012345678901234567890121234567890123456789012345678 meaning. (D) is irrelevant. The answer is (C). 123456789012345678901234567890121234567890123456789012345678 9 9 123456789012345678901234567890121234567890123456789012345678 123456789012345678901234567890121234567890123456789012345678 9 123456789012345678901234567890121234567890123456789012345678 9 123456789012345678901234567890121234567890123456789012345678 9 123456789012345678901234567890121234567890123456789012345678 9 1234567890123456789012345678901212345678901234567890123456789 48

Copyright © 2005 Thomson Peterson’s, a part of The Thomson Corporaton SAT is a registered trademark of the College Entrance Examination Board, which was not involved in the production of and does not endorse this product.

1234567890123456789012345678901212345678901234567890123456789 123456789012345678901234567890121234567890123456789012345678 9 9 123456789012345678901234567890121234567890123456789012345678 123456789012345678901234567890121234567890123456789012345678 9 9 123456789012345678901234567890121234567890123456789012345678 12. D The Mexican material is more accessible to audiences. The opposite of 9 123456789012345678901234567890121234567890123456789012345678 9 123456789012345678901234567890121234567890123456789012345678 accessible is inaccessible. (D) should look good to you right away. 9 123456789012345678901234567890121234567890123456789012345678 23456789012345678901234567890121234567890123456789012345678 9 1 Don’t get confused by (C). The emphasis in the sentence is on 9 123456789012345678901234567890121234567890123456789012345678 9 123456789012345678901234567890121234567890123456789012345678 audiences, not musicians. 9 123456789012345678901234567890121234567890123456789012345678 9 123456789012345678901234567890121234567890123456789012345678 23456789012345678901234567890121234567890123456789012345678 1 13. C You can eliminate (B) because it is too specific. You can eliminate (D) 9 9 123456789012345678901234567890121234567890123456789012345678 9 123456789012345678901234567890121234567890123456789012345678 because the author never talks about eating or brushing teeth or other 9 123456789012345678901234567890121234567890123456789012345678 9 123456789012345678901234567890121234567890123456789012345678 daily activities. (E) is also too specific. (A) is really off in emphasis; the 23456789012345678901234567890121234567890123456789012345678 9 1 9 123456789012345678901234567890121234567890123456789012345678 author focuses on describing the Earth’s appearance more than the 9 123456789012345678901234567890121234567890123456789012345678 9 trajectory of the Space Station. The answer is (C). It’s also a good 123456789012345678901234567890121234567890123456789012345678 9 123456789012345678901234567890121234567890123456789012345678 answer because it captures a little bit of the tone of the piece; the author 23456789012345678901234567890121234567890123456789012345678 9 1 9 123456789012345678901234567890121234567890123456789012345678 seems to write the passage for an average person experiencing curiosity 9 123456789012345678901234567890121234567890123456789012345678 9 123456789012345678901234567890121234567890123456789012345678 about seeing the Earth from space. 9 123456789012345678901234567890121234567890123456789012345678 23456789012345678901234567890121234567890123456789012345678 123456789012345678901234567890121234567890123456789012345678 99 123456789012345678901234567890121234567890123456789012345678 14. E The second half of the paragraph contrasts seeing the whole Earth to 9 123456789012345678901234567890121234567890123456789012345678 9 123456789012345678901234567890121234567890123456789012345678 seeing just a limited part of it. But the emphasis is on seeing the whole 123456789012345678901234567890121234567890123456789012345678 99 123456789012345678901234567890121234567890123456789012345678 Earth, because we’ve all seen limited parts of it. The answer is (E). 9 1 9 123456789012345678901234567890121234567890123456789012345678 9 123456789012345678901234567890121234567890123456789012345678 15. B Here you have to be careful because two different window views are 9 123456789012345678901234567890121234567890123456789012345678 23456789012345678901234567890121234567890123456789012345678 9 123456789012345678901234567890121234567890123456789012345678 discussed in the passage. The downward-facing window is discussed in 9 123456789012345678901234567890121234567890123456789012345678 9 123456789012345678901234567890121234567890123456789012345678 the second half of the second paragraph. And there the author 123456789012345678901234567890121234567890123456789012345678 99 123456789012345678901234567890121234567890123456789012345678 compares the view to looking at a big blue beach ball up-close, choice 9 123456789012345678901234567890121234567890123456789012345678 9 123456789012345678901234567890121234567890123456789012345678 (B). 123456789012345678901234567890121234567890123456789012345678 99 123456789012345678901234567890121234567890123456789012345678 1 16. B This is where that second window view comes in. The author first 9 9 123456789012345678901234567890121234567890123456789012345678 discusses the downward-facing window and then contrasts it with 9 123456789012345678901234567890121234567890123456789012345678 9 123456789012345678901234567890121234567890123456789012345678 looking through a sideward-facing window in the third paragraph. (B) 123456789012345678901234567890121234567890123456789012345678 9 9 123456789012345678901234567890121234567890123456789012345678 is the answer. 9 123456789012345678901234567890121234567890123456789012345678 9 123456789012345678901234567890121234567890123456789012345678 9 123456789012345678901234567890121234567890123456789012345678 17. B The passage discusses the “faint glow” in the last sentence of the third 9 123456789012345678901234567890121234567890123456789012345678 9 123456789012345678901234567890121234567890123456789012345678 paragraph. There, he attributes it to the outer rim of the atmosphere. 23456789012345678901234567890121234567890123456789012345678 123456789012345678901234567890121234567890123456789012345678 99 123456789012345678901234567890121234567890123456789012345678 (B) says the same thing. (A) mentions the atmosphere, but it designates 9 1 9 123456789012345678901234567890121234567890123456789012345678 the wrong part of the atmosphere. 123456789012345678901234567890121234567890123456789012345678 9 23456789012345678901234567890121234567890123456789012345678 9 123456789012345678901234567890121234567890123456789012345678 18. B Before the thought exercise is given the author states that, “a good way 9 123456789012345678901234567890121234567890123456789012345678 9 123456789012345678901234567890121234567890123456789012345678 to imagine our view is to stand up and look down at your feet.” In 9 123456789012345678901234567890121234567890123456789012345678 9 1 “imagining the view” through the thought exercise the reader gets a 23456789012345678901234567890121234567890123456789012345678 123456789012345678901234567890121234567890123456789012345678 99 123456789012345678901234567890121234567890123456789012345678 definite idea of the proportions of things within the space stations view 9 123456789012345678901234567890121234567890123456789012345678 9 123456789012345678901234567890121234567890123456789012345678 (e.g. where San Francisco is in relation to Denver). Choice (B) says as 9 1 23456789012345678901234567890121234567890123456789012345678 9 123456789012345678901234567890121234567890123456789012345678 much, and so is the best answer. Choice (C) is a tempting answer, 9 123456789012345678901234567890121234567890123456789012345678 9 123456789012345678901234567890121234567890123456789012345678 because it is the idea with which the paragraph begins, but it ends up 123456789012345678901234567890121234567890123456789012345678 99 1 being too narrow a description of the thought exercise. 9 123456789012345678901234567890121234567890123456789012345678 123456789012345678901234567890121234567890123456789012345678 9 9 123456789012345678901234567890121234567890123456789012345678 123456789012345678901234567890121234567890123456789012345678 9 9 123456789012345678901234567890121234567890123456789012345678 123456789012345678901234567890121234567890123456789012345678 9 123456789012345678901234567890121234567890123456789012345678 9 123456789012345678901234567890121234567890123456789012345678 9 123456789012345678901234567890121234567890123456789012345678 9 1234567890123456789012345678901212345678901234567890123456789 Copyright © 2005 Thomson Peterson’s, a part of The Thomson Corporaton SAT is a registered trademark of the College Entrance Examination Board, which was not involved in the production of and does not endorse this product.

49

1234567890123456789012345678901212345678901234567890123456789 123456789012345678901234567890121234567890123456789012345678 9 9 123456789012345678901234567890121234567890123456789012345678 123456789012345678901234567890121234567890123456789012345678 9 9 123456789012345678901234567890121234567890123456789012345678 19. D You may be tempted to answer (A), because the passage can be 9 123456789012345678901234567890121234567890123456789012345678 9 123456789012345678901234567890121234567890123456789012345678 confusing. But it’s not dry or scientific, so “technical” just isn’t a good 9 123456789012345678901234567890121234567890123456789012345678 23456789012345678901234567890121234567890123456789012345678 9 1 word for it. Choice (B) doesn’t capture it’s informality. Choice (E) is too 9 123456789012345678901234567890121234567890123456789012345678 9 123456789012345678901234567890121234567890123456789012345678 extreme—as far as poetic descriptions of the planet go, this one doesn’t 9 123456789012345678901234567890121234567890123456789012345678 9 123456789012345678901234567890121234567890123456789012345678 even rate. That leaves (C) and (D). Choice (C) isn’t quite right, either, 23456789012345678901234567890121234567890123456789012345678 9 1 9 123456789012345678901234567890121234567890123456789012345678 because it’s not irreverent, it’s just casual. Choice (D) best captures that 9 123456789012345678901234567890121234567890123456789012345678 9 123456789012345678901234567890121234567890123456789012345678 sense. 9 123456789012345678901234567890121234567890123456789012345678 23456789012345678901234567890121234567890123456789012345678 9 1 9 123456789012345678901234567890121234567890123456789012345678 20. D You don’t know a lot about what the astronauts do from the passage, 9 123456789012345678901234567890121234567890123456789012345678 9 so you’ll have to dig this answer up. Choice (A) seems silly, and would 123456789012345678901234567890121234567890123456789012345678 9 123456789012345678901234567890121234567890123456789012345678 really be a disrespectful thing to say about the author. The SAT won’t 23456789012345678901234567890121234567890123456789012345678 9 1 9 123456789012345678901234567890121234567890123456789012345678 do that. So eliminate (A). There’s no reference to gravity in the passage, 9 123456789012345678901234567890121234567890123456789012345678 9 123456789012345678901234567890121234567890123456789012345678 so eliminate (B). There’s no real reference to physical activity, so (C) 9 123456789012345678901234567890121234567890123456789012345678 23456789012345678901234567890121234567890123456789012345678 9 123456789012345678901234567890121234567890123456789012345678 seems wrong. You are left with (D) and (E). The astronaut uses a lot of 9 123456789012345678901234567890121234567890123456789012345678 9 123456789012345678901234567890121234567890123456789012345678 numbers, and throws them around as though they were quite easy. (D) 123456789012345678901234567890121234567890123456789012345678 9 9 123456789012345678901234567890121234567890123456789012345678 seems like a reasonable answer. There’s no reference in the passage to 9 123456789012345678901234567890121234567890123456789012345678 9 1 communication procedures, which means you can eliminate (E) even 9 123456789012345678901234567890121234567890123456789012345678 9 123456789012345678901234567890121234567890123456789012345678 though it’s sort of a tempting answer. 9 123456789012345678901234567890121234567890123456789012345678 23456789012345678901234567890121234567890123456789012345678 123456789012345678901234567890121234567890123456789012345678 9 123456789012345678901234567890121234567890123456789012345678 9 123456789012345678901234567890121234567890123456789012345678 9 9 123456789012345678901234567890121234567890123456789012345678 Section 2 9 123456789012345678901234567890121234567890123456789012345678 123456789012345678901234567890121234567890123456789012345678 9 9 123456789012345678901234567890121234567890123456789012345678 1. C This is the first question, so it should be one of the easiest, if not the 9 123456789012345678901234567890121234567890123456789012345678 9 123456789012345678901234567890121234567890123456789012345678 easiest, questions in the section. For this reason, you don’t have to 9 1 23456789012345678901234567890121234567890123456789012345678 9 123456789012345678901234567890121234567890123456789012345678 expect anything too tricky about this problem. Substitute x 5 2y into 9 123456789012345678901234567890121234567890123456789012345678 the first equation and solve for x: 9 1 123456789012345678901234567890121234567890123456789012345678 9 9 123456789012345678901234567890121234567890123456789012345678 9 123456789012345678901234567890121234567890123456789012345678 x 1 4y 5 3 9 123456789012345678901234567890121234567890123456789012345678 9 123456789012345678901234567890121234567890123456789012345678 2y 1 4y 5 3 9 123456789012345678901234567890121234567890123456789012345678 9 123456789012345678901234567890121234567890123456789012345678 3y 5 3 23456789012345678901234567890121234567890123456789012345678 123456789012345678901234567890121234567890123456789012345678 9 9 123456789012345678901234567890121234567890123456789012345678 3y 3 9 1 9 123456789012345678901234567890121234567890123456789012345678 5 9 123456789012345678901234567890121234567890123456789012345678 3 3 23456789012345678901234567890121234567890123456789012345678 123456789012345678901234567890121234567890123456789012345678 9 9 123456789012345678901234567890121234567890123456789012345678 y51 9 123456789012345678901234567890121234567890123456789012345678 123456789012345678901234567890121234567890123456789012345678 9 9 1 (C) is the answer. 9 123456789012345678901234567890121234567890123456789012345678 9 123456789012345678901234567890121234567890123456789012345678 2. B There are two main parts to this problem: 9 123456789012345678901234567890121234567890123456789012345678 9 123456789012345678901234567890121234567890123456789012345678 9 123456789012345678901234567890121234567890123456789012345678 1. the use of the variable d, and 23456789012345678901234567890121234567890123456789012345678 123456789012345678901234567890121234567890123456789012345678 9 123456789012345678901234567890121234567890123456789012345678 9 9 123456789012345678901234567890121234567890123456789012345678 2. the change of units from hours to minutes. 9 123456789012345678901234567890121234567890123456789012345678 9 1 23456789012345678901234567890121234567890123456789012345678 9 123456789012345678901234567890121234567890123456789012345678 Start with the variable since d is half as much as 2d, it should take him 9 1 9 123456789012345678901234567890121234567890123456789012345678 half the amount of time, or 1.5 hours. One hour is 60 minutes, and half 123456789012345678901234567890121234567890123456789012345678 9 9 123456789012345678901234567890121234567890123456789012345678 an hour is 30 minutes, so 1.5 hours is 90 minutes. That’s choice (B). 9 123456789012345678901234567890121234567890123456789012345678 123456789012345678901234567890121234567890123456789012345678 9 123456789012345678901234567890121234567890123456789012345678 9 123456789012345678901234567890121234567890123456789012345678 9 1234567890123456789012345678901212345678901234567890123456789 50

Copyright © 2005 Thomson Peterson’s, a part of The Thomson Corporaton SAT is a registered trademark of the College Entrance Examination Board, which was not involved in the production of and does not endorse this product.

Tip

1234567890123456789012345678901212345678901234567890123456789 123456789012345678901234567890121234567890123456789012345678 9 9 123456789012345678901234567890121234567890123456789012345678 123456789012345678901234567890121234567890123456789012345678 9 9 123456789012345678901234567890121234567890123456789012345678 3. A There is more than one way to solve this problem, but all paths involve 9 123456789012345678901234567890121234567890123456789012345678 9 123456789012345678901234567890121234567890123456789012345678 manipulating the equation. You can solve for f in the first equation, and 9 123456789012345678901234567890121234567890123456789012345678 23456789012345678901234567890121234567890123456789012345678 9 1 then plug that value into the second equation. You could also just leave 9 123456789012345678901234567890121234567890123456789012345678 9 123456789012345678901234567890121234567890123456789012345678 the 3f as is, and monkey with the formula this way: 9 123456789012345678901234567890121234567890123456789012345678 9 123456789012345678901234567890121234567890123456789012345678 23456789012345678901234567890121234567890123456789012345678 9 1 3f 1 15 5 27 9 123456789012345678901234567890121234567890123456789012345678 9 123456789012345678901234567890121234567890123456789012345678 3f 1 15 2 15 5 27 2 15 9 123456789012345678901234567890121234567890123456789012345678 9 123456789012345678901234567890121234567890123456789012345678 23456789012345678901234567890121234567890123456789012345678 9 1 3f 5 12 9 123456789012345678901234567890121234567890123456789012345678 9 123456789012345678901234567890121234567890123456789012345678 3f 2 6 5 12 2 6 9 123456789012345678901234567890121234567890123456789012345678 9 123456789012345678901234567890121234567890123456789012345678 3f 2 6 5 6 23456789012345678901234567890121234567890123456789012345678 9 1 9 123456789012345678901234567890121234567890123456789012345678 9 123456789012345678901234567890121234567890123456789012345678 Whichever path you take, if you manipulate the equation correctly 9 123456789012345678901234567890121234567890123456789012345678 9 123456789012345678901234567890121234567890123456789012345678 you’ll get choice (A) as your answer. 23456789012345678901234567890121234567890123456789012345678 123456789012345678901234567890121234567890123456789012345678 99 123456789012345678901234567890121234567890123456789012345678 9 123456789012345678901234567890121234567890123456789012345678 4. A The first thing to notice is that this triangle is a 30-60-90 triangle, one 9 123456789012345678901234567890121234567890123456789012345678 9 123456789012345678901234567890121234567890123456789012345678 of the SAT’s favorite triangles. Since you can read off the relationships 123456789012345678901234567890121234567890123456789012345678 99 1 between different sides of a 30-60-90 triangle, it can be readily seen that 9 123456789012345678901234567890121234567890123456789012345678 9 123456789012345678901234567890121234567890123456789012345678 the x 1 y side is half the length of the hypotenuse (which is 14). Half of 9 123456789012345678901234567890121234567890123456789012345678 23456789012345678901234567890121234567890123456789012345678 14 is 7, so (A) is the answer. 123456789012345678901234567890121234567890123456789012345678 99 123456789012345678901234567890121234567890123456789012345678 9 123456789012345678901234567890121234567890123456789012345678 5. D Translating the English into algebra is the key to all word problems. 9 123456789012345678901234567890121234567890123456789012345678 123456789012345678901234567890121234567890123456789012345678 99 123456789012345678901234567890121234567890123456789012345678 Since the snack costs twenty cents less than the drink, you can write 9 123456789012345678901234567890121234567890123456789012345678 9 123456789012345678901234567890121234567890123456789012345678 down d 2 20 5 s. Since a snack and drink together costs $1.30, you 123456789012345678901234567890121234567890123456789012345678 99 1 also know that s 1 d 5 130. You have two equations and two variables. 9 123456789012345678901234567890121234567890123456789012345678 9 123456789012345678901234567890121234567890123456789012345678 Substitute the first equation into the second and then solve: 123456789012345678901234567890121234567890123456789012345678 9 123456789012345678901234567890121234567890123456789012345678 9 9 123456789012345678901234567890121234567890123456789012345678 s 1 d 5 130 9 123456789012345678901234567890121234567890123456789012345678 9 123456789012345678901234567890121234567890123456789012345678 ~d 2 20! 1 d 5 130 9 123456789012345678901234567890121234567890123456789012345678 9 123456789012345678901234567890121234567890123456789012345678 9 123456789012345678901234567890121234567890123456789012345678 2d 2 20 5 130 23456789012345678901234567890121234567890123456789012345678 123456789012345678901234567890121234567890123456789012345678 99 123456789012345678901234567890121234567890123456789012345678 2d 2 20 1 20 5 130 1 20 9 1 9 123456789012345678901234567890121234567890123456789012345678 2d 5 150 123456789012345678901234567890121234567890123456789012345678 9 23456789012345678901234567890121234567890123456789012345678 9 123456789012345678901234567890121234567890123456789012345678 2d 150 9 123456789012345678901234567890121234567890123456789012345678 9 123456789012345678901234567890121234567890123456789012345678 5 9 123456789012345678901234567890121234567890123456789012345678 2 2 9 1 23456789012345678901234567890121234567890123456789012345678 9 123456789012345678901234567890121234567890123456789012345678 d 5 75 9 123456789012345678901234567890121234567890123456789012345678 123456789012345678901234567890121234567890123456789012345678 99 123456789012345678901234567890121234567890123456789012345678 A drink costs 75 cents ($0.75), so (D) is the answer. 9 1 23456789012345678901234567890121234567890123456789012345678 123456789012345678901234567890121234567890123456789012345678 99 123456789012345678901234567890121234567890123456789012345678 9 123456789012345678901234567890121234567890123456789012345678 For problems dealing with units of money, always decide whether you 9 123456789012345678901234567890121234567890123456789012345678 9 1 want to work in dollars or cents. If the amount of money is great, using 23456789012345678901234567890121234567890123456789012345678 9 123456789012345678901234567890121234567890123456789012345678 dollars is typically the best way to go. On this problem, converting to 9 1 9 123456789012345678901234567890121234567890123456789012345678 cents might work better since you won’t have to deal with any decimals. 9 123456789012345678901234567890121234567890123456789012345678 9 123456789012345678901234567890121234567890123456789012345678 23456789012345678901234567890121234567890123456789012345678 9 1 123456789012345678901234567890121234567890123456789012345678 9 123456789012345678901234567890121234567890123456789012345678 9 123456789012345678901234567890121234567890123456789012345678 9 1234567890123456789012345678901212345678901234567890123456789 Copyright © 2005 Thomson Peterson’s, a part of The Thomson Corporaton SAT is a registered trademark of the College Entrance Examination Board, which was not involved in the production of and does not endorse this product.

51

1234567890123456789012345678901212345678901234567890123456789 123456789012345678901234567890121234567890123456789012345678 9 9 123456789012345678901234567890121234567890123456789012345678 123456789012345678901234567890121234567890123456789012345678 9 9 123456789012345678901234567890121234567890123456789012345678 6. D This problem looks really complicated, but don’t let that ruffle your test 9 123456789012345678901234567890121234567890123456789012345678 9 123456789012345678901234567890121234567890123456789012345678 feathers. With function problems, just plug in whatever the problem 9 123456789012345678901234567890121234567890123456789012345678 23456789012345678901234567890121234567890123456789012345678 9 1 tells you to plug into the equation. Be a machine! Here, you are 9 123456789012345678901234567890121234567890123456789012345678 9 123456789012345678901234567890121234567890123456789012345678 considering when x 5 2, so f(2x) is the same thing as f(4). (Of course, 9 123456789012345678901234567890121234567890123456789012345678 9 123456789012345678901234567890121234567890123456789012345678 the question could have just said x 5 4, but that would have been too 23456789012345678901234567890121234567890123456789012345678 9 1 9 123456789012345678901234567890121234567890123456789012345678 easy). 9 123456789012345678901234567890121234567890123456789012345678 9 123456789012345678901234567890121234567890123456789012345678 9 123456789012345678901234567890121234567890123456789012345678 Now just replace every x in the problem with a 4: 23456789012345678901234567890121234567890123456789012345678 9 1 9 123456789012345678901234567890121234567890123456789012345678 x 9 123456789012345678901234567890121234567890123456789012345678 f(x) 5 x 1 x 9 123456789012345678901234567890121234567890123456789012345678 9 123456789012345678901234567890121234567890123456789012345678 4 f 4 5 4 1 4 ~ ! 23456789012345678901234567890121234567890123456789012345678 9 1 9 123456789012345678901234567890121234567890123456789012345678 9 123456789012345678901234567890121234567890123456789012345678 f ~4! 5 4 1 256 9 123456789012345678901234567890121234567890123456789012345678 9 123456789012345678901234567890121234567890123456789012345678 f 4 5 260 ~ ! 23456789012345678901234567890121234567890123456789012345678 123456789012345678901234567890121234567890123456789012345678 9 123456789012345678901234567890121234567890123456789012345678 9 9 123456789012345678901234567890121234567890123456789012345678 Your answer is (D). 9 123456789012345678901234567890121234567890123456789012345678 9 123456789012345678901234567890121234567890123456789012345678 7. D This one looks strange, and it is as straightforward to solve as it 123456789012345678901234567890121234567890123456789012345678 9 9 1 2 9 123456789012345678901234567890121234567890123456789012345678 appears. If you only consider x to be x, then you only get the = 9 123456789012345678901234567890121234567890123456789012345678 9 123456789012345678901234567890121234567890123456789012345678 extraneous solution, and not the correct one. You start by squaring 23456789012345678901234567890121234567890123456789012345678 123456789012345678901234567890121234567890123456789012345678 9 9 123456789012345678901234567890121234567890123456789012345678 both sides and then factoring. 9 123456789012345678901234567890121234567890123456789012345678 9 123456789012345678901234567890121234567890123456789012345678 2 2 9 123456789012345678901234567890121234567890123456789012345678 5 4x 1 12x 1 9 x 9 123456789012345678901234567890121234567890123456789012345678 2 9 123456789012345678901234567890121234567890123456789012345678 0 5 3x 1 12x 19 123456789012345678901234567890121234567890123456789012345678 9 9 123456789012345678901234567890121234567890123456789012345678 0 5 3~x2 1 4x 1 3! 9 1 23456789012345678901234567890121234567890123456789012345678 9 123456789012345678901234567890121234567890123456789012345678 2 9 123456789012345678901234567890121234567890123456789012345678 0 5 x 1 4x 1 3 9 1 9 123456789012345678901234567890121234567890123456789012345678 0 5 x 1 3 x 1 1 ~ !~ ! 9 123456789012345678901234567890121234567890123456789012345678 23456789012345678901234567890121234567890123456789012345678 9 123456789012345678901234567890121234567890123456789012345678 x 5 23 or x 5 21 9 123456789012345678901234567890121234567890123456789012345678 9 1 9 123456789012345678901234567890121234567890123456789012345678 Checking both solutions in the original equation, you see that only x 5 9 123456789012345678901234567890121234567890123456789012345678 23456789012345678901234567890121234567890123456789012345678 9 123456789012345678901234567890121234567890123456789012345678 21 works. This is choice (D). 9 123456789012345678901234567890121234567890123456789012345678 9 1 9 123456789012345678901234567890121234567890123456789012345678 8. E If |x 1 1| . |y|, what do you know about x and y? Not much actually. 123456789012345678901234567890121234567890123456789012345678 9 23456789012345678901234567890121234567890123456789012345678 9 123456789012345678901234567890121234567890123456789012345678 x could be 210 and y could be 5, and the inequality would be true. But 9 123456789012345678901234567890121234567890123456789012345678 9 123456789012345678901234567890121234567890123456789012345678 x could also be 10,000 and y could be 9,998, and the inequality would 123456789012345678901234567890121234567890123456789012345678 9 9 1 still be true. This means you cannot nail down the relationship between 9 123456789012345678901234567890121234567890123456789012345678 9 123456789012345678901234567890121234567890123456789012345678 x and y, so (E) is the answer. 9 123456789012345678901234567890121234567890123456789012345678 9 123456789012345678901234567890121234567890123456789012345678 123456789012345678901234567890121234567890123456789012345678 9 23456789012345678901234567890121234567890123456789012345678 123456789012345678901234567890121234567890123456789012345678 9 123456789012345678901234567890121234567890123456789012345678 9 123456789012345678901234567890121234567890123456789012345678 9 123456789012345678901234567890121234567890123456789012345678 9 9 1 9 123456789012345678901234567890121234567890123456789012345678 123456789012345678901234567890121234567890123456789012345678 9 9 123456789012345678901234567890121234567890123456789012345678 123456789012345678901234567890121234567890123456789012345678 9 9 123456789012345678901234567890121234567890123456789012345678 123456789012345678901234567890121234567890123456789012345678 9 123456789012345678901234567890121234567890123456789012345678 9 123456789012345678901234567890121234567890123456789012345678 9 123456789012345678901234567890121234567890123456789012345678 9 1234567890123456789012345678901212345678901234567890123456789 52

Copyright © 2005 Thomson Peterson’s, a part of The Thomson Corporaton SAT is a registered trademark of the College Entrance Examination Board, which was not involved in the production of and does not endorse this product.

1234567890123456789012345678901212345678901234567890123456789 123456789012345678901234567890121234567890123456789012345678 9 9 123456789012345678901234567890121234567890123456789012345678 123456789012345678901234567890121234567890123456789012345678 9 9 123456789012345678901234567890121234567890123456789012345678 9. A This question is almost all Math Speak, but hopefully you are getting 9 123456789012345678901234567890121234567890123456789012345678 9 123456789012345678901234567890121234567890123456789012345678 better at this language. The whole thing sets up an equation. “Percent” 9 123456789012345678901234567890121234567890123456789012345678 23456789012345678901234567890121234567890123456789012345678 9 1 means “divide by 100,” and “is the same value” works the same as an 9 123456789012345678901234567890121234567890123456789012345678 9 123456789012345678901234567890121234567890123456789012345678 equal sign. “What number” is Math Speak for “place a variable here.” 9 123456789012345678901234567890121234567890123456789012345678 9 123456789012345678901234567890121234567890123456789012345678 Here’s the translation: 23456789012345678901234567890121234567890123456789012345678 9 1 9 123456789012345678901234567890121234567890123456789012345678 9 123456789012345678901234567890121234567890123456789012345678 “75 percent of 104 is the same value as 60 percent of what number” 9 123456789012345678901234567890121234567890123456789012345678 9 123456789012345678901234567890121234567890123456789012345678 23456789012345678901234567890121234567890123456789012345678 9 1 75 60 9 123456789012345678901234567890121234567890123456789012345678 104 5 n ~ ! 9 123456789012345678901234567890121234567890123456789012345678 100 100 9 123456789012345678901234567890121234567890123456789012345678 9 123456789012345678901234567890121234567890123456789012345678 0.75 104 5 0.6n ~ !~ ! 23456789012345678901234567890121234567890123456789012345678 9 1 9 123456789012345678901234567890121234567890123456789012345678 9 123456789012345678901234567890121234567890123456789012345678 78 5 0.6n 9 123456789012345678901234567890121234567890123456789012345678 9 123456789012345678901234567890121234567890123456789012345678 78 0.6n 23456789012345678901234567890121234567890123456789012345678 9 123456789012345678901234567890121234567890123456789012345678 5 9 123456789012345678901234567890121234567890123456789012345678 0.6 0.6 123456789012345678901234567890121234567890123456789012345678 99 123456789012345678901234567890121234567890123456789012345678 130 5 n 9 123456789012345678901234567890121234567890123456789012345678 123456789012345678901234567890121234567890123456789012345678 99 1 Choice (A) is the answer. 9 123456789012345678901234567890121234567890123456789012345678 9 123456789012345678901234567890121234567890123456789012345678 9 123456789012345678901234567890121234567890123456789012345678 23456789012345678901234567890121234567890123456789012345678 123456789012345678901234567890121234567890123456789012345678 99 123456789012345678901234567890121234567890123456789012345678 The percentages were rewritten as decimals instead of fractions to 9 123456789012345678901234567890121234567890123456789012345678 9 make it easier to use your calculator. Dividing 78 by 0.6 is not 123456789012345678901234567890121234567890123456789012345678 9 123456789012345678901234567890121234567890123456789012345678 something most people can do easily, but for a calculator, it’s a cinch. 123456789012345678901234567890121234567890123456789012345678 99 123456789012345678901234567890121234567890123456789012345678 9 123456789012345678901234567890121234567890123456789012345678 9 123456789012345678901234567890121234567890123456789012345678 9 1 10. D Equations of lines can be put into the y 5 mx 1 b form, and then the 9 123456789012345678901234567890121234567890123456789012345678 9 123456789012345678901234567890121234567890123456789012345678 y-intercept, b, can be read off. 123456789012345678901234567890121234567890123456789012345678 9 123456789012345678901234567890121234567890123456789012345678 9 9 123456789012345678901234567890121234567890123456789012345678 3y 2 x 5 12 9 123456789012345678901234567890121234567890123456789012345678 9 123456789012345678901234567890121234567890123456789012345678 3y 2 x 1 x 5 12 1 x 9 123456789012345678901234567890121234567890123456789012345678 9 123456789012345678901234567890121234567890123456789012345678 9 123456789012345678901234567890121234567890123456789012345678 3y 5 x 1 12 23456789012345678901234567890121234567890123456789012345678 123456789012345678901234567890121234567890123456789012345678 99 123456789012345678901234567890121234567890123456789012345678 3y x 12 9 1 5 1 9 123456789012345678901234567890121234567890123456789012345678 3 3 3 123456789012345678901234567890121234567890123456789012345678 9 23456789012345678901234567890121234567890123456789012345678 9 123456789012345678901234567890121234567890123456789012345678 x 9 123456789012345678901234567890121234567890123456789012345678 y5 14 9 123456789012345678901234567890121234567890123456789012345678 3 9 123456789012345678901234567890121234567890123456789012345678 9 1 23456789012345678901234567890121234567890123456789012345678 9 123456789012345678901234567890121234567890123456789012345678 If 4 is the y-intercept, and twice this number is 8, choice (D). 9 123456789012345678901234567890121234567890123456789012345678 123456789012345678901234567890121234567890123456789012345678 99 123456789012345678901234567890121234567890123456789012345678 9 1 9 123456789012345678901234567890121234567890123456789012345678 9 123456789012345678901234567890121234567890123456789012345678 9 123456789012345678901234567890121234567890123456789012345678 9 123456789012345678901234567890121234567890123456789012345678 123456789012345678901234567890121234567890123456789012345678 9 23456789012345678901234567890121234567890123456789012345678 123456789012345678901234567890121234567890123456789012345678 99 1 9 123456789012345678901234567890121234567890123456789012345678 123456789012345678901234567890121234567890123456789012345678 9 9 123456789012345678901234567890121234567890123456789012345678 123456789012345678901234567890121234567890123456789012345678 9 123456789012345678901234567890121234567890123456789012345678 9 123456789012345678901234567890121234567890123456789012345678 9 123456789012345678901234567890121234567890123456789012345678 9 1234567890123456789012345678901212345678901234567890123456789

S D

Note

S D

Copyright © 2005 Thomson Peterson’s, a part of The Thomson Corporaton SAT is a registered trademark of the College Entrance Examination Board, which was not involved in the production of and does not endorse this product.

53

1234567890123456789012345678901212345678901234567890123456789 123456789012345678901234567890121234567890123456789012345678 9 9 123456789012345678901234567890121234567890123456789012345678 123456789012345678901234567890121234567890123456789012345678 9 9 123456789012345678901234567890121234567890123456789012345678 11. C 9 123456789012345678901234567890121234567890123456789012345678 123456789012345678901234567890121234567890123456789012345678 9 9 123456789012345678901234567890121234567890123456789012345678 123456789012345678901234567890121234567890123456789012345678 9 9 123456789012345678901234567890121234567890123456789012345678 9 123456789012345678901234567890121234567890123456789012345678 9 123456789012345678901234567890121234567890123456789012345678 9 123456789012345678901234567890121234567890123456789012345678 23456789012345678901234567890121234567890123456789012345678 9 1 9 123456789012345678901234567890121234567890123456789012345678 9 123456789012345678901234567890121234567890123456789012345678 9 123456789012345678901234567890121234567890123456789012345678 As you can see from this figure a 5 a, and b 5 b because alternate 9 123456789012345678901234567890121234567890123456789012345678 23456789012345678901234567890121234567890123456789012345678 9 1 interior angles are congruent. If two angles of a triangle are congruent 9 123456789012345678901234567890121234567890123456789012345678 9 123456789012345678901234567890121234567890123456789012345678 to two angles of another triangle, what can you deduce about the 9 123456789012345678901234567890121234567890123456789012345678 relationship between the third angles? The third angles must also be 9 123456789012345678901234567890121234567890123456789012345678 23456789012345678901234567890121234567890123456789012345678 9 1 congruent. If you don’t see this, pick values for a and b and remember 9 123456789012345678901234567890121234567890123456789012345678 9 123456789012345678901234567890121234567890123456789012345678 that the sum of the mesures of the interior angles of a triangle is 180. 9 123456789012345678901234567890121234567890123456789012345678 9 123456789012345678901234567890121234567890123456789012345678 Choice (C) is the answer. 23456789012345678901234567890121234567890123456789012345678 123456789012345678901234567890121234567890123456789012345678 9 123456789012345678901234567890121234567890123456789012345678 9 9 123456789012345678901234567890121234567890123456789012345678 12. D These next three problems test your ability to deal with unfamiliar 9 123456789012345678901234567890121234567890123456789012345678 9 123456789012345678901234567890121234567890123456789012345678 symbols. Recall that you will be told everything that you need to know 123456789012345678901234567890121234567890123456789012345678 9 9 1 about the new symbol. Carefully read the instructions and don’t get 9 123456789012345678901234567890121234567890123456789012345678 9 123456789012345678901234567890121234567890123456789012345678 rattled; everything you need to know about the new symbol will be 9 123456789012345678901234567890121234567890123456789012345678 23456789012345678901234567890121234567890123456789012345678 9 123456789012345678901234567890121234567890123456789012345678 handed to you on a platter. 9 123456789012345678901234567890121234567890123456789012345678 123456789012345678901234567890121234567890123456789012345678 9 9 123456789012345678901234567890121234567890123456789012345678 In the subtraction problem, convert each term one at a time: 9 123456789012345678901234567890121234567890123456789012345678 9 123456789012345678901234567890121234567890123456789012345678 { 2323 5 3322 and { 2321 5 1322 9 123456789012345678901234567890121234567890123456789012345678 123456789012345678901234567890121234567890123456789012345678 9 9 123456789012345678901234567890121234567890123456789012345678 (It’s helpful if you write out the middle unchanging middle digits first, 9 1 23456789012345678901234567890121234567890123456789012345678 9 123456789012345678901234567890121234567890123456789012345678 and then write the first and last digits.) 9 123456789012345678901234567890121234567890123456789012345678 9 1 9 123456789012345678901234567890121234567890123456789012345678 So { 2323 2 { 2321 5 3322 2 1322 5 2000, choice (D). 9 123456789012345678901234567890121234567890123456789012345678 23456789012345678901234567890121234567890123456789012345678 123456789012345678901234567890121234567890123456789012345678 9 9 123456789012345678901234567890121234567890123456789012345678 9 1 13. A You know that A is a two-digit number between 10 and 20, which 9 123456789012345678901234567890121234567890123456789012345678 narrows the field of possible answers to 11, 12, 13, 14, 15, 16, 17, 18, 9 123456789012345678901234567890121234567890123456789012345678 23456789012345678901234567890121234567890123456789012345678 9 123456789012345678901234567890121234567890123456789012345678 and 19. This may seem like a lot, but the second part of the problem 9 123456789012345678901234567890121234567890123456789012345678 2 2 will narrow things down. The equation ({ A) 5 {(A )looks confusing, 9 1 9 123456789012345678901234567890121234567890123456789012345678 but the key word is “equation.” The two values are equal, even though 123456789012345678901234567890121234567890123456789012345678 9 23456789012345678901234567890121234567890123456789012345678 9 123456789012345678901234567890121234567890123456789012345678 you’ve flipped the first and last digits. Flipping 19 makes 91, which is 9 123456789012345678901234567890121234567890123456789012345678 9 123456789012345678901234567890121234567890123456789012345678 2 2 5 91 . quite a difference, and it’s highly unlikely that 19 123456789012345678901234567890121234567890123456789012345678 9 9 1 9 123456789012345678901234567890121234567890123456789012345678 At this point, you might suspect that 11 was the answer because 9 123456789012345678901234567890121234567890123456789012345678 9 123456789012345678901234567890121234567890123456789012345678 reversing the digits does not change the value of the number. That is a 9 123456789012345678901234567890121234567890123456789012345678 9 123456789012345678901234567890121234567890123456789012345678 good suspicion, and if you see it, you can easily read off that (A) is the 23456789012345678901234567890121234567890123456789012345678 123456789012345678901234567890121234567890123456789012345678 9 9 123456789012345678901234567890121234567890123456789012345678 answer. If you did not have that suspicion, just dive into the problem 9 123456789012345678901234567890121234567890123456789012345678 9 123456789012345678901234567890121234567890123456789012345678 trying different choices. 9 1 23456789012345678901234567890121234567890123456789012345678 123456789012345678901234567890121234567890123456789012345678 9 9 1 9 123456789012345678901234567890121234567890123456789012345678 123456789012345678901234567890121234567890123456789012345678 9 9 123456789012345678901234567890121234567890123456789012345678 123456789012345678901234567890121234567890123456789012345678 9 123456789012345678901234567890121234567890123456789012345678 9 123456789012345678901234567890121234567890123456789012345678 9 123456789012345678901234567890121234567890123456789012345678 9 1234567890123456789012345678901212345678901234567890123456789 54

Copyright © 2005 Thomson Peterson’s, a part of The Thomson Corporaton SAT is a registered trademark of the College Entrance Examination Board, which was not involved in the production of and does not endorse this product.

Note

1234567890123456789012345678901212345678901234567890123456789 123456789012345678901234567890121234567890123456789012345678 9 9 123456789012345678901234567890121234567890123456789012345678 123456789012345678901234567890121234567890123456789012345678 9 9 123456789012345678901234567890121234567890123456789012345678 14. E Since each variable is a digit and the inequality is true, plug in some 9 123456789012345678901234567890121234567890123456789012345678 9 123456789012345678901234567890121234567890123456789012345678 numbers to try to make the smallest difference possible: 9 123456789012345678901234567890121234567890123456789012345678 23456789012345678901234567890121234567890123456789012345678 9 1 9 123456789012345678901234567890121234567890123456789012345678 A . B . C . D . E 9 123456789012345678901234567890121234567890123456789012345678 9 123456789012345678901234567890121234567890123456789012345678 5 . 4 . 3 . 2 . 1 9 123456789012345678901234567890121234567890123456789012345678 23456789012345678901234567890121234567890123456789012345678 9 1 Now look at the subtraction problem and plug in the numbers above: 9 123456789012345678901234567890121234567890123456789012345678 9 123456789012345678901234567890121234567890123456789012345678 9 123456789012345678901234567890121234567890123456789012345678 ABCD 2 {(ABCD) 5 5432 2 2435 5 2977 9 123456789012345678901234567890121234567890123456789012345678 23456789012345678901234567890121234567890123456789012345678 9 1 9 123456789012345678901234567890121234567890123456789012345678 The difference is greater than one thousand, so choice (E) is the answer. 9 123456789012345678901234567890121234567890123456789012345678 9 123456789012345678901234567890121234567890123456789012345678 9 123456789012345678901234567890121234567890123456789012345678 23456789012345678901234567890121234567890123456789012345678 9 1 There is a theoretical path that leads to the same answer on this 9 123456789012345678901234567890121234567890123456789012345678 9 123456789012345678901234567890121234567890123456789012345678 problem, but when dealing with weird symbols problems the 9 123456789012345678901234567890121234567890123456789012345678 9 123456789012345678901234567890121234567890123456789012345678 theoretical path is usually not the best one to take. As you can see, 23456789012345678901234567890121234567890123456789012345678 123456789012345678901234567890121234567890123456789012345678 99 123456789012345678901234567890121234567890123456789012345678 generating some numbers and then placing them into the equation 9 123456789012345678901234567890121234567890123456789012345678 9 123456789012345678901234567890121234567890123456789012345678 works well and didn’t take too long. 123456789012345678901234567890121234567890123456789012345678 99 123456789012345678901234567890121234567890123456789012345678 9 1 9 123456789012345678901234567890121234567890123456789012345678 15. C There’s no answer choice that says, “It cannot be determined,” so you 9 123456789012345678901234567890121234567890123456789012345678 have to realize that there is a way to determine the area of the circle. 9 123456789012345678901234567890121234567890123456789012345678 23456789012345678901234567890121234567890123456789012345678 9 123456789012345678901234567890121234567890123456789012345678 Since the area formula for a circle is A 5 pr2 this means there has to be 9 123456789012345678901234567890121234567890123456789012345678 9 123456789012345678901234567890121234567890123456789012345678 a way to find the radius of the circle, which in this case is line 9 123456789012345678901234567890121234567890123456789012345678 9 123456789012345678901234567890121234567890123456789012345678 segment RS. 123456789012345678901234567890121234567890123456789012345678 99 123456789012345678901234567890121234567890123456789012345678 9 123456789012345678901234567890121234567890123456789012345678 The two statements underneath the drawing give you the tools you 9 123456789012345678901234567890121234567890123456789012345678 9 1 need. If “line q is tangent to circle R,” then angle RST is a right angle. 23456789012345678901234567890121234567890123456789012345678 123456789012345678901234567890121234567890123456789012345678 99 123456789012345678901234567890121234567890123456789012345678 RT 9 1 , then you can determine the value of RS since the And if RS 5 9 123456789012345678901234567890121234567890123456789012345678 3 9 123456789012345678901234567890121234567890123456789012345678 23456789012345678901234567890121234567890123456789012345678 9 123456789012345678901234567890121234567890123456789012345678 problem gives you the length of RT as 18. For line segment RS, the 9 123456789012345678901234567890121234567890123456789012345678 9 1 18 9 123456789012345678901234567890121234567890123456789012345678 radius will be 6, since 5 6. If you place this value of the radius into 9 123456789012345678901234567890121234567890123456789012345678 3 23456789012345678901234567890121234567890123456789012345678 9 123456789012345678901234567890121234567890123456789012345678 the area formula for a circle, you’ll find answer (C) at the end. 9 123456789012345678901234567890121234567890123456789012345678 9 1 2 9 123456789012345678901234567890121234567890123456789012345678 A 5 pr 9 123456789012345678901234567890121234567890123456789012345678 2 23456789012345678901234567890121234567890123456789012345678 9 123456789012345678901234567890121234567890123456789012345678 A 5 p6 9 123456789012345678901234567890121234567890123456789012345678 9 123456789012345678901234567890121234567890123456789012345678 A 5 36p 123456789012345678901234567890121234567890123456789012345678 99 1 9 123456789012345678901234567890121234567890123456789012345678 16. E All of the answer choices mention August and September total sales, so 9 123456789012345678901234567890121234567890123456789012345678 9 123456789012345678901234567890121234567890123456789012345678 the first step in this problem is to figure out what these values are. In the 9 123456789012345678901234567890121234567890123456789012345678 9 table, August has two squares (1,000 each) and three triangles (50 123456789012345678901234567890121234567890123456789012345678 23456789012345678901234567890121234567890123456789012345678 9 123456789012345678901234567890121234567890123456789012345678 returns each), so total August sales are: 9 123456789012345678901234567890121234567890123456789012345678 123456789012345678901234567890121234567890123456789012345678 99 123456789012345678901234567890121234567890123456789012345678 2(1,000) 2 3(50)5 2000 2 150 5 1,850 9 1 9 123456789012345678901234567890121234567890123456789012345678 123456789012345678901234567890121234567890123456789012345678 9 9 123456789012345678901234567890121234567890123456789012345678 123456789012345678901234567890121234567890123456789012345678 9 9 123456789012345678901234567890121234567890123456789012345678 123456789012345678901234567890121234567890123456789012345678 9 123456789012345678901234567890121234567890123456789012345678 9 123456789012345678901234567890121234567890123456789012345678 9 123456789012345678901234567890121234567890123456789012345678 9 1234567890123456789012345678901212345678901234567890123456789 Copyright © 2005 Thomson Peterson’s, a part of The Thomson Corporaton SAT is a registered trademark of the College Entrance Examination Board, which was not involved in the production of and does not endorse this product.

55

1234567890123456789012345678901212345678901234567890123456789 123456789012345678901234567890121234567890123456789012345678 9 9 123456789012345678901234567890121234567890123456789012345678 123456789012345678901234567890121234567890123456789012345678 9 9 123456789012345678901234567890121234567890123456789012345678 Doing the same box-and-triangle conversion, you should find that 9 123456789012345678901234567890121234567890123456789012345678 9 123456789012345678901234567890121234567890123456789012345678 September total sales were 1950. Therefore, there were 100 more sales 9 123456789012345678901234567890121234567890123456789012345678 23456789012345678901234567890121234567890123456789012345678 9 1 in September than in August. This lets you cross out (B) and (D), 9 123456789012345678901234567890121234567890123456789012345678 9 123456789012345678901234567890121234567890123456789012345678 because they have September total sales as being less than August total 9 123456789012345678901234567890121234567890123456789012345678 9 123456789012345678901234567890121234567890123456789012345678 sales. 23456789012345678901234567890121234567890123456789012345678 9 1 9 123456789012345678901234567890121234567890123456789012345678 9 123456789012345678901234567890121234567890123456789012345678 The final three answers all have different percentages. The difference in 9 123456789012345678901234567890121234567890123456789012345678 9 123456789012345678901234567890121234567890123456789012345678 monthly sales is 100 (1950 2 1850), so what percent of 1,850 is 100? 23456789012345678901234567890121234567890123456789012345678 9 1 9 123456789012345678901234567890121234567890123456789012345678 9 123456789012345678901234567890121234567890123456789012345678 n 9 123456789012345678901234567890121234567890123456789012345678 1850 5 100 9 123456789012345678901234567890121234567890123456789012345678 100 23456789012345678901234567890121234567890123456789012345678 9 1 9 123456789012345678901234567890121234567890123456789012345678 n 9 123456789012345678901234567890121234567890123456789012345678 100 1850 5 100 100 ~ ! 100 ~ ! 9 123456789012345678901234567890121234567890123456789012345678 9 123456789012345678901234567890121234567890123456789012345678 23456789012345678901234567890121234567890123456789012345678 123456789012345678901234567890121234567890123456789012345678 9 9 123456789012345678901234567890121234567890123456789012345678 1850n 5 10000 123456789012345678901234567890121234567890123456789012345678 9 9 123456789012345678901234567890121234567890123456789012345678 1850n 10000 9 123456789012345678901234567890121234567890123456789012345678 5 9 123456789012345678901234567890121234567890123456789012345678 1850 1850 9 1 9 123456789012345678901234567890121234567890123456789012345678 n 5 5.4 9 123456789012345678901234567890121234567890123456789012345678 9 123456789012345678901234567890121234567890123456789012345678 23456789012345678901234567890121234567890123456789012345678 9 123456789012345678901234567890121234567890123456789012345678 100 is 5.4 percent of 1850, which means that September total sales 9 123456789012345678901234567890121234567890123456789012345678 9 123456789012345678901234567890121234567890123456789012345678 were 5.4 percent greater than August total sales. (E) says the same and 123456789012345678901234567890121234567890123456789012345678 9 9 123456789012345678901234567890121234567890123456789012345678 is the correct answer. 9 123456789012345678901234567890121234567890123456789012345678 123456789012345678901234567890121234567890123456789012345678 9 9 123456789012345678901234567890121234567890123456789012345678 17. C You cannot solve this problem unless you do some creative line 9 123456789012345678901234567890121234567890123456789012345678 9 1 drawing. 23456789012345678901234567890121234567890123456789012345678 123456789012345678901234567890121234567890123456789012345678 9 123456789012345678901234567890121234567890123456789012345678 9 9 1 123456789012345678901234567890121234567890123456789012345678 9 9 123456789012345678901234567890121234567890123456789012345678 9 123456789012345678901234567890121234567890123456789012345678 9 123456789012345678901234567890121234567890123456789012345678 123456789012345678901234567890121234567890123456789012345678 9 9 123456789012345678901234567890121234567890123456789012345678 9 123456789012345678901234567890121234567890123456789012345678 23456789012345678901234567890121234567890123456789012345678 123456789012345678901234567890121234567890123456789012345678 9 123456789012345678901234567890121234567890123456789012345678 9 9 1 123456789012345678901234567890121234567890123456789012345678 9 123456789012345678901234567890121234567890123456789012345678 9 9 123456789012345678901234567890121234567890123456789012345678 9 123456789012345678901234567890121234567890123456789012345678 To find the unknown lengths, you have to subtract known values of 9 123456789012345678901234567890121234567890123456789012345678 9 123456789012345678901234567890121234567890123456789012345678 opposites. Take the top and bottom sides, for instance. The larger top 123456789012345678901234567890121234567890123456789012345678 9 23456789012345678901234567890121234567890123456789012345678 9 123456789012345678901234567890121234567890123456789012345678 side is 8, and the lower part is 5, so the difference must be 3. A dashed 9 123456789012345678901234567890121234567890123456789012345678 line shows this value. Subtracting the right side (length 2) from the left 9 123456789012345678901234567890121234567890123456789012345678 9 123456789012345678901234567890121234567890123456789012345678 side (length 6), you find another dashed line that has a value of 4. 9 1 23456789012345678901234567890121234567890123456789012345678 123456789012345678901234567890121234567890123456789012345678 9 9 123456789012345678901234567890121234567890123456789012345678 From this sketch you can see that x is the hypotenuse of a 3-4-5 triangle, 9 123456789012345678901234567890121234567890123456789012345678 9 123456789012345678901234567890121234567890123456789012345678 and so is equal to 5. This means the perimeter is 26, (C). 9 1 23456789012345678901234567890121234567890123456789012345678 123456789012345678901234567890121234567890123456789012345678 9 9 1 9 123456789012345678901234567890121234567890123456789012345678 123456789012345678901234567890121234567890123456789012345678 9 9 123456789012345678901234567890121234567890123456789012345678 123456789012345678901234567890121234567890123456789012345678 9 123456789012345678901234567890121234567890123456789012345678 9 123456789012345678901234567890121234567890123456789012345678 9 123456789012345678901234567890121234567890123456789012345678 9 1234567890123456789012345678901212345678901234567890123456789

S D S D

56

Copyright © 2005 Thomson Peterson’s, a part of The Thomson Corporaton SAT is a registered trademark of the College Entrance Examination Board, which was not involved in the production of and does not endorse this product.

1234567890123456789012345678901212345678901234567890123456789 123456789012345678901234567890121234567890123456789012345678 9 9 123456789012345678901234567890121234567890123456789012345678 123456789012345678901234567890121234567890123456789012345678 9 9 123456789012345678901234567890121234567890123456789012345678 18. A Unpack the tangle of terms, and you’ll find the answer. You are looking 9 123456789012345678901234567890121234567890123456789012345678 9 123456789012345678901234567890121234567890123456789012345678 for two prime numbers that: 9 123456789012345678901234567890121234567890123456789012345678 23456789012345678901234567890121234567890123456789012345678 9 1 9 123456789012345678901234567890121234567890123456789012345678 1. When you subtract them you get 21. 9 123456789012345678901234567890121234567890123456789012345678 9 123456789012345678901234567890121234567890123456789012345678 9 123456789012345678901234567890121234567890123456789012345678 2. When you multiply them you get one of the answer choices. 23456789012345678901234567890121234567890123456789012345678 9 1 9 123456789012345678901234567890121234567890123456789012345678 It might not sound like enough information to find the answer, but it is. 9 123456789012345678901234567890121234567890123456789012345678 9 123456789012345678901234567890121234567890123456789012345678 None of the answer choices are very great, so one or both of the prime 9 123456789012345678901234567890121234567890123456789012345678 23456789012345678901234567890121234567890123456789012345678 9 1 numbers must be less than ten. If the two primes were greater, their 9 123456789012345678901234567890121234567890123456789012345678 9 123456789012345678901234567890121234567890123456789012345678 product would not be as small as the answer choices. The difference 9 123456789012345678901234567890121234567890123456789012345678 9 123456789012345678901234567890121234567890123456789012345678 between the two primes is 21, which means that one of the prime 23456789012345678901234567890121234567890123456789012345678 9 1 9 123456789012345678901234567890121234567890123456789012345678 numbers must be small. Good candidates for the lesser prime are 2, 3, 9 123456789012345678901234567890121234567890123456789012345678 9 123456789012345678901234567890121234567890123456789012345678 or 5. 9 123456789012345678901234567890121234567890123456789012345678 23456789012345678901234567890121234567890123456789012345678 123456789012345678901234567890121234567890123456789012345678 99 123456789012345678901234567890121234567890123456789012345678 Prime numbers starting in the 20s are 23, 29, 31, and 37. Use a little 9 123456789012345678901234567890121234567890123456789012345678 9 123456789012345678901234567890121234567890123456789012345678 trial and error, and you’ll realize that the difference between 23 and 2 is 123456789012345678901234567890121234567890123456789012345678 99 123456789012345678901234567890121234567890123456789012345678 21. Those are the two primes. What is 2 times 23? The answer is 46, 9 1 9 123456789012345678901234567890121234567890123456789012345678 choice (A). 9 123456789012345678901234567890121234567890123456789012345678 9 123456789012345678901234567890121234567890123456789012345678 19. B First, it always helps to draw a quick sketch of the scenario to get a 23456789012345678901234567890121234567890123456789012345678 123456789012345678901234567890121234567890123456789012345678 99 123456789012345678901234567890121234567890123456789012345678 better grasp on what is being asked. 9 123456789012345678901234567890121234567890123456789012345678 123456789012345678901234567890121234567890123456789012345678 99 123456789012345678901234567890121234567890123456789012345678 9 123456789012345678901234567890121234567890123456789012345678 9 123456789012345678901234567890121234567890123456789012345678 9 123456789012345678901234567890121234567890123456789012345678 9 123456789012345678901234567890121234567890123456789012345678 9 1 9 123456789012345678901234567890121234567890123456789012345678 9 123456789012345678901234567890121234567890123456789012345678 123456789012345678901234567890121234567890123456789012345678 9 123456789012345678901234567890121234567890123456789012345678 9 9 123456789012345678901234567890121234567890123456789012345678 9 123456789012345678901234567890121234567890123456789012345678 9 123456789012345678901234567890121234567890123456789012345678 123456789012345678901234567890121234567890123456789012345678 9 9 123456789012345678901234567890121234567890123456789012345678 9 123456789012345678901234567890121234567890123456789012345678 23456789012345678901234567890121234567890123456789012345678 123456789012345678901234567890121234567890123456789012345678 99 123456789012345678901234567890121234567890123456789012345678 9 1 123456789012345678901234567890121234567890123456789012345678 9 123456789012345678901234567890121234567890123456789012345678 9 9 123456789012345678901234567890121234567890123456789012345678 9 123456789012345678901234567890121234567890123456789012345678 9 123456789012345678901234567890121234567890123456789012345678 9 123456789012345678901234567890121234567890123456789012345678 123456789012345678901234567890121234567890123456789012345678 9 23456789012345678901234567890121234567890123456789012345678 9 123456789012345678901234567890121234567890123456789012345678 The first thing to determine is the midpoint of DE, which you can do by 9 123456789012345678901234567890121234567890123456789012345678 9 123456789012345678901234567890121234567890123456789012345678 using the midpoint formula: 123456789012345678901234567890121234567890123456789012345678 99 1 9 123456789012345678901234567890121234567890123456789012345678 x1 1 x2 y1 1 y2 0 1 5 12 1 0 5 12 9 123456789012345678901234567890121234567890123456789012345678 5 5 5 2.5,6 , , , ~ ! 9 123456789012345678901234567890121234567890123456789012345678 2 2 2 2 2 2 9 123456789012345678901234567890121234567890123456789012345678 123456789012345678901234567890121234567890123456789012345678 9 23456789012345678901234567890121234567890123456789012345678 9 123456789012345678901234567890121234567890123456789012345678 You can keep the x-value as a fraction if you like, but converting it to 9 1 9 123456789012345678901234567890121234567890123456789012345678 2.5 will make it easier to punch into your calculator. 123456789012345678901234567890121234567890123456789012345678 9 9 123456789012345678901234567890121234567890123456789012345678 123456789012345678901234567890121234567890123456789012345678 9 123456789012345678901234567890121234567890123456789012345678 9 123456789012345678901234567890121234567890123456789012345678 9 123456789012345678901234567890121234567890123456789012345678 9 1234567890123456789012345678901212345678901234567890123456789

S

D S

D S D

Copyright © 2005 Thomson Peterson’s, a part of The Thomson Corporaton SAT is a registered trademark of the College Entrance Examination Board, which was not involved in the production of and does not endorse this product.

57

1234567890123456789012345678901212345678901234567890123456789 123456789012345678901234567890121234567890123456789012345678 9 9 123456789012345678901234567890121234567890123456789012345678 123456789012345678901234567890121234567890123456789012345678 9 9 123456789012345678901234567890121234567890123456789012345678 Looking at the dashed lines, you can see a right triangle with its 9 123456789012345678901234567890121234567890123456789012345678 9 123456789012345678901234567890121234567890123456789012345678 hypotenuse from the origin to the midpoint of DE. It has sides of 2.5 9 123456789012345678901234567890121234567890123456789012345678 23456789012345678901234567890121234567890123456789012345678 9 1 and 6. Placing those values into the Pythagorean theorem yields: 9 123456789012345678901234567890121234567890123456789012345678 9 123456789012345678901234567890121234567890123456789012345678 2 2 2 9 123456789012345678901234567890121234567890123456789012345678 1 b 5 c a 9 123456789012345678901234567890121234567890123456789012345678 23456789012345678901234567890121234567890123456789012345678 9 1 2.52 1 62 5 c2 9 123456789012345678901234567890121234567890123456789012345678 9 123456789012345678901234567890121234567890123456789012345678 2 9 123456789012345678901234567890121234567890123456789012345678 6.25 1 36 5 c 9 123456789012345678901234567890121234567890123456789012345678 2 23456789012345678901234567890121234567890123456789012345678 9 1 42.25 5 c 9 123456789012345678901234567890121234567890123456789012345678 9 123456789012345678901234567890121234567890123456789012345678 9 123456789012345678901234567890121234567890123456789012345678 42.25 5 c = = 9 123456789012345678901234567890121234567890123456789012345678 23456789012345678901234567890121234567890123456789012345678 9 1 6.5 5 c 9 123456789012345678901234567890121234567890123456789012345678 9 123456789012345678901234567890121234567890123456789012345678 9 123456789012345678901234567890121234567890123456789012345678 It’s choice (B). 9 123456789012345678901234567890121234567890123456789012345678 23456789012345678901234567890121234567890123456789012345678 123456789012345678901234567890121234567890123456789012345678 9 9 123456789012345678901234567890121234567890123456789012345678 20. A It’s a big equation, but in the end it’s just that: an equation. You have to 123456789012345678901234567890121234567890123456789012345678 9 9 123456789012345678901234567890121234567890123456789012345678 pull out two different techniques—FOIL and PEMDAS—to solve it, 9 123456789012345678901234567890121234567890123456789012345678 but so long as you write out your work carefully, you will reach the 123456789012345678901234567890121234567890123456789012345678 9 9 1 right answer. 9 123456789012345678901234567890121234567890123456789012345678 9 123456789012345678901234567890121234567890123456789012345678 9 123456789012345678901234567890121234567890123456789012345678 2 23456789012345678901234567890121234567890123456789012345678 9 123456789012345678901234567890121234567890123456789012345678 2b 5 ~b 2 7!~b 1 3! 2 ~2b 1 2!~b 1 5! 9 123456789012345678901234567890121234567890123456789012345678 2 2 2 9 123456789012345678901234567890121234567890123456789012345678 2b 5 b 1 3b 2 7b 2 21 2 2b 1 10b 1 2b 1 10 ~ ! ~ ! 9 123456789012345678901234567890121234567890123456789012345678 2 2 2 9 123456789012345678901234567890121234567890123456789012345678 2b 5 b 2 4b 2 21 2 2b 1 12b 1 10 ~ ! ~ ! 123456789012345678901234567890121234567890123456789012345678 9 9 123456789012345678901234567890121234567890123456789012345678 2b2 5 b2 2 4b 2 21 2 2b2 2 12b 2 10 9 123456789012345678901234567890121234567890123456789012345678 9 123456789012345678901234567890121234567890123456789012345678 2 2 9 1 2b 5 2b 2 16b 2 31 23456789012345678901234567890121234567890123456789012345678 9 123456789012345678901234567890121234567890123456789012345678 2 2 2 2 9 123456789012345678901234567890121234567890123456789012345678 2b 1 b 5 2b 1 b 2 16b 2 31 9 1 9 123456789012345678901234567890121234567890123456789012345678 0 5 216b 2 31 9 123456789012345678901234567890121234567890123456789012345678 23456789012345678901234567890121234567890123456789012345678 123456789012345678901234567890121234567890123456789012345678 9 9 123456789012345678901234567890121234567890123456789012345678 0 1 16b 5 216b 1 16b 2 31 9 1 9 123456789012345678901234567890121234567890123456789012345678 16b 5 231 9 123456789012345678901234567890121234567890123456789012345678 23456789012345678901234567890121234567890123456789012345678 9 123456789012345678901234567890121234567890123456789012345678 16b 231 9 123456789012345678901234567890121234567890123456789012345678 5 9 1 16 16 9 123456789012345678901234567890121234567890123456789012345678 123456789012345678901234567890121234567890123456789012345678 9 23456789012345678901234567890121234567890123456789012345678 9 123456789012345678901234567890121234567890123456789012345678 31 15 9 123456789012345678901234567890121234567890123456789012345678 b 5 2 5 21 9 123456789012345678901234567890121234567890123456789012345678 16 16 123456789012345678901234567890121234567890123456789012345678 9 9 1 Choice (A) is your answer. 9 123456789012345678901234567890121234567890123456789012345678 9 123456789012345678901234567890121234567890123456789012345678 9 123456789012345678901234567890121234567890123456789012345678 21. A There’s no table given for this question, so it’s up to you to create one. 9 123456789012345678901234567890121234567890123456789012345678 9 123456789012345678901234567890121234567890123456789012345678 In this respect it’s like a diagram without a diagram given: You could 23456789012345678901234567890121234567890123456789012345678 123456789012345678901234567890121234567890123456789012345678 9 9 123456789012345678901234567890121234567890123456789012345678 conceivably answer it without the visual aids, but it’s much easier 9 123456789012345678901234567890121234567890123456789012345678 9 123456789012345678901234567890121234567890123456789012345678 with them. 9 1 23456789012345678901234567890121234567890123456789012345678 123456789012345678901234567890121234567890123456789012345678 9 9 1 For the table, start with the years: 9 123456789012345678901234567890121234567890123456789012345678 123456789012345678901234567890121234567890123456789012345678 9 9 123456789012345678901234567890121234567890123456789012345678 1950 1960 1970 1980 1990 2000 9 123456789012345678901234567890121234567890123456789012345678 123456789012345678901234567890121234567890123456789012345678 9 123456789012345678901234567890121234567890123456789012345678 9 123456789012345678901234567890121234567890123456789012345678 9 1234567890123456789012345678901212345678901234567890123456789 58

Copyright © 2005 Thomson Peterson’s, a part of The Thomson Corporaton SAT is a registered trademark of the College Entrance Examination Board, which was not involved in the production of and does not endorse this product.

1234567890123456789012345678901212345678901234567890123456789 123456789012345678901234567890121234567890123456789012345678 9 9 123456789012345678901234567890121234567890123456789012345678 123456789012345678901234567890121234567890123456789012345678 9 9 123456789012345678901234567890121234567890123456789012345678 Now place the number of people in Cree County in 1950 in the 9 123456789012345678901234567890121234567890123456789012345678 9 123456789012345678901234567890121234567890123456789012345678 appropriate place, and then start multiplying by three. 9 123456789012345678901234567890121234567890123456789012345678 23456789012345678901234567890121234567890123456789012345678 9 1 9 123456789012345678901234567890121234567890123456789012345678 1950 1960 1970 1980 1990 2000 9 123456789012345678901234567890121234567890123456789012345678 9 123456789012345678901234567890121234567890123456789012345678 1000 3000 9000 27000 9 123456789012345678901234567890121234567890123456789012345678 23456789012345678901234567890121234567890123456789012345678 9 1 9 123456789012345678901234567890121234567890123456789012345678 In the 1980s, the population fell by one-half, so divide 27,000 by 2. 9 123456789012345678901234567890121234567890123456789012345678 Then go back to multiplying by 3. 9 123456789012345678901234567890121234567890123456789012345678 9 123456789012345678901234567890121234567890123456789012345678 23456789012345678901234567890121234567890123456789012345678 9 1 1950 1960 1970 1980 1990 2000 9 123456789012345678901234567890121234567890123456789012345678 9 123456789012345678901234567890121234567890123456789012345678 1000 3000 9000 27000 13,500 40,500 9 123456789012345678901234567890121234567890123456789012345678 9 123456789012345678901234567890121234567890123456789012345678 23456789012345678901234567890121234567890123456789012345678 9 1 Your answer is choice (A). 9 123456789012345678901234567890121234567890123456789012345678 9 123456789012345678901234567890121234567890123456789012345678 9 123456789012345678901234567890121234567890123456789012345678 9 123456789012345678901234567890121234567890123456789012345678 23456789012345678901234567890121234567890123456789012345678 9 123456789012345678901234567890121234567890123456789012345678 Section 3 9 123456789012345678901234567890121234567890123456789012345678 9 123456789012345678901234567890121234567890123456789012345678 1. A Something that can be counted requires “fewer” rather than “less.” The 9 123456789012345678901234567890121234567890123456789012345678 9 123456789012345678901234567890121234567890123456789012345678 answer is (A). 123456789012345678901234567890121234567890123456789012345678 99 1 9 123456789012345678901234567890121234567890123456789012345678 2. B In for Ada and I, I is used here as the object of a preposition (for). That 9 123456789012345678901234567890121234567890123456789012345678 9 123456789012345678901234567890121234567890123456789012345678 should never be. Me is the object of a preposition, or a direct or indirect 23456789012345678901234567890121234567890123456789012345678 123456789012345678901234567890121234567890123456789012345678 99 123456789012345678901234567890121234567890123456789012345678 object. The answer is (B). 9 123456789012345678901234567890121234567890123456789012345678 123456789012345678901234567890121234567890123456789012345678 99 123456789012345678901234567890121234567890123456789012345678 3. B Is the subject in this sentence singular or plural? Team is singular. Is the 9 123456789012345678901234567890121234567890123456789012345678 9 123456789012345678901234567890121234567890123456789012345678 verb singular or plural? Compete is plural. The clause which is 123456789012345678901234567890121234567890123456789012345678 99 123456789012345678901234567890121234567890123456789012345678 comprised of four cyclists obscures the subject-verb disagreement, but 9 1 23456789012345678901234567890121234567890123456789012345678 9 123456789012345678901234567890121234567890123456789012345678 it is still there. (B) is the answer. 9 123456789012345678901234567890121234567890123456789012345678 9 1 4. E Did you hear any mistakes in this sentence? Hopefully you didn’t 9 123456789012345678901234567890121234567890123456789012345678 9 123456789012345678901234567890121234567890123456789012345678 because there are not any. (E) is the answer. 23456789012345678901234567890121234567890123456789012345678 123456789012345678901234567890121234567890123456789012345678 99 123456789012345678901234567890121234567890123456789012345678 9 1 5. A If you know that Louis and Clark were nineteenth-century explorers, 9 123456789012345678901234567890121234567890123456789012345678 9 123456789012345678901234567890121234567890123456789012345678 the verb have surveyed might sound strange even before you read the 23456789012345678901234567890121234567890123456789012345678 123456789012345678901234567890121234567890123456789012345678 99 123456789012345678901234567890121234567890123456789012345678 rest of the sentence. If not, were well prepared suggests that the first 9 1 9 123456789012345678901234567890121234567890123456789012345678 exploration happened prior to an event which is, itself, in the past. The 123456789012345678901234567890121234567890123456789012345678 9 23456789012345678901234567890121234567890123456789012345678 9 123456789012345678901234567890121234567890123456789012345678 first verb should be had surveyed, and the answer is (A). 9 123456789012345678901234567890121234567890123456789012345678 123456789012345678901234567890121234567890123456789012345678 99 123456789012345678901234567890121234567890123456789012345678 6. D There is a parallel structure problem here. Focus, a verb, should be 9 1 paired with follow up, a verb, not follow-up, which is a noun. The 23456789012345678901234567890121234567890123456789012345678 9 123456789012345678901234567890121234567890123456789012345678 9 123456789012345678901234567890121234567890123456789012345678 answer is (D). 9 123456789012345678901234567890121234567890123456789012345678 123456789012345678901234567890121234567890123456789012345678 99 1 7. C Nothing in this sentence sounds wrong, but (E) is not the answer. Here’s 9 123456789012345678901234567890121234567890123456789012345678 9 123456789012345678901234567890121234567890123456789012345678 why, to whom does the its refer? It is not clear if it refers to the Ottoman 9 123456789012345678901234567890121234567890123456789012345678 9 123456789012345678901234567890121234567890123456789012345678 or Austrian Empire. An ambiguous pronoun reference is an error, and 123456789012345678901234567890121234567890123456789012345678 9 23456789012345678901234567890121234567890123456789012345678 9 123456789012345678901234567890121234567890123456789012345678 so (C) is the answer. 9 1 9 123456789012345678901234567890121234567890123456789012345678 123456789012345678901234567890121234567890123456789012345678 9 9 123456789012345678901234567890121234567890123456789012345678 123456789012345678901234567890121234567890123456789012345678 9 123456789012345678901234567890121234567890123456789012345678 9 123456789012345678901234567890121234567890123456789012345678 9 123456789012345678901234567890121234567890123456789012345678 9 1234567890123456789012345678901212345678901234567890123456789 Copyright © 2005 Thomson Peterson’s, a part of The Thomson Corporaton SAT is a registered trademark of the College Entrance Examination Board, which was not involved in the production of and does not endorse this product.

59

1234567890123456789012345678901212345678901234567890123456789 123456789012345678901234567890121234567890123456789012345678 9 9 123456789012345678901234567890121234567890123456789012345678 123456789012345678901234567890121234567890123456789012345678 9 9 123456789012345678901234567890121234567890123456789012345678 8. A What is the subject, census (singular) or statistics (plural)? Statistics. 9 123456789012345678901234567890121234567890123456789012345678 9 123456789012345678901234567890121234567890123456789012345678 Census is used as an adjective. The plural subject means you would 9 123456789012345678901234567890121234567890123456789012345678 23456789012345678901234567890121234567890123456789012345678 9 1 need a plural verb, were. The answer is (A). 9 123456789012345678901234567890121234567890123456789012345678 9 123456789012345678901234567890121234567890123456789012345678 9 123456789012345678901234567890121234567890123456789012345678 9. B Scarcely no one is a double negative. It should be scarcely anyone. (B) is 9 123456789012345678901234567890121234567890123456789012345678 23456789012345678901234567890121234567890123456789012345678 9 1 the answer. 9 123456789012345678901234567890121234567890123456789012345678 9 123456789012345678901234567890121234567890123456789012345678 9 123456789012345678901234567890121234567890123456789012345678 10. D This is a tricky question because the error arises with the implied part of 9 123456789012345678901234567890121234567890123456789012345678 the sentence. The full sentence with the implied part written out 23456789012345678901234567890121234567890123456789012345678 9 1 9 123456789012345678901234567890121234567890123456789012345678 explicitly reads, “Even though he had the title of Vice-President of 9 123456789012345678901234567890121234567890123456789012345678 9 123456789012345678901234567890121234567890123456789012345678 Operations, his duties and responsibilities were not much greater than a 9 123456789012345678901234567890121234567890123456789012345678 23456789012345678901234567890121234567890123456789012345678 9 1 midlevel manager’s duties and responsibilities.” The short version 9 123456789012345678901234567890121234567890123456789012345678 9 123456789012345678901234567890121234567890123456789012345678 should still say manager’s. The answer is (D). 9 123456789012345678901234567890121234567890123456789012345678 9 123456789012345678901234567890121234567890123456789012345678 23456789012345678901234567890121234567890123456789012345678 9 123456789012345678901234567890121234567890123456789012345678 11. D After both, you especially need parallel structure. It should read more 9 123456789012345678901234567890121234567890123456789012345678 9 123456789012345678901234567890121234567890123456789012345678 accessible and more understandable. The other answers only confuse 123456789012345678901234567890121234567890123456789012345678 9 9 123456789012345678901234567890121234567890123456789012345678 the original. The answer is (D). 9 123456789012345678901234567890121234567890123456789012345678 9 1 9 123456789012345678901234567890121234567890123456789012345678 12. D The initial modifier should put you on the lookout for an incorrect 9 123456789012345678901234567890121234567890123456789012345678 9 123456789012345678901234567890121234567890123456789012345678 noun, or reference to the “thing” that is being modified. You should 23456789012345678901234567890121234567890123456789012345678 123456789012345678901234567890121234567890123456789012345678 9 9 123456789012345678901234567890121234567890123456789012345678 examine the noun that directly follows the modifier carefully. Does the 9 123456789012345678901234567890121234567890123456789012345678 modifier modify the noun that follows it? Nope. The poet, Gerard 9 123456789012345678901234567890121234567890123456789012345678 9 123456789012345678901234567890121234567890123456789012345678 Manley Hopkins, not his poems, is what is being modified here. The 123456789012345678901234567890121234567890123456789012345678 9 9 123456789012345678901234567890121234567890123456789012345678 possessive form (Hopkins’s) and the noun poems messes things up in 9 123456789012345678901234567890121234567890123456789012345678 9 123456789012345678901234567890121234567890123456789012345678 the original and answer choices (A), (B), and (C). To make it work, you 9 1 23456789012345678901234567890121234567890123456789012345678 9 123456789012345678901234567890121234567890123456789012345678 need to get rid of the possessive on the author’s name. Only (D) and (E) 9 123456789012345678901234567890121234567890123456789012345678 9 1 do that. (E) makes an unnecessary change to the verb. The better 123456789012345678901234567890121234567890123456789012345678 9 9 123456789012345678901234567890121234567890123456789012345678 answer is (D). 9 123456789012345678901234567890121234567890123456789012345678 9 123456789012345678901234567890121234567890123456789012345678 9 123456789012345678901234567890121234567890123456789012345678 13. E The original underlined phrase is not glaringly obtuse or a grammatical 9 123456789012345678901234567890121234567890123456789012345678 9 123456789012345678901234567890121234567890123456789012345678 disaster, but it is a little wordy. Also, this includes the Human Rights 23456789012345678901234567890121234567890123456789012345678 123456789012345678901234567890121234567890123456789012345678 9 9 123456789012345678901234567890121234567890123456789012345678 Commission is a complete sentence, so it would need to be attached 9 1 9 123456789012345678901234567890121234567890123456789012345678 with a semicolon. Only (E) addresses both problems without creating 123456789012345678901234567890121234567890123456789012345678 9 23456789012345678901234567890121234567890123456789012345678 9 123456789012345678901234567890121234567890123456789012345678 others. 9 123456789012345678901234567890121234567890123456789012345678 9 123456789012345678901234567890121234567890123456789012345678 14. C The most blatant mistake here is that, again, the clause that follows a 9 123456789012345678901234567890121234567890123456789012345678 9 1 comma is technically a complete sentence. The best way to approach 23456789012345678901234567890121234567890123456789012345678 123456789012345678901234567890121234567890123456789012345678 9 9 123456789012345678901234567890121234567890123456789012345678 this question is to read all the way through each of the answers to see if 9 123456789012345678901234567890121234567890123456789012345678 9 123456789012345678901234567890121234567890123456789012345678 they contain problems. (B) has tense problems. (D) also has two 9 1 23456789012345678901234567890121234567890123456789012345678 9 123456789012345678901234567890121234567890123456789012345678 complete sentences attached by a comma (this is called a comma splice). 9 123456789012345678901234567890121234567890123456789012345678 9 123456789012345678901234567890121234567890123456789012345678 (E) just doesn’t work. The answer is (C). 123456789012345678901234567890121234567890123456789012345678 9 9 1 9 123456789012345678901234567890121234567890123456789012345678 123456789012345678901234567890121234567890123456789012345678 9 9 123456789012345678901234567890121234567890123456789012345678 123456789012345678901234567890121234567890123456789012345678 9 9 123456789012345678901234567890121234567890123456789012345678 123456789012345678901234567890121234567890123456789012345678 9 123456789012345678901234567890121234567890123456789012345678 9 123456789012345678901234567890121234567890123456789012345678 9 123456789012345678901234567890121234567890123456789012345678 9 1234567890123456789012345678901212345678901234567890123456789 60

Copyright © 2005 Thomson Peterson’s, a part of The Thomson Corporaton SAT is a registered trademark of the College Entrance Examination Board, which was not involved in the production of and does not endorse this product.

1234567890123456789012345678901212345678901234567890123456789 123456789012345678901234567890121234567890123456789012345678 9 9 123456789012345678901234567890121234567890123456789012345678 123456789012345678901234567890121234567890123456789012345678 9 9 123456789012345678901234567890121234567890123456789012345678 15. B The underlined portion of the sentence reads awkwardly for two 9 123456789012345678901234567890121234567890123456789012345678 9 123456789012345678901234567890121234567890123456789012345678 reasons. First, it includes the phrase her plans of, which is redundant 9 123456789012345678901234567890121234567890123456789012345678 23456789012345678901234567890121234567890123456789012345678 9 1 because the list is a list of her plans. Second, the preceding plans in the 9 123456789012345678901234567890121234567890123456789012345678 9 123456789012345678901234567890121234567890123456789012345678 list are given in infinitive form (i.e. to refinance), but the last plan of 9 123456789012345678901234567890121234567890123456789012345678 9 123456789012345678901234567890121234567890123456789012345678 action is not in infinitive form. Any answer choice that deals with these 23456789012345678901234567890121234567890123456789012345678 9 1 9 123456789012345678901234567890121234567890123456789012345678 two issues will be the best choice. (B) is that answer choice. 9 123456789012345678901234567890121234567890123456789012345678 9 123456789012345678901234567890121234567890123456789012345678 9 123456789012345678901234567890121234567890123456789012345678 16. B You know the correct answer won’t be a complete sentence, because it 23456789012345678901234567890121234567890123456789012345678 9 1 9 123456789012345678901234567890121234567890123456789012345678 is attached by a comma to the main clause. Eliminate (E). You will also 9 123456789012345678901234567890121234567890123456789012345678 9 want to cut down on wordiness, rather than add to it. Eliminate (A) and 123456789012345678901234567890121234567890123456789012345678 9 123456789012345678901234567890121234567890123456789012345678 (C). Which suggests that the clause modifies a noun that comes just 23456789012345678901234567890121234567890123456789012345678 9 1 9 123456789012345678901234567890121234567890123456789012345678 before it. That isn’t the case. The answer is (B). 9 123456789012345678901234567890121234567890123456789012345678 9 123456789012345678901234567890121234567890123456789012345678 9 123456789012345678901234567890121234567890123456789012345678 17. C Again, you want to move toward parallel structure as much as possible. 23456789012345678901234567890121234567890123456789012345678 123456789012345678901234567890121234567890123456789012345678 99 123456789012345678901234567890121234567890123456789012345678 You don’t want nouns in one part of the list and verbs in another, as you 9 123456789012345678901234567890121234567890123456789012345678 9 123456789012345678901234567890121234567890123456789012345678 have now. Change the noun tolerance to a verb, tolerate. You can 123456789012345678901234567890121234567890123456789012345678 99 123456789012345678901234567890121234567890123456789012345678 narrow the field to (C) and (E). (E) is wrong because you don’t use a 9 1 9 123456789012345678901234567890121234567890123456789012345678 comma in a two-items list. 9 123456789012345678901234567890121234567890123456789012345678 9 123456789012345678901234567890121234567890123456789012345678 23456789012345678901234567890121234567890123456789012345678 9 123456789012345678901234567890121234567890123456789012345678 18. D Guess what? Parallel structure again. To commit a crime needs to be 9 123456789012345678901234567890121234567890123456789012345678 9 123456789012345678901234567890121234567890123456789012345678 followed by (to) conceal it. That leaves (D), without you even having to 123456789012345678901234567890121234567890123456789012345678 99 123456789012345678901234567890121234567890123456789012345678 know that among suggests more than two people, whereas between 9 123456789012345678901234567890121234567890123456789012345678 9 123456789012345678901234567890121234567890123456789012345678 suggests two. 123456789012345678901234567890121234567890123456789012345678 99 123456789012345678901234567890121234567890123456789012345678 1 19. C Here’s the test writers’ other favorite trick: “not only” with “but also.” 9 9 123456789012345678901234567890121234567890123456789012345678 Only (C) and (E) use this construction. (E) makes the tenses 9 123456789012345678901234567890121234567890123456789012345678 9 123456789012345678901234567890121234567890123456789012345678 unnecessarily complex. The answer is (C). 123456789012345678901234567890121234567890123456789012345678 9 9 123456789012345678901234567890121234567890123456789012345678 9 123456789012345678901234567890121234567890123456789012345678 20. D The sentence as it stands is muddy and unnecessarily wordy. These are 9 123456789012345678901234567890121234567890123456789012345678 9 123456789012345678901234567890121234567890123456789012345678 things to be avoided in good writing. “If” should be followed by a 9 123456789012345678901234567890121234567890123456789012345678 9 123456789012345678901234567890121234567890123456789012345678 “then” clause. What follows here doesn’t fit that logical pattern. So if is 23456789012345678901234567890121234567890123456789012345678 123456789012345678901234567890121234567890123456789012345678 99 123456789012345678901234567890121234567890123456789012345678 out. That leaves (D) and (E). (D) gets rid of the repetitive and vague it in 9 1 9 123456789012345678901234567890121234567890123456789012345678 the second part of the sentence. Good. The answer is (D). 123456789012345678901234567890121234567890123456789012345678 9 23456789012345678901234567890121234567890123456789012345678 9 123456789012345678901234567890121234567890123456789012345678 21. D The original version should wave a red flag at you that it is repetitive 9 123456789012345678901234567890121234567890123456789012345678 9 123456789012345678901234567890121234567890123456789012345678 and cumbersome and will need to be changed. (B) is even more vague 9 123456789012345678901234567890121234567890123456789012345678 9 1 and wordy than the original, despite being a single sentence. (C) is weak 23456789012345678901234567890121234567890123456789012345678 123456789012345678901234567890121234567890123456789012345678 99 123456789012345678901234567890121234567890123456789012345678 because it begins the essay with the word it, which is never a strong 9 123456789012345678901234567890121234567890123456789012345678 9 123456789012345678901234567890121234567890123456789012345678 opener. (E) is suspect because the essay is about the word modern not 9 1 23456789012345678901234567890121234567890123456789012345678 9 123456789012345678901234567890121234567890123456789012345678 modernity. (Yes, modernity is later introduced, but that does not 9 123456789012345678901234567890121234567890123456789012345678 9 123456789012345678901234567890121234567890123456789012345678 change the point that the essay is about the word modern.) (E) is also 123456789012345678901234567890121234567890123456789012345678 99 1 unnecessarily wordy. That leaves (D). 9 123456789012345678901234567890121234567890123456789012345678 123456789012345678901234567890121234567890123456789012345678 9 9 123456789012345678901234567890121234567890123456789012345678 123456789012345678901234567890121234567890123456789012345678 9 9 123456789012345678901234567890121234567890123456789012345678 123456789012345678901234567890121234567890123456789012345678 9 123456789012345678901234567890121234567890123456789012345678 9 123456789012345678901234567890121234567890123456789012345678 9 123456789012345678901234567890121234567890123456789012345678 9 1234567890123456789012345678901212345678901234567890123456789 Copyright © 2005 Thomson Peterson’s, a part of The Thomson Corporaton SAT is a registered trademark of the College Entrance Examination Board, which was not involved in the production of and does not endorse this product.

61

1234567890123456789012345678901212345678901234567890123456789 123456789012345678901234567890121234567890123456789012345678 9 9 123456789012345678901234567890121234567890123456789012345678 123456789012345678901234567890121234567890123456789012345678 9 9 123456789012345678901234567890121234567890123456789012345678 22. B Descartes is the thing that will link the two sentences. He is the subject 9 123456789012345678901234567890121234567890123456789012345678 9 123456789012345678901234567890121234567890123456789012345678 of the first and the object of the second. This is a prime example of 9 123456789012345678901234567890121234567890123456789012345678 23456789012345678901234567890121234567890123456789012345678 9 1 when to use whom. (B) is better than (C) and the others are longer than 9 123456789012345678901234567890121234567890123456789012345678 9 123456789012345678901234567890121234567890123456789012345678 the original, not shorter. (B) is the answer. 9 123456789012345678901234567890121234567890123456789012345678 9 123456789012345678901234567890121234567890123456789012345678 23456789012345678901234567890121234567890123456789012345678 9 1 23. E What is the logical connection between the opinions of academics other 9 123456789012345678901234567890121234567890123456789012345678 9 123456789012345678901234567890121234567890123456789012345678 than philosophers and the opinions of historians in particular? The 9 123456789012345678901234567890121234567890123456789012345678 9 123456789012345678901234567890121234567890123456789012345678 latter is an example of the former. The answer is (E). 23456789012345678901234567890121234567890123456789012345678 9 1 9 123456789012345678901234567890121234567890123456789012345678 24. B The original sentence is clunky because it has both beginning of 9 123456789012345678901234567890121234567890123456789012345678 9 123456789012345678901234567890121234567890123456789012345678 modernity and beginning of modern times. You will want to replace the 9 123456789012345678901234567890121234567890123456789012345678 23456789012345678901234567890121234567890123456789012345678 9 1 second with it. That leaves (B) and possibly (E). (E) adds unhelpful 9 123456789012345678901234567890121234567890123456789012345678 9 123456789012345678901234567890121234567890123456789012345678 extra verbiage in other places, so (B) is the best answer. 9 123456789012345678901234567890121234567890123456789012345678 9 123456789012345678901234567890121234567890123456789012345678 23456789012345678901234567890121234567890123456789012345678 9 123456789012345678901234567890121234567890123456789012345678 25. E Anything separated by commas should be detachable from the rest of 9 123456789012345678901234567890121234567890123456789012345678 9 123456789012345678901234567890121234567890123456789012345678 the sentence. Remove the phrase between commas and you get Most of 123456789012345678901234567890121234567890123456789012345678 9 9 123456789012345678901234567890121234567890123456789012345678 us. . . we are thinking. Bad. Most of us think. (E) is the only answer that 9 123456789012345678901234567890121234567890123456789012345678 9 1 addresses that problem correctly. (B) actually changes the sense by 9 123456789012345678901234567890121234567890123456789012345678 9 123456789012345678901234567890121234567890123456789012345678 eliminating the subject of “think” so that it becomes a command. 9 123456789012345678901234567890121234567890123456789012345678 23456789012345678901234567890121234567890123456789012345678 123456789012345678901234567890121234567890123456789012345678 9 9 123456789012345678901234567890121234567890123456789012345678 26. B Choices (C) and (E) have an imprecise and unclear 2ing verb, so 9 123456789012345678901234567890121234567890123456789012345678 eliminate them. (D) is just a wordier alternative to (B), so eliminate it. 9 123456789012345678901234567890121234567890123456789012345678 9 123456789012345678901234567890121234567890123456789012345678 (B) is so straightforward and clear that it is better than the original. The 123456789012345678901234567890121234567890123456789012345678 9 9 123456789012345678901234567890121234567890123456789012345678 answer is (B). 9 123456789012345678901234567890121234567890123456789012345678 123456789012345678901234567890121234567890123456789012345678 9 9 1 27. E The passage is talking about the 1920s. Be as specific as possible, 9 123456789012345678901234567890121234567890123456789012345678 9 123456789012345678901234567890121234567890123456789012345678 without being wordy as in (C) and (D). The answer is (E). 123456789012345678901234567890121234567890123456789012345678 9 123456789012345678901234567890121234567890123456789012345678 9 9 123456789012345678901234567890121234567890123456789012345678 28. C The only punctuation marks that are possible here are parentheses, a 9 123456789012345678901234567890121234567890123456789012345678 9 123456789012345678901234567890121234567890123456789012345678 semicolon or a colon. The second sentence gives more specific 123456789012345678901234567890121234567890123456789012345678 9 9 123456789012345678901234567890121234567890123456789012345678 information to elaborate on the first sentence, so a colon is better than 9 123456789012345678901234567890121234567890123456789012345678 23456789012345678901234567890121234567890123456789012345678 9 123456789012345678901234567890121234567890123456789012345678 a semicolon or parentheses. The answer is (C). 9 123456789012345678901234567890121234567890123456789012345678 9 1 29. D What’s wrong with the transition in the original? Is it a logical problem 123456789012345678901234567890121234567890123456789012345678 9 9 123456789012345678901234567890121234567890123456789012345678 or a grammatical problem? It sounds weird and wordy, so it’s a 9 123456789012345678901234567890121234567890123456789012345678 9 123456789012345678901234567890121234567890123456789012345678 grammatical problem. The sense is logical, so keep it. Eliminate (A); it 9 123456789012345678901234567890121234567890123456789012345678 9 123456789012345678901234567890121234567890123456789012345678 changes the sense. Which of the answers simplifies and clarifies the 123456789012345678901234567890121234567890123456789012345678 9 23456789012345678901234567890121234567890123456789012345678 9 123456789012345678901234567890121234567890123456789012345678 grammar? The answer is (D). 9 123456789012345678901234567890121234567890123456789012345678 123456789012345678901234567890121234567890123456789012345678 9 9 123456789012345678901234567890121234567890123456789012345678 9 1 30. A Eliminate (D) because it’s grammatically incorrect. Eliminate (C) 23456789012345678901234567890121234567890123456789012345678 9 123456789012345678901234567890121234567890123456789012345678 because it really doesn’t go with the ideas in the passage. If you look 9 123456789012345678901234567890121234567890123456789012345678 9 123456789012345678901234567890121234567890123456789012345678 back at the passage, you’ll see that the last sentence is not about Macy’s 123456789012345678901234567890121234567890123456789012345678 9 9 1 but Bloomingdales. (E) would be out of place, so eliminate it. You’re 9 123456789012345678901234567890121234567890123456789012345678 9 123456789012345678901234567890121234567890123456789012345678 left with (A) and (B). While (B) would be an adequate conclusion to the 9 123456789012345678901234567890121234567890123456789012345678 9 123456789012345678901234567890121234567890123456789012345678 third paragraph, only (A) reflects back over the entire passage. The best 9 123456789012345678901234567890121234567890123456789012345678 23456789012345678901234567890121234567890123456789012345678 9 1 answer is (A). 9 123456789012345678901234567890121234567890123456789012345678 123456789012345678901234567890121234567890123456789012345678 9 123456789012345678901234567890121234567890123456789012345678 9 1234567890123456789012345678901212345678901234567890123456789 62

Copyright © 2005 Thomson Peterson’s, a part of The Thomson Corporaton SAT is a registered trademark of the College Entrance Examination Board, which was not involved in the production of and does not endorse this product.

1234567890123456789012345678901212345678901234567890123456789 123456789012345678901234567890121234567890123456789012345678 9 9 123456789012345678901234567890121234567890123456789012345678 123456789012345678901234567890121234567890123456789012345678 9 9 123456789012345678901234567890121234567890123456789012345678 9 123456789012345678901234567890121234567890123456789012345678 Section 4 123456789012345678901234567890121234567890123456789012345678 9 9 1. D Jerome does not indulge much. He is an ascetic, (D). A teetotaler, (B), is 123456789012345678901234567890121234567890123456789012345678 9 123456789012345678901234567890121234567890123456789012345678 someone who does not consume alcohol. A gourmand, (C), is someone 9 123456789012345678901234567890121234567890123456789012345678 9 123456789012345678901234567890121234567890123456789012345678 who indulges a great deal in fine food. Neither (A) nor (E) have 9 123456789012345678901234567890121234567890123456789012345678 9 123456789012345678901234567890121234567890123456789012345678 anything to do with indulgence. 23456789012345678901234567890121234567890123456789012345678 9 1 9 123456789012345678901234567890121234567890123456789012345678 9 123456789012345678901234567890121234567890123456789012345678 2. A Begin with the first blank, since you know that it must be something 9 123456789012345678901234567890121234567890123456789012345678 9 123456789012345678901234567890121234567890123456789012345678 negative since protests were planned. (B) and (D) are not negative, so 23456789012345678901234567890121234567890123456789012345678 9 1 9 123456789012345678901234567890121234567890123456789012345678 eliminate them. Going to the second blank, organized fits well, while 9 123456789012345678901234567890121234567890123456789012345678 negotiated and theorized do not (you don’t theorize about a protest 9 123456789012345678901234567890121234567890123456789012345678 9 123456789012345678901234567890121234567890123456789012345678 event). This makes (A) the best answer. 23456789012345678901234567890121234567890123456789012345678 9 1 9 123456789012345678901234567890121234567890123456789012345678 9 123456789012345678901234567890121234567890123456789012345678 3. B We know the first word will be negative, since it engendered criticism. 9 123456789012345678901234567890121234567890123456789012345678 9 123456789012345678901234567890121234567890123456789012345678 All of the words are negative, so you can’t eliminate anything yet. But 23456789012345678901234567890121234567890123456789012345678 123456789012345678901234567890121234567890123456789012345678 99 123456789012345678901234567890121234567890123456789012345678 what would a politician be if he “seized a ceremony” having to do with 9 123456789012345678901234567890121234567890123456789012345678 9 123456789012345678901234567890121234567890123456789012345678 a “girl’s tragic death to speak out against his opponent”? Not militaris9 123456789012345678901234567890121234567890123456789012345678 tic, or unreceptive, certainly not passive or defeatist. The answer is (B). 123456789012345678901234567890121234567890123456789012345678 99 1 9 123456789012345678901234567890121234567890123456789012345678 4. E All of the options in this question have a first word that could be 9 123456789012345678901234567890121234567890123456789012345678 9 123456789012345678901234567890121234567890123456789012345678 something one software did to another. For the second word, all the 23456789012345678901234567890121234567890123456789012345678 123456789012345678901234567890121234567890123456789012345678 99 123456789012345678901234567890121234567890123456789012345678 choices, except (D), are things a user could likely do to software. 9 123456789012345678901234567890121234567890123456789012345678 9 123456789012345678901234567890121234567890123456789012345678 Preclude, (A), means make something impossible, but the “prior 123456789012345678901234567890121234567890123456789012345678 99 123456789012345678901234567890121234567890123456789012345678 version” clearly isn’t impossible so eliminate (A). If the older version is 9 123456789012345678901234567890121234567890123456789012345678 9 123456789012345678901234567890121234567890123456789012345678 outdone, you don’t want to implement, or use, it. Eliminate (B). If 9 123456789012345678901234567890121234567890123456789012345678 something is infected, you don’t want to disregard it, so eliminate (C). 9 1 23456789012345678901234567890121234567890123456789012345678 9 123456789012345678901234567890121234567890123456789012345678 The answer is (E). Supercede means replace by being better, which 9 123456789012345678901234567890121234567890123456789012345678 9 1 would mean you could throw out, or discard, the now useless thing. 123456789012345678901234567890121234567890123456789012345678 9 9 123456789012345678901234567890121234567890123456789012345678 9 123456789012345678901234567890121234567890123456789012345678 5. B Let’s start with the first blank since it has to be paired with long. Which 9 123456789012345678901234567890121234567890123456789012345678 9 123456789012345678901234567890121234567890123456789012345678 of the first answer choices pairs well with long? Only storied, 9 123456789012345678901234567890121234567890123456789012345678 9 123456789012345678901234567890121234567890123456789012345678 exemplary, and reputable do (the others either are awkward or don’t fit 23456789012345678901234567890121234567890123456789012345678 123456789012345678901234567890121234567890123456789012345678 99 123456789012345678901234567890121234567890123456789012345678 with the meaning of the sentence). Plug in the second answer choices of 9 1 9 123456789012345678901234567890121234567890123456789012345678 (A), (B), and (E) into the sentence. Can poetry initiate the dignity of 123456789012345678901234567890121234567890123456789012345678 9 23456789012345678901234567890121234567890123456789012345678 9 123456789012345678901234567890121234567890123456789012345678 humanity? No. Eliminate (A). Can it articulate, or express? Definitely. 9 123456789012345678901234567890121234567890123456789012345678 9 Choice (B) sounds too good to pass up. 123456789012345678901234567890121234567890123456789012345678 123456789012345678901234567890121234567890123456789012345678 99 1 6. C You should have a good sense of what kind of word goes in each blank 9 123456789012345678901234567890121234567890123456789012345678 9 123456789012345678901234567890121234567890123456789012345678 because each blank has a pair. The second blank has a more obvious 9 123456789012345678901234567890121234567890123456789012345678 9 123456789012345678901234567890121234567890123456789012345678 pair, so start with it. Which of the second answer choices goes with 123456789012345678901234567890121234567890123456789012345678 9 23456789012345678901234567890121234567890123456789012345678 9 123456789012345678901234567890121234567890123456789012345678 speedy? (A) and (C) are the best candidates. On the first blank, does 9 123456789012345678901234567890121234567890123456789012345678 9 123456789012345678901234567890121234567890123456789012345678 intricate and laborious or fragile and laborious sound better? Intricate 123456789012345678901234567890121234567890123456789012345678 99 1 does because it provides a better contrast with the second half of the 9 123456789012345678901234567890121234567890123456789012345678 9 123456789012345678901234567890121234567890123456789012345678 sentence. (Also, it is strange to speak of a surgery being fragile; someone 9 123456789012345678901234567890121234567890123456789012345678 9 123456789012345678901234567890121234567890123456789012345678 in surgery might be in fragile health, but the surgery itself would not be 9 123456789012345678901234567890121234567890123456789012345678 23456789012345678901234567890121234567890123456789012345678 9 1 fragile.) (C) is the answer. 9 123456789012345678901234567890121234567890123456789012345678 123456789012345678901234567890121234567890123456789012345678 99 123456789012345678901234567890121234567890123456789012345678 1234567890123456789012345678901212345678901234567890123456789 Copyright © 2005 Thomson Peterson’s, a part of The Thomson Corporaton SAT is a registered trademark of the College Entrance Examination Board, which was not involved in the production of and does not endorse this product.

63

1234567890123456789012345678901212345678901234567890123456789 123456789012345678901234567890121234567890123456789012345678 9 9 123456789012345678901234567890121234567890123456789012345678 123456789012345678901234567890121234567890123456789012345678 9 9 123456789012345678901234567890121234567890123456789012345678 7. A Despite at the beginning of the sentence is the main clue to unlocking 9 123456789012345678901234567890121234567890123456789012345678 9 123456789012345678901234567890121234567890123456789012345678 the blank. You need one positive word and one negative word. 9 123456789012345678901234567890121234567890123456789012345678 23456789012345678901234567890121234567890123456789012345678 9 1 Eliminate (B) and (C). Now what can clans do to each other? Assist, 9 123456789012345678901234567890121234567890123456789012345678 9 123456789012345678901234567890121234567890123456789012345678 possibly discount. A problem is ameliorated, not a person or group of 9 123456789012345678901234567890121234567890123456789012345678 9 123456789012345678901234567890121234567890123456789012345678 people. (A) is the better answer, especially since “feud” and “clan” are 23456789012345678901234567890121234567890123456789012345678 9 1 9 123456789012345678901234567890121234567890123456789012345678 both words that seem to refer to the past. 9 123456789012345678901234567890121234567890123456789012345678 9 123456789012345678901234567890121234567890123456789012345678 9 123456789012345678901234567890121234567890123456789012345678 8. B From the structure of the sentence, we can see that the word in the 23456789012345678901234567890121234567890123456789012345678 9 1 9 123456789012345678901234567890121234567890123456789012345678 blank goes along with humor and merriment. Which of the answer 9 123456789012345678901234567890121234567890123456789012345678 9 choices fits with these two positive words? Only (B), mirth, does. 123456789012345678901234567890121234567890123456789012345678 9 123456789012345678901234567890121234567890123456789012345678 (Irony, history, and mystery, are at best neutral terms in this context, 23456789012345678901234567890121234567890123456789012345678 9 1 9 123456789012345678901234567890121234567890123456789012345678 not positive.) 9 123456789012345678901234567890121234567890123456789012345678 9 123456789012345678901234567890121234567890123456789012345678 9 123456789012345678901234567890121234567890123456789012345678 9. A The Latin makes this sentence complicated. You can combat this by 23456789012345678901234567890121234567890123456789012345678 123456789012345678901234567890121234567890123456789012345678 9 9 123456789012345678901234567890121234567890123456789012345678 replacing the Latin words with the letters (A) and (B) in your head (i.e. 9 123456789012345678901234567890121234567890123456789012345678 9 123456789012345678901234567890121234567890123456789012345678 although (A) has more subspecies than (B) . . . ). The although is the key 123456789012345678901234567890121234567890123456789012345678 9 9 123456789012345678901234567890121234567890123456789012345678 to the logic of the sentence structure. Although (A) has more subspecies 9 1 9 123456789012345678901234567890121234567890123456789012345678 than (B), (B) has greater numbers. (A) and (E) match this pre-guess for 9 123456789012345678901234567890121234567890123456789012345678 9 123456789012345678901234567890121234567890123456789012345678 the first blank. (E) isn’t really logical (how can a region be ominous?), 23456789012345678901234567890121234567890123456789012345678 123456789012345678901234567890121234567890123456789012345678 9 9 123456789012345678901234567890121234567890123456789012345678 so the answer is (A). 9 123456789012345678901234567890121234567890123456789012345678 123456789012345678901234567890121234567890123456789012345678 9 9 123456789012345678901234567890121234567890123456789012345678 10. D Looking at the first blank, (B) and (E) can be eliminated because both 9 123456789012345678901234567890121234567890123456789012345678 9 123456789012345678901234567890121234567890123456789012345678 are not conventional English. That leaves (A), (C), and (D). (A) is 9 123456789012345678901234567890121234567890123456789012345678 suspect, though, because progression is an odd word choice for the 9 123456789012345678901234567890121234567890123456789012345678 9 1 context. (C) is illogical. (D) is a best choice because it flows well in both 23456789012345678901234567890121234567890123456789012345678 123456789012345678901234567890121234567890123456789012345678 9 9 123456789012345678901234567890121234567890123456789012345678 blanks and because development is already a word you associate with 9 1 9 123456789012345678901234567890121234567890123456789012345678 “pregnancy” and “fetus.” 9 123456789012345678901234567890121234567890123456789012345678 23456789012345678901234567890121234567890123456789012345678 123456789012345678901234567890121234567890123456789012345678 9 9 123456789012345678901234567890121234567890123456789012345678 9 1 11. C The word, Though, tells you that the ideals and the practices of the 9 123456789012345678901234567890121234567890123456789012345678 organization are at odds. So if the ideals are outward-focused then the 9 123456789012345678901234567890121234567890123456789012345678 23456789012345678901234567890121234567890123456789012345678 9 123456789012345678901234567890121234567890123456789012345678 practice must be inward-focused. Which of the answer choices matches 9 123456789012345678901234567890121234567890123456789012345678 9 1 this pre-guess? Only (C), insular (isolated, circumscribed), does. 123456789012345678901234567890121234567890123456789012345678 9 9 123456789012345678901234567890121234567890123456789012345678 Hermetic means tightly sealed. It’s not too far off in meaning, but it 9 123456789012345678901234567890121234567890123456789012345678 9 123456789012345678901234567890121234567890123456789012345678 doesn’t refer to group behavior. 9 123456789012345678901234567890121234567890123456789012345678 9 123456789012345678901234567890121234567890123456789012345678 9 123456789012345678901234567890121234567890123456789012345678 12. B On the first blank we know that the word goes with base, and we know 23456789012345678901234567890121234567890123456789012345678 9 123456789012345678901234567890121234567890123456789012345678 that the prosecuting attorney is probably not saying nice things about 9 123456789012345678901234567890121234567890123456789012345678 9 123456789012345678901234567890121234567890123456789012345678 the defendant. (A) and (B) fit this (motley doesn’t; even though it has a 9 123456789012345678901234567890121234567890123456789012345678 9 1 negative connotation, it means a random or ungainly assortment). This 23456789012345678901234567890121234567890123456789012345678 123456789012345678901234567890121234567890123456789012345678 9 9 123456789012345678901234567890121234567890123456789012345678 leaves misunderstanding the testimonials or misconstruing the testimo9 123456789012345678901234567890121234567890123456789012345678 9 123456789012345678901234567890121234567890123456789012345678 nials, for the second blank. The latter is a better choice since the 9 1 23456789012345678901234567890121234567890123456789012345678 9 123456789012345678901234567890121234567890123456789012345678 prosecuting attorney is more likely to be making the defendant sound 9 1 9 123456789012345678901234567890121234567890123456789012345678 bad on purpose than by mistake. 123456789012345678901234567890121234567890123456789012345678 9 9 123456789012345678901234567890121234567890123456789012345678 123456789012345678901234567890121234567890123456789012345678 9 123456789012345678901234567890121234567890123456789012345678 9 123456789012345678901234567890121234567890123456789012345678 9 123456789012345678901234567890121234567890123456789012345678 9 1234567890123456789012345678901212345678901234567890123456789 64

Copyright © 2005 Thomson Peterson’s, a part of The Thomson Corporaton SAT is a registered trademark of the College Entrance Examination Board, which was not involved in the production of and does not endorse this product.

1234567890123456789012345678901212345678901234567890123456789 123456789012345678901234567890121234567890123456789012345678 9 9 123456789012345678901234567890121234567890123456789012345678 123456789012345678901234567890121234567890123456789012345678 9 9 123456789012345678901234567890121234567890123456789012345678 13. B This sentence is either really positive or really negative about the new 9 123456789012345678901234567890121234567890123456789012345678 9 123456789012345678901234567890121234567890123456789012345678 drug regime. It is more likely positive since a new drug wouldn’t come 9 123456789012345678901234567890121234567890123456789012345678 23456789012345678901234567890121234567890123456789012345678 9 1 out if it was known to be really negative. So your inclination should be 9 123456789012345678901234567890121234567890123456789012345678 9 123456789012345678901234567890121234567890123456789012345678 that the blank is very positive. Only (B) fits this, and it fits well. If you 9 123456789012345678901234567890121234567890123456789012345678 9 123456789012345678901234567890121234567890123456789012345678 have extra time, you can check to see if there is a really negative 23456789012345678901234567890121234567890123456789012345678 9 1 9 123456789012345678901234567890121234567890123456789012345678 response that might fit better. Bane could work in terms of meaning, 9 123456789012345678901234567890121234567890123456789012345678 9 123456789012345678901234567890121234567890123456789012345678 but it doesn’t work syntactically. 9 123456789012345678901234567890121234567890123456789012345678 23456789012345678901234567890121234567890123456789012345678 9 1 9 123456789012345678901234567890121234567890123456789012345678 14. C The blanks in this sentence are a little more complicated because they 9 123456789012345678901234567890121234567890123456789012345678 9 must be considered together. The first blank is negative and the second 123456789012345678901234567890121234567890123456789012345678 9 123456789012345678901234567890121234567890123456789012345678 blank is even more negative (in the same way, or to a greater degree). 23456789012345678901234567890121234567890123456789012345678 9 1 9 123456789012345678901234567890121234567890123456789012345678 The second blank isn’t negative in (A) and (D), so they are not it. That 9 123456789012345678901234567890121234567890123456789012345678 9 123456789012345678901234567890121234567890123456789012345678 leaves (B), (C), and (E). Of the three, (C) is the best since self-centered 9 123456789012345678901234567890121234567890123456789012345678 23456789012345678901234567890121234567890123456789012345678 9 123456789012345678901234567890121234567890123456789012345678 and egocentric are synonyms and solipsism is a more negative form of 9 123456789012345678901234567890121234567890123456789012345678 9 123456789012345678901234567890121234567890123456789012345678 both words. You might be tempted to go with (B) because you don’t 123456789012345678901234567890121234567890123456789012345678 99 123456789012345678901234567890121234567890123456789012345678 know what solipsistic means, but you can eliminate it because 9 123456789012345678901234567890121234567890123456789012345678 9 1 ill-tempered has little to do with self-centered. 9 123456789012345678901234567890121234567890123456789012345678 9 123456789012345678901234567890121234567890123456789012345678 9 123456789012345678901234567890121234567890123456789012345678 15. B What is a good pre-guess for the blank? Really hectic (that is, opposite 23456789012345678901234567890121234567890123456789012345678 123456789012345678901234567890121234567890123456789012345678 99 123456789012345678901234567890121234567890123456789012345678 of alleviating demands upon our time). Only (B) matches this pre-guess, 9 123456789012345678901234567890121234567890123456789012345678 9 123456789012345678901234567890121234567890123456789012345678 and it fits in the flow of the sentence. 123456789012345678901234567890121234567890123456789012345678 99 123456789012345678901234567890121234567890123456789012345678 9 123456789012345678901234567890121234567890123456789012345678 16. A This is a global question since it cannot be answered by looking at one 9 123456789012345678901234567890121234567890123456789012345678 specific place in the text but has to be weighed considering the passage 9 123456789012345678901234567890121234567890123456789012345678 9 1 as a whole. Ask yourself, does the author present Dr. Rael positively or 23456789012345678901234567890121234567890123456789012345678 123456789012345678901234567890121234567890123456789012345678 99 123456789012345678901234567890121234567890123456789012345678 negatively, sympathetically or unsympathetically? The article describes 9 1 9 123456789012345678901234567890121234567890123456789012345678 Dr. Rael’s career, generally in positive terms, and no criticisms of Dr. 9 123456789012345678901234567890121234567890123456789012345678 23456789012345678901234567890121234567890123456789012345678 9 123456789012345678901234567890121234567890123456789012345678 Rael’s work is discussed. So the answer should be positive. That 9 123456789012345678901234567890121234567890123456789012345678 9 1 eliminates (C)—(E). (B) might seem appealing, but realize that an article 9 123456789012345678901234567890121234567890123456789012345678 9 123456789012345678901234567890121234567890123456789012345678 can engage a person’s work without being positive about that work. 23456789012345678901234567890121234567890123456789012345678 123456789012345678901234567890121234567890123456789012345678 99 123456789012345678901234567890121234567890123456789012345678 Also the passage more tells about Dr. Rael’s work than engages it. So 9 1 9 123456789012345678901234567890121234567890123456789012345678 (A) is the best choice. 123456789012345678901234567890121234567890123456789012345678 9 23456789012345678901234567890121234567890123456789012345678 123456789012345678901234567890121234567890123456789012345678 99 123456789012345678901234567890121234567890123456789012345678 17. C Again this is a global question. To rephrase the question, what is the 9 123456789012345678901234567890121234567890123456789012345678 9 123456789012345678901234567890121234567890123456789012345678 passage about? It tells the story of Dr. Rael’s life with specific emphasis 9 1 23456789012345678901234567890121234567890123456789012345678 9 123456789012345678901234567890121234567890123456789012345678 on his professional career. Choice (C) captures this. You might have 9 123456789012345678901234567890121234567890123456789012345678 9 123456789012345678901234567890121234567890123456789012345678 been attracted by (E), but remember the passage only discusses the Rael 123456789012345678901234567890121234567890123456789012345678 99 1 family when it contributes to the story of Dr. Rael. 9 123456789012345678901234567890121234567890123456789012345678 9 123456789012345678901234567890121234567890123456789012345678 18. B You can answer this either as a detail question or as a global question; 9 123456789012345678901234567890121234567890123456789012345678 9 123456789012345678901234567890121234567890123456789012345678 you will get the answer more quickly if you answer it as a detail 123456789012345678901234567890121234567890123456789012345678 9 23456789012345678901234567890121234567890123456789012345678 9 123456789012345678901234567890121234567890123456789012345678 question. You can find in paragraph 3 that “José Ignacio had the 9 1 9 123456789012345678901234567890121234567890123456789012345678 foresight to recognize the changes that were coming with the increasing 123456789012345678901234567890121234567890123456789012345678 9 9 123456789012345678901234567890121234567890123456789012345678 Americanization of New Mexico and realized that a fluent knowledge 9 123456789012345678901234567890121234567890123456789012345678 123456789012345678901234567890121234567890123456789012345678 9 123456789012345678901234567890121234567890123456789012345678 9 123456789012345678901234567890121234567890123456789012345678 9 1234567890123456789012345678901212345678901234567890123456789 Copyright © 2005 Thomson Peterson’s, a part of The Thomson Corporaton SAT is a registered trademark of the College Entrance Examination Board, which was not involved in the production of and does not endorse this product.

65

1234567890123456789012345678901212345678901234567890123456789 123456789012345678901234567890121234567890123456789012345678 9 9 123456789012345678901234567890121234567890123456789012345678 123456789012345678901234567890121234567890123456789012345678 9 9 123456789012345678901234567890121234567890123456789012345678 of English. . . would be necessary.” Choices (B) and (D) seem to echo 9 123456789012345678901234567890121234567890123456789012345678 9 123456789012345678901234567890121234567890123456789012345678 this sentence. But the passage does not go on to say that Dr. Rael did not 9 123456789012345678901234567890121234567890123456789012345678 23456789012345678901234567890121234567890123456789012345678 9 1 learn English. The answer is (B). If you answer it as a global question, 9 123456789012345678901234567890121234567890123456789012345678 9 123456789012345678901234567890121234567890123456789012345678 use the process of elimination. The passage doesn’t mention job 9 123456789012345678901234567890121234567890123456789012345678 9 123456789012345678901234567890121234567890123456789012345678 discrimination. The passage refers to Rael studying far from his family, 23456789012345678901234567890121234567890123456789012345678 9 1 9 123456789012345678901234567890121234567890123456789012345678 but doesn’t mention that problem in regard to teaching. It strongly 9 123456789012345678901234567890121234567890123456789012345678 9 123456789012345678901234567890121234567890123456789012345678 suggests that Rael was successful in American schools with English 9 123456789012345678901234567890121234567890123456789012345678 23456789012345678901234567890121234567890123456789012345678 9 1 names, so eliminate (D). The passage doesn’t specify what Rael studied 9 123456789012345678901234567890121234567890123456789012345678 9 123456789012345678901234567890121234567890123456789012345678 before his Ph.D. Choices (C) and (E) are hard to eliminate, but an 9 123456789012345678901234567890121234567890123456789012345678 9 123456789012345678901234567890121234567890123456789012345678 overall view of the passage should convey that the folklore was precious 23456789012345678901234567890121234567890123456789012345678 9 1 9 123456789012345678901234567890121234567890123456789012345678 and disappearing. 9 123456789012345678901234567890121234567890123456789012345678 9 123456789012345678901234567890121234567890123456789012345678 19. E Relinquished means he gave something up, but the passage does not state 9 123456789012345678901234567890121234567890123456789012345678 23456789012345678901234567890121234567890123456789012345678 9 123456789012345678901234567890121234567890123456789012345678 that Rael was relieved to give up his family duties. Eliminate (D). There is 9 123456789012345678901234567890121234567890123456789012345678 9 123456789012345678901234567890121234567890123456789012345678 no mention of sibling rivalry, so you can eliminate (A). The passage doesn’t 123456789012345678901234567890121234567890123456789012345678 9 9 123456789012345678901234567890121234567890123456789012345678 say if the family was wealthy or simply OK, so (B) is out. It certainly 9 123456789012345678901234567890121234567890123456789012345678 9 1 doesn’t say Rael had tried to be a rancher, choice (C). That leaves (E). 9 123456789012345678901234567890121234567890123456789012345678 9 123456789012345678901234567890121234567890123456789012345678 9 123456789012345678901234567890121234567890123456789012345678 20. E This question is similar to question 18, but it is definitely a global 23456789012345678901234567890121234567890123456789012345678 123456789012345678901234567890121234567890123456789012345678 9 9 123456789012345678901234567890121234567890123456789012345678 question. The basic gist of the passage is that Rael’s work was 9 123456789012345678901234567890121234567890123456789012345678 9 123456789012345678901234567890121234567890123456789012345678 important. He collected Spanish-language folklore and studied the 123456789012345678901234567890121234567890123456789012345678 9 9 123456789012345678901234567890121234567890123456789012345678 particular Spanish used in the area. The answer is (E). 9 123456789012345678901234567890121234567890123456789012345678 9 123456789012345678901234567890121234567890123456789012345678 21. C Someone you study under is your mentor. This is a vocabulary question. 9 123456789012345678901234567890121234567890123456789012345678 9 1 The answer is (C). 23456789012345678901234567890121234567890123456789012345678 123456789012345678901234567890121234567890123456789012345678 9 123456789012345678901234567890121234567890123456789012345678 9 9 1 22. B Corpus refers to a body of work. The only answer choice that reflects 123456789012345678901234567890121234567890123456789012345678 9 9 123456789012345678901234567890121234567890123456789012345678 the idea of more than one book is (B). 9 123456789012345678901234567890121234567890123456789012345678 9 123456789012345678901234567890121234567890123456789012345678 9 123456789012345678901234567890121234567890123456789012345678 23. B Looking over the choices, there is no reference to Dr. Rael enjoying 9 123456789012345678901234567890121234567890123456789012345678 9 123456789012345678901234567890121234567890123456789012345678 working on the ranch (or using his scholarly pursuits to avoid it, which 23456789012345678901234567890121234567890123456789012345678 123456789012345678901234567890121234567890123456789012345678 9 9 123456789012345678901234567890121234567890123456789012345678 would already be using logic too strained for the SAT). The passage 9 1 9 123456789012345678901234567890121234567890123456789012345678 does mention his love of Pastores. Just after the mention of Pastores, 123456789012345678901234567890121234567890123456789012345678 9 23456789012345678901234567890121234567890123456789012345678 9 123456789012345678901234567890121234567890123456789012345678 the passage says this influenced his later work. Bingo. The answer is (B). 9 123456789012345678901234567890121234567890123456789012345678 9 123456789012345678901234567890121234567890123456789012345678 24. A This is a tough one. But “diffusion of motifs” seems to be related to the 9 123456789012345678901234567890121234567890123456789012345678 9 1 number of variants of a particular story. That eliminates (C) and (E). 23456789012345678901234567890121234567890123456789012345678 123456789012345678901234567890121234567890123456789012345678 9 9 123456789012345678901234567890121234567890123456789012345678 The passage goes on to emphasize how many variants there were, 9 123456789012345678901234567890121234567890123456789012345678 9 123456789012345678901234567890121234567890123456789012345678 suggesting that the answer will emphasize difference more than 9 1 23456789012345678901234567890121234567890123456789012345678 9 123456789012345678901234567890121234567890123456789012345678 sameness. Eliminate (B) and (D). The answer is (A). 9 123456789012345678901234567890121234567890123456789012345678 123456789012345678901234567890121234567890123456789012345678 9 9 123456789012345678901234567890121234567890123456789012345678 9 1 25. B If you do not know the definition of provenance you are not lost on this 23456789012345678901234567890121234567890123456789012345678 9 123456789012345678901234567890121234567890123456789012345678 one. Replace it with each of the answer choices and see which one 9 1 9 123456789012345678901234567890121234567890123456789012345678 makes the most sense. If you do know the definition of provenance 123456789012345678901234567890121234567890123456789012345678 9 9 123456789012345678901234567890121234567890123456789012345678 (origin), then (B) pops out as the answer. 9 123456789012345678901234567890121234567890123456789012345678 123456789012345678901234567890121234567890123456789012345678 9 123456789012345678901234567890121234567890123456789012345678 9 123456789012345678901234567890121234567890123456789012345678 9 1234567890123456789012345678901212345678901234567890123456789 66

Copyright © 2005 Thomson Peterson’s, a part of The Thomson Corporaton SAT is a registered trademark of the College Entrance Examination Board, which was not involved in the production of and does not endorse this product.

1234567890123456789012345678901212345678901234567890123456789 123456789012345678901234567890121234567890123456789012345678 9 9 123456789012345678901234567890121234567890123456789012345678 123456789012345678901234567890121234567890123456789012345678 9 9 123456789012345678901234567890121234567890123456789012345678 26. D The end of the seventh paragraph states, “But inevitably the historic9 123456789012345678901234567890121234567890123456789012345678 9 123456789012345678901234567890121234567890123456789012345678 geographic approach led more to collection building than to analysis.” 9 123456789012345678901234567890121234567890123456789012345678 23456789012345678901234567890121234567890123456789012345678 9 1 This is the most critical sentence in the passage. The answer (D) 9 123456789012345678901234567890121234567890123456789012345678 9 123456789012345678901234567890121234567890123456789012345678 includes the word “analysis” and is a fair paraphrase of this sentence. 9 123456789012345678901234567890121234567890123456789012345678 9 123456789012345678901234567890121234567890123456789012345678 The answer is (D). 23456789012345678901234567890121234567890123456789012345678 9 1 9 123456789012345678901234567890121234567890123456789012345678 9 123456789012345678901234567890121234567890123456789012345678 27. B This one is also hard. First of all, do you know what formidable and 9 123456789012345678901234567890121234567890123456789012345678 9 123456789012345678901234567890121234567890123456789012345678 quixotic mean? Impressive, difficult, and errant—which is to say, 23456789012345678901234567890121234567890123456789012345678 9 1 9 123456789012345678901234567890121234567890123456789012345678 traveling a lot. This comes from Don Quixote, who traveled a lot. He 9 123456789012345678901234567890121234567890123456789012345678 9 was also a little crazy, which is why the test writers try to trick you with 123456789012345678901234567890121234567890123456789012345678 9 123456789012345678901234567890121234567890123456789012345678 (A) and (C). But these don’t apply to Dr. Rael. Remember, the overall 23456789012345678901234567890121234567890123456789012345678 9 1 9 123456789012345678901234567890121234567890123456789012345678 tone is “laudatory.” (E) tries to trick you by getting you to admit that 9 123456789012345678901234567890121234567890123456789012345678 9 123456789012345678901234567890121234567890123456789012345678 you were confused at this point in the passage. Admitting you don’t 9 123456789012345678901234567890121234567890123456789012345678 23456789012345678901234567890121234567890123456789012345678 9 123456789012345678901234567890121234567890123456789012345678 know will never be the answer on the SAT. The answer is (B). 9 123456789012345678901234567890121234567890123456789012345678 123456789012345678901234567890121234567890123456789012345678 99 123456789012345678901234567890121234567890123456789012345678 9 123456789012345678901234567890121234567890123456789012345678 9 123456789012345678901234567890121234567890123456789012345678 9 1 Section 5 9 123456789012345678901234567890121234567890123456789012345678 1. A The wrinkle in this word problem is correctly translating what a 9 123456789012345678901234567890121234567890123456789012345678 9 123456789012345678901234567890121234567890123456789012345678 “quarter” of the factory’s capacity means. A quarter is one-fourth of 23456789012345678901234567890121234567890123456789012345678 123456789012345678901234567890121234567890123456789012345678 99 123456789012345678901234567890121234567890123456789012345678 something, so a quarter of full capacity—200 sheets of paper per 9 123456789012345678901234567890121234567890123456789012345678 9 123456789012345678901234567890121234567890123456789012345678 second—is one-fourth of 200, or 50. If a quarter-capacity is 50 sheets 123456789012345678901234567890121234567890123456789012345678 99 123456789012345678901234567890121234567890123456789012345678 per second, in 12 seconds the factory would produce 600 sheets since 9 123456789012345678901234567890121234567890123456789012345678 9 123456789012345678901234567890121234567890123456789012345678 50 3 12 5 600. Choice (A) is what’s going on. 123456789012345678901234567890121234567890123456789012345678 99 1 9 123456789012345678901234567890121234567890123456789012345678 2. E This is a straightforward variable/equation substitution problem. If 9 123456789012345678901234567890121234567890123456789012345678 2x 9 123456789012345678901234567890121234567890123456789012345678 9 123456789012345678901234567890121234567890123456789012345678 Z 5 3 and Z 5 , then you substitute the value of 3 for Z in the 9 123456789012345678901234567890121234567890123456789012345678 5 23456789012345678901234567890121234567890123456789012345678 9 123456789012345678901234567890121234567890123456789012345678 second equation to get: 9 123456789012345678901234567890121234567890123456789012345678 9 1 9 123456789012345678901234567890121234567890123456789012345678 2x 9 123456789012345678901234567890121234567890123456789012345678 Z 5 23456789012345678901234567890121234567890123456789012345678 9 123456789012345678901234567890121234567890123456789012345678 5 9 123456789012345678901234567890121234567890123456789012345678 9 1 9 123456789012345678901234567890121234567890123456789012345678 2x 9 123456789012345678901234567890121234567890123456789012345678 35 23456789012345678901234567890121234567890123456789012345678 9 123456789012345678901234567890121234567890123456789012345678 5 9 123456789012345678901234567890121234567890123456789012345678 9 123456789012345678901234567890121234567890123456789012345678 2x 9 123456789012345678901234567890121234567890123456789012345678 5 3 5 5 ~ ! ~ ! 9 1 5 23456789012345678901234567890121234567890123456789012345678 123456789012345678901234567890121234567890123456789012345678 99 123456789012345678901234567890121234567890123456789012345678 15 5 2x 9 123456789012345678901234567890121234567890123456789012345678 123456789012345678901234567890121234567890123456789012345678 99 1 15 2x 9 123456789012345678901234567890121234567890123456789012345678 5 9 123456789012345678901234567890121234567890123456789012345678 2 2 9 123456789012345678901234567890121234567890123456789012345678 9 123456789012345678901234567890121234567890123456789012345678 15 9 123456789012345678901234567890121234567890123456789012345678 5 x 23456789012345678901234567890121234567890123456789012345678 9 123456789012345678901234567890121234567890123456789012345678 2 9 1 9 123456789012345678901234567890121234567890123456789012345678 9 123456789012345678901234567890121234567890123456789012345678 (E) is the answer. 9 123456789012345678901234567890121234567890123456789012345678 23456789012345678901234567890121234567890123456789012345678 9 1 123456789012345678901234567890121234567890123456789012345678 9 123456789012345678901234567890121234567890123456789012345678 9 123456789012345678901234567890121234567890123456789012345678 9 1234567890123456789012345678901212345678901234567890123456789 Copyright © 2005 Thomson Peterson’s, a part of The Thomson Corporaton SAT is a registered trademark of the College Entrance Examination Board, which was not involved in the production of and does not endorse this product.

67

Note

1234567890123456789012345678901212345678901234567890123456789 123456789012345678901234567890121234567890123456789012345678 9 9 123456789012345678901234567890121234567890123456789012345678 123456789012345678901234567890121234567890123456789012345678 9 9 123456789012345678901234567890121234567890123456789012345678 3. A There are two prime ways to approach this problem. You can do a 9 123456789012345678901234567890121234567890123456789012345678 9 123456789012345678901234567890121234567890123456789012345678 factor tree for both numbers, find the common factors, and then look 9 123456789012345678901234567890121234567890123456789012345678 23456789012345678901234567890121234567890123456789012345678 9 1 down the answer list for the greatest one. If you don’t know what a 9 123456789012345678901234567890121234567890123456789012345678 9 123456789012345678901234567890121234567890123456789012345678 “factor tree” is, you could go through the answer choices one by one to 9 123456789012345678901234567890121234567890123456789012345678 9 123456789012345678901234567890121234567890123456789012345678 see which is the greatest that divides 32 and 42. Start with choice (E), 23456789012345678901234567890121234567890123456789012345678 9 1 9 123456789012345678901234567890121234567890123456789012345678 since it’s the greatest number, and if it works, it’s the answer. 12 doesn’t 9 123456789012345678901234567890121234567890123456789012345678 9 123456789012345678901234567890121234567890123456789012345678 work, so try the next greatest one, 8, choice (D). Eight doesn’t work, 9 123456789012345678901234567890121234567890123456789012345678 23456789012345678901234567890121234567890123456789012345678 9 1 and neither does 6 or 3. This leaves (A). 9 123456789012345678901234567890121234567890123456789012345678 9 123456789012345678901234567890121234567890123456789012345678 9 4. B If you understand the idea of slope, the answer will fall into your lap. If 123456789012345678901234567890121234567890123456789012345678 9 123456789012345678901234567890121234567890123456789012345678 you have trouble with this problem, review pages 350–352 in the Math 23456789012345678901234567890121234567890123456789012345678 9 1 9 123456789012345678901234567890121234567890123456789012345678 section to make sure you are perfectly comfortable with slope and 9 123456789012345678901234567890121234567890123456789012345678 9 123456789012345678901234567890121234567890123456789012345678 linear equations in the form y 5 mx 1b. 9 123456789012345678901234567890121234567890123456789012345678 23456789012345678901234567890121234567890123456789012345678 123456789012345678901234567890121234567890123456789012345678 9 9 123456789012345678901234567890121234567890123456789012345678 Viewed from left to right, the line descends, which means the slope is 9 123456789012345678901234567890121234567890123456789012345678 9 123456789012345678901234567890121234567890123456789012345678 negative. 123456789012345678901234567890121234567890123456789012345678 9 9 123456789012345678901234567890121234567890123456789012345678 That eliminates choices (C), (D), and (E). A slope is “rise over run,” 9 1 9 123456789012345678901234567890121234567890123456789012345678 meaning you look at the change in y-values as the numerator of the 9 123456789012345678901234567890121234567890123456789012345678 9 123456789012345678901234567890121234567890123456789012345678 fraction, while the change in x-values is the denominator. The line 23456789012345678901234567890121234567890123456789012345678 123456789012345678901234567890121234567890123456789012345678 9 9 123456789012345678901234567890121234567890123456789012345678 drops two points along the y-axis for every three points it moves over 9 123456789012345678901234567890121234567890123456789012345678 9 123456789012345678901234567890121234567890123456789012345678 2 9 123456789012345678901234567890121234567890123456789012345678 , choice (B). on the x-axis, so the slope is 2 9 123456789012345678901234567890121234567890123456789012345678 3 9 123456789012345678901234567890121234567890123456789012345678 9 123456789012345678901234567890121234567890123456789012345678 5. E You need to know the total number of students in each grade to answer 9 123456789012345678901234567890121234567890123456789012345678 th 9 1 grade, so you this question. The table tells you there are 32 in the 4 23456789012345678901234567890121234567890123456789012345678 9 123456789012345678901234567890121234567890123456789012345678 rd 9 123456789012345678901234567890121234567890123456789012345678 grade. There are 16 boys and only need to determine the total in the 3 9 1 rd 9 123456789012345678901234567890121234567890123456789012345678 grade, which makes 30 total. The overall total is 32 14 girls in the 3 9 123456789012345678901234567890121234567890123456789012345678 23456789012345678901234567890121234567890123456789012345678 9 123456789012345678901234567890121234567890123456789012345678 plus 30, which is 62, choice (E). 9 123456789012345678901234567890121234567890123456789012345678 9 1 9 123456789012345678901234567890121234567890123456789012345678 9 123456789012345678901234567890121234567890123456789012345678 23456789012345678901234567890121234567890123456789012345678 9 123456789012345678901234567890121234567890123456789012345678 The table is incomplete, but don’t let this worry you. The SAT will 9 123456789012345678901234567890121234567890123456789012345678 always provide the information needed to answer the question, one 9 1 9 123456789012345678901234567890121234567890123456789012345678 way or another. 123456789012345678901234567890121234567890123456789012345678 9 23456789012345678901234567890121234567890123456789012345678 123456789012345678901234567890121234567890123456789012345678 9 123456789012345678901234567890121234567890123456789012345678 9 9 123456789012345678901234567890121234567890123456789012345678 th th 9 123456789012345678901234567890121234567890123456789012345678 grade, then there are 14 boys in 4 grade 6. D If there are 18 girls in the 4 9 1 rd 23456789012345678901234567890121234567890123456789012345678 9 123456789012345678901234567890121234567890123456789012345678 (32 2 18 5 14). The table says there are 16 boys in 3 grade, so there 9 123456789012345678901234567890121234567890123456789012345678 rd th 9 123456789012345678901234567890121234567890123456789012345678 are a total of 30 (16 1 14 5 30) in the 3 and 4 grade. Choice (D) is 123456789012345678901234567890121234567890123456789012345678 9 9 1 the answer. 9 123456789012345678901234567890121234567890123456789012345678 9 123456789012345678901234567890121234567890123456789012345678 9 123456789012345678901234567890121234567890123456789012345678 9 123456789012345678901234567890121234567890123456789012345678 123456789012345678901234567890121234567890123456789012345678 9 23456789012345678901234567890121234567890123456789012345678 123456789012345678901234567890121234567890123456789012345678 9 9 1 9 123456789012345678901234567890121234567890123456789012345678 123456789012345678901234567890121234567890123456789012345678 9 9 123456789012345678901234567890121234567890123456789012345678 123456789012345678901234567890121234567890123456789012345678 9 123456789012345678901234567890121234567890123456789012345678 9 123456789012345678901234567890121234567890123456789012345678 9 123456789012345678901234567890121234567890123456789012345678 9 1234567890123456789012345678901212345678901234567890123456789 68

Copyright © 2005 Thomson Peterson’s, a part of The Thomson Corporaton SAT is a registered trademark of the College Entrance Examination Board, which was not involved in the production of and does not endorse this product.

1234567890123456789012345678901212345678901234567890123456789 123456789012345678901234567890121234567890123456789012345678 9 9 123456789012345678901234567890121234567890123456789012345678 123456789012345678901234567890121234567890123456789012345678 9 9 123456789012345678901234567890121234567890123456789012345678 7. A Careful here. The problem asks for the value of the exponent, not the 9 123456789012345678901234567890121234567890123456789012345678 9 123456789012345678901234567890121234567890123456789012345678 simplified expression. Simplifying this expression looks a little wiggy, 9 123456789012345678901234567890121234567890123456789012345678 23456789012345678901234567890121234567890123456789012345678 9 1 but you need to be prepared to work with wacky-looking exponents 9 123456789012345678901234567890121234567890123456789012345678 9 123456789012345678901234567890121234567890123456789012345678 problems like this: 9 123456789012345678901234567890121234567890123456789012345678 9 123456789012345678901234567890121234567890123456789012345678 3 22 23456789012345678901234567890121234567890123456789012345678 9 1 9 123456789012345678901234567890121234567890123456789012345678 11 4! ~ m 1 1 1 1 24 9 123456789012345678901234567890121234567890123456789012345678 5 5 5 5 5 m 5 5 3 2 5 3 3 51313 11 9 123456789012345678901234567890121234567890123456789012345678 9 123456789012345678901234567890121234567890123456789012345678 4 4~m4! 4m4m4 4 4 m m m m m 23456789012345678901234567890121234567890123456789012345678 9 1 9 123456789012345678901234567890121234567890123456789012345678 11 9 123456789012345678901234567890121234567890123456789012345678 9 123456789012345678901234567890121234567890123456789012345678 The exponent is 2 , which is choice (A). If this makes no sense to you, 9 123456789012345678901234567890121234567890123456789012345678 4 23456789012345678901234567890121234567890123456789012345678 9 1 you’ll want to review the rules of multiplying and dividing exponents. 9 123456789012345678901234567890121234567890123456789012345678 9 123456789012345678901234567890121234567890123456789012345678 9 123456789012345678901234567890121234567890123456789012345678 8. C You can’t solve this problem without a little help from the algebraic 9 123456789012345678901234567890121234567890123456789012345678 23456789012345678901234567890121234567890123456789012345678 9 123456789012345678901234567890121234567890123456789012345678 expressions. You know that the two expressions sum to 90 (because the 9 123456789012345678901234567890121234567890123456789012345678 9 123456789012345678901234567890121234567890123456789012345678 measures of the interior angles of a triangle sum to 180), but there are 123456789012345678901234567890121234567890123456789012345678 99 123456789012345678901234567890121234567890123456789012345678 two variables and only one equation: 2x 1 y 1 3x 2 y 5 90 (They 9 123456789012345678901234567890121234567890123456789012345678 9 1 equal 90 since the right angle takes up the other 90°.) 9 123456789012345678901234567890121234567890123456789012345678 9 123456789012345678901234567890121234567890123456789012345678 If the y variables didn’t subtract out when you added the two 9 123456789012345678901234567890121234567890123456789012345678 23456789012345678901234567890121234567890123456789012345678 9 123456789012345678901234567890121234567890123456789012345678 expressions, you could not solve for x. The whole point to this problem 9 123456789012345678901234567890121234567890123456789012345678 9 123456789012345678901234567890121234567890123456789012345678 is to brain freeze students who look at it and say, “There are two 123456789012345678901234567890121234567890123456789012345678 99 123456789012345678901234567890121234567890123456789012345678 variables and only one equation. It can’t be solved!” If you just write 9 123456789012345678901234567890121234567890123456789012345678 9 123456789012345678901234567890121234567890123456789012345678 out the equation, you can avoid this type of paralysis. Writing down 123456789012345678901234567890121234567890123456789012345678 99 123456789012345678901234567890121234567890123456789012345678 your work saves the day! 9 1 9 123456789012345678901234567890121234567890123456789012345678 9 123456789012345678901234567890121234567890123456789012345678 2x 1 y 1 3x 2 y 5 90 9 123456789012345678901234567890121234567890123456789012345678 123456789012345678901234567890121234567890123456789012345678 9 9 123456789012345678901234567890121234567890123456789012345678 5x 5 90 9 123456789012345678901234567890121234567890123456789012345678 9 123456789012345678901234567890121234567890123456789012345678 90 5x 9 123456789012345678901234567890121234567890123456789012345678 5 9 123456789012345678901234567890121234567890123456789012345678 5 5 9 123456789012345678901234567890121234567890123456789012345678 23456789012345678901234567890121234567890123456789012345678 9 123456789012345678901234567890121234567890123456789012345678 x 5 18 9 123456789012345678901234567890121234567890123456789012345678 9 1 9 123456789012345678901234567890121234567890123456789012345678 (C) is the answer. 123456789012345678901234567890121234567890123456789012345678 9 23456789012345678901234567890121234567890123456789012345678 123456789012345678901234567890121234567890123456789012345678 99 123456789012345678901234567890121234567890123456789012345678 9 123456789012345678901234567890121234567890123456789012345678 9 123456789012345678901234567890121234567890123456789012345678 9 1 9 123456789012345678901234567890121234567890123456789012345678 9 123456789012345678901234567890121234567890123456789012345678 9 123456789012345678901234567890121234567890123456789012345678 9 123456789012345678901234567890121234567890123456789012345678 123456789012345678901234567890121234567890123456789012345678 9 23456789012345678901234567890121234567890123456789012345678 123456789012345678901234567890121234567890123456789012345678 99 123456789012345678901234567890121234567890123456789012345678 9 123456789012345678901234567890121234567890123456789012345678 9 123456789012345678901234567890121234567890123456789012345678 9 1 9 123456789012345678901234567890121234567890123456789012345678 123456789012345678901234567890121234567890123456789012345678 9 9 123456789012345678901234567890121234567890123456789012345678 123456789012345678901234567890121234567890123456789012345678 9 9 123456789012345678901234567890121234567890123456789012345678 123456789012345678901234567890121234567890123456789012345678 9 123456789012345678901234567890121234567890123456789012345678 9 123456789012345678901234567890121234567890123456789012345678 9 123456789012345678901234567890121234567890123456789012345678 9 1234567890123456789012345678901212345678901234567890123456789 Copyright © 2005 Thomson Peterson’s, a part of The Thomson Corporaton SAT is a registered trademark of the College Entrance Examination Board, which was not involved in the production of and does not endorse this product.

69

1234567890123456789012345678901212345678901234567890123456789 123456789012345678901234567890121234567890123456789012345678 9 9 123456789012345678901234567890121234567890123456789012345678 123456789012345678901234567890121234567890123456789012345678 9 9 123456789012345678901234567890121234567890123456789012345678 9. E For word problems, always translate the English into algebra. Call the 9 123456789012345678901234567890121234567890123456789012345678 9 123456789012345678901234567890121234567890123456789012345678 distance from Easton to Bethsaida via highway x. Traveling this route is 9 123456789012345678901234567890121234567890123456789012345678 23456789012345678901234567890121234567890123456789012345678 9 1 7 miles longer than going on surface streets, so going via surface streets 9 123456789012345678901234567890121234567890123456789012345678 9 123456789012345678901234567890121234567890123456789012345678 is x 2 7. Traveling both routes is 31 miles, which translates to: 9 123456789012345678901234567890121234567890123456789012345678 9 123456789012345678901234567890121234567890123456789012345678 x 1 (x 2 7) 5 31. Once you have an equation, you can solve for x: 23456789012345678901234567890121234567890123456789012345678 9 1 9 123456789012345678901234567890121234567890123456789012345678 9 123456789012345678901234567890121234567890123456789012345678 x 1 x 2 7 5 31 9 123456789012345678901234567890121234567890123456789012345678 9 123456789012345678901234567890121234567890123456789012345678 2x 2 7 5 31 23456789012345678901234567890121234567890123456789012345678 9 1 9 123456789012345678901234567890121234567890123456789012345678 9 123456789012345678901234567890121234567890123456789012345678 2x 2 7 1 7 5 31 1 7 9 123456789012345678901234567890121234567890123456789012345678 9 123456789012345678901234567890121234567890123456789012345678 2x 5 38 23456789012345678901234567890121234567890123456789012345678 9 1 9 123456789012345678901234567890121234567890123456789012345678 2x 38 9 123456789012345678901234567890121234567890123456789012345678 5 9 123456789012345678901234567890121234567890123456789012345678 2 2 9 123456789012345678901234567890121234567890123456789012345678 23456789012345678901234567890121234567890123456789012345678 123456789012345678901234567890121234567890123456789012345678 9 9 123456789012345678901234567890121234567890123456789012345678 x59 123456789012345678901234567890121234567890123456789012345678 9 9 123456789012345678901234567890121234567890123456789012345678 Here you might to double check that x is the highway route distance 9 123456789012345678901234567890121234567890123456789012345678 9 123456789012345678901234567890121234567890123456789012345678 and not the surface street distance. Choice (E) is the answer. 9 1 9 123456789012345678901234567890121234567890123456789012345678 9 123456789012345678901234567890121234567890123456789012345678 10. E A function is undefined at a certain value, if at that value the function 9 123456789012345678901234567890121234567890123456789012345678 23456789012345678901234567890121234567890123456789012345678 9 123456789012345678901234567890121234567890123456789012345678 does not make any sense. For this function, the most probable way that 9 123456789012345678901234567890121234567890123456789012345678 9 123456789012345678901234567890121234567890123456789012345678 it could be undefined is if the denominator is zero. 123456789012345678901234567890121234567890123456789012345678 9 123456789012345678901234567890121234567890123456789012345678 9 9 123456789012345678901234567890121234567890123456789012345678 When is the denominator zero? Solve it like a quadratic to see: 9 123456789012345678901234567890121234567890123456789012345678 9 123456789012345678901234567890121234567890123456789012345678 2 9 123456789012345678901234567890121234567890123456789012345678 1 3x 2 18 5 0 x 9 1 23456789012345678901234567890121234567890123456789012345678 9 123456789012345678901234567890121234567890123456789012345678 ~x 1 6!~x 2 3! 5 0 9 123456789012345678901234567890121234567890123456789012345678 9 1 9 123456789012345678901234567890121234567890123456789012345678 The denominator is zero if x equals 26 or 3, which is choice (E). 9 123456789012345678901234567890121234567890123456789012345678 23456789012345678901234567890121234567890123456789012345678 9 123456789012345678901234567890121234567890123456789012345678 2 9 123456789012345678901234567890121234567890123456789012345678 11. D The graph is a parabola, so it will have an x term in it. Sometimes that 9 1 9 123456789012345678901234567890121234567890123456789012345678 will help you cross out an answer choice or two, but on this question all 9 123456789012345678901234567890121234567890123456789012345678 2 23456789012345678901234567890121234567890123456789012345678 9 123456789012345678901234567890121234567890123456789012345678 answer choices have an x in them. Even so, it was a good technique; it 9 123456789012345678901234567890121234567890123456789012345678 9 didn’t work on this problem, but it will work on others. 1 123456789012345678901234567890121234567890123456789012345678 9 9 123456789012345678901234567890121234567890123456789012345678 The parabola is facing downwards, so the x2 term must have a negative 9 123456789012345678901234567890121234567890123456789012345678 9 123456789012345678901234567890121234567890123456789012345678 2 in front of it. If you don’t see why, plug some numbers into 2x . That 9 123456789012345678901234567890121234567890123456789012345678 9 123456789012345678901234567890121234567890123456789012345678 eliminates (A) and (B). 123456789012345678901234567890121234567890123456789012345678 9 23456789012345678901234567890121234567890123456789012345678 123456789012345678901234567890121234567890123456789012345678 9 9 123456789012345678901234567890121234567890123456789012345678 Our last clue will come from where the figure crosses the y-axis. At this 9 123456789012345678901234567890121234567890123456789012345678 2 9 123456789012345678901234567890121234567890123456789012345678 will also be zero. When point, x must equal zero, which means that 2x 9 1 23456789012345678901234567890121234567890123456789012345678 9 123456789012345678901234567890121234567890123456789012345678 x equals zero on the graph, the value of f(x is 22. So you are looking 9 123456789012345678901234567890121234567890123456789012345678 2 9 123456789012345678901234567890121234567890123456789012345678 and 22. Although it’s hiding a for an answer choice that has both 2x 9 123456789012345678901234567890121234567890123456789012345678 2 2 9 1 bit inside some parentheses, (D) is the answer since 2(x 1 2) 5 2x 2 2. 23456789012345678901234567890121234567890123456789012345678 123456789012345678901234567890121234567890123456789012345678 9 9 1 9 123456789012345678901234567890121234567890123456789012345678 123456789012345678901234567890121234567890123456789012345678 9 9 123456789012345678901234567890121234567890123456789012345678 123456789012345678901234567890121234567890123456789012345678 9 123456789012345678901234567890121234567890123456789012345678 9 123456789012345678901234567890121234567890123456789012345678 9 123456789012345678901234567890121234567890123456789012345678 9 1234567890123456789012345678901212345678901234567890123456789 70

Copyright © 2005 Thomson Peterson’s, a part of The Thomson Corporaton SAT is a registered trademark of the College Entrance Examination Board, which was not involved in the production of and does not endorse this product.

1234567890123456789012345678901212345678901234567890123456789 123456789012345678901234567890121234567890123456789012345678 9 9 123456789012345678901234567890121234567890123456789012345678 123456789012345678901234567890121234567890123456789012345678 9 9 123456789012345678901234567890121234567890123456789012345678 12. D With congruent triangles, corresponding angles and sides are congru9 123456789012345678901234567890121234567890123456789012345678 9 123456789012345678901234567890121234567890123456789012345678 ent. Both are right triangles, so you should smell the Pythagorean 9 123456789012345678901234567890121234567890123456789012345678 23456789012345678901234567890121234567890123456789012345678 9 1 theorem wafting about this problem as a method of solving for y. 9 123456789012345678901234567890121234567890123456789012345678 9 123456789012345678901234567890121234567890123456789012345678 2 2 2 9 123456789012345678901234567890121234567890123456789012345678 1 b 5 c a 9 123456789012345678901234567890121234567890123456789012345678 23456789012345678901234567890121234567890123456789012345678 9 1 y2 1 52 5 102 9 123456789012345678901234567890121234567890123456789012345678 9 123456789012345678901234567890121234567890123456789012345678 2 9 123456789012345678901234567890121234567890123456789012345678 y 1 25 5 100 9 123456789012345678901234567890121234567890123456789012345678 2 23456789012345678901234567890121234567890123456789012345678 9 1 y 1 25 2 25 5 100 2 25 9 123456789012345678901234567890121234567890123456789012345678 9 123456789012345678901234567890121234567890123456789012345678 2 y 5 75 9 123456789012345678901234567890121234567890123456789012345678 9 123456789012345678901234567890121234567890123456789012345678 23456789012345678901234567890121234567890123456789012345678 9 1 y 5 =75 9 123456789012345678901234567890121234567890123456789012345678 9 123456789012345678901234567890121234567890123456789012345678 9 123456789012345678901234567890121234567890123456789012345678 You might also have noticed that both figures are 30-60-90 triangles 9 123456789012345678901234567890121234567890123456789012345678 23456789012345678901234567890121234567890123456789012345678 9 123456789012345678901234567890121234567890123456789012345678 (you can infer this from the relationship between the hypotenuse and 9 123456789012345678901234567890121234567890123456789012345678 9 123456789012345678901234567890121234567890123456789012345678 the smallest side of the triangle on the left). This means y 5 5 3 = 123456789012345678901234567890121234567890123456789012345678 99 123456789012345678901234567890121234567890123456789012345678 which brings you once again to =75. Either method, choice (D) is 9 123456789012345678901234567890121234567890123456789012345678 9 1 correct. 9 123456789012345678901234567890121234567890123456789012345678 9 123456789012345678901234567890121234567890123456789012345678 9 123456789012345678901234567890121234567890123456789012345678 13. A Plug in the answer choices and see which ones make the equation true. 23456789012345678901234567890121234567890123456789012345678 123456789012345678901234567890121234567890123456789012345678 99 123456789012345678901234567890121234567890123456789012345678 Many of the numbers repeat and the equation isn’t that involved, so 9 123456789012345678901234567890121234567890123456789012345678 9 123456789012345678901234567890121234567890123456789012345678 this doesn’t take as long as you might think. 1 works, but 21 doesn’t. If 123456789012345678901234567890121234567890123456789012345678 99 123456789012345678901234567890121234567890123456789012345678 you try all the answer choices, you will see that only 1 works, which 9 123456789012345678901234567890121234567890123456789012345678 9 123456789012345678901234567890121234567890123456789012345678 makes (A) the answer. 123456789012345678901234567890121234567890123456789012345678 99 1 9 123456789012345678901234567890121234567890123456789012345678 14. B There are multiple ways to approach this problem. Possibly the 9 123456789012345678901234567890121234567890123456789012345678 9 123456789012345678901234567890121234567890123456789012345678 quickest is to count up the multiples of three that are even and 50 or less 123456789012345678901234567890121234567890123456789012345678 9 9 123456789012345678901234567890121234567890123456789012345678 (if they are even, they will be divisible by 2). This list starts 6, 12, 18, 24 9 123456789012345678901234567890121234567890123456789012345678 9 123456789012345678901234567890121234567890123456789012345678 ... 123456789012345678901234567890121234567890123456789012345678 9 9 123456789012345678901234567890121234567890123456789012345678 You might stop here and realize that the members of set Z are all 9 123456789012345678901234567890121234567890123456789012345678 23456789012345678901234567890121234567890123456789012345678 9 123456789012345678901234567890121234567890123456789012345678 multiples of 6. Even if you don’t, you’ll continue with: 30, 36, 42, and 9 123456789012345678901234567890121234567890123456789012345678 9 1 48. 54 is the next item, but it’s too great, so there are eight members of 123456789012345678901234567890121234567890123456789012345678 9 9 123456789012345678901234567890121234567890123456789012345678 set Z. (B) is the answer. 9 123456789012345678901234567890121234567890123456789012345678 9 123456789012345678901234567890121234567890123456789012345678 9 123456789012345678901234567890121234567890123456789012345678 15. D With similar figures, the ratios of each pair of corresponding sides must 9 123456789012345678901234567890121234567890123456789012345678 9 123456789012345678901234567890121234567890123456789012345678 be equal. The tricky part to this problem is the bent-leg formation of the 23456789012345678901234567890121234567890123456789012345678 123456789012345678901234567890121234567890123456789012345678 99 123456789012345678901234567890121234567890123456789012345678 given figure. The L-shape could roughly be described as, “three squares 9 123456789012345678901234567890121234567890123456789012345678 9 123456789012345678901234567890121234567890123456789012345678 up, then one to the right.” Choice (A) is too thin, i.e. there was no 9 1 23456789012345678901234567890121234567890123456789012345678 9 123456789012345678901234567890121234567890123456789012345678 corresponding increase in width even though there is an increase in 9 123456789012345678901234567890121234567890123456789012345678 9 123456789012345678901234567890121234567890123456789012345678 length. With (B), the small part of L seems to be too big, and with (C) 123456789012345678901234567890121234567890123456789012345678 99 1 the big part of the L is too large in proportion to the skinny part. 9 123456789012345678901234567890121234567890123456789012345678 Now look at choice (D). The fact that the figure has been rotated makes 9 123456789012345678901234567890121234567890123456789012345678 9 123456789012345678901234567890121234567890123456789012345678 no difference with similarity. If you draw the following dashed lines on 9 123456789012345678901234567890121234567890123456789012345678 9 123456789012345678901234567890121234567890123456789012345678 choice (D), you’ll see why it’s the right answer. 23456789012345678901234567890121234567890123456789012345678 9 1 123456789012345678901234567890121234567890123456789012345678 9 123456789012345678901234567890121234567890123456789012345678 9 123456789012345678901234567890121234567890123456789012345678 9 1234567890123456789012345678901212345678901234567890123456789 Copyright © 2005 Thomson Peterson’s, a part of The Thomson Corporaton SAT is a registered trademark of the College Entrance Examination Board, which was not involved in the production of and does not endorse this product.

71

1234567890123456789012345678901212345678901234567890123456789 123456789012345678901234567890121234567890123456789012345678 9 9 123456789012345678901234567890121234567890123456789012345678 123456789012345678901234567890121234567890123456789012345678 9 9 123456789012345678901234567890121234567890123456789012345678 9 123456789012345678901234567890121234567890123456789012345678 123456789012345678901234567890121234567890123456789012345678 9 9 123456789012345678901234567890121234567890123456789012345678 123456789012345678901234567890121234567890123456789012345678 9 9 123456789012345678901234567890121234567890123456789012345678 9 123456789012345678901234567890121234567890123456789012345678 9 123456789012345678901234567890121234567890123456789012345678 9 123456789012345678901234567890121234567890123456789012345678 23456789012345678901234567890121234567890123456789012345678 9 1 9 123456789012345678901234567890121234567890123456789012345678 9 123456789012345678901234567890121234567890123456789012345678 9 123456789012345678901234567890121234567890123456789012345678 9 123456789012345678901234567890121234567890123456789012345678 23456789012345678901234567890121234567890123456789012345678 9 1 9 123456789012345678901234567890121234567890123456789012345678 9 123456789012345678901234567890121234567890123456789012345678 9 123456789012345678901234567890121234567890123456789012345678 9 123456789012345678901234567890121234567890123456789012345678 23456789012345678901234567890121234567890123456789012345678 9 1 9 123456789012345678901234567890121234567890123456789012345678 Here you have the same figure, “three squares, then one to the right.” 9 123456789012345678901234567890121234567890123456789012345678 9 123456789012345678901234567890121234567890123456789012345678 The squares are much larger, but they are in the same corresponding 9 123456789012345678901234567890121234567890123456789012345678 23456789012345678901234567890121234567890123456789012345678 9 123456789012345678901234567890121234567890123456789012345678 proportion. Choice (D) is correct. 9 123456789012345678901234567890121234567890123456789012345678 123456789012345678901234567890121234567890123456789012345678 9 9 123456789012345678901234567890121234567890123456789012345678 16. C The figure is a square since the side lengths are all equal and all the 9 123456789012345678901234567890121234567890123456789012345678 9 123456789012345678901234567890121234567890123456789012345678 angles are ninety degrees (if you don’t see this, rotate the figure ninety 9 1 9 123456789012345678901234567890121234567890123456789012345678 degrees). To find the side length, it’s Pythagoras time, but you get a 9 123456789012345678901234567890121234567890123456789012345678 9 123456789012345678901234567890121234567890123456789012345678 shortcut since you have a special triangle. Each side length is the 23456789012345678901234567890121234567890123456789012345678 123456789012345678901234567890121234567890123456789012345678 9 9 123456789012345678901234567890121234567890123456789012345678 hypotenuse of a 45-45-90 triangle whose equal sides are of length one. 9 123456789012345678901234567890121234567890123456789012345678 9 123456789012345678901234567890121234567890123456789012345678 2. Put this into the So the hypotenuse/side length of the square is = 123456789012345678901234567890121234567890123456789012345678 9 9 123456789012345678901234567890121234567890123456789012345678 area of a square formula and you’ll find answer (C): 9 123456789012345678901234567890121234567890123456789012345678 9 123456789012345678901234567890121234567890123456789012345678 2 A5s 9 123456789012345678901234567890121234567890123456789012345678 9 1 2 23456789012345678901234567890121234567890123456789012345678 9 123456789012345678901234567890121234567890123456789012345678 A 5 ~=2! 9 123456789012345678901234567890121234567890123456789012345678 9 1 A52 9 123456789012345678901234567890121234567890123456789012345678 9 123456789012345678901234567890121234567890123456789012345678 23456789012345678901234567890121234567890123456789012345678 9 123456789012345678901234567890121234567890123456789012345678 17. B First, make sure you understand what all those words mean. You have 9 123456789012345678901234567890121234567890123456789012345678 9 1 eight numbers that are integers, and are also even, and when you add 9 123456789012345678901234567890121234567890123456789012345678 9 123456789012345678901234567890121234567890123456789012345678 them up, they sum to 50. Furthermore, only two of the eight integers 23456789012345678901234567890121234567890123456789012345678 123456789012345678901234567890121234567890123456789012345678 9 9 123456789012345678901234567890121234567890123456789012345678 can be the same value. In other words, there are never three integers 9 1 9 123456789012345678901234567890121234567890123456789012345678 that are the same. 123456789012345678901234567890121234567890123456789012345678 9 23456789012345678901234567890121234567890123456789012345678 123456789012345678901234567890121234567890123456789012345678 9 9 123456789012345678901234567890121234567890123456789012345678 To find the greatest possible integer in the set, first make all the other 9 123456789012345678901234567890121234567890123456789012345678 9 123456789012345678901234567890121234567890123456789012345678 integers as least as possible. The set could be 2, 2, 4, 4, 6, 6, 8, x. This 9 1 23456789012345678901234567890121234567890123456789012345678 9 123456789012345678901234567890121234567890123456789012345678 would maximize x. Now solve: 123456789012345678901234567890121234567890123456789012345678 9 9 123456789012345678901234567890121234567890123456789012345678 2 1 2 1 4 1 4 1 6 1 6 1 8 1 x 5 50 9 123456789012345678901234567890121234567890123456789012345678 9 1 32 1 x 5 50 23456789012345678901234567890121234567890123456789012345678 123456789012345678901234567890121234567890123456789012345678 9 123456789012345678901234567890121234567890123456789012345678 9 9 123456789012345678901234567890121234567890123456789012345678 32 2 32 1 x 5 50 2 32 9 123456789012345678901234567890121234567890123456789012345678 x 5 18 9 1 23456789012345678901234567890121234567890123456789012345678 9 123456789012345678901234567890121234567890123456789012345678 This is choice (B). 9 1 9 123456789012345678901234567890121234567890123456789012345678 You can also start with the greatet numerical answer choice and then 9 123456789012345678901234567890121234567890123456789012345678 9 123456789012345678901234567890121234567890123456789012345678 see if it works, but this will take more time than setting up a formula. 23456789012345678901234567890121234567890123456789012345678 9 1 123456789012345678901234567890121234567890123456789012345678 9 123456789012345678901234567890121234567890123456789012345678 9 123456789012345678901234567890121234567890123456789012345678 9 1234567890123456789012345678901212345678901234567890123456789 72

Copyright © 2005 Thomson Peterson’s, a part of The Thomson Corporaton SAT is a registered trademark of the College Entrance Examination Board, which was not involved in the production of and does not endorse this product.

1234567890123456789012345678901212345678901234567890123456789 123456789012345678901234567890121234567890123456789012345678 9 9 123456789012345678901234567890121234567890123456789012345678 123456789012345678901234567890121234567890123456789012345678 9 9 123456789012345678901234567890121234567890123456789012345678 18. E Time to sketch! Here’s one where two of the triangle’s vertices coincide 9 123456789012345678901234567890121234567890123456789012345678 9 123456789012345678901234567890121234567890123456789012345678 with two of the square’s corners. 9 123456789012345678901234567890121234567890123456789012345678 23456789012345678901234567890121234567890123456789012345678 9 1 9 123456789012345678901234567890121234567890123456789012345678 9 123456789012345678901234567890121234567890123456789012345678 9 123456789012345678901234567890121234567890123456789012345678 9 123456789012345678901234567890121234567890123456789012345678 23456789012345678901234567890121234567890123456789012345678 9 1 9 123456789012345678901234567890121234567890123456789012345678 9 123456789012345678901234567890121234567890123456789012345678 9 123456789012345678901234567890121234567890123456789012345678 9 123456789012345678901234567890121234567890123456789012345678 23456789012345678901234567890121234567890123456789012345678 9 1 9 123456789012345678901234567890121234567890123456789012345678 9 123456789012345678901234567890121234567890123456789012345678 9 123456789012345678901234567890121234567890123456789012345678 9 123456789012345678901234567890121234567890123456789012345678 23456789012345678901234567890121234567890123456789012345678 9 1 9 123456789012345678901234567890121234567890123456789012345678 9 123456789012345678901234567890121234567890123456789012345678 9 123456789012345678901234567890121234567890123456789012345678 9 123456789012345678901234567890121234567890123456789012345678 23456789012345678901234567890121234567890123456789012345678 123456789012345678901234567890121234567890123456789012345678 99 123456789012345678901234567890121234567890123456789012345678 9 123456789012345678901234567890121234567890123456789012345678 9 123456789012345678901234567890121234567890123456789012345678 You could also have AB run along DE. In fact, using AB you could put 9 123456789012345678901234567890121234567890123456789012345678 9 123456789012345678901234567890121234567890123456789012345678 the triangle in the square four different ways (along DF, DE, EG, GF). 9 1 9 123456789012345678901234567890121234567890123456789012345678 9 123456789012345678901234567890121234567890123456789012345678 Using BC, you could put the triangle in the square four different ways 9 123456789012345678901234567890121234567890123456789012345678 23456789012345678901234567890121234567890123456789012345678 9 123456789012345678901234567890121234567890123456789012345678 also. The same is true for AC. That is a total of 12 different ways that 9 123456789012345678901234567890121234567890123456789012345678 9 123456789012345678901234567890121234567890123456789012345678 the triangle could be placed in the square such that two angles of the 123456789012345678901234567890121234567890123456789012345678 99 123456789012345678901234567890121234567890123456789012345678 triangle coincided with two corners of the square. It means that (E) is 9 123456789012345678901234567890121234567890123456789012345678 9 the answer. 123456789012345678901234567890121234567890123456789012345678 123456789012345678901234567890121234567890123456789012345678 99 123456789012345678901234567890121234567890123456789012345678 1 19. D You only know the distances between these points. You don’t know 9 9 123456789012345678901234567890121234567890123456789012345678 their orientation in relation to each other. If T is on a line between 9 123456789012345678901234567890121234567890123456789012345678 9 123456789012345678901234567890121234567890123456789012345678 G and S, then G and T are 4 away from each other (9 2 5 5 4). That is 123456789012345678901234567890121234567890123456789012345678 9 9 123456789012345678901234567890121234567890123456789012345678 the closest that they could be to each other. But if T is on a line with 9 123456789012345678901234567890121234567890123456789012345678 9 123456789012345678901234567890121234567890123456789012345678 S and G, and S is between G and T, then they are 14 away from each 123456789012345678901234567890121234567890123456789012345678 9 9 123456789012345678901234567890121234567890123456789012345678 other (9 1 5 5 14). This is the farthest that they could be from each 9 123456789012345678901234567890121234567890123456789012345678 23456789012345678901234567890121234567890123456789012345678 9 123456789012345678901234567890121234567890123456789012345678 other. Therefore I is not possible, but II and III are. Choice (D) is 9 123456789012345678901234567890121234567890123456789012345678 9 1 the answer. 123456789012345678901234567890121234567890123456789012345678 9 123456789012345678901234567890121234567890123456789012345678 9 9 123456789012345678901234567890121234567890123456789012345678 20. C If you connect A, B, and C, you have an equilateral triangle with a side 9 123456789012345678901234567890121234567890123456789012345678 9 123456789012345678901234567890121234567890123456789012345678 BD cuts the triangle into two 30-60-90 triangles. This length of six. 9 123456789012345678901234567890121234567890123456789012345678 9 123456789012345678901234567890121234567890123456789012345678 AD 5 3—because of the relationship between the sides of a means that 23456789012345678901234567890121234567890123456789012345678 123456789012345678901234567890121234567890123456789012345678 99 123456789012345678901234567890121234567890123456789012345678 30-60-90 triangle—and so BD 5 3=3, choice (C). 9 123456789012345678901234567890121234567890123456789012345678 123456789012345678901234567890121234567890123456789012345678 99 1 21. E This problem has many steps, which is why it’s number 21. First, ask 9 123456789012345678901234567890121234567890123456789012345678 9 123456789012345678901234567890121234567890123456789012345678 yourself, “What point on the fan travels the greatest distance through 9 123456789012345678901234567890121234567890123456789012345678 9 123456789012345678901234567890121234567890123456789012345678 one revolution?” The answer is the point on the corner tip of the fan 123456789012345678901234567890121234567890123456789012345678 9 23456789012345678901234567890121234567890123456789012345678 9 123456789012345678901234567890121234567890123456789012345678 blade, as it is the farthest from the axis of rotation. In going through 9 1 9 123456789012345678901234567890121234567890123456789012345678 one revolution, this point describes a circle because it rotates about a 123456789012345678901234567890121234567890123456789012345678 9 9 123456789012345678901234567890121234567890123456789012345678 fixed point with a constant radius. 9 123456789012345678901234567890121234567890123456789012345678 123456789012345678901234567890121234567890123456789012345678 9 123456789012345678901234567890121234567890123456789012345678 9 123456789012345678901234567890121234567890123456789012345678 9 1234567890123456789012345678901212345678901234567890123456789 Copyright © 2005 Thomson Peterson’s, a part of The Thomson Corporaton SAT is a registered trademark of the College Entrance Examination Board, which was not involved in the production of and does not endorse this product.

73

1234567890123456789012345678901212345678901234567890123456789 123456789012345678901234567890121234567890123456789012345678 9 9 123456789012345678901234567890121234567890123456789012345678 123456789012345678901234567890121234567890123456789012345678 9 9 123456789012345678901234567890121234567890123456789012345678 To find out how far this point travels, you need to determine the radius 9 123456789012345678901234567890121234567890123456789012345678 9 123456789012345678901234567890121234567890123456789012345678 of this circle. This can be done with some help from the triangle shape 9 123456789012345678901234567890121234567890123456789012345678 23456789012345678901234567890121234567890123456789012345678 9 1 at the top of the fan. The dashed lines through the two sides show that 9 123456789012345678901234567890121234567890123456789012345678 9 123456789012345678901234567890121234567890123456789012345678 these sides are congruent. Combine this with the right angle, and you 9 123456789012345678901234567890121234567890123456789012345678 9 123456789012345678901234567890121234567890123456789012345678 have a 45-45-90 right triangle with two sides of length 4. The 23456789012345678901234567890121234567890123456789012345678 9 1 9 123456789012345678901234567890121234567890123456789012345678 hypotenuse of this triangle will be 4=2, and this length will also be 9 123456789012345678901234567890121234567890123456789012345678 9 123456789012345678901234567890121234567890123456789012345678 the diameter of the circle. 9 123456789012345678901234567890121234567890123456789012345678 23456789012345678901234567890121234567890123456789012345678 9 1 9 123456789012345678901234567890121234567890123456789012345678 Therefore, in one rotation that point travels the circumference of the 9 123456789012345678901234567890121234567890123456789012345678 9 123456789012345678901234567890121234567890123456789012345678 circle it describes: 9 123456789012345678901234567890121234567890123456789012345678 23456789012345678901234567890121234567890123456789012345678 9 1 9 123456789012345678901234567890121234567890123456789012345678 9 C 5 pd 5 4=2p 123456789012345678901234567890121234567890123456789012345678 9 123456789012345678901234567890121234567890123456789012345678 9 123456789012345678901234567890121234567890123456789012345678 But there’s more! In 30 seconds, the fan can do 300 revolutions. You 23456789012345678901234567890121234567890123456789012345678 123456789012345678901234567890121234567890123456789012345678 9 9 123456789012345678901234567890121234567890123456789012345678 know this because the problem states that maximum blade speed is 100 9 123456789012345678901234567890121234567890123456789012345678 9 123456789012345678901234567890121234567890123456789012345678 revolutions in 10 seconds, and you can multiply both of these numbers 123456789012345678901234567890121234567890123456789012345678 9 9 123456789012345678901234567890121234567890123456789012345678 by 3 to get 300 revolutions in 30 seconds. This means that the farthest 9 1 9 123456789012345678901234567890121234567890123456789012345678 the point could travel would be: 9 123456789012345678901234567890121234567890123456789012345678 9 123456789012345678901234567890121234567890123456789012345678 23456789012345678901234567890121234567890123456789012345678 123456789012345678901234567890121234567890123456789012345678 9 9 123456789012345678901234567890121234567890123456789012345678 300 3 4=2p 5 1200=2 9 123456789012345678901234567890121234567890123456789012345678 9 123456789012345678901234567890121234567890123456789012345678 Choice (E). 123456789012345678901234567890121234567890123456789012345678 9 123456789012345678901234567890121234567890123456789012345678 9 123456789012345678901234567890121234567890123456789012345678 9 123456789012345678901234567890121234567890123456789012345678 9 9 123456789012345678901234567890121234567890123456789012345678 9 1 Section 6 23456789012345678901234567890121234567890123456789012345678 9 123456789012345678901234567890121234567890123456789012345678 1. D The passage reads, “As he matured, however, Remington turned his 9 123456789012345678901234567890121234567890123456789012345678 9 1 attention away from illustration, concentrating instead on painting and 123456789012345678901234567890121234567890123456789012345678 9 9 123456789012345678901234567890121234567890123456789012345678 sculpture.” (lines 5–9) So the passage links Remington’s concentration 9 123456789012345678901234567890121234567890123456789012345678 9 123456789012345678901234567890121234567890123456789012345678 upon painting and sculpture as key for his maturation. Eliminate (A). 123456789012345678901234567890121234567890123456789012345678 9 9 123456789012345678901234567890121234567890123456789012345678 However, the rest of the passage focuses on his paintings, so the answer 9 123456789012345678901234567890121234567890123456789012345678 23456789012345678901234567890121234567890123456789012345678 9 123456789012345678901234567890121234567890123456789012345678 can’t just be sculpture. Eliminate (B). We know he paints nocturnal 9 123456789012345678901234567890121234567890123456789012345678 9 1 scenes, so eliminate (E) (this answer isn’t specific enough). He is a great 123456789012345678901234567890121234567890123456789012345678 9 9 123456789012345678901234567890121234567890123456789012345678 artist of the American West, painting before there were any big cities in 9 123456789012345678901234567890121234567890123456789012345678 9 123456789012345678901234567890121234567890123456789012345678 the West. The passage also implies that he paints natural scenes. The 9 123456789012345678901234567890121234567890123456789012345678 9 123456789012345678901234567890121234567890123456789012345678 best answer is (D). 123456789012345678901234567890121234567890123456789012345678 9 23456789012345678901234567890121234567890123456789012345678 9 123456789012345678901234567890121234567890123456789012345678 2. C Go back to the passage. In the same sentence with vindicates is claim to 9 123456789012345678901234567890121234567890123456789012345678 9 123456789012345678901234567890121234567890123456789012345678 the status of literature. This phrase indicates that literature has more 9 123456789012345678901234567890121234567890123456789012345678 9 1 status than fiction. You can confirm this hunch in the next sentence, 23456789012345678901234567890121234567890123456789012345678 123456789012345678901234567890121234567890123456789012345678 9 9 123456789012345678901234567890121234567890123456789012345678 too. It says a work clearly rises to the auspicious status of literature. 9 123456789012345678901234567890121234567890123456789012345678 9 123456789012345678901234567890121234567890123456789012345678 Again, the emphasis is on the loftiness of literature. Choice (C) best 9 1 23456789012345678901234567890121234567890123456789012345678 9 123456789012345678901234567890121234567890123456789012345678 captures this sense. 9 1 9 123456789012345678901234567890121234567890123456789012345678 123456789012345678901234567890121234567890123456789012345678 9 9 123456789012345678901234567890121234567890123456789012345678 123456789012345678901234567890121234567890123456789012345678 9 123456789012345678901234567890121234567890123456789012345678 9 123456789012345678901234567890121234567890123456789012345678 9 123456789012345678901234567890121234567890123456789012345678 9 1234567890123456789012345678901212345678901234567890123456789 74

Copyright © 2005 Thomson Peterson’s, a part of The Thomson Corporaton SAT is a registered trademark of the College Entrance Examination Board, which was not involved in the production of and does not endorse this product.

1234567890123456789012345678901212345678901234567890123456789 123456789012345678901234567890121234567890123456789012345678 9 9 123456789012345678901234567890121234567890123456789012345678 123456789012345678901234567890121234567890123456789012345678 9 9 123456789012345678901234567890121234567890123456789012345678 3. C The passage states that the test of time method, by definition excludes 9 123456789012345678901234567890121234567890123456789012345678 9 123456789012345678901234567890121234567890123456789012345678 contemporary works from consideration. Choice (C) is a reasonable 9 123456789012345678901234567890121234567890123456789012345678 23456789012345678901234567890121234567890123456789012345678 9 1 paraphrase. 9 123456789012345678901234567890121234567890123456789012345678 9 123456789012345678901234567890121234567890123456789012345678 9 123456789012345678901234567890121234567890123456789012345678 4. E There is only one topic that the paragraph mentions twice in relation to 9 123456789012345678901234567890121234567890123456789012345678 23456789012345678901234567890121234567890123456789012345678 9 1 the exploration of Mars: detection of life. So this topic is the best 9 123456789012345678901234567890121234567890123456789012345678 9 123456789012345678901234567890121234567890123456789012345678 candidate for what the paragraph most emphasizes as the motivation 9 123456789012345678901234567890121234567890123456789012345678 9 123456789012345678901234567890121234567890123456789012345678 for the Mars exploration. The paragraph also concludes with this point. 23456789012345678901234567890121234567890123456789012345678 9 1 9 123456789012345678901234567890121234567890123456789012345678 Choice (E) is the answer. 9 123456789012345678901234567890121234567890123456789012345678 9 123456789012345678901234567890121234567890123456789012345678 5. B Does the author of Passage 1 criticize the English system? He says it is 9 123456789012345678901234567890121234567890123456789012345678 23456789012345678901234567890121234567890123456789012345678 9 1 quirky, but he does not outright criticize it. That eliminates (D) and (E). 9 123456789012345678901234567890121234567890123456789012345678 9 123456789012345678901234567890121234567890123456789012345678 The author does not strongly praise the system either. That eliminates 9 123456789012345678901234567890121234567890123456789012345678 9 123456789012345678901234567890121234567890123456789012345678 (A). The author is also not neutral toward it (he argues that it is part of 23456789012345678901234567890121234567890123456789012345678 123456789012345678901234567890121234567890123456789012345678 99 123456789012345678901234567890121234567890123456789012345678 the inheritance of the English-speaking world). That eliminates (C). 9 123456789012345678901234567890121234567890123456789012345678 9 123456789012345678901234567890121234567890123456789012345678 Choice (B), qualified acceptance, sounds about right since the author 123456789012345678901234567890121234567890123456789012345678 99 123456789012345678901234567890121234567890123456789012345678 does accept the system but still points out its quirkiness. 9 1 9 123456789012345678901234567890121234567890123456789012345678 9 123456789012345678901234567890121234567890123456789012345678 6. D What is Henry I’s role in the passage? Not simply to demonstrate that 9 123456789012345678901234567890121234567890123456789012345678 23456789012345678901234567890121234567890123456789012345678 9 123456789012345678901234567890121234567890123456789012345678 the monarchy was involved in the development of the English system. 9 123456789012345678901234567890121234567890123456789012345678 9 123456789012345678901234567890121234567890123456789012345678 Eliminate (B). The story of Henry I is definitely not included to suggest 123456789012345678901234567890121234567890123456789012345678 99 123456789012345678901234567890121234567890123456789012345678 the practicality of the English system, since that is contrary to the basic 9 123456789012345678901234567890121234567890123456789012345678 9 123456789012345678901234567890121234567890123456789012345678 thesis of the passage. Eliminate (E). The basic point of the passage is to 9 123456789012345678901234567890121234567890123456789012345678 argue that the English system developed arbitrarily over time. Choice 9 123456789012345678901234567890121234567890123456789012345678 9 1 (D) sounds like a good answer. (A) isn’t right, since the author says the 23456789012345678901234567890121234567890123456789012345678 123456789012345678901234567890121234567890123456789012345678 99 123456789012345678901234567890121234567890123456789012345678 anecdote is surprisingly true. Answer (C) is off base, because the 9 1 9 123456789012345678901234567890121234567890123456789012345678 passage isn’t simply interested in the length, but also the quality, of the 9 123456789012345678901234567890121234567890123456789012345678 23456789012345678901234567890121234567890123456789012345678 9 123456789012345678901234567890121234567890123456789012345678 English system’s history. Choice (D) is correct. 9 123456789012345678901234567890121234567890123456789012345678 9 1 9 123456789012345678901234567890121234567890123456789012345678 7. C Earthy in context must mean something slightly better than ridiculous, 9 123456789012345678901234567890121234567890123456789012345678 23456789012345678901234567890121234567890123456789012345678 9 123456789012345678901234567890121234567890123456789012345678 because it is used in the discussion of the arbitrary nature of the English 9 123456789012345678901234567890121234567890123456789012345678 9 1 system. Choice (C) is the best answer. 123456789012345678901234567890121234567890123456789012345678 9 123456789012345678901234567890121234567890123456789012345678 9 9 123456789012345678901234567890121234567890123456789012345678 8. E Go back to the phrase in the passage. It talks about one system 9 123456789012345678901234567890121234567890123456789012345678 9 123456789012345678901234567890121234567890123456789012345678 developing in Rome and another in medieval Europe. They reconciled 9 123456789012345678901234567890121234567890123456789012345678 9 123456789012345678901234567890121234567890123456789012345678 the two by making an estimate of one (the mile) that is an even number 23456789012345678901234567890121234567890123456789012345678 9 123456789012345678901234567890121234567890123456789012345678 of the other (furlongs). Is that synonymous with determining which one 9 123456789012345678901234567890121234567890123456789012345678 9 123456789012345678901234567890121234567890123456789012345678 is accurate? No. Eliminate choice (A). With developing a more accurate 9 123456789012345678901234567890121234567890123456789012345678 9 1 system? No. (B) is out. Settling the public’s disagreement? Possibly. 23456789012345678901234567890121234567890123456789012345678 123456789012345678901234567890121234567890123456789012345678 99 123456789012345678901234567890121234567890123456789012345678 Hang on to (C). Finding a metric equivalent? Definitely not, so 9 123456789012345678901234567890121234567890123456789012345678 9 123456789012345678901234567890121234567890123456789012345678 eliminate (D). Ceasing to use two different systems? Definitely. Between 9 1 23456789012345678901234567890121234567890123456789012345678 9 123456789012345678901234567890121234567890123456789012345678 definitely (E)and possibly (C), pick definitely. The answer is (E). 9 1 9 123456789012345678901234567890121234567890123456789012345678 123456789012345678901234567890121234567890123456789012345678 9 9 123456789012345678901234567890121234567890123456789012345678 123456789012345678901234567890121234567890123456789012345678 9 123456789012345678901234567890121234567890123456789012345678 9 123456789012345678901234567890121234567890123456789012345678 9 123456789012345678901234567890121234567890123456789012345678 9 1234567890123456789012345678901212345678901234567890123456789 Copyright © 2005 Thomson Peterson’s, a part of The Thomson Corporaton SAT is a registered trademark of the College Entrance Examination Board, which was not involved in the production of and does not endorse this product.

75

1234567890123456789012345678901212345678901234567890123456789 123456789012345678901234567890121234567890123456789012345678 9 9 123456789012345678901234567890121234567890123456789012345678 123456789012345678901234567890121234567890123456789012345678 9 9 123456789012345678901234567890121234567890123456789012345678 9. B Again, use your knowledge of the main idea to help guide you. You can 9 123456789012345678901234567890121234567890123456789012345678 9 123456789012345678901234567890121234567890123456789012345678 therefore eliminate (A) and (E). Does the English system continue to 9 123456789012345678901234567890121234567890123456789012345678 23456789012345678901234567890121234567890123456789012345678 9 1 evolve? Use common sense—no. You are left with (B) and (C). The 9 123456789012345678901234567890121234567890123456789012345678 9 123456789012345678901234567890121234567890123456789012345678 passage specifically mentions a variety of sources. (B) is the best answer. 9 123456789012345678901234567890121234567890123456789012345678 9 123456789012345678901234567890121234567890123456789012345678 (C) is less good because it emphasizes the length of history instead of 23456789012345678901234567890121234567890123456789012345678 9 1 9 123456789012345678901234567890121234567890123456789012345678 the quality of that history. 9 123456789012345678901234567890121234567890123456789012345678 9 123456789012345678901234567890121234567890123456789012345678 9 123456789012345678901234567890121234567890123456789012345678 10. A You’ve established that the history of the English system is long and 23456789012345678901234567890121234567890123456789012345678 9 1 9 123456789012345678901234567890121234567890123456789012345678 colorful. In the second passage, the author is also interested in history. 9 123456789012345678901234567890121234567890123456789012345678 9 So she/he would probably not happily agree that that history is less 123456789012345678901234567890121234567890123456789012345678 9 123456789012345678901234567890121234567890123456789012345678 interesting. But there are dates in both passages. The English system 23456789012345678901234567890121234567890123456789012345678 9 1 9 123456789012345678901234567890121234567890123456789012345678 goes back to the sixteenth century. Even if you forget that’s actually the 9 123456789012345678901234567890121234567890123456789012345678 9 123456789012345678901234567890121234567890123456789012345678 1500s, you still know it’s before 1840. So the history is factually 9 123456789012345678901234567890121234567890123456789012345678 23456789012345678901234567890121234567890123456789012345678 9 123456789012345678901234567890121234567890123456789012345678 shorter. That makes the answer (A). 9 123456789012345678901234567890121234567890123456789012345678 123456789012345678901234567890121234567890123456789012345678 9 9 123456789012345678901234567890121234567890123456789012345678 11. D Go back to the reference to the French Revolution in the passage. The 9 123456789012345678901234567890121234567890123456789012345678 9 123456789012345678901234567890121234567890123456789012345678 author specifically mentions its anti-traditionalist bent. So (D) looks 9 1 9 123456789012345678901234567890121234567890123456789012345678 good. She/he doesn’t go so far as to indicate that the greatest 9 123456789012345678901234567890121234567890123456789012345678 9 123456789012345678901234567890121234567890123456789012345678 accomplishment of the French Revolution was the metric system. 23456789012345678901234567890121234567890123456789012345678 123456789012345678901234567890121234567890123456789012345678 9 9 123456789012345678901234567890121234567890123456789012345678 Eliminate (A). The author indicates that the metric system was in 9 123456789012345678901234567890121234567890123456789012345678 9 123456789012345678901234567890121234567890123456789012345678 keeping with the rest of the revolutionary thinking. So eliminate (B). 123456789012345678901234567890121234567890123456789012345678 9 9 123456789012345678901234567890121234567890123456789012345678 There’s no mention of the revolutionary calendar in the passage. 9 123456789012345678901234567890121234567890123456789012345678 9 123456789012345678901234567890121234567890123456789012345678 Eliminate (C). (E) is the only tricky wrong answer. The passage says the 123456789012345678901234567890121234567890123456789012345678 9 9 1 metric system wasn’t adopted in France until 1840, but it doesn’t say it 9 123456789012345678901234567890121234567890123456789012345678 9 wasn’t fully invented before then. The answer is (D). 123456789012345678901234567890121234567890123456789012345678 123456789012345678901234567890121234567890123456789012345678 9 9 123456789012345678901234567890121234567890123456789012345678 12. C In the sentence in question, you find that it has been called, or dubbed, 9 123456789012345678901234567890121234567890123456789012345678 23456789012345678901234567890121234567890123456789012345678 9 123456789012345678901234567890121234567890123456789012345678 “voluntary” and “preferred.” The implication is these are some sort of 9 123456789012345678901234567890121234567890123456789012345678 9 1 official designation, but they specifically are not required. Choice (C) 9 123456789012345678901234567890121234567890123456789012345678 9 123456789012345678901234567890121234567890123456789012345678 sounds like a reasonable, noncommittal paraphrase. Eliminate (D). 23456789012345678901234567890121234567890123456789012345678 123456789012345678901234567890121234567890123456789012345678 9 9 123456789012345678901234567890121234567890123456789012345678 These words also don’t suggest any pending official adoption. 9 1 9 123456789012345678901234567890121234567890123456789012345678 Eliminate (B). (A) overstates the case in the other direction: the view of 123456789012345678901234567890121234567890123456789012345678 9 23456789012345678901234567890121234567890123456789012345678 9 123456789012345678901234567890121234567890123456789012345678 the metric system reflected in the sentence in the passage is favorable. 9 123456789012345678901234567890121234567890123456789012345678 9 123456789012345678901234567890121234567890123456789012345678 (E) overstates how favorable—the words in the passage are warm but 123456789012345678901234567890121234567890123456789012345678 9 9 1 don’t suggest “superiority.” The answer is (C). 9 123456789012345678901234567890121234567890123456789012345678 9 123456789012345678901234567890121234567890123456789012345678 9 123456789012345678901234567890121234567890123456789012345678 13. E Refined in the passage is synonymous with recalculated. The passage 9 123456789012345678901234567890121234567890123456789012345678 9 123456789012345678901234567890121234567890123456789012345678 doesn’t say whether it’s to make the meter smaller or bigger. That only 23456789012345678901234567890121234567890123456789012345678 9 123456789012345678901234567890121234567890123456789012345678 leaves (E). 9 123456789012345678901234567890121234567890123456789012345678 123456789012345678901234567890121234567890123456789012345678 9 123456789012345678901234567890121234567890123456789012345678 9 9 1 9 123456789012345678901234567890121234567890123456789012345678 123456789012345678901234567890121234567890123456789012345678 9 9 123456789012345678901234567890121234567890123456789012345678 123456789012345678901234567890121234567890123456789012345678 9 9 123456789012345678901234567890121234567890123456789012345678 123456789012345678901234567890121234567890123456789012345678 9 123456789012345678901234567890121234567890123456789012345678 9 123456789012345678901234567890121234567890123456789012345678 9 123456789012345678901234567890121234567890123456789012345678 9 1234567890123456789012345678901212345678901234567890123456789 76

Copyright © 2005 Thomson Peterson’s, a part of The Thomson Corporaton SAT is a registered trademark of the College Entrance Examination Board, which was not involved in the production of and does not endorse this product.

1234567890123456789012345678901212345678901234567890123456789 123456789012345678901234567890121234567890123456789012345678 9 9 123456789012345678901234567890121234567890123456789012345678 123456789012345678901234567890121234567890123456789012345678 9 9 123456789012345678901234567890121234567890123456789012345678 14. B There is no indication of any ideological shift motivating the 1983 9 123456789012345678901234567890121234567890123456789012345678 9 123456789012345678901234567890121234567890123456789012345678 move. That eliminates (A), (C), and (E). Which is more logical: new 9 123456789012345678901234567890121234567890123456789012345678 23456789012345678901234567890121234567890123456789012345678 9 1 calculations of the Earth’s circumference or new ways to calculate 9 123456789012345678901234567890121234567890123456789012345678 9 123456789012345678901234567890121234567890123456789012345678 fractions? The former, especially where the speed of light is involved. 9 123456789012345678901234567890121234567890123456789012345678 9 123456789012345678901234567890121234567890123456789012345678 The answer is (B). 23456789012345678901234567890121234567890123456789012345678 9 1 9 123456789012345678901234567890121234567890123456789012345678 9 123456789012345678901234567890121234567890123456789012345678 15. A As soon as you find an answer that isn’t addressed in one passage, 9 123456789012345678901234567890121234567890123456789012345678 9 123456789012345678901234567890121234567890123456789012345678 you’re done. Do both passages mention kings who were ruling at the 23456789012345678901234567890121234567890123456789012345678 9 1 9 123456789012345678901234567890121234567890123456789012345678 time of their invention? No. Passage 2 talks about the French 9 123456789012345678901234567890121234567890123456789012345678 9 Revolution, which overthrew the king; it only mentions a king in 123456789012345678901234567890121234567890123456789012345678 9 123456789012345678901234567890121234567890123456789012345678 regards to the English system. Choice (A) is the answer. 23456789012345678901234567890121234567890123456789012345678 9 1 9 123456789012345678901234567890121234567890123456789012345678 9 123456789012345678901234567890121234567890123456789012345678 16. A Which of the answers is something that the author of Passage 2 would 9 123456789012345678901234567890121234567890123456789012345678 9 123456789012345678901234567890121234567890123456789012345678 agree with but does not explicitly state? Which is most plausible? 23456789012345678901234567890121234567890123456789012345678 123456789012345678901234567890121234567890123456789012345678 99 123456789012345678901234567890121234567890123456789012345678 Choice (A) is the most reasonable answer on its face. You can find 9 123456789012345678901234567890121234567890123456789012345678 9 123456789012345678901234567890121234567890123456789012345678 evidence for it when the author says It is the great asset of the metric 123456789012345678901234567890121234567890123456789012345678 99 123456789012345678901234567890121234567890123456789012345678 system, at least for scientists, that units for measuring weight and 9 1 9 123456789012345678901234567890121234567890123456789012345678 energy are also derived from the basic unit of the meter and The 9 123456789012345678901234567890121234567890123456789012345678 9 123456789012345678901234567890121234567890123456789012345678 adoption of the metric system, also known as the international system, 23456789012345678901234567890121234567890123456789012345678 123456789012345678901234567890121234567890123456789012345678 99 123456789012345678901234567890121234567890123456789012345678 or S.I., coincided with great advances in science. 9 123456789012345678901234567890121234567890123456789012345678 123456789012345678901234567890121234567890123456789012345678 99 123456789012345678901234567890121234567890123456789012345678 9 123456789012345678901234567890121234567890123456789012345678 9 123456789012345678901234567890121234567890123456789012345678 Section 7 9 123456789012345678901234567890121234567890123456789012345678 9 123456789012345678901234567890121234567890123456789012345678 1. There is nothing fancy about this problem. Substitute and solve for x. 9 1 23456789012345678901234567890121234567890123456789012345678 123456789012345678901234567890121234567890123456789012345678 99 123456789012345678901234567890121234567890123456789012345678 2y 5 12 9 1 9 123456789012345678901234567890121234567890123456789012345678 y 5 6 9 123456789012345678901234567890121234567890123456789012345678 23456789012345678901234567890121234567890123456789012345678 123456789012345678901234567890121234567890123456789012345678 99 123456789012345678901234567890121234567890123456789012345678 9 1 2=x 1 =x 5 y 9 123456789012345678901234567890121234567890123456789012345678 9 123456789012345678901234567890121234567890123456789012345678 2 x 1 x 5 6 = = 23456789012345678901234567890121234567890123456789012345678 123456789012345678901234567890121234567890123456789012345678 99 123456789012345678901234567890121234567890123456789012345678 9 1 3=x 5 6 9 123456789012345678901234567890121234567890123456789012345678 123456789012345678901234567890121234567890123456789012345678 9 23456789012345678901234567890121234567890123456789012345678 9 123456789012345678901234567890121234567890123456789012345678 =x 5 2 9 123456789012345678901234567890121234567890123456789012345678 9 123456789012345678901234567890121234567890123456789012345678 x 5 4 123456789012345678901234567890121234567890123456789012345678 99 1 9 123456789012345678901234567890121234567890123456789012345678 2. The first five even integers are 2, 4, 6, 8, and 10. Rev up that calculator start 9 123456789012345678901234567890121234567890123456789012345678 9 123456789012345678901234567890121234567890123456789012345678 multiplying. The answer is 3,840. 9 123456789012345678901234567890121234567890123456789012345678 123456789012345678901234567890121234567890123456789012345678 9 23456789012345678901234567890121234567890123456789012345678 9 123456789012345678901234567890121234567890123456789012345678 3. If the probability that a senior would be picked is three-eighths, then seniors 9 123456789012345678901234567890121234567890123456789012345678 9 123456789012345678901234567890121234567890123456789012345678 are three-eighths of the entire student body. Since Rider High has 400 9 123456789012345678901234567890121234567890123456789012345678 students, the equation would be: 9 1 23456789012345678901234567890121234567890123456789012345678 123456789012345678901234567890121234567890123456789012345678 99 1 9 123456789012345678901234567890121234567890123456789012345678 3 1200 9 123456789012345678901234567890121234567890123456789012345678 400 3 5 5 150 9 123456789012345678901234567890121234567890123456789012345678 8 8 23456789012345678901234567890121234567890123456789012345678 9 1 123456789012345678901234567890121234567890123456789012345678 9 123456789012345678901234567890121234567890123456789012345678 9 123456789012345678901234567890121234567890123456789012345678 9 1234567890123456789012345678901212345678901234567890123456789 Copyright © 2005 Thomson Peterson’s, a part of The Thomson Corporaton SAT is a registered trademark of the College Entrance Examination Board, which was not involved in the production of and does not endorse this product.

77

1234567890123456789012345678901212345678901234567890123456789 123456789012345678901234567890121234567890123456789012345678 9 9 123456789012345678901234567890121234567890123456789012345678 123456789012345678901234567890121234567890123456789012345678 9 9 123456789012345678901234567890121234567890123456789012345678 4. The volume of any rectangular solid is the length times the width times the 9 123456789012345678901234567890121234567890123456789012345678 9 123456789012345678901234567890121234567890123456789012345678 height. The figure tells us the width and the height, and you can determine 9 123456789012345678901234567890121234567890123456789012345678 23456789012345678901234567890121234567890123456789012345678 9 1 the length from what you know about the area of the shaded side. Start with 9 123456789012345678901234567890121234567890123456789012345678 9 123456789012345678901234567890121234567890123456789012345678 the area of a rectangle formula and you can find the length: 9 123456789012345678901234567890121234567890123456789012345678 9 123456789012345678901234567890121234567890123456789012345678 23456789012345678901234567890121234567890123456789012345678 9 1 A5l3h 9 123456789012345678901234567890121234567890123456789012345678 9 123456789012345678901234567890121234567890123456789012345678 24 5 4 3 l 9 123456789012345678901234567890121234567890123456789012345678 9 123456789012345678901234567890121234567890123456789012345678 23456789012345678901234567890121234567890123456789012345678 9 1 65l 9 123456789012345678901234567890121234567890123456789012345678 9 123456789012345678901234567890121234567890123456789012345678 Place this length of 6 into the volume formula for the box: 9 123456789012345678901234567890121234567890123456789012345678 9 123456789012345678901234567890121234567890123456789012345678 23456789012345678901234567890121234567890123456789012345678 9 1 V 5 l 3 h 3 w 5 3 3 4 3 6 5 72 9 123456789012345678901234567890121234567890123456789012345678 9 123456789012345678901234567890121234567890123456789012345678 9 123456789012345678901234567890121234567890123456789012345678 5. First use the area of a circle formula to determine the radius. 9 123456789012345678901234567890121234567890123456789012345678 23456789012345678901234567890121234567890123456789012345678 9 123456789012345678901234567890121234567890123456789012345678 2 9 123456789012345678901234567890121234567890123456789012345678 A 5 pr 123456789012345678901234567890121234567890123456789012345678 9 9 123456789012345678901234567890121234567890123456789012345678 16p 5 pr2 9 123456789012345678901234567890121234567890123456789012345678 9 123456789012345678901234567890121234567890123456789012345678 2 9 1 16p pr 9 123456789012345678901234567890121234567890123456789012345678 5 9 123456789012345678901234567890121234567890123456789012345678 p p 9 123456789012345678901234567890121234567890123456789012345678 2 23456789012345678901234567890121234567890123456789012345678 9 123456789012345678901234567890121234567890123456789012345678 16 5 r 9 123456789012345678901234567890121234567890123456789012345678 9 123456789012345678901234567890121234567890123456789012345678 45r 9 123456789012345678901234567890121234567890123456789012345678 123456789012345678901234567890121234567890123456789012345678 9 9 123456789012345678901234567890121234567890123456789012345678 The diameter of a circle is twice the radius, so the diameter is 8. 9 123456789012345678901234567890121234567890123456789012345678 123456789012345678901234567890121234567890123456789012345678 9 9 123456789012345678901234567890121234567890123456789012345678 6. There are some fancier ways to solve this problem, but the surest way is to 9 1 23456789012345678901234567890121234567890123456789012345678 9 123456789012345678901234567890121234567890123456789012345678 count up the options. She could have: 9 123456789012345678901234567890121234567890123456789012345678 9 1 9 123456789012345678901234567890121234567890123456789012345678 1. No toppings. 9 123456789012345678901234567890121234567890123456789012345678 23456789012345678901234567890121234567890123456789012345678 9 123456789012345678901234567890121234567890123456789012345678 2. Just a 9 123456789012345678901234567890121234567890123456789012345678 9 1 3. Just p 9 123456789012345678901234567890121234567890123456789012345678 4. Just r 9 123456789012345678901234567890121234567890123456789012345678 23456789012345678901234567890121234567890123456789012345678 9 123456789012345678901234567890121234567890123456789012345678 5. a and p 9 123456789012345678901234567890121234567890123456789012345678 9 1 6. a and r 9 123456789012345678901234567890121234567890123456789012345678 9 123456789012345678901234567890121234567890123456789012345678 7. p and r 23456789012345678901234567890121234567890123456789012345678 123456789012345678901234567890121234567890123456789012345678 9 9 123456789012345678901234567890121234567890123456789012345678 8. All the toppings. 9 123456789012345678901234567890121234567890123456789012345678 123456789012345678901234567890121234567890123456789012345678 9 9 1 That is a total of 8. Sure, there’s a fancier math way of handling this 9 123456789012345678901234567890121234567890123456789012345678 9 123456789012345678901234567890121234567890123456789012345678 problem, but since you have the right answer, what does it matter? Is your 9 123456789012345678901234567890121234567890123456789012345678 9 123456789012345678901234567890121234567890123456789012345678 SAT score in any way determined by whether you used the “fancy method” 123456789012345678901234567890121234567890123456789012345678 9 23456789012345678901234567890121234567890123456789012345678 9 123456789012345678901234567890121234567890123456789012345678 or not? 9 123456789012345678901234567890121234567890123456789012345678 123456789012345678901234567890121234567890123456789012345678 9 123456789012345678901234567890121234567890123456789012345678 9 9 1 9 123456789012345678901234567890121234567890123456789012345678 123456789012345678901234567890121234567890123456789012345678 9 9 123456789012345678901234567890121234567890123456789012345678 123456789012345678901234567890121234567890123456789012345678 9 9 123456789012345678901234567890121234567890123456789012345678 123456789012345678901234567890121234567890123456789012345678 9 123456789012345678901234567890121234567890123456789012345678 9 123456789012345678901234567890121234567890123456789012345678 9 123456789012345678901234567890121234567890123456789012345678 9 1234567890123456789012345678901212345678901234567890123456789 78

Copyright © 2005 Thomson Peterson’s, a part of The Thomson Corporaton SAT is a registered trademark of the College Entrance Examination Board, which was not involved in the production of and does not endorse this product.

1234567890123456789012345678901212345678901234567890123456789 123456789012345678901234567890121234567890123456789012345678 9 9 123456789012345678901234567890121234567890123456789012345678 123456789012345678901234567890121234567890123456789012345678 9 9 123456789012345678901234567890121234567890123456789012345678 7. If x is the sum of the five numbers, you know: 9 123456789012345678901234567890121234567890123456789012345678 123456789012345678901234567890121234567890123456789012345678 9 9 123456789012345678901234567890121234567890123456789012345678 sum 9 123456789012345678901234567890121234567890123456789012345678 5 Average 9 123456789012345678901234567890121234567890123456789012345678 number of items 9 123456789012345678901234567890121234567890123456789012345678 9 123456789012345678901234567890121234567890123456789012345678 9 123456789012345678901234567890121234567890123456789012345678 x 23456789012345678901234567890121234567890123456789012345678 9 1 5 16 9 123456789012345678901234567890121234567890123456789012345678 5 9 123456789012345678901234567890121234567890123456789012345678 9 123456789012345678901234567890121234567890123456789012345678 x 9 123456789012345678901234567890121234567890123456789012345678 5 5 5 16 ~ ! ~ ! 23456789012345678901234567890121234567890123456789012345678 9 1 5 9 123456789012345678901234567890121234567890123456789012345678 9 123456789012345678901234567890121234567890123456789012345678 x 5 80 9 123456789012345678901234567890121234567890123456789012345678 9 123456789012345678901234567890121234567890123456789012345678 23456789012345678901234567890121234567890123456789012345678 9 1 Make y the number taken away from the set y. You know that 80 minus y is 9 123456789012345678901234567890121234567890123456789012345678 9 123456789012345678901234567890121234567890123456789012345678 the new sum, and that the new average is 14. With this information you can 9 123456789012345678901234567890121234567890123456789012345678 9 123456789012345678901234567890121234567890123456789012345678 solve for y using the equation: 23456789012345678901234567890121234567890123456789012345678 123456789012345678901234567890121234567890123456789012345678 99 123456789012345678901234567890121234567890123456789012345678 9 123456789012345678901234567890121234567890123456789012345678 80 2 y 9 123456789012345678901234567890121234567890123456789012345678 5 14 9 123456789012345678901234567890121234567890123456789012345678 4 123456789012345678901234567890121234567890123456789012345678 99 1 9 123456789012345678901234567890121234567890123456789012345678 80 2 y 9 123456789012345678901234567890121234567890123456789012345678 4 4 5 14 ~ ! ~ ! 9 123456789012345678901234567890121234567890123456789012345678 4 23456789012345678901234567890121234567890123456789012345678 123456789012345678901234567890121234567890123456789012345678 99 123456789012345678901234567890121234567890123456789012345678 80 2 y 5 56 9 123456789012345678901234567890121234567890123456789012345678 9 123456789012345678901234567890121234567890123456789012345678 80 2 56 2 y 5 56 2 56 123456789012345678901234567890121234567890123456789012345678 99 123456789012345678901234567890121234567890123456789012345678 9 123456789012345678901234567890121234567890123456789012345678 24 2 y 5 0 9 123456789012345678901234567890121234567890123456789012345678 9 123456789012345678901234567890121234567890123456789012345678 24 5 y 9 1 23456789012345678901234567890121234567890123456789012345678 123456789012345678901234567890121234567890123456789012345678 99 123456789012345678901234567890121234567890123456789012345678 8. Since the triangle is a right triangle, you can use the Pythagorean theorem to 9 1 9 123456789012345678901234567890121234567890123456789012345678 determine the third side of the triangle: 9 123456789012345678901234567890121234567890123456789012345678 23456789012345678901234567890121234567890123456789012345678 9 123456789012345678901234567890121234567890123456789012345678 2 2 2 9 123456789012345678901234567890121234567890123456789012345678 c 5 a 1 b 9 1 2 9 123456789012345678901234567890121234567890123456789012345678 100 5 64 1 b 9 123456789012345678901234567890121234567890123456789012345678 23456789012345678901234567890121234567890123456789012345678 9 123456789012345678901234567890121234567890123456789012345678 36 5 b2 9 123456789012345678901234567890121234567890123456789012345678 9 1 9 123456789012345678901234567890121234567890123456789012345678 b56 123456789012345678901234567890121234567890123456789012345678 9 23456789012345678901234567890121234567890123456789012345678 9 123456789012345678901234567890121234567890123456789012345678 If 6 is the length of one side of the square, the area of the square is the square 9 123456789012345678901234567890121234567890123456789012345678 9 123456789012345678901234567890121234567890123456789012345678 of that, 36. 9 123456789012345678901234567890121234567890123456789012345678 9 1 23456789012345678901234567890121234567890123456789012345678 9 123456789012345678901234567890121234567890123456789012345678 9. You need to remember the right formula for this one. At least you know that 9 123456789012345678901234567890121234567890123456789012345678 9 123456789012345678901234567890121234567890123456789012345678 since you can’t grid in negative numbers, the slope must be positive. Here’s 123456789012345678901234567890121234567890123456789012345678 99 1 the rise over run calculation: 9 123456789012345678901234567890121234567890123456789012345678 9 123456789012345678901234567890121234567890123456789012345678 9 123456789012345678901234567890121234567890123456789012345678 2 y 22 2 1 rise y 23 3 1 2 9 123456789012345678901234567890121234567890123456789012345678 5 slope 5 5 5 5 9 123456789012345678901234567890121234567890123456789012345678 run x 2 x 21 2 3 24 4 23456789012345678901234567890121234567890123456789012345678 9 123456789012345678901234567890121234567890123456789012345678 1 2 9 1 9 123456789012345678901234567890121234567890123456789012345678 123456789012345678901234567890121234567890123456789012345678 9 9 123456789012345678901234567890121234567890123456789012345678 123456789012345678901234567890121234567890123456789012345678 9 123456789012345678901234567890121234567890123456789012345678 9 123456789012345678901234567890121234567890123456789012345678 9 123456789012345678901234567890121234567890123456789012345678 9 1234567890123456789012345678901212345678901234567890123456789 Copyright © 2005 Thomson Peterson’s, a part of The Thomson Corporaton SAT is a registered trademark of the College Entrance Examination Board, which was not involved in the production of and does not endorse this product.

79

1234567890123456789012345678901212345678901234567890123456789 123456789012345678901234567890121234567890123456789012345678 9 9 123456789012345678901234567890121234567890123456789012345678 123456789012345678901234567890121234567890123456789012345678 9 9 123456789012345678901234567890121234567890123456789012345678 10. If you work backward from what you know, this problem contains no 9 123456789012345678901234567890121234567890123456789012345678 9 123456789012345678901234567890121234567890123456789012345678 difficult steps. If Bo is 15 and Gina is five years younger than he, then Gina 9 123456789012345678901234567890121234567890123456789012345678 23456789012345678901234567890121234567890123456789012345678 9 1 is 10. And if Gina is 10 and Susan is three times the age of Gina, then Susan 9 123456789012345678901234567890121234567890123456789012345678 9 123456789012345678901234567890121234567890123456789012345678 is 30. And if Susan is 30 and Tom is twelve years older than Susan, then Tom 9 123456789012345678901234567890121234567890123456789012345678 9 123456789012345678901234567890121234567890123456789012345678 is 42. There’s your answer, 42. 23456789012345678901234567890121234567890123456789012345678 9 1 9 123456789012345678901234567890121234567890123456789012345678 9 123456789012345678901234567890121234567890123456789012345678 11. This one tries to intimidate you with a new symbol and a complicated 9 123456789012345678901234567890121234567890123456789012345678 9 123456789012345678901234567890121234567890123456789012345678 definition. By now, this sort of attempted distraction should not even faze 23456789012345678901234567890121234567890123456789012345678 9 1 9 123456789012345678901234567890121234567890123456789012345678 you, as you are well aware that everything you need to know about the new 9 123456789012345678901234567890121234567890123456789012345678 9 symbol is right in front of you. 123456789012345678901234567890121234567890123456789012345678 9 123456789012345678901234567890121234567890123456789012345678 23456789012345678901234567890121234567890123456789012345678 9 1 The phrase greatest prime divisor means the greatest number that is prime and 9 123456789012345678901234567890121234567890123456789012345678 9 123456789012345678901234567890121234567890123456789012345678 also divides the original number. So the greatest prime divisor of 15 is 5 since 9 123456789012345678901234567890121234567890123456789012345678 9 123456789012345678901234567890121234567890123456789012345678 no prime numbers greater than 5 evenly divide into 15. As for 12, the greatest 23456789012345678901234567890121234567890123456789012345678 123456789012345678901234567890121234567890123456789012345678 9 9 123456789012345678901234567890121234567890123456789012345678 prime divisor is 3. This means (15*)(12*) 5 (5)(3) 5 15. 9 123456789012345678901234567890121234567890123456789012345678 123456789012345678901234567890121234567890123456789012345678 9 9 123456789012345678901234567890121234567890123456789012345678 12. The graph might look messy, but you only need to pick out the five 9 123456789012345678901234567890121234567890123456789012345678 9 1 y-intercepts and add them up. 9 123456789012345678901234567890121234567890123456789012345678 9 123456789012345678901234567890121234567890123456789012345678 9 123456789012345678901234567890121234567890123456789012345678 Line 1: at (0, 2) 23456789012345678901234567890121234567890123456789012345678 123456789012345678901234567890121234567890123456789012345678 9 9 123456789012345678901234567890121234567890123456789012345678 Line 2: at (0, 1) 9 123456789012345678901234567890121234567890123456789012345678 Lines 3 and 4: at (0, 0) 9 123456789012345678901234567890121234567890123456789012345678 9 123456789012345678901234567890121234567890123456789012345678 Line 5: at (0, 21) 123456789012345678901234567890121234567890123456789012345678 9 123456789012345678901234567890121234567890123456789012345678 9 9 123456789012345678901234567890121234567890123456789012345678 Adding up these five y-values gives you: 2 1 1 1 0 1 0 2 1 5 2 9 123456789012345678901234567890121234567890123456789012345678 9 1 23456789012345678901234567890121234567890123456789012345678 9 123456789012345678901234567890121234567890123456789012345678 13. The problem says that the numbers are distinct, so none of the four numbers 9 123456789012345678901234567890121234567890123456789012345678 9 1 are the same. That’s your first clue. Since the sum of the four numbers is 26, the 123456789012345678901234567890121234567890123456789012345678 9 9 123456789012345678901234567890121234567890123456789012345678 numbers in the sum must be less than 21. Here’s a Rogue’s Gallery List of all 9 123456789012345678901234567890121234567890123456789012345678 9 123456789012345678901234567890121234567890123456789012345678 the less prime numbers: 2, 3, 5, 7, 11, 13, 17, and 19. To find the greatest 123456789012345678901234567890121234567890123456789012345678 9 9 123456789012345678901234567890121234567890123456789012345678 possible integer in the set, first make all the other integers as least as possible. 9 123456789012345678901234567890121234567890123456789012345678 23456789012345678901234567890121234567890123456789012345678 9 123456789012345678901234567890121234567890123456789012345678 The set could be 2, 3, 5, x. This would maximize x. Now solve: 9 123456789012345678901234567890121234567890123456789012345678 9 1 9 123456789012345678901234567890121234567890123456789012345678 2 1 31 51 x 5 21 123456789012345678901234567890121234567890123456789012345678 9 23456789012345678901234567890121234567890123456789012345678 9 123456789012345678901234567890121234567890123456789012345678 10 1 x 5 21 9 123456789012345678901234567890121234567890123456789012345678 9 123456789012345678901234567890121234567890123456789012345678 10 2 10 1 x 5 21 2 10 123456789012345678901234567890121234567890123456789012345678 9 9 1 x 5 11, also prime. 9 123456789012345678901234567890121234567890123456789012345678 9 123456789012345678901234567890121234567890123456789012345678 9 123456789012345678901234567890121234567890123456789012345678 11 must be the answer. 9 123456789012345678901234567890121234567890123456789012345678 123456789012345678901234567890121234567890123456789012345678 9 23456789012345678901234567890121234567890123456789012345678 123456789012345678901234567890121234567890123456789012345678 9 123456789012345678901234567890121234567890123456789012345678 9 9 123456789012345678901234567890121234567890123456789012345678 Section 8 9 123456789012345678901234567890121234567890123456789012345678 9 1 As you might expect, answers will vary. If possible, politely ask a teacher, 23456789012345678901234567890121234567890123456789012345678 123456789012345678901234567890121234567890123456789012345678 9 9 1 fellow student, or some other person knowledgeable about formal essay 9 123456789012345678901234567890121234567890123456789012345678 9 123456789012345678901234567890121234567890123456789012345678 writing to review your essay and provide feedback on ways in which the 9 123456789012345678901234567890121234567890123456789012345678 23456789012345678901234567890121234567890123456789012345678 9 1 essay is commendable and on areas where it could be improved. 9 123456789012345678901234567890121234567890123456789012345678 123456789012345678901234567890121234567890123456789012345678 9 123456789012345678901234567890121234567890123456789012345678 9 1234567890123456789012345678901212345678901234567890123456789 80

Copyright © 2005 Thomson Peterson’s, a part of The Thomson Corporaton SAT is a registered trademark of the College Entrance Examination Board, which was not involved in the production of and does not endorse this product.

12345678901234567890123456789012123456789012345678901234567890121234567 1234567890123456789012345678901212345678901234567890123456789012123456 7 7 12234567890123456789012345678901212345678901234567890123456789012123456 1 34567890123456789012345678901212345678901234567890123456789012123456 7 7 12234567890123456789012345678901212345678901234567890123456789012123456 7 1234567890123456789012345678901212345678901234567890123456789012123456 1 34567890123456789012345678901212345678901234567890123456789012123456 7 7 12234567890123456789012345678901212345678901234567890123456789012123456 1 34567890123456789012345678901212345678901234567890123456789012123456 7 7 12234567890123456789012345678901212345678901234567890123456789012123456 7 1234567890123456789012345678901212345678901234567890123456789012123456 7 1234567890123456789012345678901212345678901234567890123456789012123456 MATH 7 1234567890123456789012345678901212345678901234567890123456789012123456 34567890123456789012345678901212345678901234567890123456789012123456 7 1 7 12234567890123456789012345678901212345678901234567890123456789012123456 Number Number Raw 7 1234567890123456789012345678901212345678901234567890123456789012123456 7 1234567890123456789012345678901212345678901234567890123456789012123456 Correct Incorrect 5 Score 7 1234567890123456789012345678901212345678901234567890123456789012123456 34567890123456789012345678901212345678901234567890123456789012123456 7 1 7 12234567890123456789012345678901212345678901234567890123456789012123456 7 1234567890123456789012345678901212345678901234567890123456789012123456 2 (.25 3 ) 5 Section 2 7 1234567890123456789012345678901212345678901234567890123456789012123456 7 1234567890123456789012345678901212345678901234567890123456789012123456 34567890123456789012345678901212345678901234567890123456789012123456 7 1 7 12234567890123456789012345678901212345678901234567890123456789012123456 7 1234567890123456789012345678901212345678901234567890123456789012123456 Section 5 2 (.25 3 ) 5 7 1234567890123456789012345678901212345678901234567890123456789012123456 7 1234567890123456789012345678901212345678901234567890123456789012123456 34567890123456789012345678901212345678901234567890123456789012123456 1234567890123456789012345678901212345678901234567890123456789012123456 77 1234567890123456789012345678901212345678901234567890123456789012123456 7 1234567890123456789012345678901212345678901234567890123456789012123456 Section 7 2 (.25 3 ) 5 7 1234567890123456789012345678901212345678901234567890123456789012123456 1234567890123456789012345678901212345678901234567890123456789012123456 77 1234567890123456789012345678901212345678901234567890123456789012123456 7 1 CRITICAL READING 7 12234567890123456789012345678901212345678901234567890123456789012123456 7 1234567890123456789012345678901212345678901234567890123456789012123456 7 1234567890123456789012345678901212345678901234567890123456789012123456 34567890123456789012345678901212345678901234567890123456789012123456 7 1234567890123456789012345678901212345678901234567890123456789012123456 Sections 7 1234567890123456789012345678901212345678901234567890123456789012123456 7 1234567890123456789012345678901212345678901234567890123456789012123456 1, 4, and 6 2 (.25 3 ) 5 1234567890123456789012345678901212345678901234567890123456789012123456 77 1234567890123456789012345678901212345678901234567890123456789012123456 7 1234567890123456789012345678901212345678901234567890123456789012123456 7 1234567890123456789012345678901212345678901234567890123456789012123456 7 1234567890123456789012345678901212345678901234567890123456789012123456 WRITING 7 1234567890123456789012345678901212345678901234567890123456789012123456 7 1 2 34567890123456789012345678901212345678901234567890123456789012123456 7 1234567890123456789012345678901212345678901234567890123456789012123456 Section 3 2 (.25 3 ) 5 7 1234567890123456789012345678901212345678901234567890123456789012123456 7 1 1234567890123456789012345678901212345678901234567890123456789012123456 7 7 12234567890123456789012345678901212345678901234567890123456789012123456 7 1234567890123456789012345678901212345678901234567890123456789012123456 Section 8 Go to www.petersons.com/satessayedge/ 7 1234567890123456789012345678901212345678901234567890123456789012123456 7 1 34567890123456789012345678901212345678901234567890123456789012123456 for instant online scoring and feedback. 7 12234567890123456789012345678901212345678901234567890123456789012123456 7 1234567890123456789012345678901212345678901234567890123456789012123456 34567890123456789012345678901212345678901234567890123456789012123456 1234567890123456789012345678901212345678901234567890123456789012123456 77 1234567890123456789012345678901212345678901234567890123456789012123456 7 1 1234567890123456789012345678901212345678901234567890123456789012123456 7 1234567890123456789012345678901212345678901234567890123456789012123456 7 7 12234567890123456789012345678901212345678901234567890123456789012123456 7 1234567890123456789012345678901212345678901234567890123456789012123456 7 1234567890123456789012345678901212345678901234567890123456789012123456 7 1234567890123456789012345678901212345678901234567890123456789012123456 1 34567890123456789012345678901212345678901234567890123456789012123456 7 2 34567890123456789012345678901212345678901234567890123456789012123456 1234567890123456789012345678901212345678901234567890123456789012123456 77 1234567890123456789012345678901212345678901234567890123456789012123456 7 1234567890123456789012345678901212345678901234567890123456789012123456 7 1234567890123456789012345678901212345678901234567890123456789012123456 7 1 7 12234567890123456789012345678901212345678901234567890123456789012123456 7 1234567890123456789012345678901212345678901234567890123456789012123456 7 1234567890123456789012345678901212345678901234567890123456789012123456 7 1234567890123456789012345678901212345678901234567890123456789012123456 1 34567890123456789012345678901212345678901234567890123456789012123456 7 2 34567890123456789012345678901212345678901234567890123456789012123456 1234567890123456789012345678901212345678901234567890123456789012123456 77 1 7 12234567890123456789012345678901212345678901234567890123456789012123456 1 34567890123456789012345678901212345678901234567890123456789012123456 7 7 12234567890123456789012345678901212345678901234567890123456789012123456 1 34567890123456789012345678901212345678901234567890123456789012123456 7 1234567890123456789012345678901212345678901234567890123456789012123456 7 1234567890123456789012345678901212345678901234567890123456789012123456 7 1234567890123456789012345678901212345678901234567890123456789012123456 7 12345678901234567890123456789012123456789012345678901234567890121234567

Scoring Worksheet

Copyright © 2005 Thomson Peterson’s, a part of The Thomson Corporaton SAT is a registered trademark of the College Entrance Examination Board, which was not involved in the production of and does not endorse this product.

81

12345678901234567890123456789012123456789012345678901234567890121234567 1234567890123456789012345678901212345678901234567890123456789012123456 7 34567890123456789012345678901212345678901234567890123456789012123456 7 12 1234567890123456789012345678901212345678901234567890123456789012123456 7 34567890123456789012345678901212345678901234567890123456789012123456 7 12 34567890123456789012345678901212345678901234567890123456789012123456 7 12 1234567890123456789012345678901212345678901234567890123456789012123456 7 34567890123456789012345678901212345678901234567890123456789012123456 7 12 7 1234567890123456789012345678901212345678901234567890123456789012123456 MATH CRITICAL READING 34567890123456789012345678901212345678901234567890123456789012123456 7 12 34567890123456789012345678901212345678901234567890123456789012123456 7 12 34567890123456789012345678901212345678901234567890123456789012123456 7 12 Raw Math Scaled Raw Math Scaled Raw Verbal Scaled Raw Verbal Scaled 34567890123456789012345678901212345678901234567890123456789012123456 7 12 Score Score Score Score Score Score Score Score 2 34567890123456789012345678901212345678901234567890123456789012123456 7 1 34567890123456789012345678901212345678901234567890123456789012123456 7 12 78 800 37 510 60 800 28 500 34567890123456789012345678901212345678901234567890123456789012123456 7 12 34567890123456789012345678901212345678901234567890123456789012123456 7 12 77 800 36 510 59 800 27 490 34567890123456789012345678901212345678901234567890123456789012123456 7 12 76 800 35 500 2 34567890123456789012345678901212345678901234567890123456789012123456 7 1 58 790 26 490 34567890123456789012345678901212345678901234567890123456789012123456 7 12 75 790 34 500 57 770 25 480 34567890123456789012345678901212345678901234567890123456789012123456 7 12 34567890123456789012345678901212345678901234567890123456789012123456 7 12 74 780 33 490 56 760 24 470 34567890123456789012345678901212345678901234567890123456789012123456 7 12 73 770 32 490 2 34567890123456789012345678901212345678901234567890123456789012123456 7 1 55 740 23 460 34567890123456789012345678901212345678901234567890123456789012123456 7 12 72 760 31 480 34567890123456789012345678901212345678901234567890123456789012123456 7 12 54 720 22 460 34567890123456789012345678901212345678901234567890123456789012123456 7 12 71 750 30 480 34567890123456789012345678901212345678901234567890123456789012123456 7 12 53 710 21 450 2 34567890123456789012345678901212345678901234567890123456789012123456 7 1234567890123456789012345678901212345678901234567890123456789012123456 70 740 29 470 52 700 20 440 7 1234567890123456789012345678901212345678901234567890123456789012123456 69 730 28 460 7 1234567890123456789012345678901212345678901234567890123456789012123456 51 690 19 430 7 1234567890123456789012345678901212345678901234567890123456789012123456 68 720 27 460 7 1234567890123456789012345678901212345678901234567890123456789012123456 50 680 18 420 7 1234567890123456789012345678901212345678901234567890123456789012123456 67 710 26 450 7 1 49 670 17 420 66 700 25 450 34567890123456789012345678901212345678901234567890123456789012123456 7 12 48 660 16 410 34567890123456789012345678901212345678901234567890123456789012123456 7 12 65 700 24 440 34567890123456789012345678901212345678901234567890123456789012123456 7 12 47 650 15 410 2 34567890123456789012345678901212345678901234567890123456789012123456 7 1234567890123456789012345678901212345678901234567890123456789012123456 64 690 23 440 7 1234567890123456789012345678901212345678901234567890123456789012123456 46 640 14 400 7 1234567890123456789012345678901212345678901234567890123456789012123456 63 680 22 430 7 1234567890123456789012345678901212345678901234567890123456789012123456 45 630 13 390 62 670 21 420 1234567890123456789012345678901212345678901234567890123456789012123456 77 1234567890123456789012345678901212345678901234567890123456789012123456 44 620 12 380 61 670 20 410 7 1234567890123456789012345678901212345678901234567890123456789012123456 43 610 11 370 7 1234567890123456789012345678901212345678901234567890123456789012123456 60 660 19 410 7 1234567890123456789012345678901212345678901234567890123456789012123456 42 600 10 360 59 650 18 400 7 1 2 34567890123456789012345678901212345678901234567890123456789012123456 7 1234567890123456789012345678901212345678901234567890123456789012123456 41 600 9 350 58 640 17 390 7 1234567890123456789012345678901212345678901234567890123456789012123456 7 1 40 57 640 16 380 590 8 340 7 1234567890123456789012345678901212345678901234567890123456789012123456 56 630 15 380 34567890123456789012345678901212345678901234567890123456789012123456 7 12 39 580 7 330 2 34567890123456789012345678901212345678901234567890123456789012123456 7 1234567890123456789012345678901212345678901234567890123456789012123456 55 620 14 370 38 570 6 320 7 1234567890123456789012345678901212345678901234567890123456789012123456 7 1 37 54 610 13 360 560 5 310 34567890123456789012345678901212345678901234567890123456789012123456 7 12 34567890123456789012345678901212345678901234567890123456789012123456 7 12 53 610 12 360 36 560 4 300 34567890123456789012345678901212345678901234567890123456789012123456 7 12 52 600 11 350 34567890123456789012345678901212345678901234567890123456789012123456 7 12 35 550 3 280 7 1234567890123456789012345678901212345678901234567890123456789012123456 51 600 10 340 34 540 2 270 7 1234567890123456789012345678901212345678901234567890123456789012123456 7 1234567890123456789012345678901212345678901234567890123456789012123456 50 590 9 330 33 540 1 250 2 34567890123456789012345678901212345678901234567890123456789012123456 7 1234567890123456789012345678901212345678901234567890123456789012123456 49 590 8 320 7 1234567890123456789012345678901212345678901234567890123456789012123456 32 530 0 240 7 1234567890123456789012345678901212345678901234567890123456789012123456 48 580 7 310 7 1234567890123456789012345678901212345678901234567890123456789012123456 31 520 21 220 7 1 47 570 6 300 2 34567890123456789012345678901212345678901234567890123456789012123456 7 1234567890123456789012345678901212345678901234567890123456789012123456 30 510 22 210 46 570 5 290 7 1234567890123456789012345678901212345678901234567890123456789012123456 29 510 23 and 200 7 1234567890123456789012345678901212345678901234567890123456789012123456 45 560 4 270 7 1234567890123456789012345678901212345678901234567890123456789012123456 below 7 1 44 550 3 260 2 34567890123456789012345678901212345678901234567890123456789012123456 1234567890123456789012345678901212345678901234567890123456789012123456 77 1234567890123456789012345678901212345678901234567890123456789012123456 43 550 2 250 7 1234567890123456789012345678901212345678901234567890123456789012123456 42 540 1 240 7 1234567890123456789012345678901212345678901234567890123456789012123456 7 1 41 540 0 230 2 34567890123456789012345678901212345678901234567890123456789012123456 1234567890123456789012345678901212345678901234567890123456789012123456 77 1 40 530 21 220 34567890123456789012345678901212345678901234567890123456789012123456 7 12 39 520 22 210 7 1234567890123456789012345678901212345678901234567890123456789012123456 34567890123456789012345678901212345678901234567890123456789012123456 7 12 38 520 23 and 200 2 34567890123456789012345678901212345678901234567890123456789012123456 7 1 7 1234567890123456789012345678901212345678901234567890123456789012123456 below 1234567890123456789012345678901212345678901234567890123456789012123456 77 1234567890123456789012345678901212345678901234567890123456789012123456 12345678901234567890123456789012123456789012345678901234567890121234567

Score Charts

82

Copyright © 2005 Thomson Peterson’s, a part of The Thomson Corporaton SAT is a registered trademark of the College Entrance Examination Board, which was not involved in the production of and does not endorse this product.

Related Documents

Sat Practise Test 2
May 2020 12
Sat Practise Test 1
May 2020 12
Sat Test
November 2019 12
Sat Reasoning Test Lecture 2
November 2019 10
Sat 2
October 2019 24